Sie sind auf Seite 1von 286

m m m m

eb eb eb eb
oo oo oo oo
k sf ks ksf ks
re fre re fre
e. e. e. e .n e.
ne ne ne e ne
t t t t t
m m m m
eb eb eb eb
oo oo oo oo
ks ks ks ks
fre fre fre fre
e. e. e. e. e.
n et ne ne ne ne
t t t t
m m m m
eb eb eb eb
oo oo oo oo
ks ks k ks
fre f re
sf
re fre
e. e. e. e. e.
ne ne n ne ne
t t et t t
m m m m
eb eb eb eb
oo oo oo oo
k sf ks ksf ks
re fre re f re
e. e. e. e .n e.
ne ne ne e ne
t t t t t
m m m m
and SBAs
eb eb eb eb
oo oo oo oo
ks ks ks ks
fre fre fre fre
e. e. e. e. e.
n et ne ne ne ne
t t t t
m m m m
eb eb eb eb
Part 2 MRCOG: 500 EMQs

oo oo oo oo
ks ks k ks
fre f re
sf
re fre
e. e. e. e. e.
ne ne n ne ne
t t et t t
t

t
ne

ne

ne
e.

e.

e.
re

fre

fre
f Part 2 MRCOG: 500 EMQs
ks

ks

ks
oo

oo

oo
and SBAs
eb

eb

eb
m

m
t

t
e

ne

ne
.n

e.

e.
Andrew Sizer
e
re

fre

re
Shrewsbury and Telford Hospital NHS Trust and Keele University School of Medicine
sf

sf
ks
Bidyut Kumar
k

k
Wrexham Maelor Hospital, Betsi Cadwaladr University Health Board
oo

oo

oo
Guy Calcott
eb

eb

eb
Shrewsbury and Telford Hospital NHS Trust
m

m
t

et
ne

ne

n
e.

e.

e.
fre

fre

fre
ks

ks

ks
oo

oo

oo
eb

eb

eb
m

m
t

t
ne

ne

ne
e.

e.

e.
re

re

fre
sf

f
ks

ks
k
oo

oo

oo
eb

eb

eb
m

m
t

et

t
ne

ne
n
e.

e.

e.
t

t
ne

ne

ne
e.

e.

e.
re

fre

fre
f
ks

ks

ks
oo

oo

oo
University Printing House, Cambridge CB2 8BS, United Kingdom
eb

eb

eb
One Liberty Plaza, 20th Floor, New York, NY 10006, USA
m

m
477 Williamstown Road, Port Melbourne, VIC 3207, Australia
314–321, 3rd Floor, Plot 3, Splendor Forum, Jasola District Centre, New Delhi – 110025, India
t

t
79 Anson Road, #06-04/06, Singapore 079906
e

ne

ne
.n

e.

e.
Cambridge University Press is part of the University of Cambridge.
e
re

fre

re
It furthers the University’s mission by disseminating knowledge in the pursuit of education,
learning, and research at the highest international levels of excellence.
sf

sf
ks
www.cambridge.org
k

k
oo

oo

oo
Information on this title: www.cambridge.org/9781108709712
DOI: 10.1017/9781108627801
eb

eb

eb
© Andrew Sizer, Bidyut Kumar and Guy Calcott 2019
m

m
This publication is in copyright. Subject to statutory exception and to the provisions of relevant
collective licensing agreements, no reproduction of any part may take place without the written
permission of Cambridge University Press.
t

et
First published 2019
ne

ne

n
Printed and bound in Great Britain by Clays Ltd, Elcograf S.p.A.
e.

e.

e.
A catalogue record for this publication is available from the British Library.
fre

fre

re
ISBN 978–1-108–70971-2 Paperback
f
ks

ks

ks
Cambridge University Press has no responsibility for the persistence or accuracy of URLs for
oo

oo

oo

external or third-party internet websites referred to in this publication and does not guarantee
that any content on such websites is, or will remain, accurate or appropriate.
eb

eb

eb

Every effort has been made in preparing this book to provide accurate and up-to-date
m

information that is in accord with accepted standards and practice at the time of publication.
Although case histories are drawn from actual cases, every effort has been made to disguise
the identities of the individuals involved. Nevertheless, the authors, editors and publishers
can make no warranties that the information contained herein is totally free from error, not
t

t
least because clinical standards are constantly changing through research and regulation.
ne

ne

ne

The authors, editors and publishers therefore disclaim all liability for direct or consequential
damages resulting from the use of material contained in this book. Readers are strongly
e.

e.

e.

advised to pay careful attention to information provided by the manufacturer of any drugs or
re

re

fre

equipment that they plan to use.


sf

f
ks

ks
k
oo

oo

oo
eb

eb

eb
m

m
t

et

t
ne

ne
n
e.

e.

e.
t

t
ne

ne

ne
e.

e.

e.
re

fre

fre
Contents
ks
f

ks

ks
oo

oo

oo
Foreword  vii
eb

eb

eb
Preface  ix
m

m
Author profiles  xi
Acknowledgements  xiii
Normal ranges (non-pregnant) used in the MRCOG   xv
t

t
Abbreviations  xvii
e

ne

ne
.n

Introduction  xix

e.

e.
e
re

fre

re
sf

sf
Module 1 Clinical skills 1 Module 13 Gynaecological
ks
k

k
problems 155
Module 2 Teaching and
oo

oo

oo
assessment 7 Module 14 Subfertility 183
eb

eb

eb
Module 3 IT, clinical governance Module 15 Sexual and reproductive
m

m
and research 14 health 199
Module 5 Core surgical skills 21 Module 16 Early pregnancy
care 209
t

et
Module 6 Postoperative care 30
ne

ne

n
Module 17 Gynaecological
Module 7 Surgical procedures 41
e.

e.

e.
oncology 225
fre

fre

re
Module 8 Antenatal care 52
Module 18 Urogynaecology and pelvic
f
floor problems 244
ks

ks

ks
Module 9 Maternal medicine 91
oo

oo

oo

Module 10 Management of
labour 122
eb

eb

eb

Module 11 Management of Index  265


m

delivery 133
Module 12 Postpartum
problems 144
t

t
ne

ne

ne
e.

e.

e.
re

re

fre
sf

f
ks

ks
k
oo

oo

oo
eb

eb

eb
m

v
t

et

t
ne

ne
n
e.

e.

e.
t

t
ne

ne

ne
e.

e.

e.
re

fre

fre
Foreword
ks
f

ks

ks
oo

oo

oo
Membership of the Royal College of Obstetricians and Gynaecologists (MRCOG) is a
eb

eb

eb
highly regarded qualification throughout the world and confirms that the successful
candidate has achieved a widely respected standard of knowledge, skills, attitudes and
m

m
competencies in the practice of obstetrics and gynaecology. The award of MRCOG is
made after successfully passing all three parts of the MRCOG examination. The Part 2
MRCOG is designed to test the skills necessary to pass from core clinical training (ST1–
t

t
e

ne

ne
ST5) to higher specialist training (ST6 and ST7), and represents a significant hurdle in
.n

this transition.

e.

e.
e

This book of practice questions is an invaluable resource for candidates preparing


re

fre

re
for the Part 2 MRCOG examination. Written by experienced examiners and members
sf

sf
of RCOG examination subcommittees, this book gives candidates the most relevant
ks
k

k
and authentic practice in preparation for the examination of all the currently available
oo

oo

oo
resources. The authors have vast expertise in writing examination questions and coaching
candidates through courses, and therefore this book represents the most relevant exami-
eb

eb

eb
nation preparation material available to date. The authors make very clear that this book
m

m
should be used in addition to the standard revision resources as recommended by the
RCOG but have helpfully referenced each and every explanation of the correct answer to
enable the candidate to focus their revision of each particular topic.
t

et
This resource should become an essential part of examination preparation for all can-
ne

ne

didates attempting the Part 2 MRCOG examination.

n
e.

e.

e.
fre

fre

re
Dr Lisa Joels MB ChB MD FRCOG FHEA
Chair of the RCOG Examination and Assessment Committee 2015–18
f
ks

ks

ks
oo

oo

oo
eb

eb

eb
m

m
t

t
ne

ne

ne
e.

e.

e.
re

re

fre
sf

f
ks

ks
k
oo

oo

oo
eb

eb

eb
m

vii
t

et

t
ne

ne
n
e.

e.

e.
t

t
ne

ne

ne
e.

e.

e.
re

fre

fre
Preface
ks
f

ks

ks
oo

oo

oo
The current format of the Part 2 MRCOG examination is now well established with the
eb

eb

eb
change to written papers containing single best answer (SBAs) and extended matching
questions (EMQs) commencing in March 2015.
m

m
The Part 2 MRCOG examination is primarily concerned with testing candidates’
knowledge of the entire specialty of obstetrics and gynaecology as defined by the Royal
College of Obstetricians and Gynaecologists (RCOG) curriculum.
t

t
e

ne

ne
The new Part 3 examination now provides the clinical assessment.
.n

It is always preferable to enter an examination having had ample opportunity to prac-

e.

e.
e

tise the type of questions with which one will be faced. To this end, we have produced this
re

fre

re
book containing 250 SBA and 250 EMQ questions.
sf

sf
We have mapped the questions across all the modules of the curriculum that appear
ks
k

k
in the Part 2 MRCOG examination and have used the following sources as our primary
oo

oo

oo
references:
eb

eb

eb
• RCOG guidelines
• National Institute for Health and Care Excellence (NICE) guidelines
m

m
• Articles in The Obstetrician & Gynaecologist.
The styles of the 500 questions are different, but this will mimic the actual examina-
tion, since numerous authors have contributed to the Part 2 MRCOG question bank.
t

et
ne

ne

In this book, we have tried to conform to the style of questions found in the Part 2

n
MRCOG examination but have deliberately separated the questions into the different
e.

e.

e.
modules of the syllabus. In this way, candidates will be able to test their knowledge in
fre

fre

re
each of the modules after they have completed the necessary reading for that particular
f
module. For each answer, we have provided a brief explanation and a reference to allow
ks

ks

ks
further or more in-depth reading of that subject. The explanations given here are not
oo

oo

oo

meant to replace the wider reading of the subject that is required to attain the level neces-
sary to pass the Part 2 MRCOG examination.
eb

eb

eb

Knowledge accumulates, practice alters and guidelines change. We will be grateful for
m

feedback.
We hope that candidates for the Part 2 MRCOG will find this book helpful in their
preparation for the Part 2 MRCOG examination.
t

t
ne

ne

ne
e.

e.

e.
re

re

fre
sf

f
ks

ks
k
oo

oo

oo
eb

eb

eb
m

ix
t

et

t
ne

ne
n
e.

e.

e.
t

t
ne

ne

ne
e.

e.

e.
re

fre

fre
Author profiles
ks
f

ks

ks
oo

oo

oo
Andrew Sizer
eb

eb

eb
Andrew Sizer is a Consultant Obstetrician and Gynaecologist at the Shrewsbury and
m

m
Telford Hospital NHS Trust and Senior Lecturer at Keele University School of Medicine.
He is currently RCOG College Tutor for the Trust and Undergraduate Lead for Women’s
Health at the Shropshire campus for Keele University. Within the Postgraduate School
of Obstetrics and Gynaecology in Health Education England, West Midlands, he is the
t

t
e

ne

ne
Chair of Intermediate Training (ST3–5). He is the immediate past Chair of the Part 1
.n

MRCOG examination committee and is current Chair of the standard setting committee

e.

e.
e

and Honorary Deputy Director of Conferences at the RCOG. He was an examiner for the
re

fre

re
Part 2 MRCOG OSCE and is a current examiner for the Part 3 MRCOG clinical assess-
sf

sf
ks
ment. He is the lead author of two existing books for MRCOG examination preparation:
k

k
SBAs for the Part 1 MRCOG (2012) and Part 2 MRCOG: Single Best Answer Questions
oo

oo

oo
(2016). He is also the developer of the andragOG.co.uk website, where a variety of other
eb

eb

eb
questions in a similar format are available.
m

m
Bidyut Kumar
Bid Kumar was appointed as a Consultant Obstetrician and Gynaecologist in 2001.
He has been a RCOG tutor and a member of the Wales Deanery Specialty Training
t

et
Committee. He is an honorary lecturer at Cardiff University Medical School and an hon-
ne

ne

n
orary Senior Lecturer at Bangor University. He is a current Part 3 MRCOG examiner
e.

e.

e.
and has a number of current and former roles at the RCOG including the Part 2 course
fre

fre

re
faculty, Part 2 MRCOG EMQ subcommittee and Green-top Guideline committee. He is
Editor-in-Chief of Ultrasound, the journal of the British Medical Ultrasound Society, and
f
ks

ks

ks
an Associate Editor of the The Obstetrician & Gynaecologist. He actively contributes to
oo

oo

oo

the education and continued professional development of many healthcare professionals.


Bid is an editor-author of Fetal Medicine, a textbook of the RCOG’s Advanced Skills series
eb

eb

eb

(2016) and a co-author of Tasks for Part 3 MRCOG Clinical Assessment (2018). Bid also
works for the National Guideline Alliance (NICE) as a topic lead for the review of many
m

obstetric guidelines.

Guy Calcott
t

t
ne

ne

ne

Guy Calcott is a newly appointed Consultant Obstetrician and Gynaecologist at the


Shrewsbury and Telford Hospital NHS Trust with a special interest in high-risk obstetrics,
e.

e.

e.

maternal medicine and early pregnancy care. He qualified with a distinction in Medicine
re

re

fre

and Surgery from Imperial College School of Medicine in 2009 and a First Class Honours
sf

Bachelor of Science in Surgery and Anaesthesia. He completed foundation training and


ks

ks
k

early obstetrics and gynaecology training at North West Thames before relocating to the
oo

oo

oo

West Midlands in 2013. He completed the MRCOG in 2015 and has been presenting and
teaching on Part 2 MRCOG courses two to three times per year since 2016.
eb

eb

eb
m

xi
t

et

t
ne

ne
n
e.

e.

e.
t

t
ne

ne

ne
e.

e.

e.
re

fre

fre
Acknowledgements
ks
f

ks

ks
oo

oo

oo
The authors would like to acknowledge the contribution of Mr Sujeewa Fernando,
eb

eb

eb
Consultant Obstetrician and Gynaecologist, Wrexham Maelor Hospital, to the questions
included in module 18.
m

m
We would also like to thank the following doctors for being our ‘proofreaders’ dur-
ing the first drafts of the manuscript and for their useful feedback: Dr Joanne Ritchie
MRCOG, Dr Banchhita Sahu MRCOG, Dr Michael Algeo MRCOG, Dr James Castleman
t

t
e

ne

ne
MRCOG, Dr Hector Georghiu MRCOG and Dr Pedro Melo MRCOG.
.n

e.

e.
e
re

fre

re
sf

sf
ks
k

k
oo

oo

oo
eb

eb

eb
m

m
t

et
ne

ne

n
e.

e.

e.
fre

fre

f re
ks

ks

ks
oo

oo

oo
eb

eb

eb
m

m
t

t
ne

ne

ne
e.

e.

e.
re

re

fre
sf

f
ks

ks
k
oo

oo

oo
eb

eb

eb
m

xiii
t

et

t
ne

ne
n
e.

e.

e.
t

t
ne

ne

ne
e.

e.

e.
re

fre

fre
Normal ranges (non-pregnant)
ks
f

ks

ks
used in the MRCOG
oo

oo

oo
eb

eb

eb
m

m
Haematology
Haemoglobin (female): 115–160 g/l
Haematocrit (female): 37–47%
4.0 × 109–11.0 × 109/l
t

t
Total white cell count:
e

ne

ne
Platelets: 150 × 109–400 × 109/l
.n

e.

e.
e

Clinical chemistry
re

fre

re
Sodium: 135–145 mmol/l
sf

sf
ks
Potassium: 3.5–5.2 mmol/l
k

k
Urea: 2.5–7.0 mmol/l
oo

oo

oo
Creatinine: 60–120 μmol/l
eb

eb

eb
Liver function
m

m
Albumin: 35–50 g/l
Total bilirubin: 0–22 μmol/l
Alkaline phosphatase: 40–130 IU/l
t

et
Alanine aminotransferase (ALT): 0–40 IU/l
ne

ne

n
γ-Glutamyl transferase: 0–75 U/l
e.

e.

e.
Bile acids: 0–14 μmol/l
fre

fre

re
Endocrine
f
ks

ks

ks
Thyroid-stimulating hormone (TSH): 0.35–5.5 mU/l
Free T4: 11–24 pmol/l
oo

oo

oo

Follicle-stimulating hormone (FSH): 1–11 IU/l


eb

eb

eb

Luteinising hormone (LH): 2–13 IU/l


Testosterone (female): 0.5–3.0 nmol/l
m

Testosterone (male): 8–30 nmol/l


Prolactin: 0–520 mU/l
Free androgen index: 0.5–6.5%
t

t
ne

ne

ne

Sex hormone-binding globulin: 18–144 nmol/l


e.

e.

e.

Cancer antigen 125 (CA125): 0–35 IU/ml


re

re

fre

Please note: normal ranges can vary among laboratories.


sf

f
ks

ks
k
oo

oo

oo
eb

eb

eb
m

xv
t

et

t
ne

ne
n
e.

e.

e.
t

t
ne

ne

ne
e.

e.

e.
re

fre

fre
Abbreviations
ks
f

ks

ks
oo

oo

oo
ACE angiotensin-converting FIGO International Federation of
eb

eb

eb
enzyme Gynecology and Obstetrics
AED anti-epileptic drug FSH follicle-stimulating hormone
m

m
AFP α-fetoprotein FSRH Faculty of Sexual and
ALT alanine transaminase Reproductive Healthcare
AMH anti-Müllerian hormone GBS group B Streptococcus
t

t
e

ne

ne
ARB angiotensin-receptor GnRH gonadotropin-releasing
.n

blocker hormone

e.

e.
e

AREDV absent or reversed end- GTG Green-top Guideline


re

fre

re
diastolic velocity HAART highly active antiretroviral
sf

sf
BASHH British Association for treatment
ks
k

k
Sexual Health and HIV HBV hepatitis B virus
oo

oo

oo
BAUS British Association of hCG human chorionic
Urological Surgeons gonadotropin
eb

eb

eb
BHIVA British HIV Association HELLP haemolysis, elevated liver
m

m
BMI body mass index enzymes and low platelets
bpm beats per minute HFEA Human Fertilisation and
CBT cognitive behavioural Embryology Authority
t

et
therapy HIV human immunodeficiency
ne

ne

n
cCTG computerised CTG virus
e.

e.

e.
CEA carcinoembryonc antigen HRT hormone replacement
fre

fre

re
CI confidence interval therapy
COCP combined oral contraceptive HSDD hypoactive sexual desire
f
ks

ks

ks
pill disorder
CRP C-reactive protein HSG hysterosalpingogram
oo

oo

oo

CT computerised tomography HyCoSy hystero-contrast-


eb

eb

eb

CTG cardiotocograph salpingography


CTPA computed tomography IAP intrapartum antibiotic
m

pulmonary angiography prophylaxis


CXR chest X-ray ICSI intracytoplasmic sperm
DCDA dichorionic diamniotic injection
t

t
ne

ne

ne

DKA diabetic ketoacidosis IGFBP-1 insulin-like growth factor-


DVT deep vein thrombosis binding protein-1
e.

e.

e.

EFW estimated fetal weight IUCD intrauterine contraceptive


re

re

fre

EMQ extended matching question device


sf

ESHRE European Society of IVF in vitro fertilisation


ks

ks
k

Human Reproduction and LAM lactational amenorrhoea


oo

oo

oo

Embryology method
FBC full blood count LAVH laparoscopic-assisted
eb

eb

eb

FBS fetal blood sampling vaginal hysterectomy


m

FGM female genital mutilation LDH lactate dehydrogenase

xvii
t

et

t
ne

ne
n
e.

e.

e.
t

t
xviii Abbreviations

ne

ne

ne
e.

e.

e.
re

fre

fre
LFT liver function test PCOS polycystic ovarian syndrome
LH luteinising hormone PE pulmonary embolism
f
ks

ks

ks
LMWH low-molecular-weight PET positron emission
heparin tomography
oo

oo

oo
LNG-IUS levonorgestrel-releasing PGE2 prostaglandin E2
eb

eb

eb
intrauterine system PID pelvic inflammatory disease
MBRRACE Mothers and Babies: PPROM preterm prelabour rupture
m

m
Reducing Risk through of membranes
Audits and Confidential PTS post-thrombotic syndrome
Enquiries PTSD post-traumatic stress
t

t
e

ne

ne
MCA middle cerebral artery disorder
.n

MCDA monochorionic diamniotic PUQE pregnancy-unique

e.

e.
e

MOGCT malignant ovarian germ cell quantification of emesis


re

fre

re
tumour RCOG Royal College of
sf

sf
MPA medroxyprogesterone Obstetricians and
ks
k

k
acetate Gynaecologists
oo

oo

oo
MRKH Mayer–Rokitansky–Kuster– RMI risk of malignancy index
Hauser RCVS reversible cerebral
eb

eb

eb
MRI magnetic resonance imaging vascoconstriction syndrome
m

m
MRSA methicillin-resistant SBA single best answer
Staphylococcus aureus SGA small for gestational age
NAAT nucleic acid amplification ST speciality trainee
t

et
test STV short-term variation
ne

ne

NCEPOD National Confidential TCRE transcervical resection of

n
e.

e.

e.
Enquiry into Patient the endometrium
Outcome and Death TENS transcutaneous electrical
fre

fre

re
NHSLA National Health Service nerve stimulation
f
ks

ks

ks
Litigation Authority TTP thrombotic
NICE National Institute for Health thrombocytopenic purpura
oo

oo

oo

and Care Excellence TTTS twin-to-twin transfusion


eb

eb

eb

NSAID non-steroidal anti- syndrome


inflammatory drug U&E urea and electrolytes
m

NVP nausea and vomiting in UDCA ursodeoxycholic acid


pregnancy UKMEC UK Medical Eligibility
OASIS obstetric anal sphincter Criteria for Contraceptive
t

t
ne

ne

ne

injuries Use
OHSS ovarian hyperstimulation UTI urinary tract infection
e.

e.

e.

syndrome V/Q ventilation/perfusion


re

re

fre

OR odds ratio VBAC vaginal birth after a


sf

PAEC progesterone receptor caesarean


ks

ks

modulator-associated VIN vulval intraepithelial


k
oo

oo

oo

endometrial changes neoplasia


PAMG-1 placental α-microglobulin-1 VTE venous thromboembolism
eb

eb

eb

PCA patient-controlled analgesia WPBA workplace-based assessment


m

PCO2 partial pressure of carbon WHO World Health Organization


dioxide
t

et

t
ne

ne
n
e.

e.

e.
t

t
ne

ne

ne
e.

e.

e.
re

fre

fre
Introduction
ks
f

ks

ks
oo

oo

oo
Membership of the Royal College of Obstetricians and Gynaecologists (MRCOG) is an
eb

eb

eb
essential component of specialist training in obstetrics and gynaecology in the UK.
Possession of the MRCOG is also highly regarded by doctors working in other coun-
m

m
tries across the world, and many see the MRCOG as the ‘gold standard’ qualification in
obstetrics and gynaecology.
Worldwide, there are over 16,000 Fellows and Members of the RCOG.
t

t
e

ne

ne
.n

Format of the Part 2 MRCOG written examination

e.

e.
e

The Part 2 examination consists of two written papers with a short break (approximately
re

fre

re
30–60 minutes) between them.
sf

sf
ks
The two papers are identical in format and carry the same number of marks.
k

k
Each paper consists of 50 SBAs and 50 EMQs, but the weighting of the two question
oo

oo

oo
types is different, with the SBA component being worth 40% of the marks and the EMQ
eb

eb

eb
component 60%.
Each paper is of 3 hours’ duration, but in view of the weighting, the RCOG recom-
m

m
mends that candidates spend approximately 70 minutes on the SBA component and 110
minutes on the EMQ component. The only time warnings are 30 minutes and 10 min-
utes before the end of the examination, so candidates must take responsibility for their
t

et
ne

ne

own time management. Candidates must also remember to allow enough time to transfer

n
their answers onto the computer marking sheets, as there is no extra time to do this.
e.

e.

e.
Traditionally, one paper is mainly obstetrics and the other mainly gynaecology, but
fre

fre

re
there is no guarantee that this is this case, and, theoretically, any type of question or sub-
ject could appear in either paper. f
ks

ks

ks
oo

oo

oo

Using this book


eb

eb

eb

We hope that our 500 questions give a broad coverage of the syllabus and that you will
find the different styles of question writing useful. However, as obstetrics and gynaecol-
m

ogy is such a vast subject, it is not possible for 500 questions to cover every facet of the
specialty.
Core modules 4 and 19 are not covered by the Part 2 examination so no questions on
t

t
these two modules have been included.
ne

ne

ne

Different modules cover different proportions of the curriculum. The two biggest
e.

e.

e.

modules in terms of subject area are antenatal care and gynaecological problems. These
re

re

fre

modules therefore have the greatest number of questions in the book, with other modules
appropriately weighted according to their size.
sf

f
ks

ks

We hope you find this book helpful as part of your examination preparation.
k
oo

oo

oo
eb

eb

eb
m

xix
t

et

t
ne

ne
n
e.

e.

e.
t

t
ne

ne

ne
e.

e.

e.
re

fre

fre
Clinical skills
Module
f
ks

ks

ks
1
oo

oo

oo
eb

eb

eb
m

m
t

t
e

ne

ne
SBAs
.n

e.

e.
e

1. A 28-year-old woman is admitted to the gynaecology ward with persistent nausea


re

fre

re
and vomiting in early pregnancy. Her serum urea and electrolytes are as follows:
sf

sf
ks
k

k
oo

oo

oo
Sodium 130 mmol/l
Potassium 3.0 mmol/l
eb

eb

eb
Urea 4.3 mmol/l
m

m
Creatinine 100 μmol/l

What would be the most appropriate agent for electrolyte replacement therapy?
t

et
ne

ne

A. Intravenous potassium chloride 0.3% with glucose 5% solution

n
B. Intravenous potassium chloride 0.1% with sodium chloride 0.45% solution
e.

e.

e.
C. Intravenous potassium chloride 0.3% with sodium chloride 0.9% solution
fre

fre

re
D. Oral potassium bicarbonate 500 mg with potassium acid tartrate 300 mg
E. Oral potassium chloride 600 mg tablets f
ks

ks

ks
oo

oo

oo

2. What are the constituents of a litre of Hartmann’s solution (in mmol)?


eb

eb

eb

Sodium Potassium Calcium Bicarbonate Chloride


m

A 131 5 2 29 111
B 136 7.7 2 30 120
C 140 7.5 5 29 111
t

t
ne

ne

ne

D 145 5 2 29 150
e.

e.

e.

E 150 7.5 10 29 111


re

re

fre
sf

f
ks

ks
k
oo

oo

oo
eb

eb

eb
m

1
t

et

t
ne

ne
n
e.

e.

e.
t

t
2 Module 1

ne

ne

ne
e.

e.

e.
re

fre

fre
3. A 34-year-old woman is admitted for an elective caesarean section. The woman is
a known carrier of methicillin-­resistant Staphylococcus aureus (MRSA).
f
ks

ks

ks
Which are the most appropriate prophylactic antibiotics to use in this situation?
oo

oo

oo
Cefuroxime Clindamycin Metronidazole Teicoplanin Vancomycin
eb

eb

eb
A ✓ ✓
m

m
B ✓ ✓
C ✓ ✓
D ✓ ✓
t

t
e

ne

ne
E ✓ ✓
.n

e.

e.
e

4. A 32-year-old multiparous woman has undergone an elective caesarean section


re

fre

re
under spinal anaesthesia at term. The spinal anaesthetic included intrathecal
sf

sf
ks
morphine.
k

k
What is the minimum regime of postoperative clinical observations required
oo

oo

oo
for this woman?
eb

eb

eb
A. Continue observations every 30 minutes for 2 hours
B. Continue observations every 30 minutes for 6 hours
m

m
C. Continue observations every hour for 12 hours
D. Continue observations every hour for 24 hours
E. Continue observations every hour for 36 hours
t

et
ne

ne

n
5. Each year, there are approximately 700,000 deliveries in England and Wales.
e.

e.

e.
What proportion of these women will have undergone female genital mutilation
fre

fre

re
(FGM)?
A. 0.1% f
ks

ks

ks
B. 1.5%
oo

oo

oo

C. 3%
D. 4.5%
eb

eb

eb

E. 6%
m

m
t

t
ne

ne

ne
e.

e.

e.
re

re

fre
sf

f
ks

ks
k
oo

oo

oo
eb

eb

eb
m

m
t

et

t
ne

ne
n
e.

e.

e.
t

t
Clinical skills 3

ne

ne

ne
e.

e.

e.
re

fre

fre
EMQs
f
ks

ks

ks
Options for questions 6–8
oo

oo

oo
A 500–750 in 1000
eb

eb

eb
B 250 in 1000
C 10 in 1000
m

m
D 7.5 in 1000
E 5 in 1000
t

t
F 2–3 in 1000
e

ne

ne
.n

G 2 in 1000

e.

e.
e

H 1 in 1000
re

fre

re
I 0.5 in 1000
sf

sf
J 0.2 in 1000
ks
k

k
oo

oo

oo
Each of the following clinical scenarios relates to the process of consenting for a treat-
eb

eb

eb
ment procedure. For each patient, select the single most appropriate option from the list
above. Each option may be used once, more than once or not at all.
m

m
6. A 29-year-old woman attends the gynaecology clinic wishing to discuss
laparoscopic sterilisation as she wants a permanent method of contraception.
t

et
During counselling, she enquires about the risk of a serious complication.
ne

ne

n
7. A junior specialty trainee is about to see a 30-year-old woman who wishes to be
e.

e.

e.
sterilised. Only Filshie clips are used for sterilisation in the unit where he works.
fre

fre

re
What is the failure rate that should be quoted?
f
ks

ks

ks
8. A couple attend the clinic to discuss permanent methods of contraception. The
oo

oo

oo

woman is aged 35 years, has a body mass index (BMI) of 37 kg/m2 and has had
two caesarean sections. Her husband is aged 38 years. The woman is concerned
eb

eb

eb

about her risk of laparoscopic complications and wants her husband to consider
a vasectomy. He enquires about the chance of late contraceptive failure after
m

clearance for sterility is given following the vasectomy.


t

t
ne

ne

ne
e.

e.

e.
re

re

fre
sf

f
ks

ks
k
oo

oo

oo
eb

eb

eb
m

m
t

et

t
ne

ne
n
e.

e.

e.
t

t
4 Module 1

ne

ne

ne
e.

e.

e.
re

fre

fre
Options for questions 9 and 10

f
ks

ks

ks
A Amoxicillin 500 mg orally every 8 hours
oo

oo

oo
B Benzylpenicillin 3 g initially followed by 1.5 g 4 hourly
C Cefalexin 500 mg orally every 8 hours
eb

eb

eb
D Ceftriaxone 2 g intravenous once daily
m

m
E Clindamycin 900 mg intravenous every 8 hours
F Co-amoxiclav 625 mg orally every 8 hours
G Does not need antibiotic treatment
t

t
e

ne

ne
H Gentamicin 4 mg/kg intravenous in three divided doses
.n

e.

e.
I Metronidazole 500 mg intravenous every 8 hours
e
re

fre

re
J Tetracycline 250 mg orally four times a day
K Trimethoprim 200 mg orally twice daily
sf

sf
ks
k

k
L Vancomycin 1 g intravenous every 12 hours
oo

oo

oo
M Vancomycin 250 mg orally four times a day
eb

eb

eb
Each of the following clinical scenarios relates to the choice of management for the
m

m
prophylaxis or treatment of infection. For each patient, select the single most appropriate
management from the list above. Each option may be used once, more than once or not
at all.
t

et
ne

ne

9. A 24-year-old multiparous woman is in labour at 38 weeks’ gestation. A high

n
vaginal swab taken at 23 weeks of gestation grew group B Streptococcus in culture.
e.

e.

e.
She is severely allergic to penicillin.
fre

fre

10. A 67-year-old woman has been under treatment with various antibiotics for fre
ks

ks

ks
prolonged periods due to recurrent pneumonia. She now presents with diarrhoea
oo

oo

oo

and her stool culture has grown Clostridium difficile.


eb

eb

eb
m

m
t

t
ne

ne

ne
e.

e.

e.
re

re

fre
sf

f
ks

ks
k
oo

oo

oo
eb

eb

eb
m

m
t

et

t
ne

ne
n
e.

e.

e.
t

t
Clinical skills 5

ne

ne

ne
e.

e.

e.
re

fre

fre
Answers
f
ks

ks

ks
SBAs
oo

oo

oo
1. Answer  C  Intravenous potassium chloride 0.3% with sodium chloride 0.9%
solution
eb

eb

eb
Explanation
m

m
Oral preparations are unsuitable in the given circumstances. Option B has a very
low concentration of potassium unsuitable for treatment of hypokalaemia.
Option A contains glucose, and glucose infusions should not be used because they
t

t
e

ne

ne
can cause a further decrease in the plasma potassium concentration.
.n

e.

e.
Reference
e
re

fre

re
British National Formulary, 72. September 2016–March 2017.
sf

sf
ks
2. Answer  A  Sodium 131 mmol, potassium 5 mmol, calcium 2 mmol,
k

k
bicarbonate 29 mmol, chloride 111 mmol
oo

oo

oo
Explanation
eb

eb

eb
Hartmann’s solution is one of the commonest intravenous infusions used in day-
m

m
to-day practice on the ward. All grades of doctors must know its composition.
Reference
British National Formulary, 72. September 2016–March 2017.
t

et
ne

ne

n
3. Answer  D  Teicoplanin and vancomycin
e.

e.

e.
fre

fre

re
Explanation
For MRSA carriers, teicoplanin or vancomycin should be used.
f
ks

ks

ks
Reference
oo

oo

oo

British National Formulary, 72. Bacterial infection. September 2016–March 2017.


eb

eb

eb

4. Answer  D  Continue observations every hour for 24 hours


m

Explanation
For women who have had intrathecal opioids, there should be a minimum hourly
observation of respiratory rate, sedation and pain scores for at least 12 hours for
t

t
ne

ne

ne

diamorphine and 24 hours for morphine.


e.

e.

e.

Reference
re

re

fre

NICE. Caesarean section. NICE Clinial Guideline (CG 132). November 2011.
sf

f
ks

ks
k

5. Answer  B 1.5%
oo

oo

oo

Explanation
eb

eb

eb

Since 2008, it has been estimated that 1.5% of women each year giving birth in
England and Wales have undergone FGM.
m

Reference
Hussain S, Rymer J. Tackling female genital mutilation in the UK. The Obstetrician &
t

et

Gynaecologist 2017;19:273–8.
ne

ne
n
e.

e.

e.
t

t
6 Module 1

ne

ne

ne
e.

e.

e.
re

fre

fre
EMQs
f
ks

ks

ks
6. Answer  G  2 in 1000
oo

oo

oo
Explanation
Serious risks of laparoscopy include the overall risk of serious complications from
eb

eb

eb
diagnostic laparoscopy, which occur in approximately two women in every 1000.
m

m
7. Answer  F  2–3 in 1000
Explanation
t

t
The longest period of available follow-­up data for the most commonly used
e

ne

ne
.n

method in the UK, the Filshie clip, suggests a failure rate of 2–3 in 1000

e.

e.
procedures at 10 years.
e
re

fre

re
8. Answer  I  0.5 in 1000
sf

sf
ks
k

k
Explanation
oo

oo

oo
Individuals should be informed that a vasectomy has an associated failure rate and
that pregnancy can occur several years after vasectomy. The contraceptive failure
eb

eb

eb
rate should be quoted as approximately 1 in 2000 (0.05%) after clearance has been
m

m
given.
References
FSRH. Male and female sterilisation, FSRH Clinical Guidance. September 2014.
t

et
ne

ne

RCOG. Diagnostic laparoscopy. RCOG Consent Advice No. 2. June 2017.

n
e.

e.

e.
9. Answer  L  Vancomycin 1 g intravenous every 12 hours
fre

fre

re
Explanation
f
ks

ks

ks
Provided a woman has not had a severe allergy to penicillin, a cephalosporin
oo

oo

oo

should be used. If there is any evidence of a severe allergy to penicillin,


vancomycin should be used.
eb

eb

eb

Reference
m

RCOG. Prevention of early-­onset neonatal group B streptococcal disease. RCOG GTG No.
36. September 2017.
t

t
10. Answer  M  Vancomycin 250 mg orally four times a day
ne

ne

ne

Explanation
e.

e.

e.

For C. difficile infection, oral administration of vancomycin is more effective.


re

re

fre

Reference
sf

f
ks

ks

British National Formulary, 72. Infection. September 2016–March 2017.


k
oo

oo

oo
eb

eb

eb
m

m
t

et

t
ne

ne
n
e.

e.

e.
t

t
ne

ne

ne
e.

e.

e.
re

fre

fre
Teaching and assessment
Module
f
ks

ks

ks
2
oo

oo

oo
eb

eb

eb
m

m
t

t
e

ne

ne
SBAs
.n

e.

e.
e

11. In medical teaching, what is the single best determinant of expertise in a subject?
re

fre

re
A. Communication skills
sf

sf
ks
B. Knowledge
k

k
C. Organisational skills
oo

oo

oo
D. Positive role modelling
eb

eb

eb
E. Technical ability
m

m
12. The SBAR format is commonly used on the delivery suite as a method to convey
critical clinical information between different healthcare professionals.
An ST5 telephones the consultant on call with the following communication.
t

et
‘Good evening, Dr Smith. This is John, the ST5, calling from the delivery suite.
ne

ne

n
I have just been to see Mrs Jones who is in labour. The CTG is pathological. This
e.

e.

e.
woman is being monitored because of a previous stillbirth. I propose to take her to
fre

fre

re
theatre for an immediate caesarean section.’
What component of SBAR is missing from this communication?
f
ks

ks

ks
A. Achievement
oo

oo

oo

B. Action
C. Alignment
eb

eb

eb

D. Alternatives
m

E. Assessment

13. Which two tiers of Miller’s pyramid are best assessed using workplace-­based
assessment tools?
t

t
ne

ne

ne

Knows Knows how Shows how Does


e.

e.

e.
re

re

fre

A ✓ ✓
✓ ✓
sf

B
ks

ks

✓ ✓
k

C
oo

oo

oo

D ✓ ✓
eb

eb

eb

E ✓ ✓
m

7
t

et

t
ne

ne
n
e.

e.

e.
t

t
8 Module 2

ne

ne

ne
e.

e.

e.
re

fre

fre
14. Which learning method is associated with the lowest retention of knowledge/
information imparted?
f
ks

ks

ks
A. Lecture
oo

oo

oo
B. Practical demonstration
C. Problem-based learning
eb

eb

eb
D. Reading a textbook
m

m
E. Small group discussion

15. What is the only type of workplace-­based assessment that has a summative role in
obstetrics and gynaecology training?
t

t
e

ne

ne
A. Case-based discussion
.n

e.

e.
B. Mini-Clinical Examination (Mini-CEX)
e

C. Objective Structured Assessment of Technical Skills (OSATS)


re

fre

re
D. Reflective writing
sf

sf
ks
E. Team observation form
k

k
oo

oo

oo
eb

eb

eb
m

m
t

et
ne

ne

n
e.

e.

e.
fre

fre

f re
ks

ks

ks
oo

oo

oo
eb

eb

eb
m

m
t

t
ne

ne

ne
e.

e.

e.
re

re

fre
sf

f
ks

ks
k
oo

oo

oo
eb

eb

eb
m

m
t

et

t
ne

ne
n
e.

e.

e.
t

t
Teaching and assessment 9

ne

ne

ne
e.

e.

e.
re

fre

fre
EMQs
f
ks

ks

ks
Options for questions 16–18
oo

oo

oo
A Acceptability
eb

eb

eb
B Appraisability
C Appraisal
m

m
D Assessment
E Construct validity
t

t
F Content validity
e

ne

ne
.n

G Educational impact

e.

e.
e

H Evaluation
re

fre

re
I Face validity
sf

sf
J Feasibility
ks
k

k
K Feedback
oo

oo

oo
L Formative assessment
eb

eb

eb
M Reliability
m

m
N Sensitivity
O Specificity
P Summative assessment
t

et
Q Validity
ne

ne

n
e.

e.

e.
The option list above relates to concepts, measurements and activities in medical edu-
fre

fre

re
cation. For each of the following descriptions, what is the single most appropriate activity,
f
measurement or concept that is being described? Each option may be used once, more
ks

ks

ks
than once or not at all.
oo

oo

oo

16. The most important criterion for a quality assessment in medical education.
eb

eb

eb

17. Pendleton’s rules provide a model of delivering this kind of activity in medical
m

education.

18. An activity that considers personal and educational development and is not
measured against any set criteria.
t

t
ne

ne

ne
e.

e.

e.
re

re

fre
sf

f
ks

ks
k
oo

oo

oo
eb

eb

eb
m

m
t

et

t
ne

ne
n
e.

e.

e.
t

t
10 Module 2

ne

ne

ne
e.

e.

e.
re

fre

fre
Options for questions 19 and 20

f
ks

ks

ks
A Buzz groups
oo

oo

oo
B Case-illustrated learning
C Circular questioning
eb

eb

eb
D Crossover groups
m

m
E Fishbowls
F Group round
G Horseshoe groups
t

t
e

ne

ne
H Problem-based learning
.n

e.

e.
I Snowball groups
e
re

fre

re
J Spiral groups
sf

sf
ks
The option list above relates to methods of small group teaching. For each of the
k

k
oo

oo

oo
following descriptions, what is the single most appropriate small group teaching method
that is being described? Each option may be used once, more than once or not at all.
eb

eb

eb
19. The group is split into two, comprising an outer circle and an inner group. The
m

m
outer circle members observe the inner group in the discussions in order to
provide feedback.
t

et
20. The group is split into pairs and given a question or topic. Pairs combine to make
ne

ne

a four, and then again to make an eight. The topics get more complex, and the

n
e.

e.

e.
groups of eight then feedback to the whole group.
fre

fre

f re
ks

ks

ks
oo

oo

oo
eb

eb

eb
m

m
t

t
ne

ne

ne
e.

e.

e.
re

re

fre
sf

f
ks

ks
k
oo

oo

oo
eb

eb

eb
m

m
t

et

t
ne

ne
n
e.

e.

e.
t

t
Teaching and assessment 11

ne

ne

ne
e.

e.

e.
re

fre

fre
Answers
f
ks

ks

ks
SBAs
oo

oo

oo
11. Answer   B Knowledge
eb

eb

eb
Explanation
Knowledge is the single best determinant of expertise in a subject. However, a
m

m
practising obstetrician and gynaecologist also requires skills in communication,
organisational ability, technical ability and teaching skills, as well as displaying the
right attitude, to make them a good role model.
t

t
e

ne

ne
Reference
.n

e.

e.
Duthie SJ, Garden AS. The teacher, the learner and the method. The Obstetrician &
e

Gynaecologist 2010;12:273–80.
re

fre

re
sf

sf
ks
12. Answer   E Assessment
k

k
oo

oo

oo
Explanation
The SBAR format (situation, background, assessment, recommendation) is also
eb

eb

eb
of use in transmitting critical information and is now commonplace on many
m

m
delivery suites:
Situation: Good evening, Dr Smith. This is John, the ST5, calling from the
delivery suite. I have just been to see Mrs Jones who is in labour. The CTG is
t

et
pathological.
ne

ne

n
Background: This woman is being monitored because of a previous stillbirth.
e.

e.

e.
Assessment: None.
fre

fre

re
Recommendation: I propose to take her to theatre for an immediate caesarean
section.
f
ks

ks

ks
Reference
oo

oo

oo

Jackson KS, Hayes K, Hinshaw K. The relevance of non-­technical skills in obstetrics and
eb

eb

eb

gynaecology. The Obstetrician & Gynaecologist 2013;15:269–74.


m

13. Answer   E  ‘Shows how’ and ‘Does’


Explanation
As the model for progression to mastery of a skill is sought, assessment is
t

t
ne

ne

ne

tailored to examine each of the pyramid’s levels: Knows (knowledge), Knows how
(competence), Shows how (performance) and Does (action). It has been suggested
e.

e.

e.

that WPBA assesses the top two levels of the pyramid: performance and action.
re

re

fre
sf

Reference
ks

ks
k

Parry-Smith W, Mahmud A, Landau A, Hayes K. Workplace-based assessment: a new


oo

oo

oo

approach to existing tools. The Obstetrician & Gynaecologist 2014;16:281–5.


eb

eb

eb
m

m
t

et

t
ne

ne
n
e.

e.

e.
t

t
12 Module 2

ne

ne

ne
e.

e.

e.
re

fre

fre
14. Answer   A Lecture

f
Explanation
ks

ks

ks
Lectures result in only 5% retention.
oo

oo

oo
Reference
eb

eb

eb
Duthie SJ, Garden AS. The teacher, the learner and the method. The Obstetrician &
m

m
Gynaecologist 2010;12:273–280

15. Answer   C  Objective Structured Assessment of Technical Skills (OSATS)


t

t
Explanation
e

ne

ne
.n

All workplace-­based assessments have a formative role. Only OSATS have a

e.

e.
summative role.
e
re

fre

re
Reference
sf

sf
ks
Parry-Smith W, Mahmud A, Landau A, Hayes K. Workplace-based assessment: a new
k

k
approach to existing tools. The Obstetrician & Gynaecologist 2014;16:281–5.
oo

oo

oo
EMQs
eb

eb

eb
16. Answer   Q Validity
m

m
Explanation
Validity is the most important criterion of a quality assessment, i.e. the extent to
t

et
which the assessment measures what it is intended to measure.
ne

ne

n
e.

e.

e.
17. Answer   K Feedback
fre

fre

re
Explanation
The two most widely accepted models of delivering feedback are Pendleton’s rules f
ks

ks

ks
and Silverman’s agenda-­led, outcome-­based analysis. Both models provide a safe
oo

oo

oo

environment, thus reducing defensiveness and increasing constructiveness.


eb

eb

eb

18. Answer   C Appraisal


m

Explanation
Appraisal considers personal development as well as educational development
and is not measured against any set criteria, nor does it contribute to a formal
t

t
ne

ne

ne

summative assessment. Appraisal is jointly developed by the trainee and trainer,


and should be confidential and non-threatening.
e.

e.

e.

Reference
re

re

fre

Shehmar M, Khan KS. A guide to the ATSM in Medical Education. Article 2: assessment,
sf

f
ks

ks

feedback and evaluation. The Obstetrician & Gynaecologist 2010;12:119–25.


k
oo

oo

oo
eb

eb

eb
m

m
t

et

t
ne

ne
n
e.

e.

e.
t

t
Teaching and assessment 13

ne

ne

ne
e.

e.

e.
re

fre

fre
19. Answer   E Fishbowls

f Explanation
ks

ks

ks
The usual fishbowl configuration has an inner group discussing an issue or
oo

oo

oo
topic while the outer group listens, looking for themes, patterns or soundness of
argument, or uses a group behaviour checklist to give feedback to the group on its
eb

eb

eb
functioning. The roles may then be reversed.
m

m
20. Answer   I  Snowball groups
Explanation
t

t
e

ne

ne
Snowball groups (or pyramids) are an extension of buzz groups. Pairs join up to
.n

form fours, then fours to eights. These groups of eight report back to the whole

e.

e.
e

group. This developing pattern of group interaction can ensure comprehensive


re

fre

re
participation, especially when it starts with individuals writing down their ideas
sf

sf
before sharing them. To avoid students becoming bored with repeated discussion
ks
k

k
of the same points, it is a good idea to use increasingly sophisticated tasks as the
oo

oo

oo
groups gets larger.
eb

eb

eb
Reference
m

m
Jaques D. Teaching small groups. British Medical Journal 2003;326:492–4.
t

et
ne

ne

n
e.

e.

e.
fre

fre

re
f
ks

ks

ks
oo

oo

oo
eb

eb

eb
m

m
t

t
ne

ne

ne
e.

e.

e.
re

re

fre
sf

f
ks

ks
k
oo

oo

oo
eb

eb

eb
m

m
t

et

t
ne

ne
n
e.

e.

e.
t

t
ne

ne

ne
e.

e.

e.
re

fre

fre
IT, clinical governance
Module
f
ks

ks

ks
3 and research
oo

oo

oo
eb

eb

eb
m

m
t

t
e

ne

ne
SBAs
.n

e.

e.
e

21. A 45-year-old woman is admitted to the gynaecology ward for a planned


re

fre

re
hysterectomy for endometrial hyperplasia with atypia. A pregnancy test has
sf

sf
not been carried out prior to theatre, and during the course of the laparotomy a
ks
k

k
mass is detected in the right fallopian tube that has an appearance suggestive of
oo

oo

oo
an unruptured ectopic pregnancy. A catheter specimen of urine is then used to
perform a pregnancy test, which is found to be positive.
eb

eb

eb
What would be the correct course of action?
m

m
A. Contact a colleague for a second opinion
B. Contact the patient’s partner for authorisation
C. Continue with the hysterectomy including removal of the right tube
t

et
D. Remove the right fallopian tube for histological confirmation
ne

ne

n
E. Stop the surgery and reschedule
e.

e.

e.
fre

fre

re
22. What type of consent is required for medical students performing pelvic
examinations on anaesthetised women?
f
ks

ks

ks
A. Consent from the consultant responsible for the patient
oo

oo

oo

B. Formal consent is not required as this is part of clinical care


C. Verbal consent from the woman
eb

eb

eb

D. Verbal consent witnessed by a member of the theatre staff


m

E. Written consent

23. A pregnant woman attends the accident and emergency department with
abdominal pain and vaginal bleeding. It is 7 weeks since her last menstrual
t

t
ne

ne

ne

period. She subsequently becomes tachycardic and hypotensive, and collapses.


Plans are made to take her to the operating theatre.
e.

e.

e.

As per the National Confidential Enquiry into Patient Outcome and Death
re

re

fre

(NCEPOD), which category of intervention would this be?


sf

f
ks

ks

A. 1
k

B. 2
oo

oo

oo

C. 3
eb

eb

eb

D. 4
E. U
m

14
t

et

t
ne

ne
n
e.

e.

e.
t

t
IT, clinical governance and research 15

ne

ne

ne
e.

e.

e.
re

fre

fre
24. How many principal steps are there in an audit cycle?

f A. 3
ks

ks

ks
B. 4
oo

oo

oo
C. 5
D. 6
eb

eb

eb
E. 7
m

m
25. Which phase of a clinical trial compares a new treatment with the current best
available treatment?
t

t
A. 0
e

ne

ne
B. I
.n

e.

e.
C. II
e

D. III
re

fre

re
E. IV
sf

sf
ks
k

k
oo

oo

oo
eb

eb

eb
m

m
t

et
ne

ne

n
e.

e.

e.
fre

fre

f re
ks

ks

ks
oo

oo

oo
eb

eb

eb
m

m
t

t
ne

ne

ne
e.

e.

e.
re

re

fre
sf

f
ks

ks
k
oo

oo

oo
eb

eb

eb
m

m
t

et

t
ne

ne
n
e.

e.

e.
t

t
16 Module 3

ne

ne

ne
e.

e.

e.
re

fre

fre
EMQs
f
ks

ks

ks
Options for questions 26 and 27
oo

oo

oo
A Amniotic fluid embolism
eb

eb

eb
B Anaesthetic complications
C Cardiac disease
m

m
D Early pregnancy complications
E Haemorrhage
t

t
F Malignancy
e

ne

ne
.n

G Neurological disease

e.

e.
e

H Pre-eclampsia
re

fre

re
I Sepsis
sf

sf
J Suicide
ks
k

k
K Venous thromboembolism
oo

oo

oo
eb

eb

eb
From the list of options above, choose the single most appropriate cause of mater-
nal death as published in the MBRRACE (Mothers and Babies: Reducing Risk through
m

m
Audits and Confidential Enquiries) report of 2016. Each option may be chosen once,
more than once or not at all.
t

et
26. The leading cause of indirect maternal death.
ne

ne

n
27. The leading cause of direct maternal death occurring within a year of the end of
e.

e.

e.
pregnancy.
fre

fre

f re
ks

ks

ks
oo

oo

oo
eb

eb

eb
m

m
t

t
ne

ne

ne
e.

e.

e.
re

re

fre
sf

f
ks

ks
k
oo

oo

oo
eb

eb

eb
m

m
t

et

t
ne

ne
n
e.

e.

e.
t

t
IT, clinical governance and research 17

ne

ne

ne
e.

e.

e.
re

fre

fre
Options for questions 28–30

f
ks

ks

ks
A Arrange meeting with Trust lawyer
oo

oo

oo
B Audit
C High-risk case review
eb

eb

eb
D Refer case to Caldicott guardian
m

m
E Refer concerns to Medical Director
F Refer to coroner
G Refer to multidisciplinary team meeting
t

t
e

ne

ne
H Reflective entry in portfolio
.n

e.

e.
I Serious incident review
e
re

fre

re
J Service evaluation
K Team debriefing session
sf

sf
ks
k

k
L Undertake root cause analysis
oo

oo

oo
From the list of options above related to clinical governance, choose the single most
eb

eb

eb
appropriate initial course of action for each of the following scenarios. You may choose
m

m
an option once, more than once or not at all.

28. A specialty trainee is collecting data for a research project. She has scanned copies
of the patients’ notes and saved these on an unencrypted memory stick. When she
t

et
ne

ne

arrives home one evening, she cannot find the memory stick.

n
e.

e.

e.
29. A fetal medicine specialist performs an amniocentesis at 17 weeks of gestation.
fre

fre

re
Two days later, the woman has a miscarriage. The midwives feel that the particular
specialist has a high miscarriage rate following antenatal invasive tests.
f
ks

ks

ks
30. A woman is admitted for a planned caesarean section. During the course of the
oo

oo

oo

operation, she has a cardiac arrest and it is not possible to resuscitate her. She is
eb

eb

eb

thought to have had an amniotic fluid embolism.


m

m
t

t
ne

ne

ne
e.

e.

e.
re

re

fre
sf

f
ks

ks
k
oo

oo

oo
eb

eb

eb
m

m
t

et

t
ne

ne
n
e.

e.

e.
t

t
18 Module 3

ne

ne

ne
e.

e.

e.
re

fre

fre
Answers
f
ks

ks

ks
SBAs
oo

oo

oo
21. Answer   C  Continue with the hysterectomy including removal of the right
tube
eb

eb

eb
Explanation
m

m
A potentially viable pregnancy should not be terminated without the woman’s
consent and following the processes outlined in the 1967 Abortion Act. If a
pregnancy is discovered at the start of a hysterectomy, including one for cancer,
t

t
e

ne

ne
the operation should be rescheduled. An unexpected ectopic pregnancy should be
.n

removed. It is reasonable to presume that the woman would wish this and would

e.

e.
e

wish the surgeon to act in favour of lifesaving treatment.


re

fre

re
Reference
sf

sf
ks
RCOG. Obtaining valid consent. Clinical Governance Advice No. 6. 2015.
k

k
oo

oo

oo
22. Answer   E  Written consent
eb

eb

eb
Explanation
m

m
Explicit consent of women is required for the presence of students:
• During gynaecological and obstetric consultations
• In operating theatres as observers and assistants
t

et
• When performing a clinical pelvic examination.
ne

ne

n
Written consent must be obtained for pelvic examination of anaesthetised women.
e.

e.

e.
fre

fre

re
Reference
RCOG. Obtaining valid consent. Clinical Governance Advice No. 6. 2015.
f
ks

ks

ks
oo

oo

oo

23. Answer   A  Category 1


Explanation
eb

eb

eb

NCEPOD categories are:


m

Category Description Time to theatre Example


1 Immediate Minutes Haemorrhage
t

t
ne

ne

ne

2 Urgent Hours Fracture


3 Expedited Days Tendon injury
e.

e.

e.

4 Elective Planned Varicose veins


re

re

fre
sf

f
ks

ks

Reference
k
oo

oo

oo

See the National Confidential Enquiry into Patient Outcome and Death (NCEPOD)
website at www.ncepod.org.uk (accessed 25 July 2018).
eb

eb

eb
m

m
t

et

t
ne

ne
n
e.

e.

e.
t

t
IT, clinical governance and research 19

ne

ne

ne
e.

e.

e.
re

fre

fre
24. Answer   C 5

f Explanation
ks

ks

ks
An audit can be considered to have five principal steps, commonly referred to as
oo

oo

oo
the audit cycle (see Figure 1 in the reference article):
eb

eb

eb
1. Selection of a topic.
2. Identification of an appropriate standard.
m

m
3. Data collection to assess performance against the prespecified standard.
4. Implementation of changes to improve care if necessary.
5. Data collection for a second, or subsequent, time to determine whether care
t

t
e

ne

ne
has improved.
.n

e.

e.
Reference
e
re

fre

re
RCOG. Understanding audit. Clinical Governance Advice No. 5. 2003.
sf

sf
ks
25. Answer   D III
k

k
oo

oo

oo
Explanation
eb

eb

eb
Traditionally, to introduce a drug into clinical practice, it passes through four
phases:
m

m
• Phase I trials (20–80 people) to evaluate safety, determine a safe dosage range
and identify side effects in a small group of people
• Phase II trials (100–300 people) to evaluate safety and to begin to determine
t

et
ne

ne

efficacy

n
• Phase III trials (1000–3000 or more people if the chosen primary outcome
e.

e.

e.
measure has a low frequency, e.g. neonatal death) where it is compared to
fre

fre

re
existing treatments
f
• Phase IV postmarketing studies to delineate additional information, such as
ks

ks

ks
the treatment risks, benefits and optimal use.
oo

oo

oo

Reference
eb

eb

eb

RCOG. Developing new pharmaceutical treatments for obstetric conditions. Scientific


m

Impact Paper No. 50. 2015.

EMQs
t

t
26. Answer   C  Cardiac disease
ne

ne

ne

Explanation
e.

e.

e.

As in previous reports, cardiac disease remained the largest single cause of


re

re

fre

indirect maternal deaths in 2012–14.


sf

f
ks

ks
k

27. Answer  J Suicide


oo

oo

oo

Explanation
eb

eb

eb

The rate of maternal death by suicide remains unchanged since 2003, and
maternal suicides are now the leading cause of direct maternal deaths occurring
m

within a year after the end of pregnancy.


t

et

t
ne

ne
n
e.

e.

e.
t

t
20 Module 3

ne

ne

ne
e.

e.

e.
re

fre

fre
Reference
f
Knight M, Nair M, Tuffnell D, et al. (eds.) on behalf of MBRRACE-­UK. Saving Lives,
ks

ks

ks
Improving Mothers’ Care: Surveillance of Maternal Deaths in the UK 2012–14 and Lessons
oo

oo

oo
Learned to Inform Maternity Care From the UK and Ireland Confidential Enquiries into
Maternal Deaths and Morbidity 2009–14. Oxford: National Perinatal Epidemiology
eb

eb

eb
Unit, University of Oxford, 2016.
m

m
28. Answer   D  Refer case to Caldicott guardian
Explanation
The Caldicott guardian is responsible for all breaches of personal data within a
t

t
e

ne

ne
hospital Trust and should be informed immediately in such a case. Subsequent
.n

actions may include a serious incident review.

e.

e.
e
re

fre

re
Reference
sf

sf
Roch-Berry C. What is a Caldicott guardian? Postgraduate Medical Journal 2003;79:516–18.
ks
k

k
oo

oo

oo
29. Answer   B Audit
eb

eb

eb
Explanation
Audit is a process whereby performance can be compared to a prespecified
m

m
standard. All specialists will have some miscarriages after amniocentesis. An audit
will determine if this doctor’s performance is at odds with the expected rate.
The audit can be considered to have five principal steps, commonly referred to
t

et
ne

ne

as the audit cycle (see answer to question 24 for details).

n
e.

e.

e.
Reference
fre

fre

re
RCOG. Understanding audit. Clinical Governance Advice No. 5. 2003.

f
ks

ks

ks
30. Answer   F  Refer to coroner
oo

oo

oo

Explanation
A doctor may report the death to a coroner if:
eb

eb

eb

• The cause of death is unknown


m

• The death was violent or unnatural


• The death was sudden and unexplained
• The person who died was not visited by a medical practitioner during their
t

t
final illness
ne

ne

ne

• The medical certificate is not available


e.

e.

e.

• The person who died was not seen by the doctor who signed the medical
re

re

fre

certificate within 14 days before death or after they died


• The death occurred during an operation or before the person came out of
sf

f
ks

ks

anaesthetic
k
oo

oo

oo

• The medical certificate suggests that the death may have been caused by an
industrial disease or industrial poisoning.
eb

eb

eb

Reference
m

See the gov.uk website at www.gov.uk/after-a-death/when-a-death-is-reported-to-a-coroner


(accessed 25 July 2018).
t

et

t
ne

ne
n
e.

e.

e.
t

t
ne

ne

ne
e.

e.

e.
re

fre

fre
Core surgical skills
Module
f
ks

ks

ks
5
oo

oo

oo
eb

eb

eb
m

m
t

t
e

ne

ne
SBAs
.n

e.

e.
e

31. Which absorbable suture has the greatest tensile strength?


re

fre

re
A. Polydiaxonone (PDS)
sf

sf
ks
B. Polyglactic 910 (Vicryl rapide)
k

k
C. Polyglactin (Vicryl)
oo

oo

oo
D. Polyglecaprone (Monocryl)
eb

eb

eb
E. Polyglycolic acid (Dexon)
m

m
32. What proportion of ureteric injuries are recognised intraoperatively during
laparoscopic surgery?
A. 10%
t

et
ne

ne

B. 25%

n
C. 33%
e.

e.

e.
D. 50%
fre

fre

re
E. 67%
f
ks

ks

ks
33. Which nerve is particularly susceptible to damage when self-­retaining retractors
oo

oo

oo

are used in gynaecological surgery?


A. Femoral nerve
eb

eb

eb

B. Genitofemoral nerve
m

C. Iliohypogastric nerve
D. Obturator nerve
E. Pudendal nerve
t

t
ne

ne

ne

34. During a primary caesarean section, at what point during the operation is a
e.

e.

e.

bladder injury most likely to occur?


re

re

fre

A. Closure of the uterine incision


B. Delivery of the baby
sf

f
ks

ks

C. Dissection of bladder from the lower segment


k
oo

oo

oo

D. During catheterisation
E. Entry into the peritoneal cavity
eb

eb

eb
m

21
t

et

t
ne

ne
n
e.

e.

e.
t

t
22 Module 5

ne

ne

ne
e.

e.

e.
re

fre

fre
35. Taking all surgical procedures into consideration, which organism is most
commonly implicated in inpatient surgical-­site infections in England?
f
ks

ks

ks
A. Enterobacteriaceae
oo

oo

oo
B. Enterococcus spp.
C. Methicillin-resistant Staphylococcus aureus (MRSA)
eb

eb

eb
D. Pseudomonas spp.
m

m
E. Staphylococcus aureus

36. Urinary catheter size is identified by Charrière (Ch) or French gauge (Fg) or
French (F).
t

t
e

ne

ne
What does the gauge represent?
.n

e.

e.
A. External diameter
e

B. Flow rate
re

fre

re
C. Internal diameter
sf

sf
ks
D. Length
k

k
E. Volume of retaining balloon
oo

oo

oo
eb

eb

eb
37. What proportion of women in the UK have a body mass index (BMI) of
>30 kg/m2?
m

m
A. 6%
B. 11%
C. 16%
t

et
ne

ne

D. 21%

n
E. 26%
e.

e.

e.
fre

fre

re
38. For an obese woman with no co-­morbidities who has been unsuccessful in
f
reducing her weight with dietary modification and exercise, what is the threshold
ks

ks

ks
BMI where bariatric surgery should be considered?
oo

oo

oo

A. 32 kg/m2
eb

eb

eb

B. 35 kg/m2
C. 38 kg/m2
m

D. 40 kg/m2
E. 45 kg/m2
t

t
39. What is the overall approximate risk of serious complications from an abdominal
ne

ne

ne

hysterectomy?
e.

e.

e.

A. 1 in 100
re

re

fre

B. 2 in 100
sf

C. 3 in 100
ks

ks

D. 4 in 100
k
oo

oo

oo

E. 5 in 100
eb

eb

eb
m

m
t

et

t
ne

ne
n
e.

e.

e.
t

t
Core surgical skills 23

ne

ne

ne
e.

e.

e.
re

fre

fre
40. A 45-year-old nulliparous woman who is otherwise fit and well attends for an
outpatient hysteroscopy to investigate a potential polyp, which was suggested on
f
ks

ks

ks
an ultrasound scan.
oo

oo

oo
What pharmacotherapy would be recommended to reduce pain in the
immediate postoperative period?
eb

eb

eb
A. Administration of a paracervical anaesthetic block
m

m
B. Instillation of local anaesthetic gel into the cervical canal
C. Oral non-­steroidal anti-­inflammatory drug (NSAID) an hour before the
procedure
D. Oral opiate analgesia 1 hour before the procedure
t

t
e

ne

ne
E. Use of conscious sedation
.n

e.

e.
e

41. What is the most appropriate suture material to repair the anorectal mucosa in a
re

fre

re
fourth-­degree perineal tear?
sf

sf
ks
A. Polydiaxonone (PDS)
k

k
B. Polyglactic 910 (Vicryl rapide)
oo

oo

oo
C. Polyglactin (Vicryl)
eb

eb

eb
D. Polyglecaprone (Monocryl)
E. Polyglycolic acid (Dexon)
m

m
42. A 49-year-old woman is admitted to the gynaecology ward following an episode
of significant vaginal bleeding due to a uterine fibroid. On examination, she is
t

et
ne

ne

pale, well oriented and conscious.

n
The observations are:
e.

e.

e.
fre

fre

re
Temperature 37°C
Pulse 110 beats per minute
f
ks

ks

ks
Blood pressure 90/60 mmHg
oo

oo

oo

Urine Small volume of very dark urine


eb

eb

eb

Which is the most appropriate intravenous fluid to be used for immediate


m

resuscitation?
A. Albumin 5%
B. Dextrose 5%
t

t
ne

ne

ne

C. Sodium chloride 0.9%


D. Sodium chloride 0.18%/4% glucose
e.

e.

e.

E. Sodium chloride 0.45%/4% glucose


re

re

fre
sf

f
ks

ks
k
oo

oo

oo
eb

eb

eb
m

m
t

et

t
ne

ne
n
e.

e.

e.
t

t
24 Module 5

ne

ne

ne
e.

e.

e.
re

fre

fre
43. A 25-year-old woman is admitted to the gynaecology ward following an
evacuation of retained products of conception for a presumed molar pregnancy.
f
ks

ks

ks
On examination, her vital signs are within the normal range and stable. She is
oo

oo

oo
continuing to vomit and is unable to tolerate oral feed or fluids.
What volume of fluid should be used for maintenance intravenous therapy?
eb

eb

eb
A. 5–9 ml/kg/day of fluid
m

m
B. 10–14 ml/kg/day of fluid
C. 15–19 ml/kg/day of fluid
D. 20–24 ml/kg/day of fluid
E. 25–29 ml/kg/day of fluid
t

t
e

ne

ne
.n

44. What is the risk of major vessel injury with the Hasson (open) technique of

e.

e.
e

laparoscopic entry?
re

fre

re
A. <0.1 in 1000
sf

sf
ks
B. 0.2 in 1000
k

k
C. 0.5 in 1000
oo

oo

oo
D. 1 in 1000
eb

eb

eb
E. 2 in 1000
m

m
45. During primary entry with a force of 3 kg for gynaecological laparoscopy, what is
the depth below the indented umbilicus with a peritoneal insufflation pressure of
25 mmHg?
t

et
ne

ne

A. <4 cm

n
B. 4–8 cm
e.

e.

e.
C. 9–13 cm
fre

fre

re
D. 14–18 cm
E. >18 cm f
ks

ks

ks
oo

oo

oo
eb

eb

eb
m

m
t

t
ne

ne

ne
e.

e.

e.
re

re

fre
sf

f
ks

ks
k
oo

oo

oo
eb

eb

eb
m

m
t

et

t
ne

ne
n
e.

e.

e.
t

t
Core surgical skills 25

ne

ne

ne
e.

e.

e.
re

fre

fre
EMQs
f
ks

ks

ks
Options for questions 46–50
oo

oo

oo
A Azygos artery of the vagina
eb

eb

eb
B Descending cervical artery, branch of the uterine artery
C Descending cervical artery, branch of the internal iliac artery
m

m
D Inferior epigastric artery, branch of the external iliac artery
E Inferior epigastric artery, branch of the internal iliac artery
t

t
F Inferior gluteal artery
e

ne

ne
.n

G Internal pudendal artery

e.

e.
e

H Ovarian artery, branch of the abdominal aorta


re

fre

re
I Ovarian artery, branch of the internal iliac artery
sf

sf
J Superior epigastric artery, branch of the internal thoracic artery
ks
k

k
K Superior gluteal artery
oo

oo

oo
L Superior vesical artery
eb

eb

eb
M Vaginal artery, branch of the descending cervical artery
m

m
N Vaginal artery, branch of the internal iliac artery

Each of the following clinical scenarios relates to a pelvic surgical operative procedure.
t

et
For each operative step, select the single most applicable option from the above list. Each
ne

ne

option may be used once, more than once or not at all.

n
e.

e.

e.
46. A 29-year-old woman has just undergone a knife cone biopsy of her cervix. Both
fre

fre

re
angles of the cervix are bleeding profusely. A suture needs to be inserted to arrest
this bleeding. Which blood vessel is the main contributor to this bleeding? f
ks

ks

ks
oo

oo

oo

47. A 56-year-old woman is undergoing an abdominal hysterectomy for uterine


fibroids. The surgeon attempts to open the vaginal vault to introduce a clamp on
eb

eb

eb

the vaginal angles and makes a stab wound in the midline on the vaginal vault just
below and anterior to the cervix. The woman bleeds from the vaginal vault. Which
m

vessel is the main source of this bleeding?

48. During a teaching session, the trainer is demonstrating the anatomy of pelvic
t

t
ne

ne

ne

blood vessels and demonstrates an artery that passes in between the piriformis
and ischiococcygeus muscles and then round the sacrospinous ligament. There is
e.

e.

e.

another artery passing through the same space posteriorly. Which artery is this?
re

re

fre

49. During a laparoscopic salpingectomy for ectopic pregnancy, the surgeon is about
sf

f
ks

ks

to introduce a lateral laparoscopic port on the left abdominal flank. The surgeon
k

is trying to determine the correct position to make the incision and is looking for
oo

oo

oo

the course of a blood vessel. Which blood vessel is this?


eb

eb

eb

50. A surgeon is demonstrating the anatomy of a pelvic blood vessel to a trainee.


m

The surgeon traces the course of the internal iliac artery and identifies the largest
branch. Which artery is this?
t

et

t
ne

ne
n
e.

e.

e.
t

t
26 Module 5

ne

ne

ne
e.

e.

e.
re

fre

fre
Answers
f
ks

ks

ks
SBAs
oo

oo

oo
31. Answer   A  Polydiaxonone (PDS)
eb

eb

eb
Explanation
See Table 1 in the reference article.
m

m
Reference
Raghavan R, Arya P, Arya P, China S. Abdominal incisions and sutures in obstetrics and
t

t
gynaecology. The Obstetrician & Gynaecologist 2014;16:13–18.
e

ne

ne
.n

e.

e.
32. Answer   C 33%
e
re

fre

re
Explanation
sf

sf
There are seven types of ureteric injury, with transection the most commonly
ks
k

k
reported at laparoscopy. Only one-­third of such injuries are recognised
oo

oo

oo
intraoperatively.
eb

eb

eb
Reference
Minas V, Gul N, Aust T, Doyle M, Rowlands D. Urinary tract injuries in laparoscopic
m

m
gynaecological surgery; prevention, recognition and management. The Obstetrician &
Gynaecologist 2014;16:19–28.
t

et
33. Answer   A  Femoral nerve
ne

ne

n
Explanation
e.

e.

e.
In a 10-­year prospective study, Goldman et al. (1985; cited in the reference article)
fre

fre

re
reported an 8% incidence of femoral neuropathy when self-­retaining retractors
f
ks

ks

ks
were used during gynaecological surgery, compared with an incidence of <1%
when not used.
oo

oo

oo

Reference
eb

eb

eb

Kuponiyi O, Alleemudder DI, Latunde-Dada A, Eedarapalli P. Nerve injuries associated


m

with gynaecological surgery. The Obstetrician & Gynaecologist 2014;16:29–36.

34. Answer   E  Entry into the peritoneal cavity


t

t
Explanation
ne

ne

ne

In a primary caesarean section, most bladder injuries occur during peritoneal


e.

e.

e.

entry, whereas in repeat caesarean sections, most occur during dissection of the
re

re

fre

bladder from the lower uterine segment (bladder flap creation).


sf

f
ks

ks

Reference
k

Field A, Haloob R. Complications of caesarean section. The Obstetrician & Gynaecologist


oo

oo

oo

2016;18:265–72.
eb

eb

eb
m

m
t

et

t
ne

ne
n
e.

e.

e.
t

t
Core surgical skills 27

ne

ne

ne
e.

e.

e.
re

fre

fre
35. Answer   A  Enterobacteriaceae

f Explanation
ks

ks

ks
The number of surgical-­site infections with Enterobacteriaceae has continued to
oo

oo

oo
rise, whereas the proportion of other infections has remained static or fallen.
eb

eb

eb
Reference
m

m
Public Health England. Surveillance of surgical site infections in NHS hospitals in England,
2016 to 2017. December 2017. Available from: www.gov.uk/phe (accessed 25 July 2018).

36. Answer   A  External diameter


t

t
e

ne

ne
.n

Explanation

e.

e.
Catheter size is identified by Charrière (Ch) or French gauge (Fg) or French (F).
e
re

fre

re
These represent the external diameter of any catheter. It is recommended that the
smallest size should be used, e.g. for women, use 12–14 Ch. However, a larger size
sf

sf
ks
should be used to drain and clear the urinary bladder when urine contains heavy
k

k
oo

oo

oo
grit (encrustation) or debris.
eb

eb

eb
Reference
Aslam N, Moran PA. Catheter use in gynaecological practice. The Obstetrician &
m

m
Gynaecologist 2014;16:161–8.

37. Answer   E 26%


t

et
ne

ne

Explanation

n
In the UK, 26% of women have a BMI of >30 kg/m2.
e.

e.

e.
fre

fre

re
Reference

f
RCOG. The role of bariatric surgery in improving reproductive health. RCOG Scientific
ks

ks

ks
Impact Paper No. 17. October 2015.
oo

oo

oo

38. Answer   D  40 kg/m2


eb

eb

eb

Explanation
m

NICE guidelines recommend that bariatric surgery be considered when the BMI
is 40 kg/m2 or more, or for those with a BMI between 35 and 40 kg/m2 in the
presence of other co-­morbidities and where other non-­surgical methods have
t

t
proven unsuccessful.
ne

ne

ne

Reference
e.

e.

e.

RCOG. The role of bariatric surgery in improving reproductive health. RCOG Scientific
re

re

fre

Impact Paper No. 17. October 2015.


sf

f
ks

ks
k
oo

oo

oo
eb

eb

eb
m

m
t

et

t
ne

ne
n
e.

e.

e.
t

t
28 Module 5

ne

ne

ne
e.

e.

e.
re

fre

fre
39. Answer   D  4 in 100

f
Explanation
ks

ks

ks
The overall risk of serious complications from an abdominal hysterectomy is
oo

oo

oo
approximately four women in every 100.
eb

eb

eb
Reference
m

m
RCOG. Abdominal hysterectomy for benign conditions. RCOG Consent Advice No. 4.
May 2009.

40. Answer   C  Oral non-­steroidal anti-­inflammatory drug (NSAID) an hour


t

t
e

ne

ne
before the procedure
.n

e.

e.
Explanation
e
re

fre

re
Women without contraindications should be advised to consider taking standard
doses of NSAIDs around 1 hour before their scheduled outpatient hysteroscopy
sf

sf
ks
appointment with the aim of reducing pain in the immediate postoperative
k

k
oo

oo

oo
period.
Routine use of opiate analgesia before outpatient hysteroscopy should be
eb

eb

eb
avoided as it may cause adverse effects.
Routine administration of intracervical or paracervical local anaesthetic is not
m

m
indicated.
Reference
t

et
ne

ne

RCOG. Best practice in outpatient hysteroscopy. RCOG GTG No. 59. April 2011.

n
e.

e.

e.
41. Answer   C  Polyglactin (Vicryl)
fre

fre

re
Explanation
f
ks

ks

ks
Size 3-­0 polyglactin (Vicryl) should be used to repair the anorectal mucosa as it
may cause less irritation and discomfort than polydioxanone (PDS) sutures. When
oo

oo

oo

repair of the external and/or internal anal sphincter muscle is being performed,
eb

eb

eb

either monofilament sutures such as 3-­0 PDS or modern braided sutures such as
2-­0 polyglactin can be used with equivalent outcomes.
m

Reference
RCOG. The management of third- and fourth-­degree perineal tears. RCOG GTG No. 29.
June 2015.
t

t
ne

ne

ne

42. Answer   C  Sodium chloride 0.9%


e.

e.

e.
re

re

fre

Explanation
If patients need intravenous fluid resuscitation, use crystalloids that contain
sf

f
ks

ks

sodium in the range 130–154 mmol/l, with a bolus of 500 ml over <15 minutes.
k
oo

oo

oo

Reference
eb

eb

eb

NICE. Intravenous fluid therapy in adults in hospital. NICE Clinical Guidline (CG 174).
December 2013.
m

m
t

et

t
ne

ne
n
e.

e.

e.
t

t
Core surgical skills 29

ne

ne

ne
e.

e.

e.
re

fre

fre
43. Answer   E  25–29 ml/kg/day of fluid

f Explanation
ks

ks

ks
If patients need intravenous fluids for routine maintenance alone, restrict the
oo

oo

oo
initial prescription to: 25–30 ml/kg/day of water, approximately 1 mmol/kg/day
of potassium, sodium and chloride and approximately 50–100 g/day of glucose to
eb

eb

eb
limit starvation ketosis.
m

m
Reference
NICE. Intravenous fluid therapy in adults in hospital. NICE Clinical Guideline (CG 174).
December 2013.
t

t
e

ne

ne
.n

44. Answer   A  <0.1 in 1000

e.

e.
e
re

fre

re
Explanation
Hasson reviewed a number of series using the open method and found no cases of
sf

sf
ks
major vessel injury and a rate of bowel injury of only 0.1%. In a meta-­analysis of
k

k
oo

oo

oo
over 350,000 closed laparoscopic procedures, the risk of bowel damage was 0.4 in
1000 and of major vessel injuries was 0.2 in 1000.
eb

eb

eb
Reference
m

m
RCOG. Preventing entry-­related gynaecological laparoscopic injuries. RCOG GTG No. 49.
May 2008.
t

et
45. Answer   B  4–8 cm
ne

ne

n
Explanation
e.

e.

e.
If a constant force of 3 kg is applied to the abdominal wall at the umbilicus to
fre

fre

re
an abdominal cavity insufflated to a pressure of 10 mmHg, the depth under
the ‘indented’ umbilicus is only 0.6 cm. When the same force is applied to an f
ks

ks

ks
abdomen distended to 25 mmHg, the depth is 5.6 cm (range 4–8 cm).
oo

oo

oo

Reference
eb

eb

eb

RCOG. Preventing entry-­related gynaecological laparoscopic injuries. RCOG GTG No. 49.
May 2008.
m

EMQs
t

t
46. Answer   B  Descending cervical artery, branch of the uterine artery
ne

ne

ne
e.

e.

e.

47. Answer   A  Azygos artery of the vagina


re

re

fre

48. Answer   F  Inferior gluteal artery


sf

f
ks

ks
k

49. Answer   D  Inferior epigastric artery, branch of the external iliac artery
oo

oo

oo
eb

eb

eb

50. Answer   K  Superior gluteal artery


m

Reference
Standring S. (ed.) Gray’s Anatomy, 40th edn. Churchill Livingstone, 2008.
t

et

t
ne

ne
n
e.

e.

e.
t

t
ne

ne

ne
e.

e.

e.
re

fre

fre
Postoperative care
Module
f
ks

ks

ks
6
oo

oo

oo
eb

eb

eb
m

m
t

t
e

ne

ne
SBAs
.n

e.

e.
e

51. A patient is referred to the gynaecology clinic for consideration of hysterectomy


re

fre

re
following a failed endometrial ablation.
sf

sf
At what point should enhanced recovery planning start?
ks
k

k
A. Following discharge from hospital
oo

oo

oo
B. On admission
eb

eb

eb
C. Preoperative assessment clinic
D. Upon completion of the operation
m

m
E. With the initial referral

52. A woman has had a successful trial of forceps delivery under spinal anaesthetic.
t

et
What is the minimum duration that the catheter should remain in situ to
ne

ne

n
reduce the risk of asymptomatic bladder overfilling?
e.

e.

e.
A. 3 hours
fre

fre

re
B. 6 hours
C. 9 hours
f
ks

ks

ks
D. 12 hours
oo

oo

oo

E. 24 hours
eb

eb

eb

53. A 48-year-old woman has undergone a midurethral tape procedure for stress
m

urinary incontinence and is ready to be discharged.


Within what time frame should she be seen as an outpatient to exclude tape
erosion?
t

t
A. 6 weeks
ne

ne

ne

B. 2 months
e.

e.

e.

C. 3 months
D. 6 months
re

re

fre

E. 12 months
sf

f
ks

ks
k
oo

oo

oo
eb

eb

eb
m

30
t

et

t
ne

ne
n
e.

e.

e.
t

t
Postoperative care 31

ne

ne

ne
e.

e.

e.
re

fre

fre
54. A patient is seen 2 weeks after a laparoscopic hysterectomy. An intraoperative
bladder injury was noted and repaired laparoscopically by the urologist. A
f
ks

ks

ks
catheter has been left in situ for 2 weeks. A retrograde cystogram subsequently
oo

oo

oo
reports a leak from the bladder. The woman is clinically well.
What is the most appropriate management?
eb

eb

eb
A. Leave catheter in situ and repeat cystogram in 1 week
m

m
B. Perform cystoscopy to assess the defect
C. Repeat laparoscopy and secondary repair
D. Request a CT intravenous urogram to check ureters are not damaged
E. Urgent urological review
t

t
e

ne

ne
.n

55. A 38-year-old woman is admitted with nausea, vomiting and confusion following

e.

e.
e

a transcervical resection of the endometrium (TCRE) for heavy menstrual


re

fre

re
bleeding. It is noted that the fluid deficit at the time of the procedure was 1.6 l.
sf

sf
ks
What is the single most important investigation?
k

k
A. Chest X-ray
oo

oo

oo
B. CT of abdomen and pelvis
eb

eb

eb
C. Full blood count (FBC)
D. Serum urea and electrolytes
m

m
E. Ultrasound of the pelvis

56. A 49-year-old woman had a total abdominal hysterectomy and bilateral salpingo-­
t

et
ne

ne

oophorectomy for irregular menstrual bleeding and pelvic pain, related to

n
previous diagnosis of endometriosis. Her immediate postoperative recovery was
e.

e.

e.
uneventful. Seven days later, she is readmitted with a vaginal loss of watery fluid
fre

fre

re
and blood-­stained urine.
What is the most likely diagnosis?
f
ks

ks

ks
A. Detrusor overactivity
oo

oo

oo

B. Urinary retention with overflow


C. Urinary tract infection
eb

eb

eb

D. Vaginal candidiasis
m

E. Vesicovaginal fistula

57. Following surgery, how long should patients wait before they can shower?
t

t
A. 24 hours
ne

ne

ne

B. 48 hours
e.

e.

e.

C. 72 hours
re

re

fre

D. 5 days
E. 7 days
sf

f
ks

ks
k
oo

oo

oo
eb

eb

eb
m

m
t

et

t
ne

ne
n
e.

e.

e.
t

t
32 Module 6

ne

ne

ne
e.

e.

e.
re

fre

fre
58. What is the most common type of postoperative infection following an emergency
caesarean section?
f
ks

ks

ks
A. Endometritis
oo

oo

oo
B. Lower respiratory tract infection
C. Upper respiratory tract infection
eb

eb

eb
D. Urinary tract infection
m

m
E. Wound infection

59. Of women that develop pyrexia in the first 48 hours after gynaecological surgery,
what proportion do not have an identifiable cause?
t

t
e

ne

ne
A. 5%
.n

e.

e.
B. 10%
e

C. 20%
re

fre

re
D. 40%
sf

sf
ks
E. 80%
k

k
oo

oo

oo
60. Following a general anaesthetic, what is the most common cause of pyrexia in the
eb

eb

eb
immediate postoperative period?
A. Lower respiratory tract infection
m

m
B. Pulmonary atelectasis
C. Upper respiratory tract infection
D. Urinary tract infection
t

et
ne

ne

E. Wound infection

n
e.

e.

e.
fre

fre

f re
ks

ks

ks
oo

oo

oo
eb

eb

eb
m

m
t

t
ne

ne

ne
e.

e.

e.
re

re

fre
sf

f
ks

ks
k
oo

oo

oo
eb

eb

eb
m

m
t

et

t
ne

ne
n
e.

e.

e.
t

t
Postoperative care 33

ne

ne

ne
e.

e.

e.
re

fre

fre
EMQs
f
ks

ks

ks
Options for questions 61–63
oo

oo

oo
A Common peroneal nerve
eb

eb

eb
B Femoral nerve
C Genitofemoral nerve
m

m
D Iliohypogastric nerve
E Ilioinguinal nerve
t

t
F Lateral cutaneous nerve of the thigh
e

ne

ne
.n

G Obturator nerve

e.

e.
e

H Pudendal nerve
re

fre

re
I Radial nerve
sf

sf
J Tibial nerve
ks
k

k
K Ulnar nerve
oo

oo

oo
eb

eb

eb
From the list of options above, which nerve is the most likely to be damaged in each of
the following clinical scenarios? Each option may be used once, more than once or not at all.
m

m
61. Following a prolonged difficult hysterectomy for rectovaginal endometriosis, a
48-year-old woman makes a good postoperative recovery in hospital, although
t

et
she did notice some altered sensation on the medial aspect of the thigh and calf.
ne

ne

When she returns home a few days later, she is unable to climb the stairs.

n
e.

e.

e.
62. Following a sacrospinous fixation procedure, a woman returns to hospital with
fre

fre

re
worsening gluteal and vulval pain. The pain is worse when she sits down.
f
ks

ks

ks
63. A woman with a body mass index (BMI) of 38 kg/m2 is placed in the lithotomy
oo

oo

oo

position for a planned vaginal hysterectomy. It is necessary to hyperflex and abduct


her thighs in order to gain adequate access. Postoperatively, she has foot drop and
eb

eb

eb

has altered sensation on the lateral aspect of her calf and the dorsum of her foot.
m

m
t

t
ne

ne

ne
e.

e.

e.
re

re

fre
sf

f
ks

ks
k
oo

oo

oo
eb

eb

eb
m

m
t

et

t
ne

ne
n
e.

e.

e.
t

t
34 Module 6

ne

ne

ne
e.

e.

e.
re

fre

fre
Options for questions 64–66

f
ks

ks

ks
A Bladder injury
B Death
oo

oo

oo
C Femoral nerve damage
eb

eb

eb
D Haemorrhage requiring blood transfusion
m

m
E Pelvic abscess
F Prolapsed fallopian tube
G Rectovaginal fistula
t

t
e

ne

ne
H Surgical-site infection
.n

I Ureteric leakage

e.

e.
e

J Ureteric ligation
re

fre

re
K Urinary tract infection
sf

sf
ks
L Vault haematoma
k

k
oo

oo

oo
M Vault prolapse
N Venous thromboembolism
eb

eb

eb
O Vesicovaginal fistula
m

m
P Wound dehiscence

The risk of an operation can be categorised into serious risk and frequently occurring
t

et
risk. From the list of risks described above, choose the single most appropriate option
ne

ne

n
for the scenario in question. Each option may be used once, more than once or not at all.
e.

e.

e.
64. The most common serious risk of an abdominal hysterectomy.
fre

fre

65. The most common serious risk of vaginal surgery for prolapse.
fre
ks

ks

ks
66. The most common visceral injury (serious risk) associated with caesarean section.
oo

oo

oo

Options for questions 67–70


eb

eb

eb
m

A Mixed respiratory and metabolic acidosis


B Mixed respiratory and metabolic alkalosis
C No derangement of acid–base balance
t

t
D Primary metabolic acidosis
ne

ne

ne

E Primary metabolic acidosis with respiratory compensation


e.

e.

e.

F Primary metabolic alkalosis


re

re

fre

G Primary metabolic alkalosis with respiratory compensation


sf

f
ks

ks

H Primary respiratory acidosis


k

I Primary respiratory acidosis with renal compensation


oo

oo

oo

J Primary respiratory alkalosis


eb

eb

eb

K Primary respiratory alkalosis with renal compensation


m

The list above describes some derangements of acid–base balance. For each of the
following clinical scenarios, choose the single most appropriate derangement (if any) of
acid–base balance. Each option may be used once, more than once or not at all.
t

et

t
ne

ne
n
e.

e.

e.
t

t
Postoperative care 35

ne

ne

ne
e.

e.

e.
re

fre

fre
67. Following a difficult hysterectomy for large fibroids, a woman remains in recovery
for some time and needs several doses of morphine to maintain pain control. She
f
ks

ks

ks
returns to the ward with a patient-­controlled analgesia device. When reviewed by
oo

oo

oo
a nurse 1 hour later, she has a low respiratory rate and low oxygen saturation on
pulse oximetry.
eb

eb

eb
Her arterial blood gas results are:
m

m
pH 7.31
PCO2 7.2 kPa
+1.5
t

t
Base excess
e

ne

ne
.n

e.

e.
e

68. Following a difficult outpatient hysteroscopy, a woman becomes increasingly


re

fre

re
anxious. She is sweating and describes palpitations. Her ECG is normal.
sf

sf
ks
Her arterial blood gas results are:
k

k
oo

oo

oo
pH 7.49
eb

eb

eb
PCO2 4.3 kPa
Base excess −1.5
m

m
69. A woman with polycystic ovarian syndrome (PCOS) has been booked for
t

et
ne

ne

laparoscopic ovarian drilling. Her periods are infrequent and she has been

n
prescribed metformin to induce ovulation. Unbeknown to her gynaecologist, she
e.

e.

e.
has doubled the dose following internet advice that metformin induces weight
fre

fre

re
loss. Following the laparoscopy, she is clammy and sweaty in the recovery area.
She is tachypnoeic but her ECG is normal.
f
ks

ks

ks
Her results are:
oo

oo

oo

Blood sugar 3.1 mmol/l


eb

eb

eb

pH 7.28
m

PCO2 3.9 kPa


Base excess −2.8
t

t
ne

ne

ne

70. A woman is admitted for surgical evacuation of the uterus following a presumed
e.

e.

e.

diagnosis of molar pregnancy. She has had severe vomiting for several days
re

re

fre

before the procedure. In the recovery area, she has ongoing vomiting, which is
not controlled well by antiemetics. Eventually, a nasogastric tube is passed and
sf

f
ks

ks

attached to suction to keep her stomach empty. She continues to feel unwell with
k

a low respiratory rate.


oo

oo

oo

Her arterial blood gas results are:


eb

eb

eb

pH 7.49
m

PCO2 6.2 kPa


Base excess +2.5
t

et

t
ne

ne
n
e.

e.

e.
t

t
36 Module 6

ne

ne

ne
e.

e.

e.
re

fre

fre
Answers
f
ks

ks

ks
SBAs
oo

oo

oo
51. Answer   E  With the initial referral
eb

eb

eb
Explanation
The GP has referred the patient with a request to consider an operation. The
m

m
GP should therefore take steps to optimise the patient’s health prior to them
being seen in the clinic by checking for anaemia and treating accordingly, and
optimising the treatment of any other underlying disease such as hypertension or
t

t
e

ne

ne
diabetes.
.n

e.

e.
Reference
e
re

fre

re
RCOG. Enhanced recovery in gynaecology. Scientific Impact Paper No. 36. February 2013.
sf

sf
ks
52. Answer   D  12 hours
k

k
oo

oo

oo
Explanation
There is considerable variation in practice in postpartum bladder management
eb

eb

eb
in the UK. Further research is needed to develop evidence-­based guidelines.
m

m
However, at a minimum, the first void should be measured, and if retention is a
possibility, a postvoid residual should be measured to ensure that retention does
not go unrecognised. Women who have had a spinal anaesthetic or an epidural
t

et
that has been topped up for a trial should be offered an indwelling catheter for at
ne

ne

least 12 hours postdelivery to prevent asymptomatic bladder overfilling, followed

n
e.

e.

e.
by fluid balance charts to ensure good voiding volumes.
fre

fre

re
Reference
f
ks

ks

ks
RCOG. Operative vaginal delivery. RCOG GTG No. 26. February 2011.
oo

oo

oo

53. Answer   D  6 months


eb

eb

eb

Explanation
Offer a follow-­up appointment (including vaginal examination to exclude erosion)
m

within 6 months to all women who have had continence surgery.


Reference
t

t
ne

ne

ne

NICE. Urinary incontinence in women: management. NICE Clinical Guideline (CG171).


September 2013.
e.

e.

e.
re

re

fre

54. Answer   A  Leave catheter in situ and repeat cystogram in 1 week


sf

Explanation
ks

ks
k

Ideally, bladder repairs should be watertight, and leakage from the suture line
oo

oo

oo

should be tested (e.g. with methylene blue or indigo carmine). A bladder catheter
must be inserted and continuous postoperative bladder drainage should be
eb

eb

eb

allowed for 2 weeks. The above two measures (watertight closure and indwelling
m

catheter) will improve healing and reduce the risk of subsequent vesicovaginal
fistula formation. Prior to catheter removal, complete repair without leakage
t

et

t
ne

ne
n
e.

e.

e.
t

t
Postoperative care 37

ne

ne

ne
e.

e.

e.
re

fre

fre
should be confirmed by retrograde cystography. If contrast escape is noted, then
the catheter should be left in situ and the test repeated in 1 week.
f
ks

ks

ks
Reference
oo

oo

oo
Minas V, Gul N, Aust T, Doyle M, Rowlands D. Urinary tract injuries in laparoscopic
eb

eb

eb
gynaecological surgery; prevention, recognition and management. The Obstetrician &
Gynaecologist, 2014;16:19–28.
m

m
55. Answer   D  Serum urea and electrolytes
Explanation
t

t
e

ne

ne
Transurethral resection syndrome, a well-­recognised complication with first-­
.n

generation ablative techniques, occurs secondary to glycine overload. Glycine

e.

e.
e

overload can cause hyponatraemia and hyperammonaemia, congestive heart


re

fre

re
failure, haemolysis, coma and, in rare instances, death. Fluid balance is a
sf

sf
fundamental part of the operative set-­up and requires careful measurement of
ks
k

k
input and output. A deficit level of 1.5 l should be used as a reference mark for
oo

oo

oo
when to stop the procedure, because deficits below this level are not associated
with metabolic changes of transurethral resection syndrome (hyponatraemia and
eb

eb

eb
hyperammonaemia).
m

m
Nausea, vomiting, headache and confusion are early signs of hyponatraemia,
and therefore urea and electrolytes should give the answer.
Reference
t

et
ne

ne

Saraswat L, Cooper K. Surgical management of heavy menstrual bleeding: part 1. The

n
Obstetrician & Gynaecologist 2017;19:37–45.
e.

e.

e.
fre

fre

re
56. Answer   E  Vesicovaginal fistula
f
ks

ks

ks
Explanation
The presenting complaint of a constant watery loss per vaginam is suggestive of a
oo

oo

oo

fistula. Blood-­stained urine would also indicate some kind of urinary tract injury.
eb

eb

eb

57. Answer   B  48 hours


m

Explanation
Advise patients that they may shower safely 48 hours after surgery.
t

t
ne

ne

ne

Reference
NICE. Surgical site infections: prevention and treatment. NICE Clinical Guideline (CG74).
e.

e.

e.

Updated February 2017.


re

re

fre
sf

58. Answer   A Endometritis


ks

ks
k

Explanation
oo

oo

oo

Wound infection and endometritis are the commonest sites of postoperative


eb

eb

eb

infection, although the urinary tract, respiratory tract and nervous system must
also be considered. The risk of sepsis is, unsurprisingly, higher for emergency
m

compared with elective caesarean section. A 2014 Cochrane review suggested a


rate of wound infection of 97 in 1000 and 68 in 1000 for emergency and elective
t

et

t
ne

ne
n
e.

e.

e.
t

t
38 Module 6

ne

ne

ne
e.

e.

e.
re

fre

fre
caesarean section, respectively; for endometritis, the rates were 184 in 1000 versus
39 in 1000, respectively.
f
ks

ks

ks
Reference
oo

oo

oo
Field A, Haloob R. Complications of caesarean section. The Obstetrician & Gynaecologist
eb

eb

eb
2016;18:265–72.
m

m
59. Answer   D 40%
Explanation
Almost 40% of women with fever in the first 48 hours postoperatively do not have
t

t
e

ne

ne
an identifiable cause.
.n

e.

e.
Reference
e
re

fre

re
Read M, James M. Immediate postoperative problems following gynaecological surgery.
The Obstetrician & Gynaecologist 2002;4:29–34.
sf

sf
ks
k

k
oo

oo

oo
60. Answer   B  Pulmonary atelectasis
eb

eb

eb
Explanation
Postoperative pyrexia is commonly caused by pulmonary atelectasis developing
m

m
during general anaesthetic. If the pyrexia persists beyond 36 hours, a chest X-­ray
may be required.
Reference
t

et
ne

ne

Read M, James M. Immediate postoperative problems following gynaecological surgery.

n
The Obstetrician & Gynaecologist 2002;4:29–34
e.

e.

e.
fre

fre

re
EMQs
f
ks

ks

ks
61. Answer   B  Femoral nerve
oo

oo

oo

Explanation
Gynaecological surgery is the most common contributor to iatrogenic femoral
eb

eb

eb

nerve injury, and abdominal hysterectomy is mostly responsible for this.


m

Femoral neuropathy presents with weakness of hip flexion and adduction and
knee extension.
t

t
62. Answer   H  Pudendal nerve
ne

ne

ne

Explanation
e.

e.

e.

The pudendal nerve (S2–S4) exits the pelvis initially through the greater sciatic
re

re

fre

foramen below the piriformis. Importantly, it runs behind the lateral third of the
sf

sacrospinous ligament and ischial spine alongside the internal pudendal artery
ks

ks
k

and immediately re-­enters the pelvis through the lesser sciatic foramen to the
oo

oo

oo

pudendal canal (Alcock’s canal). This nerve is susceptible to entrapment injuries


during sacrospinous ligament fixation as it runs behind the lateral aspect of
eb

eb

eb

the sacrospinous ligament. The patient will report postoperative gluteal,


m

perineal and vulval pain, which worsens in the seated position if the nerve is
damaged.
t

et

t
ne

ne
n
e.

e.

e.
t

t
Postoperative care 39

ne

ne

ne
e.

e.

e.
re

fre

fre
63. Answer   A  Common peroneal nerve

f Explanation
ks

ks

ks
Foot drop is reported when the common peroneal nerve is injured, along with
oo

oo

oo
paraesthesia over the calf and dorsum of the foot.
eb

eb

eb
Reference
m

m
Kuponiyi O, Alleemudder DI, Latunde-Dada A, Eedarapalli P. Nerve injuries associated
with gynaecological surgery. The Obstetrician & Gynaecologist 2014;16:29–36.

64. Answer   D  Haemorrhage requiring blood transfusion


t

t
e

ne

ne
.n

Explanation

e.

e.
Haemorrhage requiring blood transfusion occurs in 23 women in every 1000
e
re

fre

re
abdominal hysterectomies (common). All other serious risks are uncommon
or rare.
sf

sf
ks
k

k
Reference
oo

oo

oo
RCOG. Abdominal hysterectomy for benign conditions. RCOG Consent Advice No. 4. May
eb

eb

eb
2009.
m

m
65. Answer   D  Haemorrhage requiring blood transfusion
Explanation
The risk of excessive bleeding requiring transfusion or a return to theatre is 2 in
t

et
ne

ne

every 100 (common).

n
e.

e.

e.
Reference
fre

fre

re
RCOG. Vaginal surgery for prolapse. RCOG Consent Advice No. 5. October 2009.

f
ks

ks

ks
66. Answer   A  Bladder injury
oo

oo

oo

Explanation
eb

eb

eb

The risk of bladder injury is 1 in every 1000 (rare). The risk of ureteric injury is
3 in every 10,000 (rare).
m

Reference
RCOG. Caesarean section. RCOG Consent Advice No. 7. October 2009.
t

t
ne

ne

ne

67. Answer   H  Primary respiratory acidosis


e.

e.

e.

Explanation
re

re

fre

This is a primary respiratory acidosis demonstrated by low pH and elevated


sf

partial pressure of carbon dioxide (PCO2). Base excess is normal. Opiates cause
ks

ks

respiratory depression and in an acute situation there is inadequate time for renal
k
oo

oo

oo

compensation.
eb

eb

eb
m

m
t

et

t
ne

ne
n
e.

e.

e.
t

t
40 Module 6

ne

ne

ne
e.

e.

e.
re

fre

fre
68. Answer   J  Primary respiratory alkalosis

f
Explanation
ks

ks

ks
A highly anxious woman is likely to hyperventilate. This can quickly result in
oo

oo

oo
primary respiratory alkalosis and this is reflected in the blood gases. As this is an
acute event, there is no time for renal compensation.
eb

eb

eb
m

m
69. Answer   E  Primary metabolic acidosis with respiratory compensation
Explanation
This woman is potentially taking high doses of metformin, which is associated
t

t
e

ne

ne
with lactic acidosis. She will have been starved for her operation, which has led
.n

to hypoglycaemia. Her acidotic state has resulted in increased ventilation in an

e.

e.
e

attempt to correct her acid–base disturbance via respiratory compensation (hence


re

fre

re
the reason the PCO2 is low).
sf

sf
ks
k

k
70. Answer   G  Primary metabolic alkalosis with respiratory compensation
oo

oo

oo
Explanation
eb

eb

eb
This woman has metabolic alkalosis with respiratory compensation. Prolonged
vomiting causes depletion of sodium and potassium. The kidney retains sodium
m

m
at the expense of hydrogen ions, so the pH falls. Respiratory compensation occurs
when ventilation is slowed, so the PCO2 rises.
t

et
Reference
ne

ne

n
Barrett K, Brooks H, Boitano S, Barman S. Ganong’s Review of Medical Physiology,
e.

e.

e.
25th edn. Lange Medical Books, 2015.
fre

fre

f re
ks

ks

ks
oo

oo

oo
eb

eb

eb
m

m
t

t
ne

ne

ne
e.

e.

e.
re

re

fre
sf

f
ks

ks
k
oo

oo

oo
eb

eb

eb
m

m
t

et

t
ne

ne
n
e.

e.

e.
t

t
ne

ne

ne
e.

e.

e.
re

fre

fre
Surgical procedures
Module
f
ks

ks

ks
7
oo

oo

oo
eb

eb

eb
m

m
t

t
e

ne

ne
SBAs
.n

e.

e.
e

71. A woman presents with pelvic pain and swelling and is found to have a 10 cm
re

fre

re
ovarian cyst. Imaging suggests that this is a dermoid cyst. A laparoscopy
sf

sf
is performed, but the surgeon decides the cyst is too large to remove
ks
k

k
laparoscopically and converts the procedure to a mini-laparotomy.
oo

oo

oo
When compared with a laparoscopic procedure, which two factors will be reduced?
eb

eb

eb
Operative time Postoperative Rate of spillage Recovery
m

m
discomfort of cyst contents time
A ✓ ✓
B ✓ ✓
t

et
ne

ne

C ✓ ✓

n
D ✓ ✓
e.

e.

e.
E ✓ ✓
fre

fre

f re
ks

ks

ks
72. During the course of an outpatient hysteroscopy with a 3 mm hysteroscope, a
oo

oo

oo

doctor finds that he has perforated the uterus. Patient observations are stable and
there is no evidence of any bleeding.
eb

eb

eb

What is the most appropriate management?


m

A. Admit for 24-hour observation and intravenous antibiotics


B. Arrange an urgent laparoscopy
C. Arrange an urgent laparotomy
D. Insert a urinary catheter and observe for 12 hours
t

t
ne

ne

ne

E. Reassure and discharge


e.

e.

e.

73. When comparing open myomectomy and hysterectomy for the surgical
re

re

fre

management of a fibroid uterus, which perioperative complication is more likely


sf

f
ks

ks

with hysterectomy?
k

A. Haemorrhage
oo

oo

oo

B. Infection
eb

eb

eb

C. Ovarian artery thrombosis


D. Return to theatre
m

E. Visceral damage (bladder or bowel)

41
t

et

t
ne

ne
n
e.

e.

e.
t

t
42 Module 7

ne

ne

ne
e.

e.

e.
re

fre

fre
74. Which laparoscopic procedure appears to be associated with the highest rate of
bladder injury?
f
ks

ks

ks
A. Laparoscopic-assisted vaginal hysterectomy (LAVH)
oo

oo

oo
B. Laparoscopic myomectomy
C. Laparoscopic ovarian cystectomy
eb

eb

eb
D. Subtotal laparoscopic hysterectomy
m

m
E. Total laparoscopic hysterectomy

75. A 30-year-old woman with a normal body mass index (BMI) has a laparoscopic
ovarian cystectomy for a persistent 6 cm simple ovarian cyst. Preoperative tumour
t

t
e

ne

ne
markers were normal. The tissue is placed in a retrieval bag.
.n

Ideally, through which port/incision should the tissue be removed?

e.

e.
e

A. Lateral laparoscopic port


re

fre

re
B. McBurney’s point incision
sf

sf
ks
C. Midline suprapubic port
k

k
D. Palmer’s point incision
oo

oo

oo
E. Umbilical port
eb

eb

eb
76. A 26-year-old woman has had a laparoscopic salpingotomy for an ectopic
m

m
pregnancy. She enquires about the future risk of another ectopic pregnancy.
What is the risk of having a subsequent ectopic pregnancy following a
salpingotomy?
t

et
ne

ne

A. 2%

n
B. 8%
e.

e.

e.
C. 14%
fre

fre

re
D. 20%
E. 26% f
ks

ks

ks
oo

oo

oo

77. Which operative procedure for vaginal vault prolapse results in the lowest rate of
recurrence?
eb

eb

eb

A. High uterosacral ligament suspension


m

B. High ventrofixation
C. Modified McCall culdoplasty
D. Open abdominal sacrocolpopexy
t

t
E. Sacrospinous fixation
ne

ne

ne
e.

e.

e.

78. A 32-year-old multiparous woman with a previous history of caesarean section


re

re

fre

is now at 38 weeks of gestation with placenta praevia. She has been admitted for
caesarean section. At the time of signing the consent form, she enquires about the
sf

f
ks

ks

chances of an emergency hysterectomy.


k

What is the risk of emergency hysterectomy for this woman?


oo

oo

oo

A. 27%
eb

eb

eb

B. 37%
m

C. 47%
D. 57%
E. 67%
t

et

t
ne

ne
n
e.

e.

e.
t

t
Surgical procedures 43

ne

ne

ne
e.

e.

e.
re

fre

fre
79. A woman has a laparoscopy for suspected appendicitis at 18 weeks of gestation.
What is the maximum recommended intra-­abdominal pressure during the
f
ks

ks

ks
procedure?
oo

oo

oo
A. 8 mmHg
B. 10 mmHg
eb

eb

eb
C. 12 mmHg
m

m
D. 15 mmHg
E. 20 mmHg

80. What is the risk of bladder injury during caesarean section?


t

t
e

ne

ne
A. 0.001%
.n

e.

e.
B. 0.005%
e

C. 0.01%
re

fre

re
D. 0.05%
sf

sf
ks
E. 0.1%
k

k
oo

oo

oo
eb

eb

eb
m

m
t

et
ne

ne

n
e.

e.

e.
fre

fre

f re
ks

ks

ks
oo

oo

oo
eb

eb

eb
m

m
t

t
ne

ne

ne
e.

e.

e.
re

re

fre
sf

f
ks

ks
k
oo

oo

oo
eb

eb

eb
m

m
t

et

t
ne

ne
n
e.

e.

e.
t

t
44 Module 7

ne

ne

ne
e.

e.

e.
re

fre

fre
EMQs
f
ks

ks

ks
Options for questions 81–85
oo

oo

oo
A 40 in 100 women
eb

eb

eb
B 26 in 100 women
C 20 in 100 women
m

m
D 10 in 100 women
E 8 in 100 women
t

t
F 6 in 100 women
e

ne

ne
.n

G 4 in 100 women

e.

e.
e

H 2 in 100 women
re

fre

re
I 1 in 100 women
sf

sf
J <1 in 100 women
ks
k

k
oo

oo

oo
Each of the following clinical scenarios relates to a woman who is about to have a
eb

eb

eb
surgical procedure. For each risk described, select the single most appropriate incidence
from the list above. Each option may be used once, more than once or not at all.
m

m
81. A 35-year-old woman is about to have a diagnostic laparoscopy for chronic pelvic
pain. She is concerned because her sister had an umbilical hernia following a
t

et
similar operation done through the umbilicus. During the consenting process,
ne

ne

she enquires about her risk of developing a hernia through the site of entry.

n
e.

e.

e.
82. A 35-year-old woman has suffered a third-­degree perineal tear following her
fre

fre

re
first delivery at term. Following repair of the tear, the doctor wishes to prescribe
f
antibiotics to her. She is apprehensive about taking antibiotics because of her
ks

ks

ks
previous history of Clostridium difficile infection related to antibiotic use during
oo

oo

oo

an episode of pneumonia. She asks about her risk of perineal infection following
this repair.
eb

eb

eb

83. A 36-year-old multiparous woman has just had a repair of her third-­degree
m

perineal tear following a forceps delivery. What is the incidence of faecal urgency
following such repair?
t

t
ne

ne

ne

84. A 63-year-old woman attends the gynaecology clinic for counselling and
consenting for vaginal hysterectomy for prolapse. She has an ‘unusual antibody’ in
e.

e.

e.

her blood and is worried about the prospect of needing blood transfusion during
re

re

fre

surgery. She enquires about the risk of intraoperative blood transfusion.


sf

f
ks

ks

85. A 27-year-old woman has suffered a missed miscarriage at 10 weeks of gestation.


k

Following a period of expectant management she has started bleeding and now
oo

oo

oo

wants to have a surgical evacuation of the uterus. She enquires about the risk of
eb

eb

eb

retained products of conception following the surgical evacuation.


m

m
t

et

t
ne

ne
n
e.

e.

e.
t

t
Surgical procedures 45

ne

ne

ne
e.

e.

e.
re

fre

fre
Options for questions 86–90

f
ks

ks

ks
A A small vertical incision starting at the base of the umbilicus
oo

oo

oo
B A transverse incision at the lower edge of the umbilicus
C A vertical incision in the skin below the umbilicus
eb

eb

eb
D A small incision 3 cm below the left costal margin in the midclavicular line
m

m
E Insertion into the abdomen at 90° to the skin with the operating table horizontal
F Insertion into the abdomen at 90° to the skin with the operating table in Trendelenburg position
G Insertion into abdomen at 45° to the skin aiming at the pelvis
t

t
e

ne

ne
H Insertion into abdomen at 45° to the skin with the operating table horizontal
.n

I Insertion at 90° to the horizontal plane irrespective of the position of the operating table

e.

e.
e

J Move Veress needle from side to side to ensure that the tip is free inside the abdomen
re

fre

re
K Reduce abdominal distension pressure to 15 mmHg
sf

sf
ks
L Rotate the laparoscope through 360° to visualise and exclude any adhesion of bowel
k

k
oo

oo

oo
M Start insufflation if intra-­abdominal pressure remains between 10 and 15 mmHg
N Tilt the operation table to the Trendelenburg position
eb

eb

eb
O Tilt operation table to the Trendelenburg position before starting to inflate the abdomen
m

m
P Visualisation of inferior epigastric artery

Each of the following clinical scenarios relates to actions or manoeuvres during lapa-
t

et
roscopic entry. For each patient, select the single most appropriate action from the option
ne

ne

n
list above. Each option may be used once, more than once or not at all.
e.

e.

e.
86. A 32-year-old woman with a previous history of midline laparotomy is about to
fre

fre

re
undergo laparoscopy. A senior specialty trainee (ST5) is teaching a junior specialty
trainee (ST2) to introduce a laparoscope into the abdomen. The junior trainee f
ks

ks

ks
enquires about the exact location of incision in order to introduce the Veress needle.
oo

oo

oo

87. A senior trainee doctor is about to introduce the Veress needle into the abdomen
eb

eb

eb

of a 19-year-old woman who is undergoing laparoscopy for chronic pelvic pain.


The doctor has made an incision. Which is the most appropriate manoeuvre while
m

the doctor introduces the Veress needle?


88. A senior trainee doctor is performing a diagnostic laparoscopy for a 42-year-
old woman with a history of chronic pelvic pain. The doctor has successfully
t

t
ne

ne

ne

introduced the laparoscope through the primary port. Which is the next most
appropriate manoeuvre?
e.

e.

e.
re

re

fre

89. A surgeon needs to introduce secondary ports for performing a laparoscopic


excision of a simple ovarian cyst in a 36-year-old multiparous woman. The
sf

f
ks

ks

laparoscope is in the correct position through the primary port. A 360° view
k

inside the abdomen is normal. What is the next most important step?
oo

oo

oo

90. A junior specialty trainee (ST2) is doing her first laparoscopy on a nulliparous
eb

eb

eb

woman with a BMI of 29 kg/m2 but no other known risk factors in her history.
m

The ST2 doctor has checked all the equipment and must now make an incision
for introduction of the Veress needle. Where is the most appropriate site on the
woman’s abdomen to place the incision?
t

et

t
ne

ne
n
e.

e.

e.
t

t
46 Module 7

ne

ne

ne
e.

e.

e.
re

fre

fre
Options for questions 91–95

f
ks

ks

ks
A Approximation of the uterosacral ligaments using continuous sutures, so as to obliterate the
peritoneum of the posterior cul-de-sac as high as possible (McCall culdoplasty)
oo

oo

oo
B Attaching the uterosacral and cardinal ligaments to the vaginal cuff and high circumferential
eb

eb

eb
obliteration of the pouch of Douglas
C Colposuspension along with sacrocolpopexy
m

m
D Colpocleisis
E Delayed absorbable polydioxanone sutures for uterosacral ligament suspension
t

t
F Placing concentric purse string sutures around the cul-de-sac to include the posterior vaginal wall,
e

ne

ne
the right pelvic side wall, the serosa of the sigmoid and the left pelvic side wall
.n

G Sacrospinous fixation

e.

e.
e

H Subtotal hysterectomy
re

fre

re
I Transvaginal mesh procedure
sf

sf
ks
J Uterosacral ligament suspension with delayed absorbable sutures
k

k
oo

oo

oo
K Uterosacral ligament suspension with permanent sutures
eb

eb

eb
Each of the following clinical scenarios relates to a surgical procedure. For each
m

m
scenario, select the single most appropriate procedure from the list of options above.
Each option may be used once, more than once or not at all.
t

et
91. An 89-year-old single woman with hypertension and chronic obstructive lung
ne

ne

n
disease presents with a procidentia. She is taking warfarin for atrial fibrillation.
e.

e.

e.
Examination reveals a small atrophic uterus with the vaginal vault lying
fre

fre

re
outside the introitus. What is the most appropriate surgical technique for
her prolapse?
f
ks

ks

ks
92. A 68-year-old woman is undergoing vaginal hysterectomy for a second-­degree
oo

oo

oo

uterine prolapse. The woman has a cystocele but no enterocele. What technique
eb

eb

eb

should be adopted to reduce the chances of posthysterectomy vaginal vault


prolapse?
m

93. A 65-year-old woman presents with a complete vaginal vault prolapse. She has been
informed that, following an operation for correcting her prolapse, occult urinary
t

t
incontinence may become manifest in the form of stress urinary incontinence. She
ne

ne

ne

asks whether any operative procedure could help reduce her chances of urinary
e.

e.

e.

incontinence after surgery. Which is the most appropriate technique?


re

re

fre

94. A 70-year-old woman has just had a vaginal hysterectomy for procidentia. During
sf

closure, the vaginal vault descends to just beyond the introitus. What procedure
ks

ks
k

should be considered to reduce her chances of posthysterectomy vaginal vault


oo

oo

oo

prolapse?
eb

eb

eb

95. A 59-year-old woman is undergoing a hysterectomy for uterine fibroids. The


m

surgeon wishes to reduce the woman’s risk of having a posthysterectomy vaginal


vault prolapse. Which is the most appropriate procedure?
t

et

t
ne

ne
n
e.

e.

e.
t

t
Surgical procedures 47

ne

ne

ne
e.

e.

e.
re

fre

fre
Answers
f
ks

ks

ks
SBAs
oo

oo

oo
71. Answer   D  Operative time and rate of spillage of cyst contents
eb

eb

eb
Explanation
Randomised trials have shown that, while mini-­laparotomy is associated with
m

m
a significant increase in minor postoperative discomfort and recovery time,
and more pain and the need for analgesia, as well as more aesthetic concerns,
operative times are shorter and rates of intraperitoneal spillage are significantly
t

t
e

ne

ne
reduced.
.n

e.

e.
Reference
e
re

fre

re
Stavroulis A, Memtsa M, Yoong W. Methods for specimen removal from the peritoneal
cavity after laparoscopic excision. The Obstetrician & Gynaecologist 2013;15:26–30.
sf

sf
ks
k

k
72. Answer   A  Admit for 24 hours observation and intravenous antibiotics
oo

oo

oo
Explanation
eb

eb

eb
See Figure 1 in the reference article.
m

m
Reference
Shakir F, Diab Y. The perforated uterus. The Obstetrician & Gynaecologist 2013;15:256–61.
t

et
ne

ne

73. Answer   E  Visceral damage (bladder or bowel)

n
e.

e.

e.
Explanation
Women who have a laparotomy for hysterectomy or myomectomy show similar
fre

fre

re
surgical complications, such as haemorrhage, unintended repeat surgery and
f
ks

ks

ks
rehospitalisation, while bladder and bowel injuries are more frequent with a
hysterectomy.
oo

oo

oo

Reference
eb

eb

eb

Younas K, Hadoura E, Majoko F, Bunkheila A. A review of evidence-­based management of


m

uterine fibroids. The Obstetrician & Gynaecologist 2016;18:33–42.

74. Answer   A  Laparoscopic-­assisted vaginal hysterectomy (LAVH)


t

t
ne

ne

ne

Explanation
Certain types of procedures, such as LAVH, appear to be associated with a higher
e.

e.

e.

frequency of bladder injury compared with others.


re

re

fre

Reference
sf

f
ks

ks

Minas V, Gul N, Aust T, Doyle M, Rowlands D. Urinary tract injuries in laparoscopic


k
oo

oo

oo

gynaecological surgery; prevention, recognition and management. The Obstetrician &


Gynaecologist 2014;16:19–28.
eb

eb

eb

75. Answer   E  Umbilical port


m

Explanation
Removing tissue in a tissue retrieval bag via the umbilical port has been
investigated in a randomised and large prospective trial. Removal of benign
t

et

t
ne

ne
n
e.

e.

e.
t

t
48 Module 7

ne

ne

ne
e.

e.

e.
re

fre

fre
ovarian masses via the umbilical port should be utilised where possible, as this
results in less postoperative pain and a quicker retrieval time. Avoidance of
f
ks

ks

ks
extending accessory ports is beneficial in reducing postoperative pain, as well as
oo

oo

oo
reducing the incidence of incisional hernia and the incidence of epigastric vessel
injury. It also leads to improved cosmesis.
eb

eb

eb
Reference
m

m
RCOG. Management of suspected ovarian masses in premenopausal women. RCOG GTG
No. 62. December 2011.
t

t
76. Answer   B 8%
e

ne

ne
.n

Explanation

e.

e.
e

A multicentre randomised controlled trial on 446 women with a laparoscopically


re

fre

re
confirmed tubal ectopic pregnancy and a healthy contralateral tube found that
sf

sf
the cumulative ongoing pregnancy rate was 60.7% after salpingotomy and 56.2%
ks
k

k
after salpingectomy. Persistent trophoblasts occurred more frequently in the
oo

oo

oo
salpingotomy group (14 (7%) versus 1 (<1%); relative risk 15.0, 95% confidence
interval (CI) 2.0–113.4). A repeat ectopic pregnancy occurred in 18 women (8%)
eb

eb

eb
in the salpingotomy group and 12 (5%) in the salpingectomy group (relative
m

m
risk 1.6, 95% CI 0.8–3.3). It was concluded that in women with a tubal ectopic
pregnancy and a healthy contralateral tube, salpingotomy does not significantly
improve fertility prospects compared with salpingectomy.
t

et
ne

ne

Reference

n
RCOG. Diagnosis and management of ectopic pregnancy. RCOG GTG No. 21. November
e.

e.

e.
2016.
fre

fre

77. Answer   D  Open abdominal sacrocolpopexy


re
f
ks

ks

ks
Explanation
oo

oo

oo

Open abdominal sacrocolpopexy is associated with significantly lower rates


eb

eb

eb

of recurrent vault prolapse, dyspareunia and postoperative stress urinary


incontinence when compared with sacrospinous fixation. However, this is not
m

reflected in significantly lower reoperation rates or higher patient satisfaction.


There is limited evidence on the effectiveness of robotic sacrocolpopexy; therefore,
it should only be performed in the context of research or prospective audit
t

t
ne

ne

ne

following local governance procedures. High uterosacral ligament suspension


should only be offered as first-­line management in women with posthysterectomy
e.

e.

e.

vaginal vault prolapse within the context of research or prospective audit


re

re

fre

following local governance procedures.


sf

f
ks

ks

Reference
k
oo

oo

oo

RCOG. Post hysterectomy vaginal vault prolapse. RCOG GTG No. 46. July 2015.
eb

eb

eb

78. Answer   A 27%


m

Explanation
In women with placenta praevia and a previous caesarean section, the risk of
emergency hysterectomy is up to 27 in 100 women (very common). In women
t

et

t
ne

ne
n
e.

e.

e.
t

t
Surgical procedures 49

ne

ne

ne
e.

e.

e.
re

fre

fre
with an abnormally adherent placenta (e.g. placenta accreta), the woman should
be advised that hysterectomy is highly likely.
f
ks

ks

ks
Reference
oo

oo

oo
RCOG. Caesarean section for placenta praevia. RCOG Consent Advice No. 12. December
eb

eb

eb
2010.
m

m
79. Answer   C  12 mmHg
Explanation
Surgery on a pregnant patient with acute appendicitis should include a left
t

t
e

ne

ne
lateral tilt to avoid aortocaval compression, avoidance of uterine or cervical
.n

manipulation or instrumentation, and limiting intra-­abdominal pressure to

e.

e.
e

<12 mmHg.
re

fre

re
sf

sf
Reference
ks
k

k
Weston P, Moroz P. Appendicitis in pregnancy: how to manage and whether to deliver.
oo

oo

oo
The Obstetrician & Gynaecologist 2015;17:105–10.
eb

eb

eb
80. Answer   E 0.1%.
m

m
Explanation
Serious risks of caesarean section include: bladder injury (1 in 1000, rare), ureteric
injury (3 in 10,000, rare) and death (approximately 1 in 12,000, very rare).
t

et
ne

ne

n
Reference
e.

e.

e.
RCOG. Caesarean section. RCOG Consent Advice No. 7. October 2009.
fre

fre

EMQs f re
ks

ks

ks
oo

oo

oo

81. Answer   J  <1 in 100 women


eb

eb

eb

82. Answer   E  8 in 100 women


m

83. Answer   B  26 in 100 women

84. Answer   H  2 in 100 women


t

t
ne

ne

ne

85. Answer   G  4 in 100 women


e.

e.

e.
re

re

fre

References
sf

f
ks

ks

RCOG. Vaginal surgery for prolapse. RCOG Consent Advice No. 5. October 2009.
k

Repair of third and fourth degree perineal tear following childbirth. RCOG Consent Advice
oo

oo

oo

No. 9. June 2010.


Diagnostic laparoscopy. RCOG Consent Advice No. 2. June 2017.
eb

eb

eb

Surgical management of miscarriage and removal of persistent placental or fetal remains.


RCOG Consent Advice No. 10 (joint with AEPAU). January 2018.
m

m
t

et

t
ne

ne
n
e.

e.

e.
t

t
50 Module 7

ne

ne

ne
e.

e.

e.
re

fre

fre
86. Answer   D  A small incision 3 cm below the left costal margin in the
midclavicular line
f
ks

ks

ks
Explanation
oo

oo

oo
The rate of adhesion formation at the umbilicus may be up to 50% following
midline laparotomy and 23% following low transverse incision. The umbilicus
eb

eb

eb
may not, therefore, be the most appropriate site for primary trocar insertion
m

m
following previous abdominal surgery. The most usual alternative site is in the
left upper quadrant, where adhesions rarely form, although even this may be
inappropriate if there has been previous surgery in this area or splenomegaly. The
t

t
preferred point of entry is 3 cm below the left costal margin in the midclavicular
e

ne

ne
.n

line (Palmer’s point).

e.

e.
e
re

fre

re
87. Answer   E  Insertion into abdomen at 90° to the skin with the operating table
horizontal
sf

sf
ks
k

k
Explanation
oo

oo

oo
The Veress needle should be sharp, with a good and tested spring action. A
disposable needle is recommended, as it will fulfil these criteria. The operating
eb

eb

eb
table should be horizontal (not in the Trendelenburg tilt) at the start of the
m

m
procedure. The abdomen should be palpated to check for any masses and for the
position of the aorta before insertion of the Veress needle.
The lower abdominal wall should be stabilised in such a way that the Veress
t

et
needle can be inserted at right angles to the skin and it should be pushed in just
ne

ne

sufficiently to penetrate the fascia and the peritoneum.

n
e.

e.

e.
88. Answer   L  Rotate the laparoscope through 360° to visualise and exclude any
fre

fre

re
adhesion of bowel
f
ks

ks

ks
Explanation
oo

oo

oo

Once the laparoscope has been introduced through the primary cannula, it
should be rotated through 360° to check visually for any adherent bowel. If this is
eb

eb

eb

present, it should be closely inspected for any evidence of haemorrhage, damage


m

or retroperitoneal haematoma.

89. Answer   N  Tilt the operation table to the Trendelenburg position


t

t
ne

ne

ne

Explanation
The Trendelenburg position helps the bowel to move out of the pelvis to allow
e.

e.

e.

safe entry of secondary ports. All such ports should be introduced under direct
re

re

fre

vision.
sf

f
ks

ks
k

90. Answer   A  A small vertical incision starting at the base of the umbilicus
oo

oo

oo

Explanation
eb

eb

eb

In most circumstances, the primary incision for laparoscopy should be vertical


from the base of the umbilicus (not in the skin below the umbilicus). Care should
m

be taken not to incise so deeply as to enter the peritoneal cavity.


t

et

t
ne

ne
n
e.

e.

e.
t

t
Surgical procedures 51

ne

ne

ne
e.

e.

e.
re

fre

fre
Reference
f RCOG. Preventing entry-­related gynaecological laparoscopic injuries. RCOG GTG No 49.
ks

ks

ks
May 2008.
oo

oo

oo
91. Answer   D Colpocleisis
eb

eb

eb
Explanation
m

m
Colpocleisis is a useful procedure in women with serious co-­morbidities.
Colpocleisis is a safe and effective procedure that can be considered for frail
women and/or women who do not wish to retain sexual function.
t

t
e

ne

ne
.n

92. Answer   A  Approximation of the uterosacral ligaments using continuous

e.

e.
sutures, so as to obliterate the peritoneum of the posterior cul-de-sac as high as
e
re

fre

re
possible (McCall culdoplasty)
sf

sf
ks
Explanation
k

k
A small randomised controlled trial compared vaginal Moschcowitz-­type
oo

oo

oo
operation, McCall culdoplasty and peritoneal closure of the cul-de-sac
eb

eb

eb
as preventative measures against the development of an enterocele. It
showed that McCall culdoplasty was more effective than vaginal Moschcowitz
m

m
or simple closure of the peritoneum in preventing an enterocele at the 3-­year
follow-up.
t

et
93. Answer   C  Colposuspension along with sacrocolpopexy
ne

ne

n
Explanation
e.

e.

e.
Colposuspension performed at the time of sacrocolpopexy is an effective measure
fre

fre

re
to reduce postoperative symptomatic stress urinary incontinence in previously
f
ks

ks

ks
continent women.
oo

oo

oo

94. Answer   G  Sacrospinous fixation


eb

eb

eb

Explanation
m

Sacrospinous fixation at the time of vaginal hysterectomy should be considered


when the vault descends to the introitus during closure.

95. Answer   B  Attaching the uterosacral and cardinal ligaments to the vaginal cuff
t

t
ne

ne

ne

and high circumferential obliteration of the pouch of Douglas


e.

e.

e.

Explanation
re

re

fre

Suturing the cardinal and uterosacral ligaments to the vaginal cuff at the time of
hysterectomy is effective in preventing posthysterectomy vaginal vault prolapse
sf

f
ks

ks

following both abdominal and vaginal hysterectomies.


k
oo

oo

oo

Reference
eb

eb

eb

RCOG. Post-hysterectomy vaginal vault prolapse. RCOG GTG No. 46. July 2015.
m

m
t

et

t
ne

ne
n
e.

e.

e.
t

t
ne

ne

ne
e.

e.

e.
re

fre

fre
Antenatal care
Module
f
ks

ks

ks
8
oo

oo

oo
eb

eb

eb
m

m
t

t
e

ne

ne
SBAs
.n

e.

e.
e

96. A 32-year-old woman is seen for preconception counselling. She has a history of
re

fre

re
breast cancer and has just completed a course of tamoxifen.
sf

sf
How long should she wait before she can try to conceive?
ks
k

k
A. 4 weeks
oo

oo

oo
B. 6 weeks
eb

eb

eb
C. 3 months
D. 6 months
m

m
E. 1 year

97. A 38-year-old woman is seen in antenatal clinic at 12 weeks of gestation. She has a
t

et
body mass index (BMI) of 37 kg/m2 and does not regularly undertake exercise.
ne

ne

What would the recommendation be with regard to starting exercise in

n
e.

e.

e.
pregnancy in order to control her weight?
fre

fre

re
A. Do not recommend initiating exercise in pregnancy
B. Gentle exercise for 60 minutes per day
f
ks

ks

ks
C. Gentle exercise for 60 minutes three times per week
oo

oo

oo

D. Moderate aerobic exercise for 15 minutes per day three times per week
E. Moderate aerobic exercise for 30 minutes per day
eb

eb

eb

98. A 27-year-old woman attends for her pregnancy dating scan. She is unsure of the
m

date of her last menstrual period. The following fetal measurements are obtained:
Crown–rump length 86 mm
t

t
ne

ne

ne

Biparietal diameter 18 mm
Head circumference 100 mm
e.

e.

e.

Femur length 12 mm
re

re

fre

Abdominal circumference 67 mm
sf

f
ks

ks

Which of these measurements should be used to date the pregnancy?


k
oo

oo

oo

A. Abdominal circumference
B. Biparietal diameter
eb

eb

eb

C. Crown–rump length
m

D. Femur length
E. Head circumference

52
t

et

t
ne

ne
n
e.

e.

e.
t

t
Antenatal care 53

ne

ne

ne
e.

e.

e.
re

fre

fre
99. A 42-year-old woman is seen in the booking clinic in her first pregnancy. She
has a history of chronic hypertension and is taking methyldopa. She is white
f
ks

ks

ks
Caucasian with a BMI of 24 kg/m2.
oo

oo

oo
What daily dose of vitamin D should be recommended for her throughout this
pregnancy?
eb

eb

eb
A. 400 units
m

m
B. 800 units
C. 1000 units
D. 1200 units
E. 1500 units
t

t
e

ne

ne
.n

100. With regard to the routine anomaly scan in pregnancy, what threshold of nuchal-­

e.

e.
e

fold measurement should trigger a referral to a fetal medicine specialist?


re

fre

re
A. 6 mm
sf

sf
ks
B. 7 mm
k

k
C. 8 mm
oo

oo

oo
D. 9 mm
eb

eb

eb
E. 10 mm
m

m
101. In an uncomplicated pregnancy, how often should auscultation of the fetal heart
be performed by the midwifery team?
A. Every 2 weeks from 24 weeks
t

et
ne

ne

B. Every 3 weeks from 24 weeks

n
C. Every 4 weeks from 24 weeks
e.

e.

e.
D. Every appointment after 20 weeks
fre

fre

re
E. Only when requested by the mother
f
ks

ks

ks
102. A 37-year-old pregnant woman is seen in the clinic at 42 weeks of gestation. She
oo

oo

oo

declines an offer of induction of labour, wanting to keep things ‘as natural as


possible’.
eb

eb

eb

What management should be offered?


m

A. Daily cardiotocograph (CTG), twice weekly amniotic fluid volume assessment


and umbilical artery Doppler
B. Daily CTG and twice weekly umbilical artery Doppler
t

t
C. Daily CTG, weekly amniotic fluid volume assessment and umbilical artery
ne

ne

ne

Doppler
e.

e.

e.

D. Twice weekly CTG and amniotic fluid volume assessment


re

re

fre

E. Twice weekly CTG, amniotic fluid volume assessment and umbilical artery
Doppler
sf

f
ks

ks
k
oo

oo

oo
eb

eb

eb
m

m
t

et

t
ne

ne
n
e.

e.

e.
t

t
54 Module 8

ne

ne

ne
e.

e.

e.
re

fre

fre
103. A 26-year-old woman is seen for her antenatal booking appointment. She was in
a road traffic accident 10 years ago and has a spinal cord transection at the level
f
ks

ks

ks
of T11 and is paraplegic. What additional complication is she at risk of when
oo

oo

oo
compared with women with lower spinal cord injuries?
A. Autonomic dysreflexia
eb

eb

eb
B. Inability to perceive labour pains
m

m
C. Late preterm labour
D. Malpresentation at term
E. Ventilatory dysfunction
t

t
e

ne

ne
104. A woman is seen at 24 weeks of gestation in the joint obstetric and mental health
.n

clinic with a psychiatrist. She has been referred by the community midwife

e.

e.
e

because of the woman’s concerns after watching her sister give birth. She
re

fre

re
recounts her sister having a very traumatic forceps delivery followed by a massive
sf

sf
ks
haemorrhage, describing ‘blood everywhere’. She is diagnosed with severe anxiety
k

k
and post-­traumatic stress disorder (PTSD). She is requesting an elective caesarean
oo

oo

oo
section.
eb

eb

eb
What intervention would be most appropriate?
A. Citalopram
m

m
B. Cognitive behavioural therapy (CBT)
C. Facilitated self-help
D. Fluoxetine
t

et
ne

ne

E. Offer an elective caesarean section at 39 weeks

n
e.

e.

e.
105. A 32-year-old woman is seen in the antenatal clinic. She is 16 weeks pregnant and
fre

fre

re
is planning to go to Tanzania for a safari in 4 weeks. She enquires about malaria
prophylaxis.
f
ks

ks

ks
What would be the primary recommendation?
oo

oo

oo

A. Ensure she takes chemoprophylaxis and continues after the trip


B. Keep her skin covered as much as possible
eb

eb

eb

C. Postpone the trip if possible


m

D. Take standby treatment (quinine) with her


E. Use insecticide-­impregnated mosquito nets at night
t

t
106. A woman with a monochorionic diamniotic (MCDA) twin pregnancy at 25 weeks
ne

ne

ne

of gestation has been diagnosed with severe twin-to-twin transfusion syndrome


e.

e.

e.

(TTTS) and has been referred to the regional centre.


What is the recommended first-­line management, assuming no
re

re

fre

contraindications to any treatment?


sf

f
ks

ks

A. Laser ablation of placental vessels


k
oo

oo

oo

B. Radiofrequency ablation of placental vessels


C. Selective amnioreduction
eb

eb

eb

D. Selective feticide via cord coagulation


E. Septostomy
m

m
t

et

t
ne

ne
n
e.

e.

e.
t

t
Antenatal care 55

ne

ne

ne
e.

e.

e.
re

fre

fre
107. A woman presents to the obstetric triage unit with reduced fetal movements
at 27 weeks of gestation. She has a monochorionic twin pregnancy. A CTG is
f
ks

ks

ks
performed. It is normal in the presenting twin but shows a bradycardia in twin
oo

oo

oo
2. Both twins have a cephalic presentation. Preparations are made for emergency
delivery, but during this time in the antenatal ward, the fetal heart of twin 2
eb

eb

eb
stops. An ultrasound examination confirms the absence of a fetal heartbeat in
twin 2.
m

m
What is the most appropriate action?
A. Administer steroids and commence induction of labour when appropriate
B. Administer steroids and deliver by caesarean section when appropriate
t

t
e

ne

ne
C. Continue with a category 1 caesarean section
.n

D. Postpone delivery and perform a detailed ultrasound scan after 4 weeks

e.

e.
e

E. Postpone delivery and perform fetal MRI after 4 weeks


re

fre

re
sf

sf
ks
108. A woman is referred from the antenatal clinic with reduced fetal movements at
k

k
29 weeks of gestation. Computerised CTG (cCTG) is normal with a short-­term
oo

oo

oo
variation (STV) of 8 ms. Ultrasound shows an estimated fetal weight plotting
eb

eb

eb
below the 10th centile, with reversed end-­diastolic flow in the umbilical artery.
Middle cerebral artery (MCA) Doppler and ductus venosus Doppler are normal.
m

m
She was given a course of steroids 2 weeks previously for a threatened preterm
labour.
What is the recommended management?
t

et
ne

ne

A. Category 2 caesarean section

n
B. Daily CTG and deliver if STV is <5 ms
e.

e.

e.
C. Daily Dopplers and deliver if ductus venosus becomes abnormal
fre

fre

re
D. Daily Dopplers and deliver if MCA Doppler becomes abnormal
E. Immediate induction of labour
f
ks

ks

ks
oo

oo

oo

109. A 28-year-old woman in her first pregnancy is in the antenatal clinic at 12


weeks of gestation. She has a history of female genital mutilation (FGM). On
eb

eb

eb

examination, she has type 3 FGM, and the vaginal orifice admits one finger only.
m

What would be the recommended management?


A. Anterior episiotomy at the time of delivery
B. Deinfibulation as soon as possible
t

t
C. Deinfibulation at 20 weeks
ne

ne

ne

D. Deinfibulation at 28 weeks
e.

e.

e.

E. Deinfibulation in early labour


re

re

fre

110. What is the most common adverse obstetric problem in a pregnancy complicated
sf

f
ks

ks

by unexplained antepartum haemorrhage?


k
oo

oo

oo

A. Fetal growth restriction


B. Oligohydramnios
eb

eb

eb

C. Premature rupture of membranes


D. Preterm delivery
m

E. Stillbirth
t

et

t
ne

ne
n
e.

e.

e.
t

t
56 Module 8

ne

ne

ne
e.

e.

e.
re

fre

fre
111. Which obstetric complication has a significantly increased likelihood in women
who have undergone bariatric surgery?
f
ks

ks

ks
A. Delivery by caesarean section
oo

oo

oo
B. Gestational diabetes
C. Large-for-gestational-age fetus
eb

eb

eb
D. Pre-eclampsia
m

m
E. Small-for-gestational-age (SGA) fetus

112. What is the incidence of vasa praevia in women who have had successful in vitro
fertilisation (IVF) treatment?
t

t
e

ne

ne
A. 1 in 10 pregnancies
.n

e.

e.
B. 1 in 30 pregnancies
e

C. 1 in 100 pregnancies
re

fre

re
D. 1 in 300 pregnancies
sf

sf
ks
E. 1 in 1000 pregnancies
k

k
oo

oo

oo
113. A woman at 36 weeks of gestation has prelabour rupture of membranes. She
eb

eb

eb
has a temperature of 40°C and is hypotensive with a systolic blood pressure of
85 mmHg. She has a diffuse macular rash and gives a history of vomiting and
m

m
diarrhoea. Blood tests reveal a significantly raised creatinine level, and her
bilirubin level is also raised.
What single additional feature would confirm a diagnosis of staphylococcal
t

et
ne

ne

toxic shock?

n
A. Coagulopathy with platelets <100 × 109/l
e.

e.

e.
B. Desquamation of the palms and soles
fre

fre

re
C. Isolation of Staphylococcus from a throat swab
D. Lactate >4 mmol/l f
ks

ks

ks
E. Oropharyngeal hyperaemia
oo

oo

oo

114. A woman is seen in antenatal clinic at 39 weeks. It is her first pregnancy and she
eb

eb

eb

is concerned about continuing the pregnancy beyond her estimated date of


m

delivery. She is asking about a membrane sweep.


When should a first membrane sweep be offered for this woman?
A. A membrane sweep is not routinely offered
t

t
B. Offer a membrane sweep at 39 weeks onwards
ne

ne

ne

C. Offer a membrane sweep at 40 weeks onwards


e.

e.

e.

D. Offer a membrane sweep at 41 weeks onwards


re

re

fre

E. Offer a membrane sweep prior to formal induction of labour


sf

f
ks

ks

115. What is the earliest gestational age at which amniocentesis can be performed?
k
oo

oo

oo

A. 12 weeks
B. 13 weeks
eb

eb

eb

C. 14 weeks
m

D. 15 weeks
E. 16 weeks
t

et

t
ne

ne
n
e.

e.

e.
t

t
Antenatal care 57

ne

ne

ne
e.

e.

e.
re

fre

fre
116. A 29-year-old woman at 33 weeks of gestation presents with a 24-­hour history of
symptoms suggestive of preterm prelabour rupture of membranes (PPROM).
f
ks

ks

ks
Which initial test should be performed to confirm a diagnosis of PPROM?
oo

oo

oo
A. Nitrazine test
B. Speculum examination of the vagina
eb

eb

eb
C. Test vaginal fluid for insulin-­like growth factor-­binding protein-­1 (IGFBP-1)
m

m
D. Test vaginal fluid for placental α-microglobulin-1 (PAMG-1)
E. Ultrasound scan for assessment of amniotic fluid volume

117. A 33-year-old woman has just had her second normal vaginal delivery. She is
t

t
e

ne

ne
a known group B Streptococcus (GBS) carrier, and received the loading dose
.n

of benzylpenicillin 30 minutes prior to delivery. She is otherwise low risk

e.

e.
e

obstetrically and the baby was born in good condition.


re

fre

re
What initial management of the neonate would be recommended?
sf

sf
ks
A. 48-Hour admission
k

k
B. Increased observations for 12 hours
oo

oo

oo
C. Take bloods and treat if C-­reactive protein (CRP) raised or cultures positive
eb

eb

eb
D. Treat as normal
E. Treat with prophylactic antibiotics
m

m
118. A 40-year-old woman in her first pregnancy attends antenatal clinic for advice at
10 weeks of gestation. She is fit and healthy and has no significant past medical
t

et
history. She asks whether it is safe to drink alcohol in pregnancy. Her preferred
ne

ne

n
drink is red wine.
e.

e.

e.
What would be the recommended upper limit of amount of wine that she
fre

fre

re
could drink in this pregnancy?
A. 80 ml one to two times per week
f
ks

ks

ks
B. 100 ml one to two times per week
oo

oo

oo

C. 120 ml one to two times per week


D. 140 ml one to two times per week
eb

eb

eb

E. 160 ml one to two times per week


m

119. A new screening test is being assessed in a university antenatal clinic looking
at the ability of a raised serum level of substance X to detect Down’s syndrome
in women over 45 years of age. One hundred women are entered into the trial
t

t
ne

ne

ne

and 12 test positive for raised serum levels of substance X. Of these 12, eight are
subsequently found to have babies with Down’s syndrome. From the 100 women,
e.

e.

e.

there are ten cases of Down’s syndrome in total.


re

re

fre

What is the sensitivity of elevated substance X to detect Down’s syndrome in


sf

these women?
ks

ks
k

A. 4%
oo

oo

oo

B. 8%
eb

eb

eb

C. 10%
D. 12%
m

E. 80%
t

et

t
ne

ne
n
e.

e.

e.
t

t
58 Module 8

ne

ne

ne
e.

e.

e.
re

fre

fre
120. Which three classes of antihypertensive drugs appear to be associated with an
increased risk of congenital malformations?
f
ks

ks

ks
Angiotensin- Angiotensin- β-Blockers Calcium- Thiazides
oo

oo

oo
converting enzyme receptor channel
eb

eb

eb
(ACE) inhibitors blockers (ARB) blockers
A ✓ ✓ ✓
m

m
B ✓ ✓ ✓
C ✓ ✓ ✓
t

t
D ✓ ✓ ✓
e

ne

ne
.n

E ✓ ✓ ✓

e.

e.
e
re

fre

re
121. For any potentially sensitising event, rhesus D-­negative, previously non-­sensitised
sf

sf
ks
women should receive a minimum dose of 500 IU anti-­D immunoglobulin (Ig)
k

k
intramuscularly within 72 hours of the event, regardless of whether the woman
oo

oo

oo
has already received routine antenatal anti-­D Ig prophylaxis at 28 weeks. Any
eb

eb

eb
additional dose of anti-­D Ig needed is guided by a test of maternal blood for
fetomaternal haemorrhage.
m

m
The dose calculation is based on which formula?
A. 100 IU anti-­D Ig/ml of fetal red blood cells
B. 125 IU anti-­D Ig/ml of fetal red blood cells
t

et
ne

ne

C. 150 IU anti-­D Ig/ml of fetal red blood cells

n
D. 175 IU anti-­D Ig/ml of fetal red blood cells
e.

e.

e.
E. 200 IU anti-­D Ig/ml of fetal red blood cells
fre

fre

122. A 28-year-old woman in her second pregnancy has a booking blood test at 14 f re
ks

ks

ks
weeks of gestation that shows the presence of anti-­K antibody at a titre of 1 in 16.
oo

oo

oo

What is the most appropriate action?


A. Immediate maternal blood test for fetal genotyping for K antigen
eb

eb

eb

B. Maternal blood test for fetal genotyping for K antigen after 20 weeks of
m

gestation
C. Measure titre every 2 weeks until 28 weeks
D. Measure titre every 4 weeks until 32 weeks
t

t
E. Paternal blood test for genotyping for K antigen
ne

ne

ne
e.

e.

e.

123. What is the optimum method of screening for chromosomal abnormality in a


monochorionic twin pregnancy at 13 weeks of gestation?
re

re

fre

A. Amniocentesis
sf

f
ks

ks

B. Combined screening test


k
oo

oo

oo

C. Non-invasive prenatal testing


D. Nuchal translucency measurement
eb

eb

eb

E. Quadruple test
m

m
t

et

t
ne

ne
n
e.

e.

e.
t

t
Antenatal care 59

ne

ne

ne
e.

e.

e.
re

fre

fre
124. At her booking visit with a midwife, a 27-year-old woman is offered
haemoglobinopathy screening using the Family Origin Questionnaire. She is
f
ks

ks

ks
deemed to be at low risk.
oo

oo

oo
What value of mean corpuscular haemoglobin would trigger laboratory
screening?
eb

eb

eb
A. <27 pg
m

m
B. 27–29 pg
C. 30–32 pg
D. 33–35 pg
E. >35 pg
t

t
e

ne

ne
.n

125. What is the primary reason that serological screening for hepatitis B is routinely

e.

e.
e

offered to all pregnant women?


re

fre

re
A. To help cross-­match blood for transfusion as and when needed
sf

sf
B. To implement antenatal intervention to decrease the risk of mother-to-child
ks
k

k
transmission
oo

oo

oo
C. To implement postnatal intervention to decrease the risk of mother-to-child
transmission
eb

eb

eb
D. To reduce the risk of infecting healthcare providers
m

m
E. To treat the mother to avoid worsening of hepatitis B disease
126. A high level of vitamin A intake in the first trimester of pregnancy may be
harmful to the fetus and poses a risk of teratogenicity.
t

et
ne

ne

What is the recommended upper limit of daily intake of vitamin A in early

n
pregnancy?
e.

e.

e.
A. 500 μg
fre

fre

re
B. 600 μg
f
ks

ks

ks
C. 700 μg
D. 800 μg
oo

oo

oo

E. 900 μg
eb

eb

eb

127. A 27-year-old multiparous woman had spontaneous rupture of membranes at


m

37 weeks of gestation. She is a carrier of group B Streptococcus (GBS) as detected


on a vaginal swab in the second trimester.
What is the most appropriate management?
t

t
A. Await spontaneous labour and use intrapartum antibiotic prophylaxis (IAP)
ne

ne

ne

B. Offer one dose of prophylactic antibiotic before elective caesarean delivery


e.

e.

e.

C. Offer induction of labour after 24 hours and IAP


re

re

fre

D. Offer induction of labour at 41 weeks and IAP


E. Offer induction of labour immediately and IAP
sf

f
ks

ks
k

128. The rate of survival for babies born at the extremes of prematurity (between 22
oo

oo

oo

and 26 weeks) has improved in recent years.


eb

eb

eb

What is the current rate of overall survival for extreme preterm births?
A. 25%
m

B. 37%
C. 53%
D. 63%
t

et

t
ne

ne
n
e.

e.

e.
t

t
60 Module 8

ne

ne

ne
e.

e.

e.
re

fre

fre
129. Babies born preterm have a much higher risk of suffering from disabilities
compared with those born at term.
f
ks

ks

ks
What is the most common major long-­term consequence of prematurity?
oo

oo

oo
A. Bronchopulmonary dysplasia
B. Infective morbidity
eb

eb

eb
C. Major cerebral scan abnormality
m

m
D. Neurodevelopmental disability
E. Retinopathy of prematurity

130. The risk of preterm birth is considerably higher in multiple pregnancies than in
t

t
e

ne

ne
singleton pregnancies.
.n

What proportion of twin births take place before 32 weeks of gestation?

e.

e.
e

A. 5%
re

fre

re
B. 10%
sf

sf
ks
C. 15%
k

k
D. 20%
oo

oo

oo
E. 25%
eb

eb

eb
131. A woman who is 36 weeks pregnant attends the maternity day assessment unit
m

m
with decreased fetal movements. She has no other symptoms. The CTG and
clinical observations are all normal. A dipstick test of a urine sample reveals 1+
proteinuria. The sample is sent for culture and sensitivity. The result is as follows:
t

et
ne

ne

The automated urine microscopy results are:

n
e.

e.

e.
White blood cells >100 (× 106/l)
fre

fre

re
Red blood cells >40–100 (× 106/l)
Squamous epithelial cells ≥20 (× 106/l)
f
ks

ks

ks
oo

oo

oo

The culture results are:


eb

eb

eb

Organism count 104–105/ml


Organism Group B Streptococcus
m

Antibiotic sensitivity Erythromycin, nitrofurantoin, penicillin

What is the correct management?


t

t
ne

ne

ne

A. No treatment required
B. Offer IAP only
e.

e.

e.

C. Send a further specimen and offer IAP if GBS confirmed on a second sample
re

re

fre

D. Treat with erythromycin now and offer IAP


sf

E. Treat with nitrofurantoin now and offer IAP


ks

ks
k
oo

oo

oo

132. From which gestational age is cell-­free fetal DNA present in reliably measurable
levels for aneuploidy screening?
eb

eb

eb

A. 8 weeks
m

B. 10 weeks
C. 12 weeks
D. 14 weeks
t

et

E. 16 weeks
ne

ne
n
e.

e.

e.
t

t
Antenatal care 61

ne

ne

ne
e.

e.

e.
re

fre

fre
133. A woman is found to have an adnexal cyst at her 20-­week anomaly scan. What is
the most common type of adnexal cystic lesion diagnosed at this gestation?
f
ks

ks

ks
A. Corpus luteum cyst
oo

oo

oo
B. Dermoid cyst
C. Endometrioma
eb

eb

eb
D. Fimbrial cyst
m

m
E. Follicular cyst

134. What proportion of terminations of pregnancy in the UK are carried out for fetal
abnormality?
t

t
e

ne

ne
A. 1%
.n

e.

e.
B. 2%
e

C. 5%
re

fre

re
D. 10%
sf

sf
ks
E. 20%
k

k
oo

oo

oo
135. After adjusting for confounding factors, which obstetric factors are associated
eb

eb

eb
with partner abuse during pregnancy?
m

m
Postpartum Preterm SGA fetus Urinary tract Vaginal
endometritis labour infection bleeding
A ✓ ✓ ✓
t

et
ne

ne

B ✓ ✓ ✓

n
e.

e.

e.
C ✓ ✓ ✓
fre

fre

re
D ✓ ✓ ✓
✓ ✓ ✓
E
f
ks

ks

ks
oo

oo

oo
eb

eb

eb
m

m
t

t
ne

ne

ne
e.

e.

e.
re

re

fre
sf

f
ks

ks
k
oo

oo

oo
eb

eb

eb
m

m
t

et

t
ne

ne
n
e.

e.

e.
t

t
62 Module 8

ne

ne

ne
e.

e.

e.
re

fre

fre
EMQs
f
ks

ks

ks
Options for questions 136–139
oo

oo

oo
A Amniotic fluid volume and umbilical artery Doppler in 1 week
eb

eb

eb
B Amniotic fluid volume and umbilical artery Doppler in 2 weeks
C Amniotic fluid volume and umbilical artery Doppler twice per week
m

m
D Biophysical profile
E Continue low-­risk pathway
t

t
F Ductus venosus Doppler
e

ne

ne
.n

G Growth scan in 3–4 weeks

e.

e.
e

H Growth scan, amniotic fluid volume


re

fre

re
I Growth scan, amniotic fluid volume and umbilical artery Doppler
sf

sf
J Growth scan, amniotic fluid volume and umbilical artery Doppler in 2
ks
k

weeks

k
oo

oo

oo
K Middle cerebral artery (MCA) Doppler
eb

eb

eb
L Serial growth scans from 28 weeks
M Umbilical vein Doppler
m

m
N Uterine artery Doppler at 18 weeks

For the following clinical scenarios, choose the most important ultrasound scan
t

et
ne

ne

investigation or action to take next. Assume that you are in a hospital where fetal medi-

n
cine scanning is available. Each option may be used once, more than once or not at all.
e.

e.

e.
fre

fre

re
136. A 28-year-old woman has a growth scan as her previous baby was born with a
weight below the 5th centile. She is now at 28 weeks of gestation. The growth
f
ks

ks

ks
scan for obstetric history shows that the estimated fetal weight (EFW) is on
oo

oo

oo

the 50th centile. Amniotic fluid volume and umbilical artery Doppler are
normal.
eb

eb

eb

137. A 32-year-old woman in her first pregnancy is seen at 28 weeks of gestation. She
m

has had a growth scan due to a symphysis fundal height measurement below
the 10th centile. The growth scan shows an EFW of <5th centile with normal
amniotic fluid volume and umbilical artery Doppler.
t

t
ne

ne

ne

138. A 26-year-old patient is seen in the antenatal clinic. She is at 29 + 5 weeks of


e.

e.

e.

gestation and has a symphysis fundal height below the 10th centile. She has an
re

re

fre

ultrasound scan that shows EFW below the 5th centile and a normal amniotic
fluid volume with absent end-­diastolic flow in the umbilical artery. The CTG is
sf

f
ks

ks

normal.
k
oo

oo

oo

139. An 18-year-old woman in her first pregnancy has a growth scan at 28 weeks
eb

eb

eb

of gestation due to her high BMI of 37 kg/m2. The growth scan shows an EFW
<10th centile. Amniotic fluid volume is normal and the umbilical artery Doppler
m

shows a raised pulsatility index of >2 standard deviations with end-­diastolic flow
positive.
t

et

t
ne

ne
n
e.

e.

e.
t

t
Antenatal care 63

ne

ne

ne
e.

e.

e.
re

fre

fre
Options for questions 140–144

f
ks

ks

ks
A Arterial blood gas
oo

oo

oo
B Compression duplex ultrasound
C Continue treatment and repeat scan in 7 days
eb

eb

eb
D Computed tomography pulmonary angiography (CTPA)
m

m
E D-dimer
F ECG and chest X-­ray (CXR)
G Full blood count (FBC), urea and electrolytes (U&E), liver function test (LFT), coagulation test
t

t
e

ne

ne
H Magnetic resonance venography
.n

e.

e.
I Refer back to midwife-­led care
e
re

fre

re
J Start prophylactic low-molecular-weight heparin (LMWH)
K Start therapeutic dose LMWH
sf

sf
ks
k

k
L Start unfractionated heparin
oo

oo

oo
M Stop treatment and repeat scan in 3 days
eb

eb

eb
N Stop treatment and repeat scan in 7 days
O Thrombophilia screening
m

m
P Ventilation/perfusion (V/Q) scan

For each of the following clinical scenarios, select the most appropriate next step in
t

et
ne

ne

management from the list of options above. Each option may be used once, more than

n
once or not at all.
e.

e.

e.
fre

fre

re
140. A 34-year-old patient is seen in the obstetric triage unit out of hours at 26
weeks of gestation with acute pain, tenderness and swelling of her left leg. She is
f
ks

ks

ks
otherwise well with no chest pain or shortness of breath.
oo

oo

oo

141. A 38-year-old patient is seen at 22 weeks of gestation with acute swelling and pain
eb

eb

eb

in her right calf. She was started on a therapeutic dose of LMWH and has had a
Doppler ultrasound scan that is negative for deep vein thrombosis (DVT). She
m

remains symptomatic.
142. An 18-year-old patient at 35 weeks of gestation has presented with chest pain and
shortness of breath. She has a sinus tachycardia and CXR is normal. Laboratory
t

t
ne

ne

ne

investigations are normal and she has been started on LMWH.


e.

e.

e.

143. A 42-year-old patient is seen at 18 weeks of gestation with chest pain, mild
re

re

fre

shortness of breath and a swollen left leg. Baseline investigations including


sf

bloods, CXR and ECG are normal. She has a duplex ultrasound the same day that
ks

ks
k

confirms left-­sided femoral DVT.


oo

oo

oo

144. A 23-year-old patient is referred by the midwife at 38 weeks of gestation with


eb

eb

eb

chest pain and shortness of breath. The symptoms resolve but the CTPA scan
report states: ‘No evidence of embolus in the segmental or subsegmental
m

pulmonary tree, unable to exclude smaller peripheral emboli on CTPA.’


t

et

t
ne

ne
n
e.

e.

e.
t

t
64 Module 8

ne

ne

ne
e.

e.

e.
re

fre

fre
Options for questions 145–148

f
ks

ks

ks
A Alternate days
B At least four times per day
oo

oo

oo
C Daily
eb

eb

eb
D Fortnightly
m

m
E Hourly
F More than four times per day
G Once only
t

t
e

ne

ne
H Three times per week
.n

I Twice per day

e.

e.
e

J Twice per week


re

fre

re
K Weekly
sf

sf
ks
k

k
For each of the following clinical scenarios, select the single most appropriate fre-
oo

oo

oo
quency to monitor the requested parameter. Each option may be used once, more than
once or not at all.
eb

eb

eb
145. A 33-year-old woman is in her first pregnancy at 34 weeks of gestation. She has a
m

m
blood pressure of 164/108 mmHg that is treated with oral labetalol as an inpatient.
Her urinary protein : creatinine ratio is 26 mg/mmol. How often should urinary
protein quantification be repeated?
t

et
ne

ne

n
146. A 28-year-old woman is in her first pregnancy at 32 weeks of gestation with a
e.

e.

e.
blood pressure of 154/103 mmHg with urinary protein : creatinine ratio of
fre

fre

re
22 mg/mmol. Treatment with labetalol is commenced. How often should her
blood pressure be checked?
f
ks

ks

ks
147. A 36-year-old woman is in her second pregnancy with dichorionic diamniotic
oo

oo

oo

(DCDA) twins at 30 weeks of gestation. Her blood pressure is 153/98 mmHg on


eb

eb

eb

two occasions. Her urinary protein : creatinine ratio is 89 mg/mmol. Initial blood
tests for FBC, U&E and LFT are normal. How often should these blood tests be
m

repeated?

148. A 29-year-old woman in her first pregnancy has been diagnosed with severe
t

t
gestational hypertension at 30 weeks of gestation. She has an ultrasound scan that
ne

ne

ne

shows a normal growth, amniotic fluid volume and umbilical artery Doppler.
e.

e.

e.

How often should the scan be repeated, assuming the growth remains normal and
stable?
re

re

fre
sf

f
ks

ks
k
oo

oo

oo
eb

eb

eb
m

m
t

et

t
ne

ne
n
e.

e.

e.
t

t
Antenatal care 65

ne

ne

ne
e.

e.

e.
re

fre

fre
Options for questions 149–153

f
ks

ks

ks
A 10 + 0 weeks
oo

oo

oo
B 12 + 0 weeks
C 14 + 0 weeks
eb

eb

eb
D 16 + 0 weeks
m

m
E 18 + 0 weeks
F 20 + 0 weeks
G 24 + 0 weeks
t

t
e

ne

ne
H 28 + 0 weeks
.n

32 + 0 weeks

e.

e.
I
e

34 + 0 weeks
re

fre

re
J
K 35 + 0 weeks
sf

sf
ks
36 + 0 weeks
k

k
L
oo

oo

oo
M 37 + 0 weeks
38 + 0 weeks
eb

eb

eb
N
O 39 + 0 weeks
m

m
For each of the following clinical scenarios pertaining to multiple pregnancy, choose
the single most appropriate gestational age from the list above. Each option may be used
t

et
ne

ne

more than once, more than once or not at all.

n
e.

e.

e.
149. A 24-year-old woman is referred to the antenatal clinic. She is uncertain of her
fre

fre

re
last menstrual period and is thought to be in the second trimester. An ultrasound
scan shows a twin pregnancy. Ideally, by what gestational age should chorionicity
f
ks

ks

ks
have been determined?
oo

oo

oo

150. A 34-year-old woman is found in the first trimester to have an MCDA twin
eb

eb

eb

pregnancy. From what gestational age should serial ultrasound scans


commence?
m

151. A 30-year-old woman is found in the first trimester to have a DCDA twin
pregnancy. From what gestational age should serial assessment of fetal weight
t

t
commence?
ne

ne

ne

152. A 25-year-old woman is seen in the antenatal clinic with a DCDA twin pregnancy.
e.

e.

e.

It has so far been uncomplicated. From what gestational age should delivery be
re

re

fre

offered if it remains uncomplicated?


sf

f
ks

ks

153. A 38-year-old woman is known to have an uncomplicated triplet pregnancy


k
oo

oo

oo

following IVF. From what gestational age should delivery be offered?


eb

eb

eb
m

m
t

et

t
ne

ne
n
e.

e.

e.
t

t
66 Module 8

ne

ne

ne
e.

e.

e.
re

fre

fre
Options for questions 154–157

f
ks

ks

ks
A Antenatal booking blood tests including hepatitis C screen
oo

oo

oo
B Antenatal care by consultant obstetrician
C Clitoral reconstruction
eb

eb

eb
D Immediate deinfibulation procedure
m

m
E Immediate risk assessment about child safeguarding
F Inform the police or social services
G Intrapartum deinfibulation
t

t
e

ne

ne
H Midwife-led antenatal care
.n

e.

e.
I Refer to mental health services
e
re

fre

re
J Refer to safeguarding midwife
K Review in the presence of a professional interpreter
sf

sf
ks
k

k
L Thorough examination of the genitalia
oo

oo

oo
Each of the following clinical scenarios relates to a woman with FGM in pregnancy.
eb

eb

eb
For each patient, select the single most appropriate advice about the next step in manage-
m

m
ment from the list above. Each option may be used once, more than once or not at all.

154. A 22-year-old married British woman of Somali origin attends the consultant
antenatal clinic following her routine fetal anomaly scan. She gives a history of
t

et
ne

ne

having undergone FGM as a young girl at the age of 12 years and suffers from

n
lack of sensation during sexual intercourse. Her 5-year-old son was born via
e.

e.

e.
uncomplicated vaginal birth and lives with the woman and her husband.
fre

fre

155. A 19-year-old woman in her first pregnancy is a new arrival in the UK. She
f re
ks

ks

ks
attends the early pregnancy unit complaining of lower abdominal pain and
oo

oo

oo

vaginal spotting during the past week. There is no bleeding at present. Her home
pregnancy test was positive a few weeks ago. She has not yet registered with a GP
eb

eb

eb

and does not have a community midwife. A female friend is accompanying her
m

and reveals that the woman had undergone a procedure suggestive of FGM in her
early childhood in Africa.

156. A 29-year-old woman in her first pregnancy at 40 weeks of gestation attends the
t

t
ne

ne

ne

obstetric day assessment unit following a fall onto her abdomen. She has been
booked in another hospital and is currently a visitor on holiday in the locality. She
e.

e.

e.

gives a history of FGM for which she was to undergo a deinfibulation procedure
re

re

fre

at 38 weeks of gestation in the hospital where she is booked but was unable to
sf

attend. She is not keen on undergoing an examination of her genitalia and


ks

ks
k

wishes to deliver in her own hospital. An abdominal examination and CTG


oo

oo

oo

are normal.
eb

eb

eb

157. An obstetric registrar is asked to review a woman in the hospital accident and
emergency department with a history of amenorrhoea for 2 months complaining
m

of acute-­onset lower abdominal pain. On questioning, she reveals that she was
forced to undergo FGM a week ago. Examination reveals a remarkably distended
bladder and evidence of freshly healing vulval wounds.
t

et

t
ne

ne
n
e.

e.

e.
t

t
Antenatal care 67

ne

ne

ne
e.

e.

e.
re

fre

fre
Options for questions 158–160

f
ks

ks

ks
A Combined spinal epidural analgesia
oo

oo

oo
B Inpatient treatment with oral labetalol
C Intravenous diazepam
eb

eb

eb
D Intravenous hydralazine
m

m
E Intravenous labetalol infusion
F Intravenous magnesium sulfate 2–4 g
G Intravenous magnesium sulfate 2 g loading dose, followed by infusion of 2 g/hour for 24 hours
t

t
e

ne

ne
H Intravenous magnesium sulfate 4 g loading dose, followed by infusion of 1 g/hour for 24 hours
.n

e.

e.
I Intravenous phenytoin
e
re

fre

re
J Oral nifedipine
K Preload with intravenous 500 ml crystalloid, followed by epidural analgesia
sf

sf
ks
k

k
L Preload with intravenous 500 ml crystalloid, followed by intravenous hydralazine
oo

oo

oo
Each of the following clinical scenarios relates to severe hypertension in pregnancy.
eb

eb

eb
For each patient, select the single most appropriate option of management from the list
m

m
above. Each option may be used once, more than once or not at all.

158. A 32-year-old woman at 37 weeks of gestation has had an eclamptic seizure and is
currently receiving treatment with intravenous labetalol. A magnesium
t

et
ne

ne

sulfate infusion has been running for the last 6 hours. Her blood pressure is

n
150/100 mmHg, urine output is 200 ml in the last 4 hours and her blood test
e.

e.

e.
results are within normal limits. She now has a second eclamptic seizure.
fre

fre

159. A 42-year-old woman has been admitted at 33 weeks of gestation with vomiting
f re
ks

ks

ks
and is subsequently found to have severe pre-­eclampsia. She is a known asthmatic
oo

oo

oo

on treatment with inhaled salbutamol and steroids. Her blood pressure is


170/110 mmHg and urine output is 140 ml in the last 4 hours. Her platelet count
eb

eb

eb

is 100 × 109/l. She appears well and her deep tendon reflexes are normal.
m

160. A 24-year-old woman at 32 weeks of gestation attends the midwife-­led antenatal


clinic and is found to have a blood pressure of 156/106 mmHg. Urine testing with
an automated reagent-­strip reader shows proteinuria of 2+ and a spot urinary
t

t
ne

ne

ne

protein : creatinine ratio of 30 mg/mmol.


e.

e.

e.
re

re

fre
sf

f
ks

ks
k
oo

oo

oo
eb

eb

eb
m

m
t

et

t
ne

ne
n
e.

e.

e.
t

t
68 Module 8

ne

ne

ne
e.

e.

e.
re

fre

fre
Options for questions 161–163

f
ks

ks

ks
A Abdominal examination
oo

oo

oo
B Abdominal ultrasound scan
C Bimanual vaginal examination
eb

eb

eb
D CT scan
m

m
E Imaging at 28 weeks of gestation
F Imaging at 32 weeks of gestation
G Imaging at 36 weeks of gestation
t

t
e

ne

ne
H MRI scan
.n

e.

e.
I Positron emission tomography (PET) scan
e
re

fre

re
J Speculum examination
K Transvaginal ultrasound scan
sf

sf
ks
k

k
L Ultrasound scan with colour Doppler
oo

oo

oo
M Ultrasound scan with power Doppler
eb

eb

eb
For each of the following clinical scenarios, select the single most appropriate
m

m
management option from the list above. Each option may be used once, more than once
or not at all.

161. A 32-year-old woman has her detailed anatomy scan at 20 weeks of gestation. The
t

et
ne

ne

placenta is situated posteriorly, and it is suspected that the placental edge covers

n
the internal os of the cervix.
e.

e.

e.
fre

fre

re
162. A 32-year-old woman with a BMI of 35 kg/m2 who has had two previous
f
caesarean sections is found to have an anterior placenta at her 20-­week scan and
ks

ks

ks
further imaging is arranged at 32 weeks of gestation. The findings are similar but
oo

oo

oo

the quality of the ultrasound image is poor.


eb

eb

eb

163. A 28-year-old woman in her first pregnancy has a detailed scan at 20 weeks of
gestation. This suggests that the placental edge is 1 cm from the internal cervical
m

os. These findings are confirmed by a transvaginal scan. The pregnancy is


otherwise uncomplicated.
t

t
ne

ne

ne
e.

e.

e.
re

re

fre
sf

f
ks

ks
k
oo

oo

oo
eb

eb

eb
m

m
t

et

t
ne

ne
n
e.

e.

e.
t

t
Antenatal care 69

ne

ne

ne
e.

e.

e.
re

fre

fre
Options for questions 164–166

f
ks

ks

ks
A Chlamydia trachomatis
oo

oo

oo
B Cytomegalovirus
C Group A Streptococcus
eb

eb

eb
D Group B Streptococcus
m

m
E Hepatitis B virus
F Hepatitis C virus
G Herpes simplex virus type 1
t

t
e

ne

ne
H Herpes simplex virus type 2
.n

e.

e.
I Human immunodeficiency virus (HIV)
e
re

fre

re
J Human parvovirus B19
K Listeria monocytogenes
sf

sf
ks
k

k
L Neisseria gonorrhoeae
oo

oo

oo
M Rubella virus
eb

eb

eb
N Toxoplasma gondii
O Treponema pallidum
m

m
P Varicella-zoster virus
Q Zika virus
t

et
ne

ne

For each of the following clinical scenarios, select the organism that is the most likely

n
cause of infection from the list above. Each option may be used once, more than once or
e.

e.

e.
not at all.
fre

fre

f
164. A woman experiences low-­grade fever in the first trimester of pregnancy but
re
ks

ks

ks
does not seek medical advice. Later during her antenatal care, the fetus is noted
oo

oo

oo

to be SGA and is delivered at 38 weeks of gestation following induction of


labour. At birth, the infant is noted to be jaundiced with a petechial rash. There
eb

eb

eb

is hepatosplenomegaly and microcephaly.


m

165. A woman who is an asylum seeker in the UK first presents for antenatal care at
36 weeks of gestation. Although it is difficult to determine the precise gestational
age, the fetus appears to be small for dates. The woman delivers vaginally at
t

t
ne

ne

ne

39 weeks of gestation. The fetus has congenital cataracts and microphthalmia.


An echocardiogram shows pulmonary artery stenosis with a patent ductus
e.

e.

e.

arteriosus.
re

re

fre

166. A British couple visit Cuba for their honeymoon and shortly afterwards the
sf

f
ks

ks

woman finds she is pregnant. For religious beliefs, the couple decline all screening
k
oo

oo

oo

tests. The woman delivers at term. The baby is found to have microcephaly and
ventriculomegaly.
eb

eb

eb
m

m
t

et

t
ne

ne
n
e.

e.

e.
t

t
70 Module 8

ne

ne

ne
e.

e.

e.
re

fre

fre
Options for questions 167–171

f
ks

ks

ks
A Autosomal dominant
oo

oo

oo
B Autosomal recessive
C Balanced translocation
eb

eb

eb
D Maternal non-disjunction
m

m
E Mitochondrial inheritance
F Mosaicism
G Paternal non-disjunction
t

t
e

ne

ne
H Robertsonian translocation
.n

e.

e.
I Sporadic mutation
e
re

fre

re
J Triploidy
K Unbalanced translocation
sf

sf
ks
k

k
L X-linked dominant
oo

oo

oo
M X-linked recessive
eb

eb

eb
N Y-linked
m

m
For each of the following conditions, give the most common genetic aetiology from
the list above. Each option may be used once, more than once or not at all.
t

et
167. Down’s syndrome
ne

ne

n
168. Haemophilia A
e.

e.

e.
fre

fre

re
169. β-Thalassaemia
f
ks

ks

ks
170. Huntington’s disease
oo

oo

oo

171. Duchenne muscular dystrophy


eb

eb

eb
m

m
t

t
ne

ne

ne
e.

e.

e.
re

re

fre
sf

f
ks

ks
k
oo

oo

oo
eb

eb

eb
m

m
t

et

t
ne

ne
n
e.

e.

e.
t

t
Antenatal care 71

ne

ne

ne
e.

e.

e.
re

fre

fre
Options for questions 172–175

f
ks

ks

ks
A 1
oo

oo

oo
B 2
C 3
eb

eb

eb
D 4
m

m
E 5
F 6
G 7
t

t
e

ne

ne
H 8
.n

e.

e.
I 10
e
re

fre

re
J 20
K 25
sf

sf
ks
k

k
L 30
oo

oo

oo
M 65
eb

eb

eb
N 70
O 100
m

m
P 105
Q 110
R 120
t

et
ne

ne

n
e.

e.

e.
For the following patients with antenatal haematological problems, choose the cor-
rect value from the list above to answer the question. The required unit is indicated in the
fre

fre

re
question. Each option may be used once, more than once or not at all.
f
ks

ks

ks
172. A 21-year-old woman is readmitted with an antepartum haemorrhage with a
oo

oo

oo

low placenta. She had a recent blood sample sent to the laboratory for group and
screen during her last admission. Within how many days should this sample have
eb

eb

eb

been sent to be used for the provision of blood?


m

173. A 22-year-old woman has been found to be anaemic at 28 weeks of gestation.


Below what threshold of haemoglobin (in g/l) should supplementation with oral
iron be commenced?
t

t
ne

ne

ne

174. A 38-year-old woman with β-­thalassaemia trait has a haemoglobin level of 95 g/l.
e.

e.

e.

Haematinic studies are requested on a sample of her blood. Below what threshold
re

re

fre

of ferritin (in μg/l) should iron supplementation be commenced?


sf

f
ks

ks

175. A 28-year-old woman is seen in the antenatal clinic. Her booking blood tests show
k

a normal haemoglobin, but she is found to be iron deficient with a ferritin level of
oo

oo

oo

5 μg /l. What daily dose (in mg) of elemental iron should be prescribed?
eb

eb

eb
m

m
t

et

t
ne

ne
n
e.

e.

e.
t

t
72 Module 8

ne

ne

ne
e.

e.

e.
re

fre

fre
Answers
f
ks

ks

ks
SBAs
oo

oo

oo
96. Answer  C  3 months
eb

eb

eb
Explanation
Conception during tamoxifen therapy should be avoided because of potential
m

m
teratogenicity, and a ‘washout period’ of 2–3 months is advised.
Reference
t

t
RCOG. Pregnancy and breast cancer. RCOG GTG No. 12. March 2011.
e

ne

ne
.n

e.

e.
97. Answer  D  Moderate aerobic exercise for 15 minutes per day three times per week
e
re

fre

re
Explanation
sf

sf
Women who have not exercised routinely should begin with 15 minutes of
ks
k

k
continuous exercise three times weekly, increasing to daily 30-­minute sessions.
oo

oo

oo
Reference
eb

eb

eb
Kuhrt K, Hezelgrave NL, Shennan AH. Exercise in pregnancy. The Obstetrician &
Gynaecologist 2015;17:281–7.
m

98. Answer  E  Head circumference


m
t

et
Explanation
ne

ne

Crown–rump length measurement should be used to determine gestational age.

n
e.

e.

e.
If the crown–rump length is >84 mm, the gestational age should be estimated
using the head circumference.
fre

fre

Reference f re
ks

ks

ks
NICE. Antenatal care for uncomplicated pregnancies. NICE Clinical Guideline (CG62).
oo

oo

oo

Updated January 2017.


eb

eb

eb

99. Answer  B  800 units


m

Explanation
According to NICE guidelines, this patient has two moderate and one high r­ isk
factors for pre-­eclampsia and therefore requires 800 units of vitamin D with
t

t
ne

ne

ne

calcium supplementation. See Table 1 in the reference article.


e.

e.

e.

Reference
re

re

fre

RCOG. Vitamin D in pregnancy. RCOG Scientific Impact Paper No. 43. June 2014.
sf

f
ks

ks

100. Answer  A  6 mm
k
oo

oo

oo

Explanation
eb

eb

eb

The presence of an increased nuchal fold (≥6 mm) or two or more soft markers on
the routine anomaly scan should prompt the offer of a referral to a fetal medicine
m

specialist or an appropriate healthcare professional with a special interest in fetal


medicine.
t

et

t
ne

ne
n
e.

e.

e.
t

t
Antenatal care 73

ne

ne

ne
e.

e.

e.
re

fre

fre
Reference
f NICE. Antenatal care for uncomplicated pregnancies. NICE Clinical Guideline (CG62).
ks

ks

ks
Updated January 2017.
oo

oo

oo
101. Answer  E  Only when requested by the mother
eb

eb

eb
Explanation
m

m
Auscultation of the fetal heart may confirm that the fetus is alive but is unlikely
to have any predictive value, and routine listening is therefore not recommended.
However, when requested by the mother, auscultation of the fetal heart may
t

t
provide reassurance.
e

ne

ne
.n

Reference

e.

e.
e

NICE. Antenatal care for uncomplicated pregnancies. NICE Clinical Guideline (CG62).
re

fre

re
Updated January 2017.
sf

sf
ks
k

k
102. Answer  D  Twice weekly CTG and amniotic fluid volume assessment
oo

oo

oo
Explanation
eb

eb

eb
From 42 weeks, women who decline induction of labour should be offered
increased antenatal monitoring consisting of at least twice weekly CTG and
m

m
ultrasound estimation of maximum amniotic pool depth.
Reference
t

et
ne

ne

NICE. Antenatal care for uncomplicated pregnancies. NICE Clinical Guideline (CG62).

n
Updated January 2017.
e.

e.

e.
fre

fre

re
103. Answer  D  Malpresentation at term
Explanation f
ks

ks

ks
See Figure 1 in the reference article.
oo

oo

oo

Reference
eb

eb

eb

Dawood R, Altanis E, Ribes-Pastor P, Ashworth F. Pregnancy and spinal cord injury. The
m

Obstetrician & Gynaecologist 2014;16:99–107.

104. Answer  B  Cognitive behavioural therapy (CBT)


t

t
Explanation
ne

ne

ne

For a woman with anxiety disorder in pregnancy or the postnatal period, offer
e.

e.

e.

a low-­intensity psychological intervention (e.g. facilitated self-­help) or a high-­


re

re

fre

intensity psychological intervention (e.g. CBT) as the initial treatment in line with
the recommendations set out in the NICE guideline for the specific mental health
sf

f
ks

ks

problem and be aware that:


k
oo

oo

oo

• Only high-­intensity psychological interventions are recommended for PTSD


• High-intensity psychological interventions are recommended for the initial
eb

eb

eb

treatment of social anxiety disorder.


m

Reference
NICE. Antenatal and postnatal mental health: clinical management and service guidance.
NICE Clinical Guideline (CG192). Updated August 2017.
t

et

t
ne

ne
n
e.

e.

e.
t

t
74 Module 8

ne

ne

ne
e.

e.

e.
re

fre

fre
105. Answer  C  Postpone the trip if possible

f
Explanation
ks

ks

ks
A health professional advising a prospective UK resident who is pregnant or
oo

oo

oo
thinking about becoming pregnant and who is intending to go to a malaria-­
endemic area should suggest that the woman considers not going or postponing
eb

eb

eb
her trip until she is no longer pregnant.
m

m
Reference
RCOG. Prevention of malaria in pregnancy. RCOG GTG No. 54a. April 2010.
t

t
e

ne

ne
106. Answer  A  Laser ablation of placental vessels
.n

e.

e.
Explanation
e
re

fre

re
TTTS presenting before 26 weeks of gestation should be treated by fetoscopic
laser ablation rather than amnioreduction or septostomy. There is evidence that
sf

sf
ks
the fetoscopic laser ablative method should be the Solomon technique.
k

k
oo

oo

oo
The conclusion of a Cochrane review was that endoscopic laser coagulation of
anastomotic vessels should continue to be considered in the treatment of all stages
eb

eb

eb
of TTTS to improve neurodevelopmental outcomes in the child. When compared
m

m
with amnioreduction, treatment with laser coagulation does not appear to
increase or reduce the overall risk of death (stillbirth, neonatal and postneonatal)
in this condition, but it appears to result in more children being alive without
neurological abnormality.
t

et
ne

ne

Amnioreduction can be retained as a treatment option for those situations

n
where the expertise in laser coagulation is not available, pending transfer to a unit
e.

e.

e.
where such treatment can be obtained or when the condition is diagnosed after 26
fre

fre

re
weeks of pregnancy. However, this may complicate future treatment if associated
with inadvertent septostomy. f
ks

ks

ks
oo

oo

oo

Reference
RCOG. Management of monochorionic twin pregnancy. RCOG GTG No. 51. November
eb

eb

eb

2016.
m

107. Answer  E  Postpone delivery and perform fetal MRI after 4 weeks
Explanation
t

t
Clinical management is complex and should be overseen by fetal medicine experts
ne

ne

ne

with the knowledge and experience to advise parents about the advantages and
e.

e.

e.

disadvantages of different approaches. Rapid delivery is usually unwise, unless at


re

re

fre

term, as fetal brain injury of the surviving twin occurs at the time of demise of
the co-­twin. Therefore, immediate delivery only adds prematurity to the possible
sf

f
ks

ks

hypotensive cerebral injury the surviving twin may have already sustained.
k
oo

oo

oo

Serious compromise of the surviving fetus may be anticipated and this should be
discussed with parents, including the significant risk of long-­term morbidity.
eb

eb

eb

A conservative management policy is often appropriate, with serial fetal brain


ultrasound imaging and a fetal cranial MRI scan planned, commonly 4 weeks
m

after the ‘sentinel event’. The appearances of intracranial neurological morbidity


on ultrasound are variable and may take up to 4 weeks to develop. Fetal MRI
t

et

t
ne

ne
n
e.

e.

e.
t

t
Antenatal care 75

ne

ne

ne
e.

e.

e.
re

fre

fre
provides earlier and more detailed information about brain lesions (haemorrhagic
or ischaemic) in the surviving fetus than ultrasound, and its use should be
f
ks

ks

ks
considered.
oo

oo

oo
Reference
eb

eb

eb
RCOG. Management of monochorionic twin pregnancy. RCOG GTG No. 51. November
2016.
m

m
108. Answer  C  Daily Dopplers and deliver if ductus venosus becomes abnormal
Explanation
t

t
e

ne

ne
In the preterm small-for-gestational-age (SGA) fetus with umbilical artery with
.n

absent or reversed end-­diastolic velocity (AREDV) detected prior to 32 weeks

e.

e.
e

of gestation, delivery is recommended when ductus venosus Doppler becomes


re

fre

re
abnormal or umbilical vein pulsations appear, provided the fetus is considered
sf

sf
viable and after completion of steroids. Even when venous Doppler is normal,
ks
k

k
delivery is recommended by 32 weeks of gestation and should be considered
oo

oo

oo
between 30 and 32 weeks of gestation.
Unlike conventional CTG, which has high intra- and interobserver variability,
eb

eb

eb
cCTG is objective and consistent. Normal ranges for cCTG parameters
m

m
throughout gestation are available. Fetal heart rate variation is the most useful
predictor of fetal well-­being in SGA fetuses; an STV of ≤3 ms (within 24 hours of
delivery) has been associated with a higher rate of metabolic acidaemia
t

et
(54.2% versus 10.5%) and early neonatal death (8.3% versus 0.5%).
ne

ne

While cCTG can be used to time delivery where ductus venosus measurement

n
e.

e.

e.
is not available, an STV of ≤3 ms is considered abnormal.
fre

fre

re
Reference
f
ks

ks

ks
RCOG. Small-for-gestational-age fetus, investigation and management. RCOG GTG No. 31.
March 2013.
oo

oo

oo

109. Answer  C  Deinfibulation at 20 weeks


eb

eb

eb

Explanation
m

For women with type 3 FGM, where adequate vaginal assessment in labour
is unlikely to be possible, deinfibulation should be recommended antenatally,
usually in the second trimester, typically at around 20 weeks of gestation.
t

t
ne

ne

ne

Antenatal deinfibulation as an elective procedure ensures that the procedure is


performed by an appropriately trained midwife or obstetrician. However, women
e.

e.

e.

may prefer deinfibulation during labour, as this is the usual practice in some
re

re

fre

countries where FGM is prevalent.


sf

f
ks

ks

Reference
k
oo

oo

oo

RCOG. Female genital mutilation and its management. RCOG GTG No. 53. July 2015.
eb

eb

eb

110. Answer  B Oligohydramnios


m

Explanation
A meta-­analysis of unexplained antepartum haemorrhage identified ten relevant
studies in the previous 38 years, with a limited number of cases; preterm delivery
t

et

(odds ratio (OR) 3.17, 95% CI 2.76–3.64), stillbirth (OR 2.09, 95% CI 1.43–3.06)
ne

ne
n
e.

e.

e.
t

t
76 Module 8

ne

ne

ne
e.

e.

e.
re

fre

fre
and fetal anomalies (OR 1.42, 95% CI 1.07–1.87) appeared to be increased in
frequency.
f
ks

ks

ks
An epidemiological study of women with unexplained antepartum
oo

oo

oo
haemorrhage demonstrated an increased risk of oligohydramnios (OR 6.2, 95% CI
3.1–12.7), prelabour rupture of membranes (OR 3.4, 95% CI 1.8–6.2), fetal growth
eb

eb

eb
restriction (OR 5.6, 95% CI 2.5–12.2), preterm labour and caesarean delivery
(OR 4.0, 95% CI 2.4–6.6).
m

m
Reference
RCOG. Antepartum haemorrhage. RCOG GTG No. 63. December 2011.
t

t
e

ne

ne
.n

111. Answer  E  Small-for-gestational-age (SGA) fetus

e.

e.
e

Explanation
re

fre

re
In one study, 339 women with a singleton delivery after bariatric surgery (84.4%
sf

sf
ks
gastric bypass) were matched to 1277 unexposed women (after adjusting for BMI,
k

k
parity, age, date of delivery and smoking). Infants in the first group had a shorter
oo

oo

oo
mean gestational age (274 versus 278 days), a higher risk of being SGA (adjusted
eb

eb

eb
OR (aOR) 2.29, 95% CI 1.32–3.96) and a lower mean birthweight (3312 versus
3585 g), but had a lower risk of being large for gestational age (aOR 0.31, 95% CI
m

m
0.15–0.65). When analysing data from women with a gastric bypass alone (n =
286), the risk of SGA was even higher (aOR 2.78, 95% CI 1.56–4.96). With respect
to other outcomes, in contrast to smaller studies, no statistically significant
t

et
differences were found between the groups regarding the risk of gestational
ne

ne

n
diabetes mellitus, pre-­eclampsia, labour induction, caesarean section, postpartum
e.

e.

e.
haemorrhage, Apgar score <7, admission to the neonatal intensive care unit or
fre

fre

re
perinatal death.
f
ks

ks

ks
Reference
RCOG. The role of bariatric surgery in improving reproductive health. RCOG Scientific
oo

oo

oo

Impact Paper No. 17. October 2015.


eb

eb

eb

112. Answer  D  1 in 300 pregnancies


m

Explanation
Risk factors for vasa praevia include placental anomalies such as a bilobed
placenta or succenturiate lobes where the fetal vessels run through the membranes
t

t
ne

ne

ne

joining the separate lobes together, a history of a low-­lying placenta in the second
trimester, multiple pregnancy and IVF, where the incidence of vasa praevia has
e.

e.

e.

been reported to be as high as 1 in 300.


re

re

fre

The reasons for this association are not clear, but disturbed orientation of the
sf

blastocyst at implantation, vanishing embryos and the increased frequency of


ks

ks
k

placental morphological variations in IVF pregnancies have all been postulated.


oo

oo

oo

Reference
eb

eb

eb

RCOG. Placenta praevia, placenta praevia accreta and vasa praevia: diagnosis and
m

management. RCOG GTG No. 27. January 2011.


t

et

t
ne

ne
n
e.

e.

e.
t

t
Antenatal care 77

ne

ne

ne
e.

e.

e.
re

fre

fre
113. Answer  B  Desquamation of palms and soles

f Explanation
ks

ks

ks
Desquamation of the palms and soles is the final (fifth) feature that would confirm
oo

oo

oo
the diagnosis. See Appendix 1 in the reference article.
eb

eb

eb
Reference
m

m
RCOG. Bacterial sepsis in pregnancy. RCOG GTG No. 64a. April 2012.

114. Answer  C  Offer a membrane sweep at 40 weeks onwards


t

t
Explanation
e

ne

ne
.n

Prior to formal induction of labour, women should be offered a vaginal

e.

e.
e

examination for membrane sweeping at the 40- and 41-­week antenatal visits for
re

fre

re
nulliparous women and at the 41-­week antenatal visit for parous women.
When a vaginal examination is carried out to assess the cervix, the
sf

sf
ks
opportunity should be taken to offer the woman a membrane sweep. Additional
k

k
oo

oo

oo
membrane sweeping may be offered if labour does not start spontaneously.
eb

eb

eb
Reference
NICE. Induction of labour. NICE Clinical Guideline (CG70). July 2008.
m

115. Answer 
Explanation
D  15 weeks
m
t

et
ne

ne

Amniocentesis should be performed after 15 (15 + 0) weeks of gestation.

n
Amniocentesis before 14 (14 + 0) weeks of gestation (early amniocentesis) has
e.

e.

e.
a higher fetal loss rate and increased incidence of fetal talipes and respiratory
fre

fre

re
morbidity compared with other procedures.
f
ks

ks

ks
Reference
oo

oo

oo

RCOG. Amniocentesis and chorionic villus sampling. RCOG GTG No. 8. June 2010.
eb

eb

eb

116. Answer  B  Speculum examination of vagina


m

Explanation
In a woman reporting symptoms suggestive of PPROM, offer a speculum
examination to look for pooling of amniotic fluid. If pooling of amniotic fluid is
t

t
ne

ne

ne

observed, do not perform any diagnostic test but offer care consistent with the
woman having PPROM. If pooling of amniotic fluid is not observed, consider
e.

e.

e.

performing an IGFBP-­1 test or PAMG-­1 test of vaginal fluid.


re

re

fre

Do not use a nitrazine test for diagnosis of PPROM.


sf

f
ks

ks

Reference
k
oo

oo

oo

NICE. Preterm labour and birth. NICE Clinical Guideline (CG25). November 2015.
eb

eb

eb

117. Answer  B  Increased observations for 12 hours


m

Explanation
Term babies who are clinically well at birth and whose mothers have received
intrapartum antibiotic prophylaxis (IAP) for prevention of early-­onset GBS
t

et

disease >4 hours before delivery do not require special observation.


ne

ne
n
e.

e.

e.
t

t
78 Module 8

ne

ne

ne
e.

e.

e.
re

fre

fre
How should well babies at risk of early-­onset GBS disease whose mothers have
not received adequate IAP be monitored? Well babies should be evaluated at birth
f
ks

ks

ks
for clinical indicators of neonatal infection and have their vital signs checked at 0,
oo

oo

oo
1 and 2 hours, and then 2 hourly until 12 hours.
Postnatal antibiotic prophylaxis is not recommended for asymptomatic term
eb

eb

eb
infants without known antenatal risk factors.
m

m
Reference
RCOG. Early-onset neonatal group B streptococcal disease. RCOG GTG No. 36. September
2017.
t

t
e

ne

ne
118. Answer  E  160 ml one to two times per week
.n

e.

e.
e

Explanation
re

fre

re
If women choose to drink alcohol during pregnancy, they should be advised
sf

sf
to drink no more than 1–2 UK units once or twice a week (1 unit equals half a
ks
k

k
pint of ordinary-­strength lager or beer, or one shot (25 ml) of spirits. One small
oo

oo

oo
(125 ml) glass of wine is equal to 1.5 UK units). Although there is uncertainty
regarding a safe level of alcohol consumption in pregnancy, at this low level there
eb

eb

eb
is no evidence of harm to the unborn baby.
m

m
Two units of normal-­strength wine would equate to approximately 160 ml.
Reference
NICE. Antenatal care for uncomplicated pregnancies. NICE Clinical Guideline (CG62).
t

et
ne

ne

March 2008.

n
e.

e.

e.
119. Answer  E 80%
fre

fre

re
Explanation
f
ks

ks

ks
Sensitivity is the ability of an assay under evaluation to identify correctly true
positive (reference assay positive) samples. Therefore, sensitivity is the number
oo

oo

oo

of true positive samples (A) correctly identified by the assay under evaluation
eb

eb

eb

divided by the total number of true positive samples (i.e. those positive by the
reference assays = A + C), expressed as a percentage, as indicated in the table.
m

It is expressed as: sensitivity = A/(A + C).

Down’s syndrome Down’s syndrome Total


t

t
present absent
ne

ne

ne

Test positive A=8 B=4 12


e.

e.

e.

(true positive) (false positive)


re

re

fre

Test negative C=2 D = 86 88


(false negative) (true negative)
sf

f
ks

ks

Total 10 90 100
k

A+B+C+D=N
oo

oo

oo

(total number of tests


in study)
eb

eb

eb
m

Reference
Public Health England. UK Standards for Microbiology Investigations: evaluations,
validations and verifications of diagnostic tests. Public Health England Quality Guidance
t

et

1, Issue 5. June 2014.


ne

ne
n
e.

e.

e.
t

t
Antenatal care 79

ne

ne

ne
e.

e.

e.
re

fre

fre
120. Answer  E  Angiotensin-­converting enzyme (ACE) inhibitors, angiotensin-­
receptor blockers (ARBs) and thiazides
f
ks

ks

ks
Explanation
oo

oo

oo
Tell women who take antihypertensive treatments other than ACE inhibitors,
ARBs or chlorothiazide that the limited evidence available has not shown an
eb

eb

eb
increased risk of congenital malformation with such treatments.
m

m
Reference
NICE. Hypertension in pregnancy: diagnosis and management. NICE Clinical Guideline
t

t
e

ne

ne
(CG107). Updated January 2011.
.n

e.

e.
e

121. Answer  B  125 IU anti-­D Ig/ml of fetal red blood cells


re

fre

re
Explanation
sf

sf
ks
A dose of 500 IU anti-­D Ig intramuscularly is considered sufficient to treat a
k

k
fetomaternal haemorrhage of up to 4 ml of fetal red blood cells. Where it is
oo

oo

oo
necessary to give additional doses of anti-­D Ig, as guided by tests for fetomaternal
eb

eb

eb
haemorrhage, the dose calculation is traditionally based on 125 IU anti-­D Ig/ml
of fetal red blood cells for intramuscular administration. However, healthcare
m

m
professionals should refer to the manufacturer’s guidance depending on which
product is used.
t

et
ne

ne

Reference

n
Qureshi H, Massey E, Kirwan D, et al. BCSH guideline for the use of anti-­D
e.

e.

e.
immunoglobulin for the prevention of haemolytic disease of the fetus and newborn.
fre

fre

re
Transfusion Medicine 2014;24:8–20.
f
ks

ks

ks
122. Answer  B  Maternal blood test for fetal genotyping for K antigen after 20
oo

oo

oo

weeks of gestation
eb

eb

eb

Explanation
Non-invasive fetal genotyping using maternal blood is now possible for D,
m

C, c, E, e and K antigens. This should be performed in the first instance for


the relevant antigen when maternal red blood cell antibodies are present.
Genotyping can be undertaken from 16 weeks of gestation for all except K, which
t

t
ne

ne

ne

can be undertaken from 20 weeks, due to the risk of a false-­negative result if


performed earlier in pregnancy. Although anti-­K titres do not correlate well with
e.

e.

e.

either the development or severity of fetal anaemia, titres should nevertheless be


re

re

fre

measured every 4 weeks up to 28 weeks of gestation and then every 2 weeks until
sf

delivery.
ks

ks
k

Reference
oo

oo

oo

RCOG. Management of red cell antibodies in pregnancy. RCOG GTG No. 65.
eb

eb

eb

May 2014.
m

m
t

et

t
ne

ne
n
e.

e.

e.
t

t
80 Module 8

ne

ne

ne
e.

e.

e.
re

fre

fre
123. Answer  B  Combined screening test

f
Explanation
ks

ks

ks
Women with monochorionic twins who wish to have aneuploidy screening should
oo

oo

oo
be offered nuchal translucency measurements in conjunction with first-­trimester
serum markers (combined screening test) at 11 + 0 weeks to 13 + 6 weeks of
eb

eb

eb
gestation (crown–rump length 45–84 mm).
m

m
In women with monochorionic twin pregnancies who ‘miss’ or who have
unsuccessful first-­trimester screening for aneuploidy, second-­trimester screening
by the quadruple test should be offered.
t

t
Early data with non-­invasive prenatal testing are encouraging, but results
e

ne

ne
.n

should be interpreted with caution until larger studies have been carried out.

e.

e.
e

Reference
re

fre

re
RCOG. Management of monochorionic twin pregnancy. RCOG GTG No. 51. November
sf

sf
ks
2016.
k

k
oo

oo

oo
124. Answer  A  <27 pg
eb

eb

eb
Explanation
Where prevalence of sickle-­cell disease is low (fetal prevalence ≤1.5 cases
m

m
in 10,000 pregnancies), all pregnant women should be offered screening for
haemoglobinopathies using the Family Origin Questionnaire.
If the Family Origin Questionnaire indicates a high risk of sickle-­
t

et
ne

ne

cell disorders, laboratory screening (preferably high-­performance liquid

n
chromatography) should be offered.
e.

e.

e.
If the mean corpuscular haemoglobin is <27 pg, laboratory screening
fre

fre

re
(preferably high-­performance liquid chromatography) should be offered.
f
ks

ks

ks
Reference
oo

oo

oo

NICE. Antenatal care for uncomplicated pregnancies. NICE Clinical Guideline (CG62).
March 2008.
eb

eb

eb

125. Answer  C  To implement postnatal intervention to decrease the risk of


m

mother-to-child transmission.
Explanation
t

t
Serological screening for hepatitis B virus should be offered to pregnant women so
ne

ne

ne

that effective postnatal intervention can be offered to infected women to decrease


e.

e.

e.

the risk of mother-to-child transmission.


re

re

fre

Reference
sf

f
ks

ks

NICE. Antenatal care for uncomplicated pregnancies. NICE Clinical Guideline (CG62).
k

March 2008.
oo

oo

oo

126. Answer  C 700 μg


eb

eb

eb

Explanation
m

The intake of vitamin A during pregnancy should be limited to the recommended


daily amount, which, in Europe, is 2310 IU, equivalent to 700 μg. As liver and
liver products contain variable and sometimes very high amounts of vitamin A
t

et

t
ne

ne
n
e.

e.

e.
t

t
Antenatal care 81

ne

ne

ne
e.

e.

e.
re

fre

fre
avoided in pregnancy. The consumption of liver and liver products by pregnant
women (and particularly the intake of >700 μg) is associated with an increase in
f
ks

ks

ks
the risk of certain congenital malformations.
oo

oo

oo
Reference
eb

eb

eb
NICE. Antenatal care for uncomplicated pregnancies. NICE Clinical Guideline (CG62).
March 2008.
m

m
127. Answer  E  Offer induction of labour immediately and IAP
Explanation
t

t
e

ne

ne
For women undergoing spontaneous rupture of membranes prior to elective
.n

caesarean section, IAP for GBS should be offered. The time interval between

e.

e.
e

pregnancies is predictive of recurrent GBS colonisation. Women with GBS


re

fre

re
colonisation with spontaneous rupture of membranes at term should be offered
sf

sf
immediate induction of labour.
ks
k

k
oo

oo

oo
Reference
RCOG. Early-onset neonatal group B streptococcal disease. RCOG GTG No. 36. September
eb

eb

eb
2017.
m

m
128. Answer  C 53%
Explanation
t

et
Babies born preterm (i.e. before 37 + 0 weeks of pregnancy) have high rates of
ne

ne

n
early, late and postneonatal mortality, with the risk of mortality being inversely
e.

e.

e.
proportional to gestational age at birth. Babies who survive have increased rates
fre

fre

re
of disability compared with babies who are not born preterm. Recent UK studies
comparing cohorts born in 1995 and 2006 have shown improved rates of survival
f
ks

ks

ks
(from 40% to 53%) for extreme preterm births (born between 22 and 26 weeks).
oo

oo

oo

Reference
eb

eb

eb

NICE. Preterm labour and birth. NICE Clinical Guideline (CG25). November 2015.
m

129. Answer  D  Neurodevelopmental disability


Explanation
The major long-­term consequence of prematurity is neurodevelopmental
t

t
ne

ne

ne

disability. This can range from severe motor abnormalities, such as cerebral palsy,
through to less severe cognitive abnormalities.
e.

e.

e.
re

re

fre

Reference
sf

NICE. Preterm labour and birth. NICE Clinical Guideline (CG25). November 2015.
ks

ks
k

130. Answer  B 10%


oo

oo

oo

Explanation
eb

eb

eb

The risk of preterm birth is also considerably higher in multiple pregnancies than
m

in singleton pregnancies, occurring in 50% of twin pregnancies (10% of twin


births take place before 32 weeks of gestation). The significantly higher preterm
delivery rates in twin and triplet pregnancies mean there is increased demand for
specialist neonatal resources.
t

et

t
ne

ne
n
e.

e.

e.
t

t
82 Module 8

ne

ne

ne
e.

e.

e.
re

fre

fre
Reference
f
NICE. Multiple pregnancy: antenatal care for twin and triplet pregnancies. NICE Clinical
ks

ks

ks
Guideline (CG129). September 2011.
oo

oo

oo
131. Answer  B  Offer IAP only
eb

eb

eb
Explanation
m

m
Clinicians should offer IAP to women with GBS bacteriuria identified during the
current pregnancy.
Women with a GBS urinary tract infection (growth of >105 colony-­forming
units/ml) during pregnancy should receive appropriate treatment at the time of
t

t
e

ne

ne
diagnosis, as well as IAP.
.n

e.

e.
e

Reference
re

fre

re
RCOG. Prevention of early-­onset neonatal group B streptococcal disease. RCOG GTG
sf

sf
No. 36. September 2017.
ks
k

k
132. Answer  B  10 weeks
oo

oo

oo
Explanation
eb

eb

eb
Cell-free fetal DNA is present in reliably measurable levels for aneuploidy
m

m
screening from 10 weeks of gestation.
Reference
Mackie FL, Allen S, Morris RK, Kilby MD. Cell-free fetal DNA-­based noninvasive prenatal
t

et
ne

ne

testing of aneuploidy. The Obstetrician & Gynaecologist 2017;19:211–8.

n
e.

e.

e.
133. Answer  B  Dermoid cyst
fre

fre

re
Explanation
Dermoid cysts are the most common adnexal cystic lesions diagnosed after 16 f
ks

ks

ks
weeks of gestation.
oo

oo

oo

Reference
eb

eb

eb

Alalade AO, Maraj H. Management of adnexal masses in pregnancy. The Obstetrician &


Gynaecologist 2017;19:317–25.
m

134. Answer  B 2%


Explanation
t

t
ne

ne

ne

These are classified as ground E abortions (risk that the child would be born
e.

e.

e.

‘seriously handicapped’). In England and Wales in 2015, 3213 abortions (2%) were
carried out under ground E.
re

re

fre
sf

Reference
ks

ks
k

Department of Health. Abortion Statistics, England and Wales: 2015. June 2016.
oo

oo

oo

135. Answer  E  Preterm labour, urinary tract infection and vaginal bleeding
eb

eb

eb

Explanation
m

After adjusting for confounding factors, studies have demonstrated an association


between partner abuse during pregnancy and vaginal bleeding, kidney infections
and preterm labour.
t

et

t
ne

ne
n
e.

e.

e.
t

t
Antenatal care 83

ne

ne

ne
e.

e.

e.
re

fre

fre
Reference
f Gottlieb AS. Domestic violence: a clinical guide for women’s health care providers. The
ks

ks

ks
Obstetrician & Gynaecologist 2012;14:197–202.
oo

oo

oo
EMQs
eb

eb

eb
136. Answer  G  Growth scan in 3–4 weeks
m

m
137. Answer  J  Growth, amniotic fluid volume and umbilical artery Doppler in 2
weeks
t

t
e

ne

ne
F  Ductus venosus Doppler
.n

138. Answer 

e.

e.
e
re

fre

re
139. Answer  C  Amniotic fluid volume and umbilical artery Doppler twice per
week
sf

sf
ks
k

k
Explanation
oo

oo

oo
See Appendix III in the reference article.
eb

eb

eb
Reference
m

m
RCOG. The investigation and management of the small-for-gestational-age fetus. RCOG
GTG No. 31. March 2013.

140. Answer  G  Full blood count (FBC), urea and electrolytes (U&E), liver function
t

et
ne

ne

test (LFT), coagulation test

n
e.

e.

e.
Explanation
fre

fre

re
Before anticoagulant therapy is commenced, blood should be taken for a FBC,
coagulation screen, U&E and LFT. Performing a thrombophilia screen prior to
f
ks

ks

ks
therapy is not recommended.
oo

oo

oo

141. Answer  M  Stop treatment and repeat scan in 3 days


eb

eb

eb

Explanation
m

If the ultrasound is negative and there is a low level of clinical suspicion,


anticoagulant treatment can be discontinued. If the ultrasound is negative
and a high level of clinical suspicion exists, anticoagulant treatment should be
t

t
discontinued but the ultrasound should be repeated on days 3 and 7.
ne

ne

ne
e.

e.

e.

142. Answer  P  Ventilation/perfusion (V/Q) scan


re

re

fre

Explanation
sf

In women with suspected pulmonary embolism (PE) without symptoms and signs
ks

ks
k

of a DVT, a V/Q lung scan or CTPA should be performed.


oo

oo

oo

It would be prudent to recommend that lung perfusion scans should be


considered the investigation of first choice for young women, especially if there is
eb

eb

eb

a family history of breast cancer or the patient has had a previous chest CT scan.
m

m
t

et

t
ne

ne
n
e.

e.

e.
t

t
84 Module 8

ne

ne

ne
e.

e.

e.
re

fre

fre
143. Answer  K  Start therapeutic dose LMWH

f
Explanation
ks

ks

ks
In women with suspected PE who also have symptoms and signs of DVT,
oo

oo

oo
a compression duplex ultrasound should be performed. If compression
ultrasonography confirms the presence of DVT, no further investigation is
eb

eb

eb
necessary and treatment for venous thromboembolism (VTE) should commence.
m

m
144. Answer  I  Refer back to midwife-­led care
Explanation
t

t
e

ne

ne
Alternative or repeat testing should be carried out where the V/Q scan or CTPA
.n

is normal but the clinical suspicion of PE remains. Anticoagulant treatment

e.

e.
e

should be continued until PE is definitively excluded. The report given with


re

fre

re
resolved symptoms can be considered normal and therefore treatment can be
sf

sf
discontinued.
ks
k

k
oo

oo

oo
Reference
RCOG. The acute management of thrombosis and embolism during pregnancy and the
eb

eb

eb
puerperium. RCOG GTG No. 37b. April 2015.
m

m
145. Answer  C Daily
Explanation
t

et
For severe gestational hypertension, test for proteinuria daily using an automated
ne

ne

reagent-­strip reading device or the urinary protein : creatinine ratio.

n
e.

e.

e.
J  Twice per week
fre

fre

re
146. Answer 
Explanation f
ks

ks

ks
For moderate hypertension, measure blood pressure at least twice a week.
oo

oo

oo

147. Answer  H  Three times per week


eb

eb

eb

Explanation
m

For moderate pre-­eclampsia, carry out blood tests and monitor using the
following tests three times per week: kidney function, electrolytes, FBC,
transaminases and bilirubin.
t

t
ne

ne

ne

148. Answer  D Fortnightly


e.

e.

e.

Explanation
re

re

fre

If conservative management of severe gestational hypertension or pre-­eclampsia is


sf

f
ks

ks

planned, carry out the following tests at diagnosis:


k
oo

oo

oo

• Ultrasound fetal growth and amniotic fluid volume assessment


• Umbilical artery Doppler velocimetry.
eb

eb

eb

If the results of all fetal monitoring are normal in women with severe
m

gestational hypertension or pre-­eclampsia, do not routinely repeat CTG more


than weekly.
t

et

t
ne

ne
n
e.

e.

e.
t

t
Antenatal care 85

ne

ne

ne
e.

e.

e.
re

fre

fre
In women with severe gestational hypertension or pre-­eclampsia, do not
routinely repeat ultrasound fetal growth and amniotic fluid volume assessment or
f
ks

ks

ks
umbilical artery Doppler velocimetry more than every 2 weeks.
oo

oo

oo
Reference
eb

eb

eb
NICE. Hypertension in pregnancy: diagnosis and management. NICE Clinical Guideline
(CG107). Updated January 2011.
m

m
149. Answer  C 14 + 0 weeks
Explanation
t

t
e

ne

ne
Offer women with twin and triplet pregnancies a first-­trimester ultrasound scan
.n

when the crown–rump length measures from 45 to 84 mm (at approximately

e.

e.
e

11 + 0 weeks to 13 + 6 weeks) to estimate gestational age, determine chorionicity


re

fre

re
and screen for Down’s syndrome (ideally, these should all be performed at the
sf

sf
same scan).
ks
k

k
oo

oo

oo
150. Answer  D 16 + 0 weeks
eb

eb

eb
Explanation
For MCDA twin pregnancies, combine appointments with scans when the crown–
m

m
rump length measures from 45 to 84 mm (at approximately 11 + 0 weeks to
13 + 6 weeks) and then at estimated gestations of 16, 18, 20, 22, 24, 28, 32
and 34 weeks.
t

et
ne

ne

n
151. Answer  F 20 + 0 weeks
e.

e.

e.
fre

fre

re
Explanation
For DCDA twin pregnancies, combine appointments with scans when the
f
ks

ks

ks
crown–rump length measures from 45 to 84 mm (at approximately 11 + 0 weeks
to 13 + 6 weeks) and then at estimated gestations of 20, 24, 28, 32 and 36 weeks.
oo

oo

oo

Estimate fetal weight discordance using two or more biometric parameters at each
eb

eb

eb

ultrasound scan from 20 weeks.


m

152. Answer  M 37 + 0 weeks


Explanation
t

t
Offer women with uncomplicated monochorionic twin pregnancies elective birth
ne

ne

ne

from 36 + 0 weeks, after a course of antenatal corticosteroids has been offered. For
e.

e.

e.

uncomplicated dichorionic twin pregnancies, offer elective birth from


37 + 0 weeks.
re

re

fre
sf

f
ks

ks

153. Answer  K 35 + 0 weeks


k
oo

oo

oo

Explanation
Offer women with uncomplicated triplet pregnancies elective birth from
eb

eb

eb

35 + 0 weeks, after a course of antenatal corticosteroids has been offered.


m

Reference
NICE. Multiple pregnancy: antenatal care for twin and triplet pregnancies. NICE
Clinical Guideline (CG129). September 2011.
t

et

t
ne

ne
n
e.

e.

e.
t

t
86 Module 8

ne

ne

ne
e.

e.

e.
re

fre

fre
154. Answer  H  Midwife-­led care

f
Explanation
ks

ks

ks
This woman had an uncomplicated vaginal birth despite having undergone FGM
oo

oo

oo
earlier in her childhood. This does not necessarily constitute an indication for
high-­risk consultant-­led care.
eb

eb

eb
If a baby girl is delivered, the safeguarding assessment tool should be filled out
m

m
and the maternal history of FGM recorded in the child’s personal health record
(‘red book’).
t

t
155. Answer  L  Thorough examination of the genitalia
e

ne

ne
.n

Explanation

e.

e.
e

Any history suggestive of FGM with a history of bleeding in an unbooked woman


re

fre

re
in her first pregnancy should be an indication to examine her genitalia. Her
sf

sf
attendance at the early pregnancy unit should be seen as an opportunity to
ks
k

k
do that.
oo

oo

oo
156. Answer  G  Intrapartum deinfibulation
eb

eb

eb
Explanation
m

m
It is reasonable to carry out deinfibulation in labour. Deinfibulation may be
performed antenatally, in the first stage of labour or at the time of delivery, and
can usually be performed under local anaesthetic in a delivery suite room. It can
t

et
ne

ne

also be performed perioperatively after a caesarean section.

n
e.

e.

e.
157. Answer  D  Immediate deinfibulation procedure
fre

fre

re
Explanation
f
ks

ks

ks
This woman appears to have urinary retention secondary to a recent FGM
procedure. The priority is to deal with the acute issues. The police and social
oo

oo

oo

services must then be informed.


eb

eb

eb

References
m

RCOG. Female genital mutilation and its management. RCOG GTG No. 53. July 2015.
Hussain S, Rymer J. Tackling female genital mutilation in the UK. The Obstetrician &
Gynaecologist 2017;19:273–8.
t

t
ne

ne

ne

158. Answer  F  Intravenous magnesium sulfate 2–4 g


e.

e.

e.

Explanation
re

re

fre

Recurrent seizures should be treated with a further dose of 2–4 g given over
sf

5 minutes.
ks

ks

Do not use diazepam, phenytoin or a lytic cocktail as an alternative to


k
oo

oo

oo

magnesium sulfate in women with eclampsia.


eb

eb

eb
m

m
t

et

t
ne

ne
n
e.

e.

e.
t

t
Antenatal care 87

ne

ne

ne
e.

e.

e.
re

fre

fre
159. Answer  L  Preload with intravenous 500 ml crystalloid, followed by
intravenous hydralazine
f
ks

ks

ks
Explanation
oo

oo

oo
Hydralazine is a vasodilator, and its administration must be preceded by an
intravenous fluid preload. In this case, magnesium sulfate is not the priority, and
eb

eb

eb
in any case, the doses mentioned in the list of options are incorrect. Labetalol is
m

m
contraindicated, and even if indicated, the preferred method at this level of blood
pressure and at this gestational age would be intravenous administration. While
nifedipine is also an option in this scenario, the fact she has been vomiting means
t

t
it would not be the best option.
e

ne

ne
.n

e.

e.
160. Answer  B  Inpatient treatment with oral labetalol
e
re

fre

re
Explanation
sf

sf
This woman needs admission as she has pre-­eclampsia. After admission, a plan for
ks
k

k
management can be made. None of the other options is appropriate.
oo

oo

oo
Reference
eb

eb

eb
NICE. Hypertension in pregnancy: diagnosis and management. NICE Clinical Guideline
(CG107). Updated January 2011.
m

161. Answer  K  Transvaginal ultrasound scan


m
t

et
Explanation
ne

ne

Transvaginal scans improve the accuracy of placental localisation and are safe, so

n
e.

e.

e.
the suspected diagnosis of placenta praevia at 20 weeks of gestation by abdominal
fre

fre

re
scan should be confirmed by a transvaginal scan.

f
ks

ks

ks
162. Answer  H  MRI scan
oo

oo

oo

Explanation
Antenatal sonographic imaging can be complemented by MRI in equivocal cases
eb

eb

eb

to distinguish those women at special risk of placenta accreta.


m

163. Answer  F  Imaging at 32 weeks of gestation


Explanation
t

t
ne

ne

ne

If the placenta is thought to be low lying (less than 20 mm from the internal
os) or praevia (covering the os) at the routine fetal anomaly scan, a follow-up
e.

e.

e.

ultrasound examination including a TVS is recommended at 32 weeks of gestation


re

re

fre

to diagnose a persistent low-lying placenta and/or placenta praevia.


sf

f
ks

ks

Reference
k
oo

oo

oo

RCOG. Placenta praevia and placenta accreta: diagnosis and management. RCOG GTG
No. 27a. September 2018.
eb

eb

eb
m

m
t

et

t
ne

ne
n
e.

e.

e.
t

t
88 Module 8

ne

ne

ne
e.

e.

e.
re

fre

fre
164. Answer  B Cytomegalovirus

f
Explanation
ks

ks

ks
Cytomegalovirus, a member of the human herpesvirus family, is the most
oo

oo

oo
common viral cause of congenital infection, affecting 0.2–2.2% of all live
births.
eb

eb

eb
The clinical features of congenital cytomegalovirus at birth include jaundice,
m

m
petechial rash, hepatosplenomegaly, microcephaly and infants born SGA.
Overall, 13% of babies born with congenital cytomegalovirus infection will be
symptomatic at birth.
t

t
e

ne

ne
.n

165. Answer  M Rubella

e.

e.
e

Explanation
re

fre

re
Congenital rubella syndrome involves a wide spectrum of clinical features. In
sf

sf
ks
order of decreasing frequency, manifestations include: hearing loss, learning
k

k
disability, cardiac malformations and ocular defects.
oo

oo

oo
eb

eb

eb
166. Answer  Q  Zika virus
m

m
Explanation
Cuba is a high-­risk country for Zika virus. The most common congenital
anomalies resulting from Zika virus infection include microcephaly,
ventriculomegaly and cerebral/ocular calcifications, as well as fetal growth
t

et
ne

ne

restriction, oligohydramnios and talipes.

n
e.

e.

e.
References
fre

fre

re
RCOG. Congenital cytomegalovirus infection: update on treatment. RCOG Scientific
Impact Paper No. 56. November 2017. f
ks

ks

ks
RCOG/RCM/PHE/HPS. Zika virus infection and pregnancy. Interim RCOG/RCM/PHE/
oo

oo

oo

HPS Clinical Guidelines. Updated July 2017.


To M, Kidd M, Maxwell D. Prenatal diagnosis and management of fetal infections. The
eb

eb

eb

Obstetrician & Gynaecologist 2009;11:108–16.


m

167. Answer  D  Maternal non-dysjunction


Explanation
t

t
The majority (96%) of Down’s syndrome (trisomy 21) arises from non-­dysjunction
ne

ne

ne

in meiosis. This arises from the maternal cell line in 85% of cases and from the
e.

e.

e.

father in 15% of cases. Overall, 2–3% of cases arise from a parental balanced
re

re

fre

translocation involving chromosome 21 or as a result of a de novo translocation.


The final 1% are mosaics.
sf

f
ks

ks
k
oo

oo

oo

168. Answer  M  X-­linked recessive


eb

eb

eb

Explanation
Clinicians should be aware that haemophilia is an X-­linked condition associated
m

with the reduction or absence of clotting factor VIII (haemophilia A) or IX


(haemophilia B), causing bleeding symptoms.
t

et

t
ne

ne
n
e.

e.

e.
t

t
Antenatal care 89

ne

ne

ne
e.

e.

e.
re

fre

fre
169. Answer  B  Autosomal recessive

f Explanation
ks

ks

ks
In theory, genetic counselling for couples who are both carriers of α- or
oo

oo

oo
β-thalassaemia is relatively straightforward, in that both conditions are inherited
in a simple autosomal-­recessive fashion, and fetal testing by chorionic villus
eb

eb

eb
sampling, amniocentesis or fetal blood sampling is available.
m

m
170. Answer  A  Autosomal dominant
Explanation
t

t
e

ne

ne
Huntington’s disease is inherited as an autosomal-­dominant condition.
.n

e.

e.
171. Answer  M  X-­linked recessive
e
re

fre

re
Explanation
sf

sf
ks
Duchenne muscular dystrophy is inherited as an X-­linked recessive trait and,
k

k
in the absence of any other family history, the mother of an affected son has
oo

oo

oo
a two-­thirds risk of being a carrier.
eb

eb

eb
References
m

m
Eissa AA, Tuck SM. Sickle cell disease and β-­thalassaemia major in pregnancy. The
Obstetrician & Gynaecologist 2013;15:71–8.
RCOG. Management of inherited bleeding disorders in pregnancy. RCOG GTG No. 71.
April 2017.
t

et
ne

ne

Tobias ES, Connor JM. Medical Genetics for the MRCOG and Beyond. Cambridge:

n
Cambridge University Press, 2014.
e.

e.

e.
fre

fre

re
172. Answer  C  3 (days)
Explanation
f
ks

ks

ks
Group and screen samples used for provision of blood in pregnancy should be
oo

oo

oo

<3 days old.


eb

eb

eb

173. Answer  P  105 (g/l)


m

Explanation
Anaemia is defined by a haemoglobin level of <110 g/l in the first trimester,
<105 g/l in the second and third trimesters, and <100 g/l in the postpartum
t

t
period.
ne

ne

ne

174. Answer  L  30 (μg/l)


e.

e.

e.
re

re

fre

Explanation
sf

Women with known haemoglobinopathy should have serum ferritin checked and
ks

ks

be offered oral supplements if their ferritin level is <30 μg/l.


k
oo

oo

oo
eb

eb

eb
m

m
t

et

t
ne

ne
n
e.

e.

e.
t

t
90 Module 8

ne

ne

ne
e.

e.

e.
re

fre

fre
175. Answer  M  65 (mg)

f
Explanation
ks

ks

ks
Women with established iron-­deficiency anaemia should be given 100–200 mg
oo

oo

oo
of elemental iron daily. They should be advised on the correct administration to
optimise absorption.
eb

eb

eb
Non-anaemic women identified to be at increased risk of iron deficiency
m

m
should have their serum ferritin checked early in pregnancy and be offered oral
supplements if their ferritin is <30 μg/l.
Non-anaemic iron-­deficient women should be offered 65 mg of elemental iron
t

t
daily, with a repeat haemoglobin and serum ferritin test after 8 weeks.
e

ne

ne
.n

References

e.

e.
e

Pavord S, Myers B, Robinson S, et al. on behalf of the British Committee for Standards
re

fre

re
in Haematology. UK guidelines on the management of iron deficiency in pregnancy.
sf

sf
British Journal of Haematology 2012;156:588–600.
ks
k

k
RCOG. Blood transfusion in obstetrics. RCOG GTG No. 47. May 2015.
oo

oo

oo
eb

eb

eb
m

m
t

et
ne

ne

n
e.

e.

e.
fre

fre

f re
ks

ks

ks
oo

oo

oo
eb

eb

eb
m

m
t

t
ne

ne

ne
e.

e.

e.
re

re

fre
sf

f
ks

ks
k
oo

oo

oo
eb

eb

eb
m

m
t

et

t
ne

ne
n
e.

e.

e.
t

t
ne

ne

ne
e.

e.

e.
re

fre

fre
Maternal medicine
Module
f
ks

ks

ks
9
oo

oo

oo
eb

eb

eb
m

m
t

t
e

ne

ne
SBAs
.n

e.

e.
e

176. A 26-year-old woman is seen in the combined obstetric cardiology booking clinic
re

fre

re
at 14 weeks of gestation. She has a history of tetralogy of Fallot that was repaired
sf

sf
in childhood.
ks
k

k
What is the main cardiac issue that she potentially faces in pregnancy?
oo

oo

oo
A. Left ventricular dysfunction
eb

eb

eb
B. Paradoxical embolism
C. Profound cyanosis
m

m
D. Pulmonary hypertension
E. Right ventricular dysfunction
t

et
177. A 25-year-old woman is seen in the antenatal clinic at 7 weeks of gestation. She
ne

ne

is seen regularly in the congenital heart disease clinic because of Eisenmenger’s

n
e.

e.

e.
syndrome secondary to a ventriculoseptal defect.
What would be the most appropriate advice in terms of management?
fre

fre

A. Bed rest and home oxygen therapy


f re
ks

ks

ks
B. Commence oral labetalol
oo

oo

oo

C. Commence oral sildenafil


D. Commence thromboprophylaxis with low-molecular-weight heparin (LMWH)
eb

eb

eb

E. Termination of pregnancy
m

178. A 28-year-old woman who is currently 12 weeks pregnant has been referred to
a breast surgeon after finding a lump in the right breast. Following appropriate
imaging and cellular analysis, she is thought to have a stage 1 cancer (no evidence
t

t
ne

ne

ne

of spread) that is positive for the human epidermal growth factor receptor 2
(HER2) and oestrogen receptor. She declines termination of pregnancy.
e.

e.

e.

What first-­line treatment is most likely to be offered?


re

re

fre

A. Conventional anthracycline chemotherapy


sf

B. Local radiotherapy with fetal shielding


ks

ks
k

C. Primary surgery
oo

oo

oo

D. Tamoxifen
E. Trastuzumab
eb

eb

eb
m

91
t

et

t
ne

ne
n
e.

e.

e.
t

t
92 Module 9

ne

ne

ne
e.

e.

e.
re

fre

fre
179. A woman presents at 28 weeks of gestation with itching. On examination, she is
found to have urticarial-­type papules and plaques on her abdomen and thighs.
f
ks

ks

ks
The woman states that she first noticed these lesions in her umbilicus. She also has
oo

oo

oo
four 1 cm blisters on her inner thighs. Liver function tests (LFTs) are normal. She
is referred to a dermatologist.
eb

eb

eb
What is the most likely diagnosis?
m

m
A. Atopic eruption of pregnancy
B. Obstetric cholestasis
C. Pemphigoid gestationis
D. Polymorphic eruption of pregnancy
t

t
e

ne

ne
E. Psoriasis
.n

e.

e.
e

180. A 36-year-old woman is seen in the booking clinic. She has a history of
re

fre

re
inflammatory bowel disease and is taking sulfasalazine 4 g/day and folic acid
sf

sf
ks
5 mg/day.
k

k
What is the main fetal risk if she continues this treatment?
oo

oo

oo
A. Bloody diarrhoea in the newborn
eb

eb

eb
B. Exomphalos
C. Intrauterine death
m

m
D. Intrauterine growth restriction
E. Nephrotoxicity
t

et
ne

ne

181. A woman who has sickle-­cell disease is seen in the preconception clinic. Her

n
vaccination history is reviewed.
e.

e.

e.
How often should the pneumococcal vaccine be administered to ensure that she
fre

fre

re
is protected?
f
ks

ks

ks
A. Annually
B. Every 2 years
oo

oo

oo

C. Every 5 years
eb

eb

eb

D. Every 10 years
E. Once only
m

182. A woman who is 18 weeks pregnant attends a regional tropical diseases clinic
with fever, chills and malaise. She has just returned to the UK from working in
t

t
East Africa. A blood film is examined and a diagnosis of malaria is made. The
ne

ne

ne

organism is identified as Plasmodium vivax.


e.

e.

e.

What is the treatment of choice?


re

re

fre

A. Chloroquine
sf

B. Clindamycin
ks

ks
k

C. Mefloquine
oo

oo

oo

D. Primaquine
E. Quinine
eb

eb

eb
m

m
t

et

t
ne

ne
n
e.

e.

e.
t

t
Maternal medicine 93

ne

ne

ne
e.

e.

e.
re

fre

fre
183. A woman who has previously had genital herpes presents with preterm prelabour
rupture of membranes (PPROM) at 32 weeks of gestation. Clinical examination
f
ks

ks

ks
reveals that she has typical lesions of recurrent genital herpes. Assessments of fetal
oo

oo

oo
and maternal well-­being are otherwise normal.
What is the most appropriate management plan?
eb

eb

eb
Management of Use of antenatal Delivery
m

m
genital herpes corticosteroids
A Oral acyclovir Avoid Expectant management
t

t
B Oral acyclovir Usual course Expectant management
e

ne

ne
.n

C Oral acyclovir Avoid Expedite with caesarean section

e.

e.
e

D Intravenous acyclovir Usual course Expedite with caesarean section


re

fre

re
E Intravenous acyclovir Avoid Expedite with caesarean section
sf

sf
ks
k

k
184. Which two anti-­epileptic drugs, when used as low-­dose monotherapy, are
oo

oo

oo
associated with the lowest risk of congenital malformations?
eb

eb

eb
Carbamazepine Lamotrigine Phenytoin Pregabalin Sodium Topiramate
m

m
valproate
A ✓ ✓
B ✓ ✓
t

et
ne

ne

C ✓ ✓

n
e.

e.

e.
D ✓ ✓
fre

fre

re
E ✓ ✓

f
ks

ks

ks
185. A woman is screened within a clinical trial and is found to be positive for thyroid
oo

oo

oo

peroxidase antibodies but is euthyroid. She opts out of randomisation and


becomes pregnant shortly afterwards.
eb

eb

eb

By what factor is she at increased risk of preterm labour compared with women
m

who do not have autoimmune thyroid disease?


A. 1–1.5-fold
B. 2–4-fold
t

t
C. 6–8-fold
ne

ne

ne

D. 10–15-fold
e.

e.

e.

E. 20–30-fold
re

re

fre

186. A pregnant woman with type 2 diabetes is seeing the diabetes specialist nurse for
sf

f
ks

ks

advice about glucose monitoring in pregnancy.


k

What would be her target capillary plasma glucose level 2 hours after a meal?
oo

oo

oo

A. 5.3 mmol/l
eb

eb

eb

B. 5.6 mmol/l
C. 6.4 mmol/l
m

D. 7.2 mmol/l
E. 7.8 mmol/l
t

et

t
ne

ne
n
e.

e.

e.
t

t
94 Module 9

ne

ne

ne
e.

e.

e.
re

fre

fre
187. Which opiate should be avoided as analgesia in labour for pregnant women with
epilepsy?
f
ks

ks

ks
A. Codeine
oo

oo

oo
B. Diamorphine
C. Morphine
eb

eb

eb
D. Pethidine
m

m
E. Tramadol

188. A 26-year-old woman with epilepsy has just given birth. Her seizures are well
controlled with carbamazepine monotherapy. She wishes to have a discussion
t

t
e

ne

ne
about postnatal contraception.
.n

Which hormonal methods of contraception would be considered reliable in this

e.

e.
e

situation?
re

fre

re
sf

sf
ks
Combined Levonorgestrel- Medroxyprogesterone Progestogen- Progestogen
k

k
hormonal releasing acetate (MPA) only pill implant
oo

oo

oo
contraceptive intrauterine injection
eb

eb

eb
system
(LNG-IUS)
m

m
A ✓ ✓
B ✓ ✓
✓ ✓
t

et
C
ne

ne

✓ ✓

n
D
e.

e.

e.
E ✓ ✓
fre

fre

f re
ks

ks

ks
189. A 23-year-old woman with β-­thalassaemia major attends the antenatal clinic at 8
weeks of gestation. Cardiac function tests performed 12 weeks ago were normal.
oo

oo

oo

She has had a splenectomy.


eb

eb

eb

Her current blood test results are:


m

Haemoglobin 105 g/l


Platelets 650 × 109/l
Serum fructosamine 275 mmol/l
t

t
ne

ne

ne

What antenatal thromboprophylaxis would be recommended?


e.

e.

e.

A. Anti-embolic stockings
re

re

fre

B. Aspirin 75 mg daily
sf

C. Aspirin 75 mg daily plus LMWH


ks

ks

D. No thromboprophylaxis required
k
oo

oo

oo

E. Warfarin
eb

eb

eb
m

m
t

et

t
ne

ne
n
e.

e.

e.
t

t
Maternal medicine 95

ne

ne

ne
e.

e.

e.
re

fre

fre
190. A 32-year-old woman with a body mass index (BMI) of 35 kg/m2 has just
delivered vaginally at term. She is a known carrier of a prothrombin gene
f
ks

ks

ks
mutation.
oo

oo

oo
For how many days postnatally should she have thromboprophylaxis?
A. Until mobile
eb

eb

eb
B. 3 days
m

m
C. 7 days
D. 10 days
E. 6 weeks
t

t
e

ne

ne
191. A 33-year-old woman who takes lithium for bipolar disorder is admitted in labour
.n

at 39 weeks of gestation. Vaginal examination reveals a fully effaced and 2 cm

e.

e.
e

dilated cervix with intact membranes.


re

fre

re
How should her lithium administration be managed in labour?
sf

sf
ks
A. Continue treatment with lithium throughout labour
k

k
B. Stop lithium and check plasma levels every 2 hours
oo

oo

oo
C. Stop lithium and check plasma levels 12 hours after the last dose
eb

eb

eb
D. Stop lithium and restart after birth
E. Substitute lithium with a benzodiazepine during labour
m

m
192. What are the categories of body weight that require routine measurement of
peak anti-­Xa activity for women who are under treatment with LMWH for acute
t

et
ne

ne

venous thromboembolism (VTE) in pregnancy?

n
e.

e.

e.
Less than (kg) More than (kg)
fre

fre

re
A 45 85
f
ks

ks

ks
B 50 90
C 55 95
oo

oo

oo

D 60 100
eb

eb

eb

E 65 110
m

193. What is the incidence of post-­thrombotic syndrome (PTS) following a deep


venous thrombosis (DVT) in pregnancy?
t

t
ne

ne

ne

A. 32%
B. 42%
e.

e.

e.

C. 52%
re

re

fre

D. 62%
sf

E. 72%
ks

ks
k
oo

oo

oo
eb

eb

eb
m

m
t

et

t
ne

ne
n
e.

e.

e.
t

t
96 Module 9

ne

ne

ne
e.

e.

e.
re

fre

fre
194. A 35-year-old woman is diagnosed with intrahepatic cholestasis of pregnancy and
is commenced on therapy with ursodeoxycholic acid (UDCA). Unfortunately,
f
ks

ks

ks
there is no change in either symptoms or biochemical profile.
oo

oo

oo
Which second-­line drug should be considered?
A. Chlorpheniramine
eb

eb

eb
B. Cholestyramine
m

m
C. Dexamethasone
D. Rifampicin
E. Vitamin K
t

t
e

ne

ne
195. What proportion of pregnancies are affected by gestational hyperthyroidism?
.n

e.

e.
A. 0.1–0.3%
e

B. 0.5–0.7%
re

fre

re
C. 1–3%
sf

sf
ks
D. 5–7%
k

k
E. 10–30%
oo

oo

oo
eb

eb

eb
196. LFTs may be deranged in 40% of women with hyperemesis gravidarum.
Which component of LFTs is most frequently abnormal?
m

m
A. Alanine transaminase (ALT)
B. Alkaline phosphatase
C. Bile acids
t

et
ne

ne

D. Bilirubin

n
E. γ-Glutamyl transferase
e.

e.

e.
fre

fre

re
197. What is the most common cause of acute kidney injury in pregnancy?
f
ks

ks

ks
A. Haemorrhage
B. Hyperemesis gravidarum
oo

oo

oo

C. Non-steroidal anti-­inflammatory drugs (NSAIDs)


eb

eb

eb

D. Pre-eclampsia
E. Urinary retention
m

198. What proportion of cases of HELLP syndrome will result in an acute kidney
injury?
t

t
ne

ne

ne

A. 0.3–1.5%
B. 3–15%
e.

e.

e.

C. 23–35%
re

re

fre

D. 43–55%
sf

E. 63–75%
ks

ks
k
oo

oo

oo

199. A woman with type 1 diabetes presents at 11 weeks of gestation with diabetic
ketoacidosis following several days of vomiting.
eb

eb

eb

What is the intravenous fluid of choice for initial fluid replacement?


m

A. 5% albumin solution
B. 20% albumin solution
C. 5% dextrose
t

et

D. Hartmann’s solution
ne

ne
n
e.

e.

e.
t

t
Maternal medicine 97

ne

ne

ne
e.

e.

e.
re

fre

fre
200. A woman with type 1 diabetes presents with suspected diabetic ketoacidosis
(DKA) following a protracted period of vomiting.
f
ks

ks

ks
Initial investigations gave the following results:
oo

oo

oo
Serum bicarbonate 13 mmol/l
eb

eb

eb
Serum glucose 18 mmol/l
m

m
What would be the threshold value for serum ketones to confirm the diagnosis
of DKA?
t

t
A. 3 nmol/l
e

ne

ne
.n

B. 30 nmol/l

e.

e.
C. 300 nmol/l
e
re

fre

re
D. 3 mmol/l
E. 30 mmol/l
sf

sf
ks
k

k
oo

oo

oo
eb

eb

eb
m

m
t

et
ne

ne

n
e.

e.

e.
fre

fre

f re
ks

ks

ks
oo

oo

oo
eb

eb

eb
m

m
t

t
ne

ne

ne
e.

e.

e.
re

re

fre
sf

f
ks

ks
k
oo

oo

oo
eb

eb

eb
m

m
t

et

t
ne

ne
n
e.

e.

e.
t

t
98 Module 9

ne

ne

ne
e.

e.

e.
re

fre

fre
EMQs
f
ks

ks

ks
Options for questions 201–203
oo

oo

oo
A Magnesium sulfate infusion
eb

eb

eb
B No treatment required
C Start amlodipine
m

m
D Start atenolol
E Start bendroflumethiazide
t

t
F Start chlorothiazide
e

ne

ne
.n

G Start hydralazine

e.

e.
e

H Start labetalol
re

fre

re
I Start lisinopril
sf

sf
J Start losartan
ks
k

k
K Start low-­dose aspirin
oo

oo

oo
L Start low-molecular-weight heparin (LMWH)
eb

eb

eb
M Start vitamins C and E
m

m
N Stop all antihypertensive medication
O Switch to clopidogrel
P Switch to enalapril
t

et
ne

ne

n
For each of the following clinical scenarios, choose the single most appropriate phar-
e.

e.

e.
macological management from the list of options above. Each option may be chosen
fre

fre

re
once, more than once or not at all.
f
ks

ks

ks
201. A 39-year-old woman is seen on the postnatal ward just prior to discharge. She
had a forceps delivery 2 days earlier. She is planning to continue breastfeeding for
oo

oo

oo

at least 6 months. She suffers with chronic hypertension and was taking enalapril
eb

eb

eb

prior to pregnancy. Her medication was changed to methyldopa once she had a
positive pregnancy test and her blood pressure was stable during pregnancy.
m

202. A 42-year-old woman with chronic hypertension is seen in the booking clinic
at 12 weeks of gestation. She was taking losartan prior to pregnancy and was
t

t
switched to labetalol once she had a positive pregnancy test at 5 weeks of
ne

ne

ne

gestation.
e.

e.

e.

203. A woman is seen in the obstetric day assessment unit at 28 weeks of gestation. She
re

re

fre

is generally fit and well. She has been referred by her community midwife because
sf

her blood pressure is persistently 150/100 mmHg. She has no proteinuria.


ks

ks
k
oo

oo

oo
eb

eb

eb
m

m
t

et

t
ne

ne
n
e.

e.

e.
t

t
Maternal medicine 99

ne

ne

ne
e.

e.

e.
re

fre

fre
Options for questions 204–206

f
ks

ks

ks
A Avoidable death
oo

oo

oo
B Coincidental death
C Coroner’s death
eb

eb

eb
D Critical death
m

m
E Direct death
F Early death
G Fetal death
t

t
e

ne

ne
H Fortuitous death
.n

e.

e.
I Indirect death
e
re

fre

re
J Late death
K Unavoidable death
sf

sf
ks
k

k
oo

oo

oo
For each of the following clinical scenarios, choose the single most appropriate cat-
egory of death according to the classification by the World Health Organization from the
eb

eb

eb
list of options above. Each option may be used once, more than once or not at all.
m

m
204. Following an uneventful delivery where the labour was augmented with
oxytocin (Syntocinon), a woman suddenly collapses and has a cardiac arrest.
Despite appropriate resuscitation, it is not possible to revive her. A post mortem
t

et
ne

ne

examination records the cause of death as an amniotic fluid embolism.

n
e.

e.

e.
205. A 25-year-old woman is a poor attender at the antenatal clinic and is known to be
fre

fre

re
a victim of domestic violence. Four weeks after delivery, she is murdered by her
ex-partner.
f
ks

ks

ks
206. A 30-year-old woman with epilepsy that is well controlled with lamotrigine is seen
oo

oo

oo

regularly in the antenatal clinic. She fails to follow advice regarding showering
eb

eb

eb

rather than bathing. She suffers a seizure while in the bath and drowns.
m

m
t

t
ne

ne

ne
e.

e.

e.
re

re

fre
sf

f
ks

ks
k
oo

oo

oo
eb

eb

eb
m

m
t

et

t
ne

ne
n
e.

e.

e.
t

t
100 Module 9

ne

ne

ne
e.

e.

e.
re

fre

fre
Options for questions 207–209

f
ks

ks

ks
A Carbamazepine
oo

oo

oo
B Eslicarbazepine
C Gabapentin
eb

eb

eb
D Lamotrigine
m

m
E Levetiracetam
F Oxcarbazepine
G Phenobarbital
t

t
e

ne

ne
H Phenytoin
.n

e.

e.
I Pregabalin
e
re

fre

re
J Primidone
K Sodium valproate
sf

sf
ks
k

k
L Tiagabine
oo

oo

oo
M Topiramate
eb

eb

eb
N Vigabatrin
m

m
For each of the following clinical scenarios, choose the single most appropriate anti-­
epileptic drug from the list of options above. Each option may be used once, more than
once or not at all.
t

et
ne

ne

207. A woman taking anti-­epileptic medication attends for a routine fetal anomaly

n
e.

e.

e.
scan at 20 weeks of gestation. The fetus is found to have spina bifida and a cleft lip.
Which medication is she most likely to be taking?
fre

fre

f
208. A pregnant woman attends antenatal clinic and is taking a single anti-­epileptic
re
ks

ks

ks
drug. She has been informed that the drug she is taking has two main advantages:
oo

oo

oo

(1) it carries the lowest risk of congenital malformations; and (2) it does not
increase the risk of haemolytic disease of the newborn. Which drug is she most
eb

eb

eb

likely to be taking?
m

209. A woman with epilepsy has a seizure in labour. Benzodiazepines are administered,
but the seizures continue. Which second-­line therapy should now be
administered?
t

t
ne

ne

ne
e.

e.

e.
re

re

fre
sf

f
ks

ks
k
oo

oo

oo
eb

eb

eb
m

m
t

et

t
ne

ne
n
e.

e.

e.
t

t
Maternal medicine 101

ne

ne

ne
e.

e.

e.
re

fre

fre
Options for questions 210–212

f
ks

ks

ks
A Chlorpropamide
oo

oo

oo
B Glibenclamide
C Gliclazide
eb

eb

eb
D Insulin aspart
m

m
E Insulin detemir
F Insulin glargine
G Insulin lispro
t

t
e

ne

ne
H Isophane insulin (NPH insulin)
.n

e.

e.
I Metformin
e
re

fre

re
J No treatment required
K Phenformin
sf

sf
ks
k

k
L Pioglitazone
oo

oo

oo
M Rosiglitazone
eb

eb

eb
N Sitagliptin
O Tolbutamide
m

m
P Troglitazone

For each of the following clinical scenarios, choose the single most appropriate medi-
t

et
ne

ne

cation from the list of options above. Each option may be used once, more than once or

n
not at all.
e.

e.

e.
fre

fre

re
210. A 34-year-old woman is newly diagnosed with gestational diabetes at 24 weeks of
f
gestation. She has modified her diet and undertaken an exercise regime but her
ks

ks

ks
plasma glucose levels remain slightly elevated after 2 weeks of this new regime.
oo

oo

oo

Which medication is the most appropriate treatment?


eb

eb

eb

211. A woman with type 2 diabetes was taking sitagliptin prior to pregnancy as she
could not tolerate the gastrointestinal side effects of metformin. She was switched
m

to insulin therapy in the antenatal period. She has now delivered and wishes to
breastfeed. If oral hypoglycaemic agents are required, what treatment would be
recommended?
t

t
ne

ne

ne

212. A woman with a BMI of 32 kg/m2 and persistent glycosuria underwent a glucose
e.

e.

e.

tolerance test at 24 weeks of gestation.


re

re

fre

Her results are as follows:


sf

f
ks

ks

Fasting glucose 7.1 mmol/l


k
oo

oo

oo

2-Hour glucose 10.2 mmol/l


eb

eb

eb

What immediate treatment is recommended?


m

m
t

et

t
ne

ne
n
e.

e.

e.
t

t
102 Module 9

ne

ne

ne
e.

e.

e.
re

fre

fre
Options for questions 213–215

f
ks

ks

ks
A <50 human immunodeficiency virus (HIV) RNA copies/ml
oo

oo

oo
B 50–399 HIV RNA copies/ml
C ≥400 HIV RNA copies/ml
eb

eb

eb
D >1000 HIV RNA copies/ml
m

m
E >100,000 HIV RNA copies/ml
F CD4 cell count <350 cells/μl
G CD4 cell count ≥350 cells/μl
t

t
e

ne

ne
H CD4 cell count between 350 and 500 cells/μl
.n

e.

e.
I CD4 cell count <500 cells/μl
e
re

fre

re
J CD4 cell count >500 cells/μl
sf

sf
K Hepatitis B virus (HBV) DNA >2000 IU/ml
ks
k

k
oo

oo

oo
For each of the following clinical scenarios, choose the single most applicable test
eb

eb

eb
result from the list of options above. Each option may be used once, more than once or
not at all.
m

m
213. An HIV-­positive woman who presented late for antenatal care at 34 weeks of
gestation is now in labour. What would be the threshold parameter for offering
t

et
intrapartum intravenous zidovudine therapy?
ne

ne

n
214. A 27-year-old woman is reviewed in the antenatal clinic at 36 weeks in her
e.

e.

e.
first pregnancy. She has HIV and is on highly active antiretroviral treatment
fre

fre

re
(HAART). What is the threshold parameter for which delivery by elective
caesarean section would be recommended? f
ks

ks

ks
oo

oo

oo

215. An HIV-­positive woman presents with spontaneous rupture of membranes at


term. What is the threshold parameter to recommend induction of labour?
eb

eb

eb
m

m
t

t
ne

ne

ne
e.

e.

e.
re

re

fre
sf

f
ks

ks
k
oo

oo

oo
eb

eb

eb
m

m
t

et

t
ne

ne
n
e.

e.

e.
t

t
Maternal medicine 103

ne

ne

ne
e.

e.

e.
re

fre

fre
Options for questions 216–218

f
ks

ks

ks
A Antiphospholipid syndrome
oo

oo

oo
B Cerebral infarction
C Cerebral venous thrombosis
eb

eb

eb
D Drug or alcohol withdrawal
m

m
E Eclampsia
F Haemorrhagic stroke
G Hypocalcaemia
t

t
e

ne

ne
H Hypoglycaemia
.n

e.

e.
I Hyponatraemia
e
re

fre

re
J Idiopathic epilepsy
sf

sf
K Non-epileptic seizure disorder
ks
k

k
L Postdural puncture
oo

oo

oo
M Secondary epilepsy
eb

eb

eb
N Thrombotic thrombocytopenic purpura (TTP)
m

m
For each of the following clinical scenarios, choose the most likely cause of the con-
vulsion described from the list above. Each option may be used once, more than once or
not at all.
t

et
ne

ne

n
216. A 27-year-old woman in her first pregnancy is brought in by ambulance at 37
e.

e.

e.
weeks of gestation having had a seizure. She was visiting her sister and forgot
fre

fre

re
her handheld notes. Her blood pressure is 135/89 mmHg, with a temperature of
f
37.9°C. A urine dipstick shows 1+ protein and the urine is noted to be very dark.
ks

ks

ks
Her blood results are:
oo

oo

oo

Reticulocyte count High


eb

eb

eb

Platelets 62 × 109/l
m

LFTs Normal
Creatinine 102 μmol/l
Coagulation Normal
t

t
ne

ne

ne
e.

e.

e.

217. A 23-year-old woman in her first pregnancy is admitted with convulsions,


headache and vomiting at 36 weeks of gestation. Her blood pressure is 147/104
re

re

fre

mmHg and a urine dipstick shows 2+ proteinuria.


sf

f
ks

ks

Her blood results are:


k
oo

oo

oo

Haemoglobin 82 g/l
eb

eb

eb

Platelets 92 × 109/l
m

Alanine transaminase 97 IU/l


Creatinine 86 μmol/l
t

et

t
ne

ne
n
e.

e.

e.
t

t
104 Module 9

ne

ne

ne
e.

e.

e.
re

fre

fre
218. A 32-year-old woman in her first pregnancy is admitted after a seizure that lasted
8 minutes. Her antenatal booking notes describe a past history of seizures. On
f
ks

ks

ks
examination, she appears drowsy, with normal reflexes and her plantar reflexes are
oo

oo

oo
downgoing. It is somewhat difficult to open her eyes to check her pupils.
eb

eb

eb
Options for questions 219 and 220
m

m
A Biliary cystadenoma
B Cholangiocarcinoma
t

t
C Focal fatty change
e

ne

ne
D Focal nodular hyperplasia
.n

e.

e.
E Hepatic adenoma
e
re

fre

re
F Hepatic haemangioma
sf

sf
G Hepatocellular carcinoma
ks
k

k
H Leiomyoma
oo

oo

oo
I Lipoma
eb

eb

eb
J Macroregenerative nodule
K Mesenchymal hamartoma
m

m
The options list contains lesions of the liver occasionally identified during pregnancy.
For each of the following clinical scenarios, choose the most likely lesion from the list.
t

et
ne

ne

Each option may be used once, more than once or not at all.

n
e.

e.

e.
219. A 25-year-old patient undergoes a liver ultrasound at 28 weeks of gestation for
fre

fre

re
the investigation of epigastric pain. She is found to have a single lesion in the liver
measuring 10 cm in diameter. f
ks

ks

ks
oo

oo

oo

220. A 29-year-old woman, known to have hepatitis B, undergoes liver screening at


36 weeks of gestation after the finding of abnormal LFTs. She is found to have a
eb

eb

eb

lesion of the liver, and on biochemical testing has as an α-­fetoprotein level of


1273 ng/ml.
m

m
t

t
ne

ne

ne
e.

e.

e.
re

re

fre
sf

f
ks

ks
k
oo

oo

oo
eb

eb

eb
m

m
t

et

t
ne

ne
n
e.

e.

e.
t

t
Maternal medicine 105

ne

ne

ne
e.

e.

e.
re

fre

fre
Options for questions 221–223

f
ks

ks

ks
A Acute fatty liver of pregnancy
oo

oo

oo
B Alcoholic liver disease
C Autoimmune hepatitis
eb

eb

eb
D Cytomegalovirus hepatitis
m

m
E Epstein–Barr virus hepatitis
F Haemochromatosis
G Hepatitis A
t

t
e

ne

ne
H Hepatitis B
.n

e.

e.
I Hepatitis C
e
re

fre

re
J Hepatitis E
K Hepatocellular carcinoma
sf

sf
ks
k

k
L Intrahepatic cholestasis of pregnancy
oo

oo

oo
M McArdle’s disease
eb

eb

eb
N Primary biliary cholangitis
O Primary sclerosing cholangitis
m

m
P Sjögren’s syndrome
R β-Thalassaemia
Q Wilson’s disease
t

et
ne

ne

n
e.

e.

e.
For each of the following clinical scenarios, choose the single most likely cause of liver
disease presenting in pregnancy. Each option may be used once, more than once or not
fre

fre

re
at all.
f
ks

ks

ks
221. A 30-year-old woman in her third pregnancy arrives unbooked from Mumbai
oo

oo

oo

at 28 weeks of gestation. She begins to feel unwell at 32 weeks of gestation and


develops jaundice, severe fatigue and nausea. She rapidly develops fulminant
eb

eb

eb

hepatic failure and encephalopathy. She is transferred to a liver unit, but dies,
m

despite intensive therapy.

222. A 34-year-old woman starts to develop severe pruritus on her limbs and trunk
at 34 weeks of gestation. On examination, there is evidence of scratching but
t

t
ne

ne

ne

no rash. She is afebrile and her urine is dark. LFTs are abnormal with an alanine
transaminase (ALT) level of 451 U/l with normal bile acids. Standard liver
e.

e.

e.

serological screening is normal.


re

re

fre
sf

f
ks

ks
k
oo

oo

oo
eb

eb

eb
m

m
t

et

t
ne

ne
n
e.

e.

e.
t

t
106 Module 9

ne

ne

ne
e.

e.

e.
re

fre

fre
223. A 23-year-old woman presents with ongoing pruritus that has worsened from
12 weeks of pregnancy.
f
ks

ks

ks
Her blood results are:
oo

oo

oo
Albumin 32 g/l
eb

eb

eb
Serum alkaline phosphatase 320 U/l
m

m
Serum ALT 35 U/l
Serum bilirubin 20 μmol/l
γ-Glutamyl transferase 40 U/l
t

t
e

ne

ne
Anti-nuclear antibodies Negative
.n

Anti-mitochondrial antibodies Positive

e.

e.
e

Anti-smooth muscle antibodies Negative


re

fre

re
Lupus anticoagulant Negative
sf

sf
ks
k

k
oo

oo

oo
Options for questions 224–226
eb

eb

eb
A Fasting glucose 4–7 mmol/l, preprandial glucose 5–7 mmol/l
m

m
B Fasting glucose 5–7 mmol/l, preprandial glucose 4–7 mmol/l
C Fasting glucose ≤5.3 mmmol/l, 1-­hour postprandial glucose ≤7.8 mmol/l, 2-­hour postprandial
glucose ≤6.4 mmol/l
Fasting glucose ≤5.6 mmmol/l, 1-hour postprandial glucose ≤7.2 mmol/l, 2-­hour postprandial
t

et
D
ne

ne

glucose ≤6.4 mmol/l

n
E Fasting glucose >5.6 mmol/l and 2-­hour postprandial glucose >7.8 mmol/l
e.

e.

e.
F Fasting glucose >5.6 mmol/l or 2-­hour postprandial glucose >7.8 mmol/l
fre

fre

re
G Fasting glucose >6 mmol/l
f
ks

ks

ks
H Glucose levels between 4 and 7 mmol/l
oo

oo

oo

I Glucose levels >4 mmol/l


J One-hour postprandial glucose 7.8 mmol/l
eb

eb

eb

K Preprandial glucose 4–6 mmol/l


m

L Two-hour postprandial glucose 6.4 mmol/l

Each of the following clinical scenarios relates to a woman with diabetes. From the
t

t
ne

ne

ne

list of options above, for each woman select the single most appropriate test result. Each
option may be used once, more than once or not at all.
e.

e.

e.
re

re

fre

224. A 29-year-old diabetic woman is seen in the preconception clinic prior to her first
pregnancy. She suffers from type 1 diabetes. She is concerned about the ill effects
sf

f
ks

ks

of diabetes on her baby and asks for advice about the blood glucose levels she
k
oo

oo

oo

should aim for before she gets pregnant.


eb

eb

eb

225. A 32-year-old woman at 32 weeks of gestation is seen in the combined antenatal


clinic following a diagnosis of gestational diabetes. What would be the
m

recommendation for target levels of blood glucose during the pregnancy?


t

et

t
ne

ne
n
e.

e.

e.
t

t
Maternal medicine 107

ne

ne

ne
e.

e.

e.
re

fre

fre
226. A 35-year-old woman who had gestational diabetes and gave birth to an infant
weighing 4.2 kg has returned to the clinic for a review at 6 weeks postnatal. She
f
ks

ks

ks
has just had a blood test and the result indicates that she will need further testing
oo

oo

oo
to see if she has type 2 diabetes.
What test result is she likely to have had?
eb

eb

eb
Options for questions 227–230
m

m
A Anti-embolic stockings only
B Commence low-molecular-weight heparin (LMWH) in the first trimester and continue for 10
t

t
days postpartum
e

ne

ne
.n

C Commence LMWH in the first trimester and continue for 6 weeks postpartum

e.

e.
e

D Commence LMWH at 28 weeks of gestation and continue for 10 days postpartum


re

fre

re
E Commence LMWH at 28 weeks of gestation and continue for 6 weeks postpartum
sf

sf
ks
F Commence LMWH at delivery and continue for 10 days postpartum
k

k
G Commence LMWH at delivery and continue for 6 weeks postpartum
oo

oo

oo
H LMWH at 75% of treatment dose from the first trimester and continue until 6 weeks postpartum
eb

eb

eb
I Send thrombophilia screen and commence LMWH in the first trimester if positive
m

m
J Short-term LMWH during pro-­thrombotic period
K Thromboprophylaxis not required
t

et
Each of the following clinical scenarios relates to thromboprophylaxis in pregnancy.
ne

ne

n
For each patient, select the single most appropriate management from the list above. Each
e.

e.

e.
option may be used once, more than once or not at all.
fre

fre

re
227. A 36-year-old woman, para 3, with a BMI of 32 kg/m2 is carrying a twin
f
ks

ks

ks
pregnancy at 11 weeks of gestation and attends the antenatal clinic for advice. She
smokes two to five cigarettes a day. Her previous pregnancies were uneventful and
oo

oo

oo

resulted in healthy newborn infants at term.


eb

eb

eb

228. A 31-year-old nulliparous woman with a BMI of 29 kg/m2 at 26 weeks of gestation


m

asks for advice about thromboprophylaxis because she will be flying to Dubai (a
7-­hour flight).

229. A 26-year-old nulliparous woman attends the antenatal clinic at 14 weeks of


t

t
ne

ne

ne

gestation. Her BMI is 29 kg/m2 and she gives a history of previous venous
thromboembolism (VTE) during the postoperative recovery period following a
e.

e.

e.

hernia operation.
re

re

fre
sf

230. A 23-year-old woman with a previous history of unprovoked VTE attends your
ks

ks

antenatal clinic at 10 weeks of gestation. She is known to have antithrombin


k
oo

oo

oo

deficiency.
eb

eb

eb
m

m
t

et

t
ne

ne
n
e.

e.

e.
t

t
108 Module 9

ne

ne

ne
e.

e.

e.
re

fre

fre
Answers
f
ks

ks

ks
SBAs
oo

oo

oo
176. Answer  E  Right ventricular dysfunction
eb

eb

eb
Explanation
Women with repaired tetralogy of Fallot usually tolerate pregnancy well; the
m

m
main issue is right ventricular dysfunction, which can deteriorate in view of the
pulmonary regurgitation resulting from earlier surgery.
t

t
Reference
e

ne

ne
.n

Nelson-Piercy C. Handbook of Obstetric Medicine. 5th edn. Boca Raton: CRC Press, 2015.

e.

e.
e

E  Termination of pregnancy
re

fre

re
177. Answer 
sf

sf
Explanation
ks
k

k
Review of the literature between 1997 and 2007 showed maternal death rates
oo

oo

oo
of 17% in idiopathic pulmonary arterial hypertension, 28% in congenital heart
disease-­associated pulmonary hypertension and 33% in other forms. Therefore,
eb

eb

eb
such women should be actively advised against pregnancy and adequate
m

m
contraception recommended, such as the subdermal progestogen-­only implant.
If they do become pregnant, termination should be offered. Termination itself is
associated with maternal mortality in up to 7%, but this is less than that associated
t

et
with such a pregnancy that is allowed to progress.
ne

ne

n
Reference
e.

e.

e.
Nelson-Piercy C. Handbook of Obstetric Medicine. 5th edn. Boca Raton: CRC Press, 2015.
fre

fre

178. Answer  C  Primary surgery f re


ks

ks

ks
oo

oo

oo

Explanation
Surgical treatment including locoregional clearance can be undertaken in all
eb

eb

eb

trimesters. Breast-­conserving surgery or mastectomy can be considered, based


on tumour characteristics and breast size, following multidisciplinary team
m

discussion. Reconstruction should be delayed to avoid prolonged anaesthesia and


to allow optimal symmetrisation of the breasts after delivery.
Radiotherapy is contraindicated until delivery unless it is lifesaving or to
t

t
ne

ne

ne

preserve organ function (e.g. spinal cord compression). If necessary, radiotherapy


can be considered with fetal shielding or, depending on gestational age, early
e.

e.

e.

elective delivery could be discussed. Routine breast/chest wall radiotherapy can be


re

re

fre

deferred until after delivery.


sf

Systemic chemotherapy is contraindicated in the first trimester because of a


ks

ks
k

high rate of fetal abnormality, but is safe from the second trimester and should
oo

oo

oo

be offered according to protocols defined by the risk of breast cancer relapse and
mortality.
eb

eb

eb

Tamoxifen and trastuzumab are contraindicated in pregnancy and should not


m

be used.
Reference
RCOG. Pregnancy and breast cancer. RCOG GTG No. 12. April 2011.
t

et

t
ne

ne
n
e.

e.

e.
t

t
Maternal medicine 109

ne

ne

ne
e.

e.

e.
re

fre

fre
179. Answer  C  Pemphigoid gestationis

f Explanation
ks

ks

ks
Pemphigoid gestationis presents with intense pruritus followed by urticarial
oo

oo

oo
papules and plaques, which typically develop on the abdomen and mostly within
the umbilical region. Lesions may involve the entire body surface and usually
eb

eb

eb
progress to tense blisters. Improvement in late pregnancy is often followed by
m

m
postpartum flare (75% of cases), after which lesions usually resolve within weeks
to months.
Polymorphic eruption of pregnancy starts within the striae distensae on the
t

t
abdomen, with severely pruritic urticarial papules that coalesce into plaques,
e

ne

ne
.n

spreading to the buttocks and proximal thighs and in severe cases becoming

e.

e.
generalised. In contrast to pemphigoid gestationis, the umbilical region is
e
re

fre

re
typically spared.
sf

sf
Reference
ks
k

k
Vaughan Jones S, Ambros-Rudolph C, Nelson-Piercy C. Skin disease in pregnancy. British
oo

oo

oo
Medical Journal 2014;348:g3489.
eb

eb

eb
180. Answer  E Nephrotoxicity
m

m
Explanation
Aminosalicylates (sulfasalazine and mesalazine) do not significantly increase
the rates of miscarriage, birth defects, low birth weight, stillbirth or preterm
t

et
ne

ne

delivery, but doses >3 g/day should be avoided because of the risk of fetal

n
nephrotoxicity.
e.

e.

e.
High-dose folic acid supplementation (5 mg/day) is recommended with
fre

fre

re
sulfasalazine use.
Watch for bloody diarrhoea in infants with mesalazine use. f
ks

ks

ks
oo

oo

oo

Reference
Kapoor D, Teahon K, Wallace SVF. Inflammatory bowel disease in pregnancy. The
eb

eb

eb

Obstetrician & Gynaecologist 2016;18:205–12.


m

181. Answer  C  Every 5 years


Explanation
t

t
Women should be given Haemophilus influenzae type b and the conjugated
ne

ne

ne

meningococcal C vaccine as a single dose if they have not received it as part of


e.

e.

e.

primary vaccination. The pneumococcal vaccine (Pneumovax, Sanofi Pasteur


MSD Ltd, Maidenhead, UK) should be given every 5 years.
re

re

fre
sf

Reference
ks

ks
k

RCOG. Management of sickle cell disease in pregnancy. RCOG GTG No. 61. August 2011.
oo

oo

oo
eb

eb

eb
m

m
t

et

t
ne

ne
n
e.

e.

e.
t

t
110 Module 9

ne

ne

ne
e.

e.

e.
re

fre

fre
182. Answer  A Chloroquine

f
Explanation
ks

ks

ks
Intravenous artesunate is the treatment of choice for severe Plasmodium
oo

oo

oo
falciparum malaria. Use intravenous quinine if artesunate is not available.
Use quinine and clindamycin to treat uncomplicated P. falciparum (or mixed
eb

eb

eb
infections, such as P. falciparum and P. vivax). Use chloroquine to treat P. vivax,
m

m
P. ovale or P. malariae. Primaquine should not be used in pregnancy
Reference
RCOG. The diagnosis and treatment of malaria in pregnancy. RCOG GTG No. 54b. April
t

t
e

ne

ne
2010.
.n

e.

e.
e

183. Answer  B  Oral acyclovir, usual course of antenatal steroids and expectant
re

fre

re
management
sf

sf
ks
Explanation
k

k
oo

oo

oo
In the case of PPROM before 34 weeks, there is evidence to suggest that expectant
management is appropriate, including oral acyclovir 400 mg three times daily
eb

eb

eb
for the mother. After this gestation, it is recommended that management is
undertaken in accordance with relevant RCOG guidelines on PPROM and
m

m
antenatal corticosteroid administration to reduce neonatal morbidity and
mortality, and is not materially influenced by the presence of recurrent genital
herpes lesions.
t

et
ne

ne

n
Reference
e.

e.

e.
BASHH/RCOG. Management of genital herpes in pregnancy. BASHH/RCOG Guideline.
fre

fre

re
October 2014.

f
ks

ks

ks
184. Answer  A  Carbamazepine and lamotrigine
oo

oo

oo

Explanation
In women with epilepsy who are taking anti-­epileptic drugs, the risk of major
eb

eb

eb

congenital malformation to the fetus is dependent on the type, number and


m

dose of the anti-­epileptic drug. Among anti-­epileptic drugs, lamotrigine, and


carbamazepine monotherapy at lower doses have the least risk of major congenital
malformation in the offspring.
t

t
ne

ne

ne

Reference
RCOG. Epilepsy in pregnancy. RCOG GTG No. 68. June 2016.
e.

e.

e.
re

re

fre

185. Answer  B 2–4-fold


sf

f
ks

ks

Explanation
k
oo

oo

oo

Studies have suggested that euthyroid women with autoimmune thyroid disease
have a 2–4-­fold increased risk of preterm labour.
eb

eb

eb

Reference
m

Jefferys A, Vanderpump M, Yasmin E. Thyroid dysfunction and reproductive health. The


Obstetrician & Gynaecologist 2015;17:39–45.
t

et

t
ne

ne
n
e.

e.

e.
t

t
Maternal medicine 111

ne

ne

ne
e.

e.

e.
re

fre

fre
186. Answer  C  6.4 mmol/l

f Explanation
ks

ks

ks
Pregnant women with any form of diabetes should be advised to maintain their
oo

oo

oo
capillary plasma glucose below the following target levels if these are achievable
without causing problematic hypoglycaemia: fasting: 5.3 mmol/l, and 1 hour after
eb

eb

eb
meals: 7.8 mmol/l or 2 hours after meals: 6.4 mmol/l.
m

m
Reference
NICE. Diabetes in pregnancy: management from preconception to the postnatal period.
NICE Guideline (NG3). August 2015.
t

t
e

ne

ne
.n

187. Answer  D Pethidine

e.

e.
e
re

fre

re
Explanation
Diamorphine should be used in preference to pethidine for analgesia in labour.
sf

sf
ks
Pethidine is metabolised to norpethidine, which is known to be epileptogenic
k

k
oo

oo

oo
when administered in high doses to patients with normal renal function.
Pethidine should therefore be avoided or used with caution. Other methods of
eb

eb

eb
analgesia including transcutaneous electrical nerve stimulation (TENS), Entonox
and regional analgesia are safe in labour in an epileptic woman.
m

m
Reference
RCOG. Epilepsy in pregnancy. RCOG GTG No. 68. June 2016.
t

et
ne

ne

n
188. Answer  B  Levonorgestrel-­releasing intrauterine system (LNG-­IUS) and
e.

e.

e.
medroxyprogesterone acetate (MPA) injection
fre

fre

re
Explanation
f
ks

ks

ks
Copper intrauterine contraceptive devices (IUCDs), the LNG-­IUS and MPA
injections should be promoted as reliable methods of contraception that are not
oo

oo

oo

affected by enzyme-­inducing anti-­epileptic drugs.


eb

eb

eb

Reference
m

RCOG. Epilepsy in pregnancy. RCOG GTG No. 68. June 2016.

189. Answer  C  Aspirin 75 mg daily plus LMWH


t

t
ne

ne

ne

Explanation
Women with thalassaemia who have undergone splenectomy and have a platelet
e.

e.

e.

count above 600 × 109/l should be offered LMWH thromboprophylaxis as well as


re

re

fre

low-­dose aspirin (75 mg/day).


sf

f
ks

ks

Reference
k

RCOG. Management of beta thalassaemia in pregnancy. RCOG GTG No. 66. March 2014.
oo

oo

oo
eb

eb

eb
m

m
t

et

t
ne

ne
n
e.

e.

e.
t

t
112 Module 9

ne

ne

ne
e.

e.

e.
re

fre

fre
190. Answer  D  10 days

f
Explanation
ks

ks

ks
During the postnatal period, women with two low-­risk factors should
oo

oo

oo
have thromboprophylaxis for 10 days. Risk factors for this woman are a BMI
>30 kg/m2 and the prothrombin gene mutation, which is a low-­risk
eb

eb

eb
thrombophilia.
m

m
Reference
RCOG. Reducing the risk of venous thromboembolism during pregnancy and the
puerperium. RCOG GTG No37a. April 2015.
t

t
e

ne

ne
.n

191. Answer  C  Stop lithium and check plasma levels 12 hours after the last dose

e.

e.
e
re

fre

re
Explanation
Ensure monitoring by the obstetric team when labour starts, including checking
sf

sf
ks
plasma lithium levels and fluid balance because of the risk of dehydration and
k

k
oo

oo

oo
lithium toxicity. Stop lithium during labour and check plasma lithium levels 12
hours after her last dose.
eb

eb

eb
Reference
m

m
NICE. Antenatal and postnatal mental health: clinical management and service. NICE
Clinical Guideline (CG192). August 2017.
t

et
192. Answer  B  Less than 50 kg and more than 90 kg
ne

ne

n
Explanation
e.

e.

e.
Routine measurement of peak anti-­Xa activity for patients on LMWH for
fre

fre

re
treatment of acute VTE in pregnancy or postpartum is not recommended
except in women at extremes of body weight (<50 kg and ≥90 kg) or with other f
ks

ks

ks
complicating factors (e.g. with renal impairment or recurrent VTE).
oo

oo

oo

Reference
eb

eb

eb

RCOG. Thromboembolic disease in pregnancy and the puerperium: acute management.


RCOG GTG No. 37b. April 2015.
m

193. Answer  B 42%


t

t
Explanation
ne

ne

ne

PTS is characterised by chronic persistent leg swelling, pain, a feeling of heaviness,


e.

e.

e.

dependent cyanosis, telangiectasia, chronic pigmentation, eczema, associated


re

re

fre

varicose veins and, in the most severe cases, venous ulceration. A case–control
study from Norway found a prevalence of PTS of 42% following DVT in
sf

f
ks

ks

pregnancy.
k
oo

oo

oo

Reference
eb

eb

eb

RCOG. Thromboembolic disease in pregnancy and the puerperium: acute management.


RCOG GTG No. 37b. April 2015.
m

m
t

et

t
ne

ne
n
e.

e.

e.
t

t
Maternal medicine 113

ne

ne

ne
e.

e.

e.
re

fre

fre
194. Answer  D Rifampicin

f Explanation
ks

ks

ks
Given that not all women treated with UDCA have biochemical or symptomatic
oo

oo

oo
improvement, a second-­line treatment is sometimes considered. Rifampicin is a
choleretic antibiotic that has been shown to reduce pruritus and enhance bile acid
eb

eb

eb
excretion in primary biliary cirrhosis when used in conjunction with UDCA.
m

m
Reference
Geenes V, Williamson C, Chappell LC. Intrahepatic cholestasis of pregnancy. The
Obstetrician & Gynaecologist 2016;18:273–81.
t

t
e

ne

ne
.n

195. Answer  C 1–3%

e.

e.
e
re

fre

re
Explanation
Graves’ disease is the most common cause of hyperthyroidism in pregnancy,
sf

sf
ks
affecting up to 1% of pregnancies. Often, the diagnosis will already have been
k

k
oo

oo

oo
made, but for those in whom the diagnosis of hyperthyroidism is made in
pregnancy, it can be difficult to differentiate from gestational hyperthyroidism,
eb

eb

eb
which affects between 1% and 3% of all pregnancies and occurs because of
stimulation of thyroid-­stimulating hormone receptors by β-­human chorionic
m

m
gonadotropin (β-hCG).
Reference
t

et
ne

ne

Jefferys A, Vanderpump M, Yasmin E. Thyroid dysfunction and reproductive health. The

n
Obstetrician & Gynaecologist 2015;17:39–45.
e.

e.

e.
fre

fre

re
196. Answer  A  Alanine transaminase
Explanation f
ks

ks

ks
LFTs are abnormal in up to 40% of women with hyperemesis gravidarum, with
oo

oo

oo

the most likely abnormality being a rise in transaminases. Bilirubin levels can be
slightly raised but without jaundice, and amylase levels can be slightly raised too.
eb

eb

eb

These abnormalities improve as the hyperemesis gravidarum resolves.


m

Reference
RCOG. The management of nausea and vomiting of pregnancy and hyperemesis
gravidarum. RCOG GTG No. 69. June 2016.
t

t
ne

ne

ne

197. Answer  D Pre-eclampsia


e.

e.

e.

Explanation
re

re

fre

Acute kidney injury complicates 1.4% of obstetric admissions in the UK and the
sf

f
ks

ks

most common cause is pre-eclampsia.


k
oo

oo

oo

Reference
Wiles KS, Banerjee A. Acute kidney injury in pregnancy and the use of non-­steroidal anti-­
eb

eb

eb

inflammatory drugs. The Obstetrician & Gynaecologist 2016;18:127–35.


m

m
t

et

t
ne

ne
n
e.

e.

e.
t

t
114 Module 9

ne

ne

ne
e.

e.

e.
re

fre

fre
198. Answer  B 3–15%

f
Explanation
ks

ks

ks
HELLP syndrome is characterised by haemolysis, elevated liver enzymes and a
oo

oo

oo
low platelet count. The incidence of renal impairment is higher in HELLP than in
pre-­eclampsia, with acute kidney injury complicating 3–15% of cases. The risk of
eb

eb

eb
acute kidney injury in HELLP increases if abruption, disseminated intravascular
m

m
coagulation, sepsis, haemorrhage or intrauterine death occur, and acute kidney
injury in the context of HELLP worsens prognosis.
Reference
t

t
e

ne

ne
Wiles KS, Banerjee A. Acute kidney injury in pregnancy and the use of non-­steroidal anti-­
.n

inflammatory drugs. The Obstetrician & Gynaecologist 2016;18:127–35.

e.

e.
e
re

fre

re
199. Answer  E  0.9% saline
sf

sf
ks
Explanation
k

k
Fluid replacement should be commenced by infusing isotonic saline (0.9%), as
oo

oo

oo
most patients have a negative fluid balance of about 100 ml/kg of body weight.
eb

eb

eb
This represents a total fluid deficit of approximately 6–10 l.
m

m
Reference
Mohan M, Baagar KAM, Lindow S. Management of diabetic ketoacidosis in pregnancy. The
Obstetrician & Gynaecologist 2017;19:55–62.
t

et
ne

ne

200. Answer  D  3 mmol/l

n
e.

e.

e.
Explanation
fre

fre

re
The Joint British Diabetes Societies Inpatient Care Group guidelines state the
following diagnostic criteria for DKA:
f
ks

ks

ks
• Blood ketone level ≥3.0 mmol/l or urine ketone level >2+
oo

oo

oo

• Blood glucose level >11.0 mmol/l or known diabetes mellitus


• Bicarbonate level <15.0 mmol/l and/or venous pH <7.3.
eb

eb

eb

Reference
m

Mohan M, Baagar KAM, Lindow S. Management of diabetic ketoacidosis in pregnancy. The


Obstetrician & Gynaecologist 2017;19:55–62.
t

t
ne

ne

ne

EMQs
e.

e.

e.

201. Answer  P  Switch to enalapril


re

re

fre

Explanation
sf

If a woman has taken methyldopa to treat chronic hypertension during pregnancy,


ks

ks
k

stop within 2 days of birth and restart the antihypertensive treatment the woman
oo

oo

oo

was taking before she planned the pregnancy.


Tell women who still need antihypertensive treatment in the postnatal period
eb

eb

eb

that the following antihypertensive drugs have no known adverse effects on babies
m

receiving breast milk: enalapril, labetalol, nifedipine, captopril, atenolol and


metoprolol.
t

et

t
ne

ne
n
e.

e.

e.
t

t
Maternal medicine 115

ne

ne

ne
e.

e.

e.
re

fre

fre
202. Answer  K  Start low-­dose aspirin

f Explanation
ks

ks

ks
Advise women at high risk of pre-­eclampsia to take 75 mg of aspirin daily from
oo

oo

oo
12 weeks until the birth of the baby. Women at high risk are those with any of the
following:
eb

eb

eb
• Hypertensive disease during a previous pregnancy
m

m
• Chronic kidney disease
• Autoimmune disease such as systemic lupus erythematosus or
antiphospholipid syndrome
t

t
e

ne

ne
• Type 1 or 2 diabetes
.n

• Chronic hypertension.

e.

e.
e
re

fre

re
203. Answer  H  Start labetalol
sf

sf
ks
Explanation
k

k
For the management of pregnancy with moderate gestational hypertension
oo

oo

oo
(150/100–159/109 mmHg), treat with oral labetalol as the first-­line treatment
eb

eb

eb
to keep the diastolic blood pressure between 80 and 100 mmHg and the systolic
blood pressure <150 mmHg.
m

m
Reference
NICE. Hypertension in pregnancy: diagnosis and management. NICE Clinical Guideline
t

et
(CG107). Updated January 2011.
ne

ne

n
e.

e.

e.
204. Answer  E  Direct death
fre

fre

re
Explanation
The death is a direct result of the pregnancy. f
ks

ks

ks
oo

oo

oo

205. Answer  B  Coincidental death


eb

eb

eb

Explanation
The death is not a direct or indirect consequence of the pregnancy.
m

206. Answer  I  Indirect death


t

t
Explanation
ne

ne

ne

The death is related to the pre-­existing disease.


e.

e.

e.

The definitions of pregnancy-­related deaths by the World Health Organization


(2010) are as follows:
re

re

fre

• Maternal: death of a women while pregnant or within 42 days of the end


sf

f
ks

ks

of the pregnancy. This includes death as a result of giving birth, ectopic


k
oo

oo

oo

pregnancy, miscarriage or termination of pregnancy from any cause related to


or aggravated by the pregnancy or its management, but not from accidental or
eb

eb

eb

incidental causes.
• Direct: deaths resulting from obstetric complications of the pregnant state
m

(pregnancy, labour and puerperium), from interventions, omissions, incorrect


treatment, or from a chain of events resulting from any of the above.
t

et

t
ne

ne
n
e.

e.

e.
t

t
116 Module 9

ne

ne

ne
e.

e.

e.
re

fre

fre
• Indirect: deaths resulting from previous existing disease, or disease that
developed during pregnancy and which was not the result of direct obstetric
f
ks

ks

ks
causes, but which was aggravated by the physiological effects of pregnancy.
oo

oo

oo
• Late: deaths occurring between 42 days and 1 year after the end of pregnancy
(includes giving birth, ectopic pregnancy, miscarriage or termination of
eb

eb

eb
pregnancy) that are the result of direct or indirect maternal causes.
• Coincidental: deaths from unrelated causes that happen to occur in pregnancy
m

m
or the puerperium.

Reference
t

t
e

ne

ne
Knight M, Nair M, Tuffnell D, et al. (eds.) on behalf of MBRRACE-­UK. Saving Lives,
.n

Improving Mothers’ Care: Surveillance of Maternal Deaths in the UK 2012–14 and Lessons

e.

e.
e

Learned to Inform Maternity Care From the UK and Ireland Confidential Enquiries into
re

fre

re
Maternal Deaths and Morbidity 2009–14. Oxford: National Perinatal Epidemiology
Unit, University of Oxford, 2016.
sf

sf
ks
k

k
207. Answer  K  Sodium valproate
oo

oo

oo
Explanation
eb

eb

eb
Sodium valproate is associated with neural tube defects, a facial cleft and hypospadias.
m

m
208. Answer  D Lamotrigine
Explanation
t

et
In women with epilepsy who are taking anti-­epileptic drugs (AEDs), the risk of
ne

ne

n
major congenital malformation to the fetus is dependent on the type, number
e.

e.

e.
and dose of AEDs. Among AEDs, lamotrigine and carbamazepine monotherapy
fre

fre

re
at lower doses have the lowest risk of major congenital malformation in the
offspring.
f
ks

ks

ks
Enzyme-inducing AEDs (carbamazepine, phenytoin, phenobarbital,
oo

oo

oo

primidone, oxcarbazepine, topiramate and eslicarbazepine) are considered to


competitively inhibit the precursors of clotting factors and affect fetal microsomal
eb

eb

eb

enzymes that degrade vitamin K, thereby increasing the risk of haemorrhagic


m

disease of the newborn.


The drug that fulfils both of these characteristics is therefore lamotrigine.

209. Answer  H Phenytoin


t

t
ne

ne

ne

Explanation
e.

e.

e.

If seizures are not controlled, consider administration of phenytoin or


re

re

fre

fosphenytoin. The loading dose of phenytoin is 10–15 mg/kg by intravenous


infusion, with the usual dosage for an adult being about 1000 mg.
sf

f
ks

ks
k

Reference
oo

oo

oo

RCOG. Epilepsy in pregnancy. RCOG GTG No. 68. June 2016.


eb

eb

eb
m

m
t

et

t
ne

ne
n
e.

e.

e.
t

t
Maternal medicine 117

ne

ne

ne
e.

e.

e.
re

fre

fre
210. Answer  I Metformin

f Explanation
ks

ks

ks
Offer metformin to women with gestational diabetes if blood glucose targets are
oo

oo

oo
not met using changes in diet and exercise within 1–2 weeks.
eb

eb

eb
211. Answer  B Glibenclamide
m

m
Explanation
Women with pre-­existing type 2 diabetes who are breastfeeding can resume or
continue to take metformin and glibenclamide immediately after birth, but should
t

t
e

ne

ne
avoid other oral blood glucose-­lowering agents while breastfeeding.
.n

e.

e.
e

212. Answer  H  Isophane insulin (NPH insulin)


re

fre

re
Explanation
sf

sf
ks
Offer immediate treatment with insulin, with or without metformin, as well as
k

k
changes in diet and exercise, to women with gestational diabetes who have a
oo

oo

oo
fasting plasma glucose level of ≥7.0 mmol/l at diagnosis. Isophane insulin is the
eb

eb

eb
only short-­acting insulin given in the list.
m

m
Reference
NICE. Diabetes in pregnancy: management from preconception to the postnatal period.
NICE Guideline (NG3). August 2015.
t

et
ne

ne

213. Answer  D  >1000 HIV RNA copies/ml

n
e.

e.

e.
Explanation
fre

fre

re
Intrapartum intravenous zidovudine infusion is recommended for women
f
with a viral load of >1000 HIV RNA copies/ml of plasma who present in labour
ks

ks

ks
or with ruptured membranes, or who are admitted for a planned caesarean
oo

oo

oo

section.
eb

eb

eb

214. Answer  C  ≥400 HIV RNA copies/ml


m

Explanation
Where the viral load is ≥400 HIV RNA copies/ml at 36 weeks, a planned lower-­
segment caesarean section is recommended. For a viral load of 50–399 copies/ml,
t

t
ne

ne

ne

planned lower-­segment caesarean section would be offered, and <50 copies/ml


vaginal birth would be recommended.
e.

e.

e.
re

re

fre

215. Answer  A  <50 HIV RNA copies/ml


sf

f
ks

ks

Explanation
k

For the management of spontaneous rupture of membranes:


oo

oo

oo

• If the maternal HIV viral load is <50 HIV RNA copies/ml, immediate
eb

eb

eb

induction of labour is recommended, with a low threshold for treatment of


intrapartum pyrexia
m

• If the viral load is >50 HIV RNA copies/ml, then an immediate caesarean
section would be offered.
t

et

t
ne

ne
n
e.

e.

e.
t

t
118 Module 9

ne

ne

ne
e.

e.

e.
re

fre

fre
Reference
f
BHIVA. British HIV Association guidelines for the management of HIV infection in
ks

ks

ks
pregnant women 2012 (2014 interim review). HIV Medicine 2014;15(Suppl. 4):1–77.
oo

oo

oo
216. Answer  N  Thrombotic thrombocytopenic purpura (TTP)
eb

eb

eb
Explanation
m

m
The classical pentad of TTP is:
• Microangiopathic haemolytic anaemia
• Thrombocytopenia
t

t
e

ne

ne
• Fever
.n

• Neurological manifestations

e.

e.
e

• Renal impairment/acute kidney injury.


re

fre

re
The clinical features of TTP/haemolytic–uraemic syndrome (HUS) may be
sf

sf
confused with pre-­eclampsia and particularly HELLP syndrome. However,
ks
k

k
hypertension is not common in TTP/HUS and there is no coagulopathy. Features
oo

oo

oo
include headache, irritability, drowsiness, seizures, coma and fever.
eb

eb

eb
217. Answer  E Eclampsia
m

m
Explanation
The features of pre-­eclampsia may be mild or delayed.
t

et
ne

ne

218. Answer  K  Non-­epileptic seizure disorder

n
e.

e.

e.
Explanation
fre

fre

re
Useful distinguishing features to differentiate a ‘psychogenic’ non-­epileptic seizure
from organic non-­epileptic seizure or epilepsy include:
f
ks

ks

ks
• Prolonged/repeated seizures without cyanosis
oo

oo

oo

• Resistance to passive eye opening


• Downgoing plantar reflexes
eb

eb

eb

• Persistence of a positive conjunctival reflex.


m

Reference
Nelson-Piercy C. Handbook of Obstetric Medicine. 5th edn. Boca Raton: CRC Press, 2015.
t

t
ne

ne

ne

219. Answer  F  Hepatic haemangioma


e.

e.

e.

Explanation
re

re

fre

Hepatic haemangiomas are the most common benign tumour of the liver and
are present in 2–20% of healthy individuals. They are well-­circumscribed lesions
sf

f
ks

ks

that arise from vascular endothelial cells with multiple, large vascular channels
k
oo

oo

oo

supported by collagenous walls. The blood supply arises from the hepatic artery,
and the lesion can grow up to 20 cm and cause symptoms.
eb

eb

eb
m

m
t

et

t
ne

ne
n
e.

e.

e.
t

t
Maternal medicine 119

ne

ne

ne
e.

e.

e.
re

fre

fre
220. Answer  G  Hepatocellular carcinoma

f Explanation
ks

ks

ks
The incidence of hepatocellular carcinoma is increasing in the UK because of
oo

oo

oo
cirrhosis linked to obesity and alcohol use. However, the highest rates are found
in women from Asian backgrounds with early-­onset cirrhosis associated with
eb

eb

eb
chronic viral hepatitis B and C. During pregnancy, it has been reported that a
m

m
serum α-­fetoprotein level of >1000 ng/ml can be considered diagnostic; however,
this is inconsistent.
Survival appears to be lower in pregnant women, which may be due to the
t

t
hormonal and immunological influences of pregnancy.
e

ne

ne
.n

Reference

e.

e.
e

Milburn J, Black M, Ahmed I, et al. Diagnosis and management of liver masses in


re

fre

re
pregnancy. The Obstetrician & Gynaecologist 2016;18:43–51.
sf

sf
ks
k

k
221. Answer  J  Hepatitis E
oo

oo

oo
Explanation
eb

eb

eb
Hepatitis E virus is spread by the faecal–oral route and has been associated with
drinking contaminated water. The incubation period is 3–8 weeks, with most
m

m
presenting at 5–6 weeks. The mortality rate is greatly elevated in pregnant women,
especially if acquired in the third trimester. While acute fatty liver of pregnancy is
a differential, this typically presents later in the third trimester and would be less
t

et
ne

ne

likely in a multiparous patient.

n
e.

e.

e.
222. Answer  L  Intrahepatic cholestasis of pregnancy
fre

fre

re
Explanation
f
ks

ks

ks
This is a typical presentation of intrahepatic cholestasis of pregnancy. Most cases
present in the third trimester. The woman is afebrile, so an infective cause is less
oo

oo

oo

likely.
eb

eb

eb

223. Answer  N  Primary biliary cholangitis


m

Explanation
The ongoing pruritus suggests pre-­existing disease. Primary biliary cholangitis
t

t
may result in worsening pruritus in pregnancy. Alkaline phosphatase and
ne

ne

ne

γ-­glutamyl transferase levels tend to be raised (although alkaline phosphatase


e.

e.

e.

is already raised in pregnancy). The finding of positive anti-­mitochondrial


antibodies clinches the diagnosis.
re

re

fre
sf

Reference
ks

ks
k

Nelson-Piercy C. Handbook of Obstetric Medicine. 5th edn. Boca Raton: CRC Press, 2015.
oo

oo

oo
eb

eb

eb
m

m
t

et

t
ne

ne
n
e.

e.

e.
t

t
120 Module 9

ne

ne

ne
e.

e.

e.
re

fre

fre
224. Answer  B  Fasting glucose 5–7 mmol/l, preprandial glucose 4–7 mmol/l

f
Explanation
ks

ks

ks
Advise women with diabetes who are planning to become pregnant to aim for the
oo

oo

oo
same capillary plasma glucose target ranges as recommended for all people with
type 1 diabetes: a fasting plasma glucose level of 5–7 mmol/l on waking and a
eb

eb

eb
plasma glucose level of 4–7 mmol/l before meals at other times of the day.
m

m
225. Answer  C  Fasting glucose ≤5.3 mmmol/l, 1-­hour postprandial glucose
≤7.8 mmol/l, 2-­hour postprandial glucose ≤6.4 mmol/l
t

t
e

ne

ne
Explanation
.n

Advise pregnant women with any form of diabetes to maintain their capillary

e.

e.
e

plasma glucose below the following target levels, if these are achievable without
re

fre

re
causing problematic hypoglycaemia: fasting glucose of 5.3 mmol/l and glucose
sf

sf
1 hour after meals of 7.8 mmol/l or glucose 2 hours after meals of 6.4 mmol/l.
ks
k

k
oo

oo

oo
226. Answer  G  Fasting glucose >6 mmol/l
eb

eb

eb
Explanation
For women who were diagnosed with gestational diabetes and whose blood
m

m
glucose levels returned to normal after the birth, offer lifestyle advice (including
weight control, diet and exercise) and a fasting plasma glucose test 6–13 weeks
after the birth to exclude diabetes (for practical reasons this might take place at
t

et
ne

ne

the 6-­week postnatal check).

n
Advise women with a fasting plasma glucose level <6.0 mmol/l that they have
e.

e.

e.
a low probability of having diabetes at present and they should continue to follow
fre

fre

re
the lifestyle advice.
f
ks

ks

ks
Reference
oo

oo

oo

NICE. Diabetes in pregnancy: management from preconception to the period. NICE


Guideline (NG3). Updated August 2015.
eb

eb

eb

227. Answer  C  Commence LMWH in the first trimester and continue for 6 weeks
m

postpartum
Explanation
t

t
This woman has five antenatal risk factors. Prophylaxis should commence in the
ne

ne

ne

first trimester and continue until 6 weeks postpartum.


e.

e.

e.

228. Answer  A  Anti-­embolic stockings only


re

re

fre
sf

Explanation
ks

ks
k

Any pregnant woman who will be undertaking air travel of >4 hours’ duration
oo

oo

oo

should be advised to use anti-­embolic stockings exerting a calf pressure of


14–15 mmHg.
eb

eb

eb
m

m
t

et

t
ne

ne
n
e.

e.

e.
t

t
Maternal medicine 121

ne

ne

ne
e.

e.

e.
re

fre

fre
229. Answer  E  Commence LMWH at 28 weeks of gestation and continue for 6
weeks postpartum
f
ks

ks

ks
Explanation
oo

oo

oo
In women in whom the original VTE was provoked by major surgery from which
they have recovered, and who have no other risk factors, thromboprophylaxis
eb

eb

eb
with LMWH can be withheld antenatally until 28 weeks provided no additional
m

m
risk factors are present (in which case they should be offered LMWH). They
require close surveillance for the development of other risk factors.
t

t
230. Answer  H  LMWH at 75% of treatment dose from the first trimester and
e

ne

ne
.n

continue until 6 weeks postpartum

e.

e.
e

Explanation
re

fre

re
Women with previous VTE associated with antithrombin deficiency
sf

sf
(who will often be on long-­term oral anticoagulation) should be offered
ks
k

k
thromboprophylaxis with higher-­dose LMWH (either 50%, 75% or full
oo

oo

oo
treatment dose) antenatally and for 6 weeks postpartum or until returned to oral
anticoagulant therapy after delivery.
eb

eb

eb
Reference
m

m
RCOG. Reducing the risk of venous thromboembolism during pregnancy and the
puerperium. RCOG GTG No37a. April 2015
t

et
ne

ne

n
e.

e.

e.
fre

fre

f re
ks

ks

ks
oo

oo

oo
eb

eb

eb
m

m
t

t
ne

ne

ne
e.

e.

e.
re

re

fre
sf

f
ks

ks
k
oo

oo

oo
eb

eb

eb
m

m
t

et

t
ne

ne
n
e.

e.

e.
t

t
ne

ne

ne
e.

e.

e.
re

fre

fre
Module
f Management of labour
ks

ks

ks
10
oo

oo

oo
eb

eb

eb
m

m
t

t
e

ne

ne
SBAs
.n

e.

e.
e

231. In a 2012 report by the National Health Service Litigation Authority (NHSLA),
re

fre

re
what was the most common contributor to medicolegal obstetric claims in the
sf

sf
UK?
ks
k

k
A. Failure to act
oo

oo

oo
B. Failure to monitor adequately
eb

eb

eb
C. Failure to recognise an abnormal cardiotocograph (CTG)
D. Failure to refer
m

m
E. Inappropriate use of oxytocin (Syntocinon)

232. A 24-year-old woman in her first pregnancy attends for induction of labour at
t

et
ne

ne

term + 13. She had membrane sweeps at 40 and 41 weeks. She is examined and

n
the cervix is found to be 2 cm dilated and 2 cm long with the fetal head engaged.
e.

e.

e.
She does not report any contractions.
fre

fre

re
What would be the preferred method of induction of labour?
f
ks

ks

ks
A. Amniotomy ± oxytocin (Syntocinon)
B. Balloon catheter
oo

oo

oo

C. Membrane sweep
eb

eb

eb

D. Misoprostol
E. Vaginal prostaglandin
m

233. A woman is admitted to the labour ward with intermittent lower abdominal pain
and is found to be in preterm labour. Her uterus is contracting three times in 10
t

t
ne

ne

ne

minutes and the cervix is 4 cm dilated.


Until which gestational age should magnesium sulfate be offered?
e.

e.

e.

A. 29 + 6 weeks
re

re

fre

B. 30 + 6 weeks
sf

f
ks

ks

C. 31 + 6 weeks
k

D. 32 + 6 weeks
oo

oo

oo

E. 33 + 6 weeks
eb

eb

eb
m

122
t

et

t
ne

ne
n
e.

e.

e.
t

t
Management of labour 123

ne

ne

ne
e.

e.

e.
re

fre

fre
234. A 26-year-old woman is seen complaining of regular contractions (one every 10
minutes) at 30 + 3 weeks of gestation. A speculum examination is performed
f
ks

ks

ks
and the cervix appears long but slightly dilated. There is no history of ruptured
oo

oo

oo
membranes.
What investigation should be considered first in this situation?
eb

eb

eb
A. Fetal fibronectin
m

m
B. Full blood count (FBC) and C-­reactive protein (CRP)
C. No test required – treat as preterm labour
D. Test for insulin-­like growth factor-­binding protein-­1 (IGFBP-­1) or placental
α-microglobulin-1 (PAMG-1)
t

t
e

ne

ne
E. Transvaginal ultrasound
.n

e.

e.
e

235. While in labour, a woman in her first pregnancy at term is deemed to have an
re

fre

re
abnormal CTG at 6 cm dilation but is making good progress in labour. Fetal
sf

sf
ks
blood sampling (FBS) is performed and the lactate level is 4.9 mmol/l. There was a
k

k
small acceleration in the fetal heart rate during the process of obtaining the blood
oo

oo

oo
sample. The cervix is 7 cm dilated after the FBS.
eb

eb

eb
What is the recommended management?
A. Caesarean section
m

m
B. Continue with labour
C. Repeat the FBS in 30 minutes
D. Repeat the FBS in 1 hour
t

et
ne

ne

E. Repeat the FBS in 1 hour or sooner if the CTG deteriorates

n
e.

e.

e.
236. A 30-year-old woman in her first pregnancy is in spontaneous labour and has
fre

fre

re
been using nitrous oxide for analgesia. Vaginal examination a few minutes ago
revealed that she is now fully dilated. She has no urge to push.
f
ks

ks

ks
What is the most appropriate plan of action?
oo

oo

oo

A. Advise the woman to start active pushing


B. Reassess in 1 hour
eb

eb

eb

C. Reassess in 2 hours
m

D. Reassess in 4 hours
E. Start an oxytocin (Syntocinon) infusion
t

t
237. A 41-year-old woman is seen at 36 weeks of gestation. This is her first pregnancy,
ne

ne

ne

and a plan for induction of labour at 39 weeks has been made. She is anxious
e.

e.

e.

about induction of labour.


What intrapartum risk is increased for women in this age group who undergo
re

re

fre

induction of labour compared with expectant management?


sf

f
ks

ks

A. Caesarean section
k
oo

oo

oo

B. Failed induction of labour


C. Fetal hypoxia
eb

eb

eb

D. Instrumental delivery
E. No additional risk
m

m
t

et

t
ne

ne
n
e.

e.

e.
t

t
124 Module 10

ne

ne

ne
e.

e.

e.
re

fre

fre
238. A woman in labour requests an epidural for analgesia, but unfortunately during
the siting of the epidural there is inadvertent puncture of the dura mater.
f
ks

ks

ks
How likely is it that she will develop a postdural puncture headache?
oo

oo

oo
A. 5–10%
B. 15–25%
eb

eb

eb
C. 30–45%
m

m
D. 70–80%
E. 100%

239. Concerning the use of FBS to assess fetal acidaemia, what is the main benefit of
t

t
e

ne

ne
using lactate alone as opposed to pH?
.n

e.

e.
A. Improved cord gases at delivery
e

B. Improved success rate


re

fre

re
C. Reduced caesarean section rate
sf

sf
ks
D. Reduced hypoxic ischaemic encephalopathy
k

k
E. Reduced instrumental delivery rate
oo

oo

oo
eb

eb

eb
240. What is the most common cardiac adverse effect associated with the use of
atosiban?
m

m
A. Chest pain
B. Hypotension
C. Palpitations
t

et
ne

ne

D. Pulmonary oedema

n
E. Tachycardia
e.

e.

e.
fre

fre

f re
ks

ks

ks
oo

oo

oo
eb

eb

eb
m

m
t

t
ne

ne

ne
e.

e.

e.
re

re

fre
sf

f
ks

ks
k
oo

oo

oo
eb

eb

eb
m

m
t

et

t
ne

ne
n
e.

e.

e.
t

t
Management of labour 125

ne

ne

ne
e.

e.

e.
re

fre

fre
EMQs
f
ks

ks

ks
Options for questions 241–244
oo

oo

oo
A Amniotomy
eb

eb

eb
B Amniotomy and vacuum extraction of the baby
C Amniotomy followed by oxytocin (Syntocinon) infusion
m

m
D Assess in 1 hour
E Assess in 2 hours
t

t
F Assess in 4 hours
e

ne

ne
.n

G Deliver by caesarean section

e.

e.
e

H Midcavity forceps delivery


re

fre

re
I Oxytocin (Syntocinon) infusion with dose increments every 30 minutes
sf

sf
J Oxytocin (Syntocinon) infusion with dose increments every 15 minutes
ks
k

k
K Oxytocin (Syntocinon) infusion until there are two to three contractions every 10 minutes
oo

oo

oo
eb

eb

eb
Each of the following clinical scenarios relates to the management of delivery. For
each patient, select the single most appropriate management plan from the options above.
m

m
Each option may be used once, more than once or not at all.

241. A 23-year-old low-­risk woman in her first pregnancy is now in established labour
t

et
following spontaneous rupture of membranes. Vaginal examination at 07:30 hours
ne

ne

revealed cephalic presentation and a fully effaced, 4 cm dilated cervix. There

n
e.

e.

e.
are regular uterine contractions of increasing intensity at three to four every 10
fre

fre

re
minutes. At 11:30 hours, her cervix is 5 cm dilated with the sagittal suture in the
transverse position and no further descent of the fetal head. There is no evidence
f
ks

ks

ks
of meconium or caput, and auscultation of the fetal heart is a normal pattern.
oo

oo

oo

242. A 27-year-old woman is in labour for the second time. Clinical findings on
eb

eb

eb

vaginal examination are as follows. At 06:30 hours, there is cephalic presentation,


and a 4 cm dilated, 0.5 cm long central, soft cervix. At 10:30 hours, there is
m

cervical dilation of 5 cm and a fully effaced cervix and intact membranes. At 12:30
hours, the cervical findings are unchanged, but no membranes are felt.

243. A 32-year-old woman in her first pregnancy at term has been in established
t

t
ne

ne

ne

labour for 14 hours before a vaginal examination at 12:30 hours finds a fully
dilated cervix, clear amniotic fluid, and the fetal head at 0 station and in the right
e.

e.

e.

occiput anterior position. At 13:30 hours, she is unable to resist the urge to push
re

re

fre

and starts voluntary efforts.


sf

f
ks

ks
k

244. A 35-year-old woman in her pregnancy at term is in the first stage of labour.
oo

oo

oo

She underwent spontaneous rupture of membranes at 19:30 hours. At 21:00


hours, vaginal examination revealed that the cervix was fully effaced and 4 cm
eb

eb

eb

dilated. At 01:00 hours, the partogram shows that uterus has been contracting
m

two to three times every 10 minutes and the cervical findings remain the same.
The fetal head is at −1 station with the sagittal suture in the transverse
position.
t

et

t
ne

ne
n
e.

e.

e.
t

t
126 Module 10

ne

ne

ne
e.

e.

e.
re

fre

fre
Options for questions 245–247

f
ks

ks

ks
A Acupuncture
oo

oo

oo
B Birthing balls and upright posture
C Codeine
eb

eb

eb
D Combined spinal–epidural anaesthesia
m

m
E Continuous one-to-one support
F Diamorphine
G Epidural anaesthesia
t

t
e

ne

ne
H Nitrous oxide
.n

e.

e.
I Paracetamol
e
re

fre

re
J Pethidine
K Remifentanil
sf

sf
ks
k

k
L Spinal anaesthesia
oo

oo

oo
M Transcutaneous electrical nerve stimulation (TENS)
eb

eb

eb
N Water immersion
m

m
For each of the following clinical scenarios, choose the single most appropriate option
from the list above. Each option may be used once, more than once or not at all.
t

et
ne

ne

245. A 26-year-old woman is being seen in the antenatal clinic with a history of anxiety.

n
She wants to avoid any sort of medication in labour. What single non-­invasive
e.

e.

e.
measure is likely to provide her with the lowest perception of pain in labour?
fre

fre

f
246. A 22-year-old woman with a history of epilepsy presents in labour. She has been
re
ks

ks

ks
seizure free for 7 years while not taking any anti-­epileptic medications. What
oo

oo

oo

method of pain relief should be avoided in this situation?


eb

eb

eb

247. A 27-year-old woman presents in labour at 6 cm dilated. She is a smoker and


reports a 1-­week history of productive cough and pleuritic-­type chest pain.
m

Oxygen saturation is 94% at rest. What labour analgesia in particular should be


avoided for this woman?
t

t
ne

ne

ne
e.

e.

e.
re

re

fre
sf

f
ks

ks
k
oo

oo

oo
eb

eb

eb
m

m
t

et

t
ne

ne
n
e.

e.

e.
t

t
Management of labour 127

ne

ne

ne
e.

e.

e.
re

fre

fre
Options for questions 248–250

f
ks

ks

ks
A Abnormal
oo

oo

oo
B Accelerative
C Non-reactive
eb

eb

eb
D Non-reassuring
m

m
E Normal
F Pathological
G Reactive
t

t
e

ne

ne
H Reassuring
.n

e.

e.
I Sinusoidal
e
re

fre

re
J Suspicious
sf

sf
ks
For each of the following CTG descriptions, choose the most appropriate overall catego-
k

k
oo

oo

oo
risation of CTG trace. Each description relates to a woman in her first pregnancy in the spon-
taneous first stage of labour. Each option may be used once, more than once or none at all.
eb

eb

eb
248. The CTG shows a baseline rate of 150 beats per minute (bpm). Accelerations are
m

m
absent and variability is 7 bpm. There are shallow decelerations occurring with
contractions, and the fetal heart rate is falling by 20 bpm from the baseline and
lasting 30 seconds, mirroring each contraction for 80 minutes. Contractions are
t

et
ne

ne

four every 10 minutes. What is the overall classification?

n
e.

e.

e.
249. The CTG shows a baseline rate of 140 bpm. Accelerations are absent. Variability
has been 3 bpm for 25 minutes. There are variable decelerations present, each
fre

fre

re
lasting 70 seconds with every contraction for a duration of 40 minutes, with a
f
ks

ks

ks
delayed recovery and no shouldering. Contractions are three every 10 minutes.
How should the decelerations in the CTG trace be described?
oo

oo

oo

250. The CTG shows a baseline rate of 150 bpm and accelerations are present.
eb

eb

eb

Baseline variability is 8 bpm, and there are decelerations, with the fetal heart rate
m

dropping by 50 bpm and lasting 70 seconds. The decelerations start following each
contraction for the last 25 minutes in a 40-­minute trace. Contraction frequency is
four every 10 minutes. What is the overall categorisation of the CTG?
t

t
ne

ne

ne
e.

e.

e.
re

re

fre
sf

f
ks

ks
k
oo

oo

oo
eb

eb

eb
m

m
t

et

t
ne

ne
n
e.

e.

e.
t

t
128 Module 10

ne

ne

ne
e.

e.

e.
re

fre

fre
Answers
f
ks

ks

ks
SBAs
oo

oo

oo
231. Answer   C  Failure to recognise an abnormal cardiotocograph (CTG)
eb

eb

eb
Explanation
See Figure 5 in the reference article.
m

m
Reference
Oláh KSJ, Steer PJ. The use and abuse of oxytocin. The Obstetrician & Gynaecologist
t

t
2015;17:265–71.
e

ne

ne
.n

e.

e.
232. Answer   E  Vaginal prostaglandin
e
re

fre

re
Explanation
sf

sf
Vaginal prostaglandin E2 (PGE2) is the preferred method of induction of labour,
ks
k

k
unless there are specific clinical reasons for not using it (in particular, the risk of
oo

oo

oo
uterine hyperstimulation). It should be administered as a gel, tablet or controlled-­
release pessary. Costs may vary over time, and Trusts/units should take this
eb

eb

eb
into consideration when prescribing PGE2. For doses, refer to the Summary of
m

m
Product Characteristics. The recommended regimens are:
• One cycle of vaginal PGE2 tablets or gel: one dose, followed by a second
dose after 6 hours if labour is not established (up to a maximum of two
t

et
doses) or
ne

ne

n
• One cycle of vaginal PGE2 controlled-­release pessary: one dose over
e.

e.

e.
24 hours.
fre

fre

re
Amniotomy, alone or with oxytocin (Syntocinon), should not be used as a
primary method of induction of labour unless there are specific clinical reasons f
ks

ks

ks
for not using vaginal PGE2, in particular the risk of uterine hyperstimulation.
oo

oo

oo

Reference
eb

eb

eb

NICE. Inducing labour. NICE Clinical Guideline (CG70). July 2008.


m

233. Answer   A 29 + 6 weeks


Explanation
t

t
Intravenous magnesium sulfate should be offered for neuroprotection of the baby
ne

ne

ne

to women between 24 weeks and 29 + 6 weeks of pregnancy who are:


e.

e.

e.

• In established preterm labour or


re

re

fre

• Having a planned preterm birth within 24 hours.


sf

Intravenous magnesium sulfate should also be considered for neuroprotection


ks

ks
k

of the baby for women between 30+0 and 33+6 weeks of pregnancy who are:
oo

oo

oo

• In established preterm labour or


eb

eb

eb

• Having a planned preterm birth within 24 hours.


m

Reference
NICE. Preterm labour and birth. NICE Guidelines (NG25). November 2015.
t

et

t
ne

ne
n
e.

e.

e.
t

t
Management of labour 129

ne

ne

ne
e.

e.

e.
re

fre

fre
234. Answer   E  Transvaginal ultrasound

f Explanation
ks

ks

ks
If the clinical assessment suggests that the woman is in suspected preterm labour
oo

oo

oo
and she is 29+6 weeks pregnant or less, advise treatment for preterm labour.
If the clinical assessment suggests that the woman is in suspected preterm
eb

eb

eb
labour and she is 30+0 weeks pregnant or more, consider transvaginal ultrasound
m

m
measurement of cervical length as a diagnostic test to determine the likelihood of
birth within 48 hours.
Reference
t

t
e

ne

ne
NICE. Preterm labour and birth. NICE Guidelines (NG25). November 2015.
.n

e.

e.
e

235. Answer   A  Caesarean section


re

fre

re
Explanation
sf

sf
ks
k

k
oo

oo

oo
Lactate (mmol/l) pH Interpretation
≤4.1 ≥7.25
eb

eb

eb
Normal
4.2–4.8 7.21–7.24 Borderline
m

m
≥4.9 ≤7.20 Abnormal
t

et
ne

ne

Reference

n
NICE. Intrapartum care for healthy women and babies. NICE Clinical Guideline (CG190).
e.

e.

e.
Updated February 2017.
fre

fre

236. Answer   B  Reassess in 1 hour


f re
ks

ks

ks
Explanation
oo

oo

oo

If full dilatation of the cervix has been confirmed in a woman without regional
eb

eb

eb

analgesia but she does not get an urge to push, carry out further assessment after
1 hour. Consideration should be given to the use of oxytocin (Syntocinon),
m

with the offer of regional analgesia, for nulliparous women if contractions are
inadequate at the onset of the second stage.
t

t
Reference
ne

ne

ne

NICE. Intrapartum care for healthy women and babies. NICE Clinical Guideline (CG190).
e.

e.

e.

Updated February 2017.


re

re

fre

237. Answer   E  No additional risk


sf

f
ks

ks

Explanation
k
oo

oo

oo

UK data from 2009–10 suggest that if all women aged ≥40 years with a singleton
pregnancy had been induced at 39 weeks of gestation instead of at 41 weeks of
eb

eb

eb

gestation, 17 stillbirths could have been prevented. This equates to inducing an


m

extra 9350 women, or 550 women to prevent one stillbirth. Inducing at 40 weeks
of gestation instead of 41 weeks would prevent seven stillbirths and require an
extra 4750 women to be induced.
t

et

t
ne

ne
n
e.

e.

e.
t

t
130 Module 10

ne

ne

ne
e.

e.

e.
re

fre

fre
In a study by Walker et al. (2016), a total of 619 women aged ≥35 years
underwent randomisation with respect to induction of labour or expectant
f
ks

ks

ks
management. In an intention-to-treat analysis, there were no significant between-­
oo

oo

oo
group differences in the percentage of women who underwent a caesarean section
(98 of 304 women in the induction group (32%) and 103 of 314 women in the
eb

eb

eb
expectant-­management group (33%); relative risk 0.99, 95% confidence interval
(CI) 0.87–1.14) or in the percentage of women who had a vaginal delivery with the
m

m
use of forceps or vacuum (115 of 304 women (38%) and 104 of 314 women (33%),
respectively; relative risk 1.30, 95% CI 0.96–1.77). There were no maternal or infant
deaths and no significant between-­group differences in the women’s experience of
t

t
e

ne

ne
childbirth or in the frequency of adverse maternal or neonatal outcomes.
.n

e.

e.
References
e
re

fre

re
RCOG. Induction of labour at term in older mothers. RCOG Scientific Impact Paper No. 34.
February 2013.
sf

sf
ks
Walker KF, Bugg GJ, Macpherson M, et al. Randomized trial of labor induction in women
k

k
35 years of age or older. New England Journal of Medicine 2016;374:813–22.
oo

oo

oo
eb

eb

eb
238. Answer   D 70–80%
m

m
Explanation
See Box 2 in the reference article.
Reference
t

et
ne

ne

Revell K, Morrish P. Headaches in pregnancy. The Obstetrician & Gynaecologist

n
2014;16:179–84.
e.

e.

e.
fre

fre

re
239. Answer   B  Improved success rate
f
ks

ks

ks
Explanation
A Cochrane meta-­analysis compared analysis of lactate and pH in fetal scalp
oo

oo

oo

blood during labour and found no differences in fetal/neonatal outcome or


eb

eb

eb

operative interventions but a significantly higher success rate with lactate


compared with pH (risk ratio 1.10; 95% CI 1.08–1.12). Only two trials were
m

included in this meta-­analysis, and approximately 90% of the cases were from one
randomised controlled trial. However, in a recently published large observational
study, where the above clinical guidelines were used, the FBS frequency was 11%,
t

t
ne

ne

ne

and out of these 9% were acidaemic (>4.8 mmol/l). This implies that only 1% of
all deliveries had an FBS lactate indicating operative/instrumental delivery.
e.

e.

e.

Reference
re

re

fre

RCOG. Is it time for UK obstetricians to accept fetal scalp lactate as an alternative to scalp
sf

f
ks

ks

pH? RCOG Scientific Impact Paper 47. January 2015.


k
oo

oo

oo

240. Answer   E Tachycardia


eb

eb

eb

Explanation
m

Atosiban has a better maternal cardiac adverse effect profile than β-­mimetics
in terms of tachycardia (5.5% versus 75.5%), chest pain (1.1% versus 4.8%) and
palpitations (2.2% versus 15.6%). Pulmonary oedema occurred in two women in
the β-­mimetic group.
t

et

t
ne

ne
n
e.

e.

e.
t

t
Management of labour 131

ne

ne

ne
e.

e.

e.
re

fre

fre
Reference
f Groom KM, Bennett PR. Tocolysis for the treatment of preterm labour – a clinically based
ks

ks

ks
review. The Obstetrician & Gynaecologist 2004;6:1.
oo

oo

oo
EMQs
eb

eb

eb
241. Answer   E  Assess in 2 hours
m

m
242. Answer   I  Oxytocin (Syntocinon) infusion with dose increments every 30
minutes
t

t
e

ne

ne
D  Assess in 1 hour
.n

243. Answer  

e.

e.
e
re

fre

re
244. Answer   I  Oxytocin (Syntocinon) infusion with dose increments every 30
minutes
sf

sf
ks
k

k
Explanation
oo

oo

oo
If a delay in the established first stage is suspected, assess all aspects of progress in
labour when diagnosing delay, including:
eb

eb

eb
• Cervical dilation of <2 cm in 4 hours for first labours
m

m
• Cervical dilation of <2 cm in 4 hours or a slowing in the progress of labour for
second or subsequent labours.
Whether or not a woman has agreed to an amniotomy, advise all women
t

et
ne

ne

with suspected delay in the established first stage of labour to have a vaginal

n
examination 2 hours later, and diagnose delay if progress is <1 cm.
e.

e.

e.
If oxytocin is used, ensure that the time between increments of the dose is no
fre

fre

re
more frequent than every 30 minutes. Increase oxytocin until there are four to five
contractions every 10 minutes. f
ks

ks

ks
For a nulliparous woman, suspect delay if progress (in terms of rotation and/or
oo

oo

oo

descent of the presenting part) is inadequate after 1 hour of active second stage.
Offer a vaginal examination and then offer amniotomy if the membranes are
eb

eb

eb

intact.
m

For a multiparous woman, suspect delay if progress (in terms of rotation


and/or descent of the presenting part) is inadequate after 30 minutes of active
second stage. Offer a vaginal examination and then offer amniotomy if the
t

t
membranes are intact.
ne

ne

ne

References
e.

e.

e.

NICE. Intrapartum care for healthy women and babies. NICE Clinical Guideline (CG190).
re

re

fre

Updated February 2017.


sf

RCOG. Operative vaginal delivery. RCOG GTG No. 26. January 2011.
ks

ks
k
oo

oo

oo

245. Answer   E  Continuous one-to-one support


eb

eb

eb

Explanation
Evidence suggests that pain perception is strongly influenced by the attitude and
m

behaviour of the woman’s caregiver, which is probably the single most important
factor in a woman’s perception of pain during labour and childbirth.
t

et

t
ne

ne
n
e.

e.

e.
t

t
132 Module 10

ne

ne

ne
e.

e.

e.
re

fre

fre
246. Answer   J Pethidine

f
Explanation
ks

ks

ks
Diamorphine should be used in preference to pethidine for analgesia in labour.
oo

oo

oo
Pethidine is metabolised to norpethidine, which is known to be epileptogenic
when administered in high doses to patients with normal renal function.
eb

eb

eb
Pethidine should therefore be avoided or used with caution.
m

m
247. Answer   K Remifentanil
Explanation
t

t
e

ne

ne
Widespread national and international implementation has been hampered by
.n

concerns over the safety of remifentanil patient-­controlled analgesia (PCA).

e.

e.
e

Respiratory depression is reported in up to 32% of patients and 5% encounter


re

fre

re
oxygen saturations <90%. In a UK survey, 11% of respondents reported critical
sf

sf
incidences relating to the use of remifentanil and respiratory depression.
ks
k

k
A comprehensive list of relative contraindications to remifentanil PCA is yet
oo

oo

oo
to be established but could include morbid obesity, clinical features of a chest
infection and women in their first pregnancy who are at risk of prolonged labour.
eb

eb

eb
References
m

m
Alleemudder DI, Kuponiyi Y, Kuponiyi C, et al. Analgesia for labour: an evidence based
insight for the obstetrician. The Obstetrician & Gynaecologist 2015;17:147–55.
RCOG. Epilepsy in pregnancy. RCOG GTG No. 68. June 2016.
t

et
ne

ne

n
248. Answer   E Normal
e.

e.

e.
Explanation
fre

fre

re
The description given is of early decelerations, which can be considered normal in
f
ks

ks

ks
an otherwise normal CTG.
oo

oo

oo

249. Answer   A Abnormal


eb

eb

eb

Explanation
m

Variable decelerations with any concerning characteristics in >50% of


contractions for 30 minutes (or less if there are any maternal or fetal clinical risk
factors) would be classed as an abnormal feature.
t

t
ne

ne

ne

250. Answer   J Suspicious


e.

e.

e.

Explanation
re

re

fre

This describes late decelerations, which would be classified as non-­reassuring up


sf

to 30 minutes and abnormal after 30 minutes. One non-­reassuring feature would


ks

ks

make the overall classification suspicious.


k
oo

oo

oo

Reference
eb

eb

eb

NICE. Intrapartum care for healthy women and babies. NICE Clinical Guideline (CG190).
Updated February 2017.
m

m
t

et

t
ne

ne
n
e.

e.

e.
t

t
ne

ne

ne
e.

e.

e.
re

fre

fre
Management of delivery
Module
f
ks

ks

ks
11
oo

oo

oo
eb

eb

eb
m

m
t

t
e

ne

ne
SBAs
.n

e.

e.
e

251. A woman delivered her first baby spontaneously 40 minutes ago and had oxytocin
re

fre

re
(Syntocinon) 10 IU intramuscularly for active management of the third stage of
sf

sf
labour. The placenta has still not delivered, with no signs of separation. She is not
ks
k

k
bleeding, has intravenous access in situ and is haemodynamically stable.
oo

oo

oo
What would be the appropriate action?
eb

eb

eb
A. Manual removal of the placenta
B. Oxytocin (Syntocinon) 20 IU in 20 ml of saline into the umbilical vein
m

m
C. Oxytocin (Syntocinon) 40 IU intravenous infusion at 125 ml/hour
D. Oxytocin (Syntocinon) 5 IU intravenously
E. Oxytocin/ergometrine 5 IU/500 μg (Syntometrine) intramuscularly
t

et
ne

ne

n
252. A 31-year-old woman presents in preterm labour at 34 weeks of gestation. Her
e.

e.

e.
labour progresses quickly and she delivers a baby boy. Both mother and baby
fre

fre

re
appear to be in good health, and the woman requests delayed cord clamping.
What time frame would be recommended for delayed cord clamping in this
f
ks

ks

ks
situation?
oo

oo

oo

A. 10–30 seconds
B. 30 seconds–3 minutes
eb

eb

eb

C. 4–6 minutes
m

D. 6–10 minutes
E. Do not recommend delayed cord clamping

253. A woman in the first stage of labour is diagnosed with inadequate progress. The
t

t
ne

ne

ne

CTG trace is classified as suspicious, and a plan is made to conduct a category 2


caesarean section.
e.

e.

e.

Within how many minutes from the decision should the baby be delivered?
re

re

fre

A. 30 minutes
sf

f
ks

ks

B. 45 minutes
k

C. 60 minutes
oo

oo

oo

D. 75 minutes
eb

eb

eb

E. 90 minutes
m

133
t

et

t
ne

ne
n
e.

e.

e.
t

t
134 Module 11

ne

ne

ne
e.

e.

e.
re

fre

fre
254. A woman has been pushing in the second stage of labour for 2 hours. The head
is two-­fifths palpable abdominally, at −1 station, with caput and moulding. A
f
ks

ks

ks
decision is taken to proceed to delivery by caesarean section.
oo

oo

oo
Which complication is more likely for this woman when compared with a
caesarean section in the first stage of labour?
eb

eb

eb
A. Bladder injury
m

m
B. Blood loss >1000 ml
C. Intraoperative uterine trauma
D. Perinatal asphyxia
E. Venous thromboembolism
t

t
e

ne

ne
.n

255. What is the recommended uterotonic regime for routine management of the third

e.

e.
e

stage at caesarean section?


re

fre

re
A. Oxytocin (Syntocinon) 10 IU intravenously
sf

sf
ks
B. Oxytocin (Syntocinon) 5 IU intravenously
k

k
C. Oxytocin (Syntocinon) 5 IU intravenously with 40 IU oxytocin infusion
oo

oo

oo
D. Oxytocin/ergometrine 5 IU/500 μg (Syntometrine) intramuscularly
eb

eb

eb
E. Oxytocin/ergometrine 5 IU/500 μg (Syntometrine) with 40 IU oxytocin
(Syntocinon) infusion
m

m
256. What is the most common reason for litigation following a shoulder dystocia?
A. Birth asphyxia
t

et
ne

ne

B. Brachial plexus injury

n
C. Clavicular fracture
e.

e.

e.
D. Humeral fracture
fre

fre

re
E. Maternal trauma
f
ks

ks

ks
257. What proportion of women who are planning a vaginal breech birth will require
oo

oo

oo

an emergency caesarean section?


eb

eb

eb

A. 10%
B. 20%
m

C. 30%
D. 40%
E. 50%
t

t
ne

ne

ne

258. For women with a breech presentation after 39 weeks of gestation, how many
e.

e.

e.

times higher is the perinatal mortality rate associated with a planned vaginal
re

re

fre

breech birth compared with an elective caesarean section?


sf

A. Two times
ks

ks

B. Three times
k
oo

oo

oo

C. Four times
D. Five times
eb

eb

eb

E. Ten times
m

m
t

et

t
ne

ne
n
e.

e.

e.
t

t
Management of delivery 135

ne

ne

ne
e.

e.

e.
re

fre

fre
259. What is the single best predictor of a successful vaginal birth after a caesarean
section?
f
ks

ks

ks
A. Gestation when in labour
oo

oo

oo
B. History of a previous vaginal birth
C. Maternal age
eb

eb

eb
D. Scar thickness measured by ultrasound
m

m
E. Type of caesarean section

260. Following a prolonged second stage of labour, a woman is taken to theatre for a
trial of forceps delivery. Examination in theatre reveals the fetal head to be one-­fifth
t

t
e

ne

ne
palpable per abdomen. On vaginal examination, the station of the fetal head is +1.
.n

How would the forceps delivery be classified?

e.

e.
e

A. High
re

fre

re
B. Unclassifiable
sf

sf
ks
C. Low
k

k
D. Mid
oo

oo

oo
E. Outlet
eb

eb

eb
m

m
t

et
ne

ne

n
e.

e.

e.
fre

fre

f re
ks

ks

ks
oo

oo

oo
eb

eb

eb
m

m
t

t
ne

ne

ne
e.

e.

e.
re

re

fre
sf

f
ks

ks
k
oo

oo

oo
eb

eb

eb
m

m
t

et

t
ne

ne
n
e.

e.

e.
t

t
136 Module 11

ne

ne

ne
e.

e.

e.
re

fre

fre
EMQs
f
ks

ks

ks
Options for questions 261–265
oo

oo

oo
A Attach fetal scalp electrode
eb

eb

eb
B Delivery by caesarean section
C Digital examination of the vagina and cervix
m

m
D Fetal scalp blood sampling
E Induction of labour
t

t
F Intermittent auscultation of the fetal heartbeat
e

ne

ne
.n

G Magnesium sulfate infusion

e.

e.
e

H Nifedipine for tocolysis


re

fre

re
I Nitrazine test
sf

sf
J Rescue cervical cerclage
ks
k

k
K Ritodrine infusion
oo

oo

oo
L Test for insulin-­like growth factor-­binding protein-­1 (IGFBP-1)
eb

eb

eb
M Ultrasound scan
m

m
Each of the following clinical scenarios relates to management of delivery. For each
clinical scenario, select the single most appropriate management option from the list
t

et
above. Each option may be used once, more than once or not at all.
ne

ne

n
261. A 45-year-old woman, a mother of two children, has been admitted with
e.

e.

e.
symptoms suggestive of preterm rupture of membranes 12 hours ago at 29 weeks
fre

fre

re
of gestation. She complains of intermittent abdominal pain. A sterile speculum
examination of the vagina is inconclusive because of laxity of the vaginal wall, f
ks

ks

ks
which obscures the view of the cervix. There is no obvious collection or pooling
oo

oo

oo

of amniotic fluid in the vagina. The uterus appears irritable and the fetus is in
cephalic presentation.
eb

eb

eb

262. A 28-year-old woman in her third pregnancy at 32 + 3 weeks of gestation is


m

in labour following the spontaneous onset of uterine contractions 2 days ago.


A vaginal examination reveals intact forewaters, and a fully effaced and 7 cm
dilated cervix with the fetal head at 0 station. Her body mass index (BMI) is 32
t

t
ne

ne

ne

kg/m2 and the CTG has been reassuring since admission. In the last hour, it has
become difficult to interpret the CTG because of difficulty in monitoring the fetal
e.

e.

e.

heartbeat due to many episodes of loss of contact.


re

re

fre
sf

263. A woman in her first pregnancy is admitted to the labour ward with severe
ks

ks

pre-­eclampsia at 29 weeks of gestation. She has commenced treatment with


k
oo

oo

oo

intravenous labetalol. Her uterine contraction frequency is about three every 10


minutes and the fetus is in cephalic presentation with the fetal head three-­fifths
eb

eb

eb

palpable abdominally. A vaginal examination reveals a fully effaced and 4 cm


m

dilated cervix.
t

et

t
ne

ne
n
e.

e.

e.
t

t
Management of delivery 137

ne

ne

ne
e.

e.

e.
re

fre

fre
264. A 35-year-old woman with a history of recurrent miscarriage and no living
children was admitted to the ward a week ago with symptoms of threatened
f
ks

ks

ks
preterm labour at 26 weeks of gestation. She was treated with tocolytics, and
oo

oo

oo
prophylactic steroids were administered. She is now at 27 + 3 weeks of gestation
and is very anxious because of the prospects of poor prognosis following a preterm
eb

eb

eb
birth. There has been no uterine activity over the last 5 days. A vaginal speculum
examination reveals intact membranes and a 50% effaced cervix dilated to 5 cm. A
m

m
vaginal swab taken a week ago has shown no growth of any pathological microbes.

265. A 28-year-old woman, the mother of three children, presents with symptoms
t

t
of threatened preterm labour at 30 weeks of gestation. She gives a history
e

ne

ne
.n

of a previous preterm birth at 32 weeks of gestation. Following a speculum

e.

e.
examination, the fetal fibronectin concentration is found to be 50 ng/ml. A
e
re

fre

re
transvaginal ultrasound scan reveals the cervical length to be 15 mm.
sf

sf
ks
Options for questions 266–268
k

k
oo

oo

oo
A Delivery of the posterior arm
eb

eb

eb
B Downward traction following delivery of the head
C Kiwi delivery
m

m
D Manual rotation of the fetal head
E Primary caesarean section
t

et
F Rotational forceps
ne

ne

n
G Rubin maneouvre
e.

e.

e.
H Traction forceps in occipitoanterior position
fre

fre

re
I Traction forceps in occipitoposterior position

f
ks

ks

ks
J Vaginal breech delivery
K Vaginal disimpaction of the fetal head
oo

oo

oo

L Ventouse delivery
eb

eb

eb
m

For each of the following childbirth-­related complications, choose the single most
likely method of delivery or manoeuvre associated with that complication. Each option
may be used once, more than once or not at all.
t

t
ne

ne

ne

266. Subgaleal haematoma.


e.

e.

e.

267. Obstetric anal sphincter injuries (OASIS).


re

re

fre

268. Reversion to malposition.


sf

f
ks

ks
k
oo

oo

oo
eb

eb

eb
m

m
t

et

t
ne

ne
n
e.

e.

e.
t

t
138 Module 11

ne

ne

ne
e.

e.

e.
re

fre

fre
Options for questions 269–270

f
ks

ks

ks
A Carbetocin
oo

oo

oo
B Carboprost (Hemabate)
C Ergometrine
eb

eb

eb
D Misoprostol
m

m
E Oxytocin (Syntocinon) 5 IU intramuscularly
F Oxytocin (Syntocinon) 10 IU intramuscularly
G Oxytocin (Syntocinon) 5 IU intravenous injection
t

t
e

ne

ne
H Oxytocin (Syntocinon) 10 IU intravenous injection
.n

e.

e.
I Oxytocin (Syntocinon) infusion
e
re

fre

re
J Oxytocin/ergometrine (Syntometrine)
K Physiological third stage
sf

sf
ks
k

k
L Uterine massage
oo

oo

oo
From the list of options above, choose the most appropriate uterotonic agent for each
eb

eb

eb
of the following clinical scenarios? Each option may be used once, more than once or not
m

m
at all.

269. A low-­risk woman in her first pregnancy has an uncomplicated labour and
delivers by spontaneous vaginal delivery.
t

et
ne

ne

n
270. A woman with gestational diabetes, but no other medical problems, has just
e.

e.

e.
delivered twins. Twin 1 was delivered by forceps due to a delay in the second stage
fre

fre

re
of labour. Twin 2 delivered spontaneously.
f
ks

ks

ks
oo

oo

oo
eb

eb

eb
m

m
t

t
ne

ne

ne
e.

e.

e.
re

re

fre
sf

f
ks

ks
k
oo

oo

oo
eb

eb

eb
m

m
t

et

t
ne

ne
n
e.

e.

e.
t

t
Management of delivery 139

ne

ne

ne
e.

e.

e.
re

fre

fre
Answers
f
ks

ks

ks
SBAs
oo

oo

oo
251. Answer  A  Manual removal of the placenta
eb

eb

eb
Explanation
With respect to a retained placenta:
m

m
• Do not use umbilical vein agents if the placenta is retained
• Do not use intravenous oxytocic agents routinely to deliver a retained placenta
• Give intravenous oxytocic agents if the placenta is retained and the woman is
t

t
e

ne

ne
bleeding excessively.
.n

e.

e.
If the placenta is retained and there is concern about the woman’s condition:
e
re

fre

re
• Offer a vaginal examination to assess the need to undertake manual removal
of the placenta
sf

sf
ks
• Explain that this assessment can be painful and advise her to have analgesia.
k

k
oo

oo

oo
Do not carry out uterine exploration or manual removal of the placenta
without an anaesthetic.
eb

eb

eb
Reference
m

m
NICE. Intrapartum care for healthy women and babies. NICE Clinical Guideline (CG190).
Updated February 2017.
t

et
ne

ne

252. Answer  B  30 seconds–3 minutes

n
Explanation
e.

e.

e.
If a preterm baby (born vaginally or by caesarean section) needs to be
fre

fre

re
moved away from the mother for resuscitation, or there is significant maternal
f
ks

ks

ks
bleeding:
• Consider milking the cord and
oo

oo

oo

• Clamp the cord as soon as possible.


eb

eb

eb

Wait at least 30 seconds, but no longer than 3 minutes, before clamping the
m

cord of preterm babies if the mother and baby are stable. Position the baby at or
below the level of the placenta before clamping the cord.
Reference
t

t
ne

ne

ne

NICE. Preterm labour and birth. NICE Guideline (NG25). November 2015.
e.

e.

e.

253. Answer  D  75 minutes


re

re

fre

Explanation
sf

f
ks

ks

Perform a category 2 caesarean section in most situations within 75 minutes of


k

making the decision.


oo

oo

oo

Reference
eb

eb

eb

NICE. Caesarean section. NICE Clinical Guideline (CG132). August 2012.


m

m
t

et

t
ne

ne
n
e.

e.

e.
t

t
140 Module 11

ne

ne

ne
e.

e.

e.
re

fre

fre
254. Answer  B  Blood loss >1000 ml
f
Explanation
ks

ks

ks
In the referenced study, compared with caesarean section in the first stage of
oo

oo

oo
labour, women undergoing caesarean section at the second stage were 4.6 times
eb

eb

eb
more likely to have composite intraoperative complications (95% confidence
interval (CI) 2.7–7.9, P < 0.001), 3.1 times more likely to have blood loss >1000 ml
m

m
(95% CI 1.3–7.4, P = 0.01), and 2.9 times more likely to have complications of
intraoperative trauma (relative risk 2.6, P < 0.001) and infants with perinatal
asphyxia (relative risk 1.5, P < 0.05).
t

t
e

ne

ne
Reference
.n

e.

e.
Tempest N, Navaratnam K, Hapangama DK. Management of delivery when malposition of
e

the fetal head complicates the second stage of labour. The Obstetrician & Gynaecologist
re

fre

re
2015;17:273–80.
sf

sf
ks
k

k
255. Answer  B  Oxytocin (Syntocinon) 5 IU intravenously
oo

oo

oo
Explanation
eb

eb

eb
Oxytocin 5 IU by slow intravenous injection should be used during caesarean
section to encourage contraction of the uterus and decrease blood loss.
m

Reference
m
NICE. Caesarean section. NICE Clinical Guideline (CG132). August 2012.
t

et
ne

ne

n
256. Answer  B  Brachial plexus injury
e.

e.

e.
Explanation
fre

fre

re
Neonatal brachial plexus injury is the most common cause for litigation related to
f
ks

ks

ks
shoulder dystocia and is the third most litigated obstetric-­related complication in
the UK.
oo

oo

oo

Reference
eb

eb

eb

RCOG. Shoulder dystocia. RCOG GTG No. 42. March 2012.


m

257. Answer  D 40%


Explanation
t

t
ne

ne

ne

Women should be informed that maternal complications are least with successful
vaginal birth; a planned caesarean section carries a higher risk, but the risk is
e.

e.

e.

highest with emergency caesarean section, which is needed in approximately 40%


re

re

fre

of women planning a vaginal breech birth.


sf

f
ks

ks

Reference
k
oo

oo

oo

RCOG. Management of breech presentation. RCOG GTG No. 20b. March 2017.
eb

eb

eb
m

m
t

et

t
ne

ne
n
e.

e.

e.
t

t
Management of delivery 141

ne

ne

ne
e.

e.

e.
re

fre

fre
258. Answer  C  Four times

f Explanation
ks

ks

ks
Women should be informed that when planning delivery for a breech baby, the
oo

oo

oo
risk of perinatal mortality is approximately 0.5 in 1000 with caesarean section
after 39 + 0 weeks of gestation and approximately 2.0 in 1000 with a planned
eb

eb

eb
vaginal breech birth.
m

m
Reference
RCOG. Management of breech presentation. RCOG GTG No. 20b. March 2017.
t

t
e

ne

ne
259. Answer  B  History of a previous vaginal birth
.n

e.

e.
Explanation
e
re

fre

re
Women with one or more previous vaginal births should be informed that a
previous vaginal delivery, particularly previous vaginal birth after a caesarean
sf

sf
ks
(VBAC), is the single best predictor of successful VBAC and is associated with a
k

k
oo

oo

oo
planned VBAC success rate of 85–90%.
eb

eb

eb
Reference
RCOG. Birth after previous caesarean birth. RCOG GTG No. 45. October 2015.
m

260. Answer 
Explanation
D Mid
m
t

et
ne

ne

For operative vaginal delivery, the classification of a midcavity forceps delivery is

n
when the fetal head is no more than one-­fifth palpable per abdomen.
e.

e.

e.
The leading point of the skull is above station plus 2 cm but not above the
fre

fre

re
ischial spines.
f
ks

ks

ks
Reference
oo

oo

oo

RCOG. Operative vaginal delivery. RCOG GTG No. 26. February 2011.
eb

eb

eb

EMQs
m

261. Answer  L  Test for insulin-­like growth factor-­binding protein-­1 (IGFBP-1)


Explanation
A test for IGFBP-­1 has a high sensitivity and specificity for detection of PROM.
t

t
ne

ne

ne

Digital examination risks the introduction of infection. It is important to know


e.

e.

e.

about PROM before making any further management plans.


re

re

fre

262. Answer  F  Intermittent auscultation of the fetal heartbeat


sf

f
ks

ks

Explanation
k
oo

oo

oo

Intermittent auscultation is an option for fetal monitoring in established preterm


labour.
eb

eb

eb
m

m
t

et

t
ne

ne
n
e.

e.

e.
t

t
142 Module 11

ne

ne

ne
e.

e.

e.
re

fre

fre
263. Answer  G  Magnesium sulfate infusion

f
Explanation
ks

ks

ks
The magnesium sulfate is for neuroprotection as the woman seems to be
oo

oo

oo
in established preterm labour but is going to take a few hours before she
delivers.
eb

eb

eb
m

m
264. Answer  J  Rescue cervical cerclage
Explanation
With this history, it will be worth taking the small risk associated with rescue
t

t
e

ne

ne
cerclage. The uterus is relaxed and there is no evidence of infection.
.n

e.

e.
e

265. Answer  H  Nifedipine for tocolysis


re

fre

re
Explanation
sf

sf
ks
The fibronectin test is negative and the cervical length is short, although it is not
k

k
<15 mm. However, her history of a previous preterm birth is the greatest risk
oo

oo

oo
factor and she should be offered treatment with nifedipine.
eb

eb

eb
References
m

m
NICE. Preterm labour and birth. NICE Guidelines (NG25). November 2015.
RCOG. The investigation and management of the small-for-gestational age fetus. RCOG
GTG No. 31. March 2013.
t

et
ne

ne

266. Answer  L  Ventouse delivery

n
e.

e.

e.
Explanation
fre

fre

re
Although the ventouse has been popular for managing fetal malposition in recent
f
years, it is associated with a significantly higher risk of failure (22.4% versus
ks

ks

ks
3.7% for Kielland forceps) and increased admissions to the neonatal unit (12.1%
oo

oo

oo

versus 10.3% for Kielland forceps). Scalp lacerations and cephalohaematoma are
common, and subaponeurotic, subgaleal and intracranial haemorrhage also occur.
eb

eb

eb

267. Answer  F  Rotational forceps


m

Explanation
Of all operative vaginal delivery methods, Kielland forceps are associated with
t

t
the highest risk of OASIS. However, it has been demonstrated that this risk can be
ne

ne

ne

mitigated and OASIS rates kept lower than national UK level 5 with high levels of
e.

e.

e.

training and supervision. Kielland forceps require advanced operator skills, including
both decision-­making and technical components. If those requirements are fulfilled,
re

re

fre

Kielland forceps are likely to be the most effective method to manage malposition of
sf

f
ks

ks

the fetal head in the second stage of labour and reduce primary emergency caesarean
k

section and emergency caesarean section for failed rotational ventouse.


oo

oo

oo
eb

eb

eb
m

m
t

et

t
ne

ne
n
e.

e.

e.
t

t
Management of delivery 143

ne

ne

ne
e.

e.

e.
re

fre

fre
268. Answer  D  Manual rotation of the fetal head

f Explanation
ks

ks

ks
Problems associated with this method in particular include the baby rotating back
oo

oo

oo
to a malposition between manual rotation and forceps or ventouse being applied,
resulting in potential inaccurate positioning of the forceps/ventouse and the
eb

eb

eb
risk of cord prolapse following disimpaction of the head, leading to an obstetric
m

m
emergency.
Reference
Tempest N, Navaratnam K, Hapangama DK. Management of delivery when malposition of
t

t
e

ne

ne
the fetal head complicates the second stage of labour. The Obstetrician & Gynaecologist
.n

2015;17:273–80.

e.

e.
e
re

fre

re
269. Answer  F  Oxytocin (Syntocinon) 10 IU intramuscularly
sf

sf
Explanation
ks
k

k
For women without risk factors for postpartum haemorrhage delivering vaginally,
oo

oo

oo
oxytocin (10 IU by intramuscular injection) is the agent of choice for prophylaxis
in the third stage of labour. A higher dose of oxytocin is unlikely to be beneficial.
eb

eb

eb
m

m
270. Answer  J  Oxytocin/ergometrine (Syntometrine)
Explanation
Oxytocin/ergometrine may be used in the absence of hypertension in women
t

et
at increased risk of haemorrhage as it reduces the risk of minor postpartum
ne

ne

n
haemorrhage (500–1000 ml of blood).
e.

e.

e.
Reference
fre

fre

re
RCOG. Prevention and management of postpartum haemorrhage. RCOG GTG No. 52.
f
ks

ks

ks
December 2016
oo

oo

oo
eb

eb

eb
m

m
t

t
ne

ne

ne
e.

e.

e.
re

re

fre
sf

f
ks

ks
k
oo

oo

oo
eb

eb

eb
m

m
t

et

t
ne

ne
n
e.

e.

e.
t

t
ne

ne

ne
e.

e.

e.
re

fre

fre
Postpartum problems
Module
f
ks

ks

ks
12
oo

oo

oo
eb

eb

eb
m

m
t

t
e

ne

ne
SBAs
.n

e.

e.
e

271. Regarding postpartum family planning, when is the soonest after delivery that a
re

fre

re
copper intrauterine contraceptive device (IUCD) can be inserted?
sf

sf
ks
A. Immediately after the third stage of labour is complete
k

k
B. Just prior to leaving the labour ward/delivery room
oo

oo

oo
C. Just prior to discharge from hospital
eb

eb

eb
D. 1 week postnatally
E. 6 weeks postnatally
m

m
272. According to the published literature, what is the only obstetric factor that is
consistently associated with an increased risk of postpartum psychosis?
t

et
A. Male infant
ne

ne

n
B. Maternal age >45 years
e.

e.

e.
C. Premature delivery
fre

fre

re
D. Primiparity
E. Ventouse delivery
f
ks

ks

ks
273. Following an episiotomy repair under local anaesthetic, a midwife requests a pre-
oo

oo

oo

scription for a non-­steroidal anti-­inflammatory drug (NSAID) for ongoing analgesia.


eb

eb

eb

Which NSAID carries the greatest risk of the development of acute renal failure?
A. Diclofenac
m

B. Ibuprofen
C. Indomethacin
D. Naproxen
t

t
ne

ne

ne

E. Paracetamol
e.

e.

e.

274. A woman who was delivered at 35 weeks of gestation due to pre-­eclampsia has
re

re

fre

ongoing hypertension requiring treatment postpartum. The woman suffers with


asthma but has never been hospitalised.
sf

f
ks

ks

What would be the most appropriate antihypertensive agent?


k
oo

oo

oo

A. Atenolol
B. Bendroflumethiazide
eb

eb

eb

C. Enalapril
m

D. Labetalol
E. Nifedipine

144
t

et

t
ne

ne
n
e.

e.

e.
t

t
Postpartum problems 145

ne

ne

ne
e.

e.

e.
re

fre

fre
275. During a massive postpartum haemorrhage, a woman is found to have a
fibrinogen level of 1.5 g/l.
f
ks

ks

ks
What is the most appropriate product to elevate her fibrinogen levels?
oo

oo

oo
A. Cryoprecipitate
B. Fibrinogen concentrate
eb

eb

eb
C. Fresh frozen plasma
m

m
D. Tranexamic acid
E. Whole blood

276. Following delivery, when is the peak in the absolute risk of venous
t

t
e

ne

ne
thromboembolism?
.n

e.

e.
A. 1–3 weeks
e

B. 4–6 weeks
re

fre

re
C. 6–12 weeks
sf

sf
ks
D. 3–6 months
k

k
E. 6–12 months
oo

oo

oo
eb

eb

eb
277. Methyldopa is often discontinued in the postnatal period due to its association
with postnatal depression.
m

m
What other side effects are often found with this medication in the postpartum
period?
t

et
ne

ne

Postural Flushing Renal Cough Sedation

n
hypotension impairment
e.

e.

e.
A ✓ ✓
fre

fre

re
B ✓ ✓
f
ks

ks

ks
C ✓ ✓
oo

oo

oo

D ✓ ✓
E ✓ ✓
eb

eb

eb
m

278. A 25-year-old woman who previously used the combined oral contraceptive
pill (COCP) has just delivered and has requested contraceptive advice prior to
discharge from hospital. She is fit and well. She has decided not to breastfeed.
t

t
ne

ne

ne

At what point in the postpartum period could she restart the COCP?
e.

e.

e.

A. Immediately
re

re

fre

B. 1 week
C. 3 weeks
sf

f
ks

ks

D. 6 weeks
k

E. 6 months
oo

oo

oo
eb

eb

eb
m

m
t

et

t
ne

ne
n
e.

e.

e.
t

t
146 Module 12

ne

ne

ne
e.

e.

e.
re

fre

fre
279. A 38-year-old woman has had her first baby by caesarean section 7 days ago. She
is seen by her community midwife and appears to be low in mood with reduced
f
ks

ks

ks
appetite and is sent to hospital for review. There is no suicidal ideation or thoughts
oo

oo

oo
to harm her baby, and she is diagnosed with mild depression. The woman’s case
notes indicate that her mother suffered from bipolar disorder and committed
eb

eb

eb
suicide.
What would be the most appropriate action?
m

m
A. Admit for observation
B. Commence fluoxetine
C. Discharge to GP
t

t
e

ne

ne
D. Offer counselling
.n

E. Refer to perinatal mental health services

e.

e.
e
re

fre

re
280. Following an eclamptic fit, a woman who is 2 days postpartum develops an
sf

sf
aspiration pneumonia and, despite intensive care treatment, dies.
ks
k

k
How would this maternal death be classified according to World Health
oo

oo

oo
Organization (WHO) criteria?
eb

eb

eb
A. Coincidental
B. Direct
m

m
C. Fortuitous
D. Indirect
E. Late
t

et
ne

ne

n
e.

e.

e.
fre

fre

f re
ks

ks

ks
oo

oo

oo
eb

eb

eb
m

m
t

t
ne

ne

ne
e.

e.

e.
re

re

fre
sf

f
ks

ks
k
oo

oo

oo
eb

eb

eb
m

m
t

et

t
ne

ne
n
e.

e.

e.
t

t
Postpartum problems 147

ne

ne

ne
e.

e.

e.
re

fre

fre
EMQs
f
ks

ks

ks
Options for questions 281–283
oo

oo

oo
A Cerebral tumour
eb

eb

eb
B Cerebral venous sinus thrombosis
C Dural puncture headache
m

m
D Idiopathic intracranial hypertension
E Meningitis
t

t
F Migraine
e

ne

ne
.n

G Posterior reversible encephalopathy syndrome

e.

e.
e

H Pre-eclampsia
re

fre

re
I Reversible cerebral vasoconstriction syndrome (RCVS)
sf

sf
J Simple headache
ks
k

k
K Sinusitis
oo

oo

oo
L Subarachnoid haemorrhage
eb

eb

eb
M Tension headache
m

m
For each of the following clinical descriptions of headaches in the postpartum period,
select the most likely cause from the list of options above. Each option may be used once,
more than once or not at all.
t

et
ne

ne

n
281. A 23-year-old woman with a body mass index (BMI) of 45 kg/m2 is now 36
e.

e.

e.
hours postemergency caesarean section for failure to progress in labour. She is
fre

fre

re
complaining of a frontal headache that developed over the last few hours. The
headache is worse every time she stands up to care for her baby. A neurological
f
ks

ks

ks
examination and ophthalmic fundoscopy are both normal.
oo

oo

oo

282. A 33-year-old woman with a BMI of 32 kg/m2 is 4 days postnormal vaginal


eb

eb

eb

delivery on the midwife-­led unit. She complains of a pulsating left-­sided headache


that has developed over the last 30 minutes. She is nauseous and has vomited
m

twice, and is asking to lie still in a darkened room. Observations are normal, and a
neurological examination and ophthalmic fundoscopy are normal.
t

t
283. A 31-year-old woman, now 14 days postdelivery, presents for the fourth time
ne

ne

ne

postpartum with a sudden-­onset headache, like being hit over the head, associated
e.

e.

e.

with vomiting, an aversion to light and blurred vision. She had a CT scan after the
first episode that was normal.
re

re

fre
sf

f
ks

ks
k
oo

oo

oo
eb

eb

eb
m

m
t

et

t
ne

ne
n
e.

e.

e.
t

t
148 Module 12

ne

ne

ne
e.

e.

e.
re

fre

fre
Options for questions 284–286

f
ks

ks

ks
A Amoxicillin
oo

oo

oo
B Benzylpenicillin
C Cefuroxime
eb

eb

eb
D Clarithromycin
m

m
E Clindamycin
F Co-amoxiclav
G Erythromycin
t

t
e

ne

ne
H Gentamicin
.n

e.

e.
I Meropenem
e
re

fre

re
J Metronidazole
K Piperacillin
sf

sf
ks
k

k
L Vancomycin
oo

oo

oo
For each of the following clinical scenarios, select the single most appropriate choice
eb

eb

eb
of antibiotic based on the information provided. Each option may be chosen once, more
m

m
than once or not at all.

284. A 26-year-old woman is diagnosed with severe pyelonephritis 2 days after a


caesarean section. Culture of a midstream urine sample grows an extended-­
t

et
ne

ne

spectrum β-­lactamase (ESBL)-producing organism. She has no known allergies,

n
and is given a single dose of gentamicin. What antibiotic should also be
e.

e.

e.
commenced?
fre

fre

285. A 24-year-old woman is 6 days postpartum. She complains of having had a


f re
ks

ks

ks
sore throat for the last 2 days, and culture of a throat swab has confirmed the
oo

oo

oo

presence of group A Streptococcus. It is suspected that she is developing toxic


shock syndrome. Which antibiotic should be used in order to switch off exotoxin
eb

eb

eb

production?
m

286. A 32-year-old woman develops sepsis 2 days after a caesarean section. She is a
nurse and is known to carry methicillin-­resistant Staphylococcus aureus (MRSA).
Which antibiotic should be used to ensure that MRSA is treated effectively?
t

t
ne

ne

ne
e.

e.

e.
re

re

fre
sf

f
ks

ks
k
oo

oo

oo
eb

eb

eb
m

m
t

et

t
ne

ne
n
e.

e.

e.
t

t
Postpartum problems 149

ne

ne

ne
e.

e.

e.
re

fre

fre
Options for questions 287–290

f
ks

ks

ks
A Alcohol withdrawal
oo

oo

oo
B Amphetamine overdose
C Baby blues
eb

eb

eb
D Bipolar disorder
m

m
E Borderline personality disorder
F Depression
G Early-onset severe panic disorder
t

t
e

ne

ne
H Generalised anxiety disorder
.n

e.

e.
I Obsessive compulsive disorder
e
re

fre

re
J Postnatal depression
K Postpartum confusional state
sf

sf
ks
k

k
L Puerperal psychosis
oo

oo

oo
M Schizophrenia
eb

eb

eb
For each of the following clinical scenarios, choose the single most appropriate diag-
m

m
nosis from the list of options above. Each option may be chosen once, more than once or
not at all.

287. A 23-year-old woman is seen 4 days postnatally in obstetric triage after self-­
t

et
ne

ne

presenting after cutting her wrists at 03:00 hours. There is no ongoing bleeding

n
from her injuries. She reports that she had got upset as her boyfriend had been
e.

e.

e.
out and came home late. They are now joking with each other and appear very
fre

fre

re
affectionate.
f
ks

ks

ks
288. A 31-year-old woman is 3 days postdelivery and presents with anxiety. She
oo

oo

oo

appears very frightened and exhibits paranoid ideation that her family is trying to
take her baby away. She has a history of schizophrenia but has been stable without
eb

eb

eb

treatment for 5 years.


m

289. A 40-year-old woman is 2 weeks postnatal after her first pregnancy. She had a
caesarean section and she has no past medical or mental health history. She has
not been sleeping, and has been feeling very low and crying a lot. The baby has
t

t
ne

ne

ne

been readmitted due to excessive weight loss.


e.

e.

e.

290. A 24-year-old woman has delivered her first baby by caesarean section. She is
re

re

fre

seen by the midwife 3 days postpartum with emotional instability, insomnia and
agitation. She seems to improve a little over the following 72 hours but continues
sf

f
ks

ks

to have similar symptoms intermittently over the next 6 weeks. The baby is well
k
oo

oo

oo

looked after.
eb

eb

eb
m

m
t

et

t
ne

ne
n
e.

e.

e.
t

t
150 Module 12

ne

ne

ne
e.

e.

e.
re

fre

fre
Answers
f
ks

ks

ks
SBAs
oo

oo

oo
271. Answer   A  Immediately after the third stage of labour is complete
eb

eb

eb
Explanation
IUCDs can be inserted following expulsion of the placenta. It is most convenient
m

m
and best practice to insert them immediately after the placenta has been delivered.
Reference
t

t
RCOG. Best practice in postpartum family planning. RCOG Best Practice Paper 1. June
e

ne

ne
2015.
.n

e.

e.
e

272. Answer   D Primiparity


re

fre

re
sf

sf
Explanation
ks
k

k
An increased risk of postpartum psychosis has been reported with a number of
oo

oo

oo
obstetric factors including: primiparity, pregnancy and delivery complications,
delivery by caesarean section, having a female baby and a shorter gestation period.
eb

eb

eb
However, findings are consistent only for primiparity.
m

m
Reference
Di Florio A, Smith S, Jones I. Postpartum psychosis. The Obstetrician & Gynaecologist
2013;15:145–50.
t

et
ne

ne

n
273. Answer   D Naproxen
e.

e.

e.
Explanation
fre

fre

re
Comparative data regarding the most ‘renal-­friendly’ NSAIDs are limited.
f
ks

ks

ks
Naproxen has been shown to carry the highest risk of acute renal failure.
Indomethacin has been associated with more nephrotoxicity than other NSAIDs
oo

oo

oo

in terms of nephritis and hyperkalaemia, and fenoprofen has been associated with
eb

eb

eb

the highest risk of nephrosis.


m

Reference
Wiles KS, Banerjee A. Acute kidney injury in pregnancy and the use of non-­steroidal anti-­
inflammatory drugs. The Obstetrician & Gynaecologist 2016;18:127–35.
t

t
ne

ne

ne

274. Answer   E Nifedipine


e.

e.

e.

Explanation
re

re

fre

A suggested regimen might be labetalol (provided there is no history of asthma)


sf

with second- and third-­line agents of calcium antagonist and an angiotensin-


ks

ks

converting-enzyme (ACE) inhibitor (such as enalapril), respectively.


k
oo

oo

oo

Reference
eb

eb

eb

Smith M, Waugh J, Nelson-Piercy C. Management of postpartum hypertension. The


Obstetrician & Gynaecologist 2013;15:45–50
m

m
t

et

t
ne

ne
n
e.

e.

e.
t

t
Postpartum problems 151

ne

ne

ne
e.

e.

e.
re

fre

fre
275. Answer   A Cryoprecipitate

f Explanation
ks

ks

ks
Cryoprecipitate should be used for fibrinogen replacement.
oo

oo

oo
The appropriate fibrinogen intervention trigger or target level is unknown. A
pragmatic view based on available evidence is that, during continuing postpartum
eb

eb

eb
haemorrhage, cryoprecipitate or fibrinogen concentrate should be used to
m

m
maintain a fibrinogen level of at least 2 g/l, even if the prothrombin time (PT)
or activated partial thromboplastin time (aPTT) is normal. Fibrinogen loss can
be replaced by cryoprecipitate or fibrinogen concentrate, although fibrinogen
t

t
concentrate is not licensed for acquired hypofibrinogenaemia in the UK.
e

ne

ne
.n

Reference

e.

e.
e

RCOG. Prevention and management of postpartum haemorrhage. RCOG GTG No. 52.
re

fre

re
December 2016.
sf

sf
ks
k

k
276. Answer   A  1–3 weeks
oo

oo

oo
Explanation
eb

eb

eb
The relative risk postpartum is 5-­fold higher compared with antepartum, and a
systematic review of the risk of postpartum venous thromboembolism (VTE)
m

m
found that the risk varied from 21- to 84-­fold from the baseline non-­pregnant,
non-­postpartum state in studies that included an internal reference group. The
absolute risk peaked in the first 3 weeks postpartum (421 per 100,000 person-­
t

et
ne

ne

years; 22-­fold increase in risk).

n
e.

e.

e.
Reference
fre

fre

re
RCOG. Reducing the risk of venous thromboembolism during pregnancy and the
puerperium. RCOG GTG No. 37a. April 2015.
f
ks

ks

ks
E  Postural hypotension and sedation
oo

277. Answer  
oo

oo

Explanation
eb

eb

eb

While methyldopa remains a safe option for treatment of hypertension in the


m

postnatal period, particularly in women who have had good antenatal control with the
agent, most authorities advise that it should be discontinued because of its maternal
side effects, in particular sedation, postural hypotension and postnatal depression.
t

t
ne

ne

ne

Reference
Smith M, Waugh J, Nelson-Piercy C. Management of postpartum hypertension. The
e.

e.

e.

Obstetrician & Gynaecologist 2013;15:45–50.


re

re

fre
sf

278. Answer   C  3 weeks


ks

ks
k

Explanation
oo

oo

oo

Women who are not breastfeeding may start the COCP at 3 weeks postpartum
eb

eb

eb

unless they have additional risk factors for VTE, in which case they should not
start the COCP until 6 weeks after childbirth.
m

Reference
RCOG. Best practice in postpartum family planning. RCOG Best Practice Paper No. 1. June
t

et

2015.
ne

ne
n
e.

e.

e.
t

t
152 Module 12

ne

ne

ne
e.

e.

e.
re

fre

fre
279. Answer   E  Refer to perinatal mental health services

f
Explanation
ks

ks

ks
The good practice guidelines developed by the RCOG suggest that the following
oo

oo

oo
scenarios are indications for referral to specialised perinatal mental health
services where available, or otherwise to general psychiatry services:
eb

eb

eb
• Current severe psychiatric symptoms
m

m
• A history of serious postpartum illness, bipolar disorder or schizophrenia
• On complex psychotropic medication schemes.
Moreover, the guidelines suggest that referral should be considered for those
t

t
e

ne

ne
with moderate symptoms developed in late pregnancy or early postpartum,
.n

or with mild symptoms and a family history of bipolar disorder or puerperal

e.

e.
e

psychosis.
re

fre

re
Reference
sf

sf
ks
Di Florio A, Smith S, Jones I. Postpartum psychosis. The Obstetrician & Gynaecologist
k

k
oo

oo

oo
2013;15:145–50.
eb

eb

eb
280. Answer   B Direct
m

m
Explanation
Direct maternal deaths are classified as deaths resulting from obstetric
complications of the pregnant state (pregnancy, labour and puerperium), from
t

et
interventions, omissions, incorrect treatment, or from a chain of events resulting
ne

ne

from any of the above.

n
e.

e.

e.
Reference
fre

fre

re
Knight M, Nair M, Tuffnell D, et al. (eds.) on behalf of MBRRACE-­UK. Saving Lives,
f
Improving Mothers’ Care: Surveillance of Maternal Deaths in the UK 2012–14 and Lessons
ks

ks

ks
Learned to Inform Maternity Care From the UK and Ireland Confidential Enquiries into
oo

oo

oo

Maternal Deaths and Morbidity 2009–14. Oxford: National Perinatal Epidemiology


Unit, University of Oxford, 2016.
eb

eb

eb

EMQs
m

281. Answer   C  Dural puncture headache


Explanation
t

t
ne

ne

ne

Puncture of the dura occurs in 0.5–2.5% of epidurals. If accidental dural puncture


occurs with an epidural needle, there is a 70–80% chance of a postdural puncture
e.

e.

e.

headache. The headache is usually in the fronto‐occipital regions and radiates to


re

re

fre

the neck; it is characteristically worse on standing and typically develops 24–48


sf

hours postpuncture.
ks

ks
k

Conservative management includes hydration and simple analgesics.


oo

oo

oo

Untreated, the headache typically lasts for 7–10 days but can last up to 6 weeks.
An epidural blood patch has a 60–90% cure rate.
eb

eb

eb
m

m
t

et

t
ne

ne
n
e.

e.

e.
t

t
Postpartum problems 153

ne

ne

ne
e.

e.

e.
re

fre

fre
282. Answer   F Migraine

f Explanation
ks

ks

ks
A migraine is classically:
oo

oo

oo
• Unilateral
eb

eb

eb
• Pulsating
• Builds up over minutes to hours
m

m
• Moderate to severe in intensity
• Associated with nausea and/or vomiting and/or sensitivity to light and/or
sensitivity to sound
t

t
e

ne

ne
• Disabling
.n

• Aggravated by routine physical activity.

e.

e.
e

Migraine is classified by the presence or absence of aura. Pregnancy can alter


re

fre

re
migraine with aura and trigger attacks of aura without headache as a result of high
sf

sf
plasma concentrations of oestrogen.
ks
k

k
oo

oo

oo
283. Answer   I  Reversible cerebral vasoconstriction syndrome (RCVS)
eb

eb

eb
Explanation
RCVS is a cerebrovascular disorder associated with multifocal arterial
m

m
constriction and dilation. It has a significant association with the
postpartum period. RCVS is characterised by recurrent sudden-­onset
and severe headaches over 1–3 weeks, often accompanied by nausea,
t

et
ne

ne

vomiting, photophobia, confusion and blurred vision. Diagnosis requires

n
the demonstration of diffuse arterial beading on cerebral angiography
e.

e.

e.
with resolution within 1–3 months. It is in the differential of a postpartum
fre

fre

re
thunderclap headache often made after subarachnoid haemorrhage has been
excluded but the headaches recur.
f
ks

ks

ks
Reference
oo

oo

oo

Morrish P. Headaches in pregnancy. The Obstetrician & Gynaecologist 2014;16:179–84.


eb

eb

eb

284. Answer   I Meropenem


m

Explanation
Treat acute pyelonephritis with gentamicin and meropenem.
t

t
ne

ne

ne

285. Answer   E Clindamycin


e.

e.

e.

Explanation
re

re

fre

Clindamycin is not nephrotoxic and switches off the production of superantigens


and other exotoxins.
sf

f
ks

ks
k

286. Answer   L Vancomycin


oo

oo

oo

Explanation
eb

eb

eb

MRSA may be resistant to clindamycin; hence, if the woman is or is highly likely


m

to be MRSA-­positive, a glycopeptide such as vancomycin or teicoplanin may be


added until sensitivity is known.
Reference
t

et

t
ne

ne
n
e.

e.

e.
t

t
154 Module 12

ne

ne

ne
e.

e.

e.
re

fre

fre
287. Answer   E  Borderline personality disorder

f
Explanation
ks

ks

ks
Borderline personality disorder is characterised by significant instability of
oo

oo

oo
interpersonal relationships, self-­image and mood, and impulsive behaviour. There
is a pattern of sometimes rapid fluctuation from periods of confidence to despair,
eb

eb

eb
with fear of abandonment and rejection, and a strong tendency towards suicidal
m

m
thinking and self-­harm. Transient psychotic symptoms, including brief delusions
and hallucinations, may also be present. It is also associated with substantial
impairment of social, psychological and occupational functioning and quality of
t

t
life. People with borderline personality disorder are particularly at risk of suicide.
e

ne

ne
.n

e.

e.
288. Answer   L  Puerperal psychosis
e
re

fre

re
Explanation
sf

sf
Puerperal psychosis is, by definition, a psychotic illness that arises in a previously
ks
k

k
well woman within a defined period after childbirth. The term covers those
oo

oo

oo
women who have a lifetime first-­onset psychotic illness following childbirth. It
also covers women who have previously had a psychotic illness but have been well
eb

eb

eb
in the years preceding their pregnancy.
m

m
289. Answer   J  Postnatal depression
Explanation
t

et
Severe postnatal depressive illness has an early onset in the first few weeks
ne

ne

n
following birth, but, unlike puerperal psychosis, this tends to be gradual and does
e.

e.

e.
not clearly manifest until 4–6 weeks postpartum or later.
fre

fre

re
The core symptoms are the same as depressive illness at other times. These
include the so-­called ‘biological syndrome’, now referred to as the ‘somatic
f
ks

ks

ks
subtype’, of early morning wakening, diurnal variation of mood, slowing of mental
oo

oo

oo

functioning, impaired concentration, and overvalued ideas of incompetence and


guilt. These are often accompanied by loss of appetite and weight, loss of spontaneity
eb

eb

eb

and enjoyment (anhedonia) and difficulty in coping with the tasks of everyday life.
The baby has been readmitted, suggesting that day-to-day life and care of the
m

newborn has been affected.

290. Answer   C  Baby blues


t

t
ne

ne

ne

Explanation
e.

e.

e.

The ‘blues’ affect 50–80% of all new mothers. This condition is self-­limiting and
lasts for approximately 48 hours, but it can recur periodically over the next 6–8
re

re

fre

weeks, particularly when the mother is very tired.


sf

f
ks

ks
k

References
oo

oo

oo

NICE. Borderline personality disorder: recognition and management. NICE Clinical


Guideline (CG78). January 2009.
eb

eb

eb

Oates M. Postnatal affective disorders. Part 1: an introduction. The Obstetrician &


Gynaecologist 2008;10:145–50.
m

m
t

et

t
ne

ne
n
e.

e.

e.
t

t
ne

ne

ne
e.

e.

e.
re

fre

fre
Gynaecological problems
Module
f
ks

ks

ks
13
oo

oo

oo
eb

eb

eb
m

m
t

t
e

ne

ne
SBAs
.n

e.

e.
e

291. A 52-year-old woman with a body mass index (BMI) of 32 kg/m2 presents
re

fre

re
with irregular vaginal bleeding. An ultrasound scan demonstrates a thickened
sf

sf
endometrium, and an endometrial biopsy confirms endometrial hyperplasia
ks
k

k
without atypia. A levonorgestrel-­releasing intrauterine system (LNG-­IUS) is
oo

oo

oo
inserted.
What follow-­up should be arranged?
eb

eb

eb
A. Annual endometrial biopsies for life
m

m
B. Endometrial biopsy at 3 and 6 months and discharge if normal
C. Endometrial biopsy at 6 and 12 months and discharge if normal
D. Endometrial biopsy at 6 and 12 months and then annually for life
t

et
E. No follow-­up required once the LNG-­IUS is inserted
ne

ne

n
e.

e.

e.
292. During an MRI scan for back pain, a 68-year-old woman was found to have
fre

fre

re
an incidental finding of a 4 cm simple ovarian cyst. This was confirmed by a
transvaginal ultrasound scan. She had a hysterectomy at the age of 38 years for
f
ks

ks

ks
heavy menstrual bleeding but her ovaries were conserved. Her serum CA125 level
oo

oo

oo

was 17 IU/l.
What is the most appropriate management?
eb

eb

eb

A. Arrange a follow-­up ultrasound scan in 4 months


m

B. Laparotomy and bilateral salpingo-oophorectomy


C. Perform an ultrasound-­guided aspiration
D. Reassure and discharge
E. Refer to multidisciplinary team meeting
t

t
ne

ne

ne

293. A healthy 40-year-old woman presents to the gynaecology clinic with heavy
e.

e.

e.

menstrual bleeding. History, examination and an ultrasound scan suggest the


re

re

fre

presence of adenomyosis, but no other pathology.


sf

f
ks

ks

What would be considered the first-­line treatment?


k

A. Cyclical oral progestogens


oo

oo

oo

B. Endometrial ablation
eb

eb

eb

C. Insertion of an LNG-IUS
D. Total abdominal hysterectomy
m

E. Total laparoscopic hysterectomy

155
t

et

t
ne

ne
n
e.

e.

e.
t

t
156 Module 13

ne

ne

ne
e.

e.

e.
re

fre

fre
294. A 35-year-old woman presents to the gynaecology clinic with heavy menstrual
bleeding and opts to have an LNG-­IUS inserted. She is warned of the possibility of
f
ks

ks

ks
changes to her bleeding pattern.
oo

oo

oo
For how long should she be advised to persevere with any unwanted changes to
her bleeding pattern in order to see the maximum benefits of treatment?
eb

eb

eb
A. Three cycles
m

m
B. Six cycles
C. Nine cycles
D. 12 cycles
E. No specified time
t

t
e

ne

ne
.n

295. What is the most common subtype of vulval lichen planus to cause vulval symptoms?

e.

e.
e

A. Classical
re

fre

re
B. Erosive
sf

sf
ks
C. Hypertrophic
k

k
D. Hypotrophic hyperpigmented
oo

oo

oo
E. Non-classical
eb

eb

eb
296. What are the three most prevalent symptoms in women with peritoneal
m

m
endometriosis?

Chronic Cyclical Deep Dysmenorrhoea Heavy Infertility


pelvic intestinal dyspareunia menstrual
t

et
ne

ne

pain complaints bleeding

n
e.

e.

e.
A ✓ ✓ ✓
fre

fre

re
B ✓ ✓ ✓
C ✓ ✓
f

ks

ks

ks
D ✓ ✓ ✓
oo

oo

oo

E ✓ ✓ ✓
eb

eb

eb

297. A 38-year-old woman presents to the gynaecology clinic with a 6-­month history
m

of amenorrhoea and hot flushes. A pregnancy test is negative. Her serum follicle-­
stimulating hormone (FSH) is measured and found to be 45 IU/l.
What further result would confirm a diagnosis of premature ovarian
t

t
ne

ne

ne

insufficiency (POI)?
A. Anti-Müllerian hormone (AMH) in 2 weeks’ time <1.1 pmol/l
e.

e.

e.

B. Further FSH in 2 weeks’ time >50 IU/l


re

re

fre

C. Further FSH in 4 weeks’ time >25 IU/l


sf

D. Immediate AMH <1.1 pmol/l


ks

ks
k

E. Immediate luteinising hormone (LH) level >50 IU/l


oo

oo

oo
eb

eb

eb
m

m
t

et

t
ne

ne
n
e.

e.

e.
t

t
Gynaecological problems 157

ne

ne

ne
e.

e.

e.
re

fre

fre
298. Ulipristal acetate has previously been used for the management of uterine fibroids.
What proportion of women using this treatment will develop progesterone
f
ks

ks

ks
receptor modulator-­associated endometrial changes (PAEC)?
oo

oo

oo
A. 10%
B. 25%
eb

eb

eb
C. 40%
m

m
D. 60%
E. 90%

299. Where in the vagina is a transverse vaginal septum most commonly found?
t

t
e

ne

ne
A. Equal distribution throughout the vagina
.n

e.

e.
B. Introitus
e

C. Lower vagina
re

fre

re
D. Mid-vagina
sf

sf
ks
E. Upper vagina
k

k
oo

oo

oo
300. Which hormone is implicated in the development of endometrial polyps in obese
eb

eb

eb
postmenopausal women?
A. Androstenedione
m

m
B. Oestradiol
C. Oestriol
D. Oestrone
t

et
ne

ne

E. Testosterone

n
e.

e.

e.
301. What are the five ultrasound parameters used to classify ovarian cysts in the risk
fre

fre

re
of malignancy index (RMI)?
f
ks

ks

ks
Ascites Bilateral Metastases Mixed Multilocular Positive Solid
oo

oo

oo

masses hyper- and colour areas


hypoechoic Doppler
eb

eb

eb

areas flow
m

A ✓ ✓ ✓ ✓ ✓
B ✓ ✓ ✓ ✓ ✓
C ✓ ✓ ✓ ✓ ✓
t

t
ne

ne

ne

D ✓ ✓ ✓ ✓ ✓
E ✓ ✓ ✓ ✓ ✓
e.

e.

e.
re

re

fre
sf

f
ks

ks
k
oo

oo

oo
eb

eb

eb
m

m
t

et

t
ne

ne
n
e.

e.

e.
t

t
158 Module 13

ne

ne

ne
e.

e.

e.
re

fre

fre
302. A 50-year-old woman is referred to the gynaecology clinic with symptoms of
abdominal bloating, loss of appetite and lower abdominal discomfort. Her serum
f
ks

ks

ks
CA125 had been measured at the GP practice and was found to be 40 IU/l.
oo

oo

oo
An urgent ultrasound scan is arranged, but this is reported as normal with no
evidence of uterine or ovarian pathology. The patient’s symptoms continue.
eb

eb

eb
What is the most appropriate management?
m

m
A. Arrange urgent CT scan of pelvis
B. Arrange urgent MRI scan of pelvis
C. Assess for other clinical causes of her symptoms
D. Refer to the gynaecology multidisciplinary team meeting in view of her raised
t

t
e

ne

ne
CA125
.n

E. Repeat CA125 in 3 months’ time

e.

e.
e
re

fre

re
303. What proportion of women who have treatment with gonadotropin-­releasing
sf

sf
ks
hormone (GnRH) analogues for the symptoms of premenstrual syndrome (PMS)
k

k
will report an improvement in symptoms?
oo

oo

oo
A. 5–12%
eb

eb

eb
B. 22–26%
C. 31–38%
m

m
D. 45–52%
E. 60–75%
t

et
ne

ne

304. What is the most common benign tumour in females?

n
A. Breast fibroadenoma
e.

e.

e.
B. Lipoma
fre

fre

re
C. Renal adenoma
D. Thyroid adenoma f
ks

ks

ks
E. Uterine leiomyoma
oo

oo

oo

305. Which pharmacological treatment is most commonly used as a first-­line therapy


eb

eb

eb

in the treatment of endometriosis in women with chronic pelvic pain?


m

A. Aromatase inhibitors
B. Combined oral contraceptive pill (COCP)
C. GnRH analogue
t

t
D. LNG-IUS
ne

ne

ne

E. Oral progestogens
e.

e.

e.
re

re

fre
sf

f
ks

ks
k
oo

oo

oo
eb

eb

eb
m

m
t

et

t
ne

ne
n
e.

e.

e.
t

t
Gynaecological problems 159

ne

ne

ne
e.

e.

e.
re

fre

fre
306. A 63-year-old woman with no significant past medical history presents with
vaginal soreness, dyspareunia and dysuria, and is found to have atrophic vaginitis.
f
ks

ks

ks
She is commenced on topical vaginal oestriol cream and reports a marked
oo

oo

oo
improvement in her symptoms.
What advice should be given regarding the use of systemic progestogen for
eb

eb

eb
endometrial protection?
m

m
A. Depot medroxyprogesterone acetate every 12 weeks indefinitely
B. Insertion of LNG-­IUS for duration of treatment
C. No progestogen required
D. Oral progestogen for 14 days per month
t

t
e

ne

ne
E. Transdermal progestogen for 14 days per month
.n

e.

e.
e

307. What proportion of postmenopausal women with bleeding have co-­existing


re

fre

re
cervical and endometrial polyps?
sf

sf
ks
A. 4–7%
k

k
B. 14–17%
oo

oo

oo
C. 24–27%
eb

eb

eb
D. 34–37%
E. 44–47%
m

m
308. A woman returns to the clinic for a review following an endometrial ablation
procedure and concomitant sterilisation. She is dissatisfied due to ongoing
t

et
ne

ne

symptoms.

n
What are the main symptoms associated with postablation tubal sterilisation
e.

e.

e.
syndrome (PATSS)?
fre

fre

f re
Constant Cyclical Deep Premature Vaginal
ks

ks

ks
unilateral pelvic dyspareunia menopause spotting
oo

oo

oo

pelvic pain pain


A ✓ ✓
eb

eb

eb

B ✓ ✓
m

C ✓ ✓
D ✓ ✓
E ✓ ✓
t

t
ne

ne

ne
e.

e.

e.

309. During an endometrial ablation procedure, what depth of myometrium needs to


re

re

fre

be destroyed to prevent endometrial regeneration from basal glands?


sf

f
ks

ks

A. 1 mm
k

B. 2 mm
oo

oo

oo

C. 3 mm
eb

eb

eb

D. 4 mm
E. 5 mm
m

m
t

et

t
ne

ne
n
e.

e.

e.
t

t
160 Module 13

ne

ne

ne
e.

e.

e.
re

fre

fre
310. What is the most common congenital abnormality of fusion in the female genital
tract?
f
ks

ks

ks
A. Bicornuate uterus
oo

oo

oo
B. Mayer–Rokitansky–Küster–Hauser (MRKH) syndrome
C. Transverse vaginal septum
eb

eb

eb
D. Uterine septum
m

m
E. Uterus didelphys

311. A 48-year-old woman presents with irregular heavy menstrual bleeding, and
an endometrial biopsy confirms endometrial hyperplasia without atypia. The
t

t
e

ne

ne
woman has a normal BMI and the only significant past medical history is a
.n

laparoscopic sterilisation at the age of 35. An LNG-­IUS is inserted for 6 months,

e.

e.
e

after which point it is removed. An endometrial biopsy at that time is normal, and
re

fre

re
a subsequent biopsy 6 months later is also normal. The woman continues to have
sf

sf
ks
ongoing bleeding.
k

k
What is the appropriate management?
oo

oo

oo
A. Offer endometrial ablation
eb

eb

eb
B. Offer hysterectomy
C. Reassure and discharge
m

m
D. Re-insert the LNG-­IUS and reassess after 1 year
E. Repeat the endometrial biopsy after 1 year
t

et
ne

ne

312. What is the primary aim of medical interventions in the management of heavy

n
menstrual bleeding?
e.

e.

e.
A. To improve haemoglobin levels
fre

fre

re
B. To improve serum ferritin levels
C. To improve the woman’s quality of life f
ks

ks

ks
D. To reduce blood loss as assessed by a menstrual calendar
oo

oo

oo

E. To reduce blood loss as assessed by pictorial charts


eb

eb

eb

313. The most common cause of postmenopausal bleeding is atrophic vaginitis/


m

endometritis.
What is the second most common cause?
A. Cervical polyp
t

t
B. Endometrial carcinoma
ne

ne

ne

C. Endometrial hyperplasia
e.

e.

e.

D. Endometrial polyp
re

re

fre

E. Exogenous oestrogens
sf

f
ks

ks
k
oo

oo

oo
eb

eb

eb
m

m
t

et

t
ne

ne
n
e.

e.

e.
t

t
Gynaecological problems 161

ne

ne

ne
e.

e.

e.
re

fre

fre
314. A 55-year-old woman presents with postmenopausal bleeding. A pipelle
endometrial biopsy is performed, and she is found to have endometrial
f
ks

ks

ks
hyperplasia without atypia. It is explained to her that she will need to have
oo

oo

oo
treatment for 6 months followed by a repeat biopsy.
Which treatment is most likely to result in a histologically normal endometrium
eb

eb

eb
after 6 months?
m

m
A. Conservative management
B. Continuous oral medroxyprogesterone acetate (MPA)
C. Cyclical oral MPA
D. GnRH analogues
t

t
e

ne

ne
E. Insertion of an LNG-IUS
.n

e.

e.
e

315. What proportion of circulating testosterone in the female is produced in the


re

fre

re
ovary?
sf

sf
ks
A. 5%
k

k
B. 10%
oo

oo

oo
C. 15%
eb

eb

eb
D. 20%
E. 25%
m

m
t

et
ne

ne

n
e.

e.

e.
fre

fre

f re
ks

ks

ks
oo

oo

oo
eb

eb

eb
m

m
t

t
ne

ne

ne
e.

e.

e.
re

re

fre
sf

f
ks

ks
k
oo

oo

oo
eb

eb

eb
m

m
t

et

t
ne

ne
n
e.

e.

e.
t

t
162 Module 13

ne

ne

ne
e.

e.

e.
re

fre

fre
EMQs
f
ks

ks

ks
Options for questions 316–318
oo

oo

oo
A Androgen insensitivity syndrome
eb

eb

eb
B Congenital adrenal hyperplasia
C Imperforate hymen
m

m
D Kallmann syndrome
E Mayer–Rokitansky–Küster–Hauser (MRKH) syndrome
t

t
F Polycystic ovarian syndrome (PCOS)
e

ne

ne
.n

G Swyer syndrome

e.

e.
e

H Transverse vaginal septum


re

fre

re
I Triple X syndrome (47,XXX)
sf

sf
J Turner syndrome
ks
k

k
K Vaginal atresia
oo

oo

oo
eb

eb

eb
For each of the following clinical descriptions, what is the most likely diagnosis from
the option list above? Each option may be used once, more than once or not at all.
m

m
316. A 16-year-old girl presents with primary amenorrhoea. She has normal secondary
sexual development. Clinical examination reveals a short, blind-­ending vagina.
t

et
Her karyotype is 46,XX.
ne

ne

n
317. A 15-year-old girl presents with amenorrhoea and abdominal pain. Clinical
e.

e.

e.
examination reveals a pelvic mass and vaginal inspection demonstrates a pink
fre

fre

re
bulging membrane.
f
ks

ks

ks
318. A 16-year-old girl presents with amenorrhoea and delayed puberty.
oo

oo

oo

Phenotypically, she is female and the uterus is present. Her serum FSH level is
elevated. Her karyotype is 46,XY.
eb

eb

eb
m

m
t

t
ne

ne

ne
e.

e.

e.
re

re

fre
sf

f
ks

ks
k
oo

oo

oo
eb

eb

eb
m

m
t

et

t
ne

ne
n
e.

e.

e.
t

t
Gynaecological problems 163

ne

ne

ne
e.

e.

e.
re

fre

fre
Options for questions 319–321

f
ks

ks

ks
A Chronic vulvovaginal candidiasis
oo

oo

oo
B Eczema
C Idiopathic
eb

eb

eb
D Lichen planus
m

m
E Lichen sclerosus
F Lichen simplex
G Paget’s disease
t

t
e

ne

ne
H Psoriasis
.n

e.

e.
I Recurrent herpes infection
e
re

fre

re
J Systemic lupus erythematosus
K Vestibulodynia
sf

sf
ks
k

k
L Vulval intraepithelial neoplasia (VIN)
oo

oo

oo
M Vulvodynia
eb

eb

eb
For each of the following clinical descriptions, what is the most likely diagnosis from
m

m
the option list above? Each option may be used once, more than once or not at all.

319. A 58-year-old woman presents with dyspareunia, vulval itching and soreness.
A biopsy is taken from atrophic areas on the vulva. The histology report states:
t

et
ne

ne

‘There is epidermal atrophy, hyperkeratosis with subepidermal hyalinisation of

n
collagen and lichenoid infiltrate.’
e.

e.

e.
fre

fre

re
320. A 23-year-old woman with schizophrenia presents with vaginal itch and soreness.
f
On examination, there are marked excoriations and lichenification. Most of her
ks

ks

ks
pubic hair is absent.
oo

oo

oo

321. A 40-year-old woman presents with vaginal soreness, discharge and dyspareunia.
eb

eb

eb

On examination, the mucosal surfaces of the vagina are eroded. At the edges of
the erosions, the epithelium is mauve/grey, and Wickham’s striae are noted.
m

m
t

t
ne

ne

ne
e.

e.

e.
re

re

fre
sf

f
ks

ks
k
oo

oo

oo
eb

eb

eb
m

m
t

et

t
ne

ne
n
e.

e.

e.
t

t
164 Module 13

ne

ne

ne
e.

e.

e.
re

fre

fre
Options for questions 322–324

f
ks

ks

ks
A Combined oral contraceptive pill (COCP)
oo

oo

oo
B Daily progestogen on days 5–25 of cycle
C Daily progestogen in the luteal phase
eb

eb

eb
D Danazol
m

m
E Etamsylate
F GnRH agonist
G Hysteroscopic myomectomy
t

t
e

ne

ne
H Levonorgestrel-releasing intrauterine system (LNG-IUS)
.n

e.

e.
I Mefenamic acid
e
re

fre

re
J No treatment required
K Open myomectomy
sf

sf
ks
k

k
L Second-generation endometrial ablation
oo

oo

oo
M Subtotal hysterectomy
eb

eb

eb
N Total abdominal hysterectomy
O Tranexamic acid
m

m
P Transcervical resection of endometrium

For each of the following clinical scenarios, what would be the most appropriate treat-
t

et
ne

ne

ment option from the list above? Each option may be used once, more than once or not at all.

n
e.

e.

e.
322. A 38-year-old woman with heavy menstrual bleeding is seen in the gynaecology
fre

fre

re
clinic. Her medical history is unremarkable other than mild renal impairment.
The woman is keen to avoid hormonal treatments.
f
ks

ks

ks
323. A 35-year-old woman attends the gynaecology clinic with heavy menstrual bleeding
oo

oo

oo

and dysmenorrhoea. Her history is otherwise unremarkable, but a pelvic examination


eb

eb

eb

demonstrates an enlarged uterus with possible fibroids. A pelvic ultrasound scan is


arranged, but the woman is desperate to start treatment immediately.
m

324. A 37-year-old woman with a BMI of 38 kg/m2 returns to the clinic for a review.
She is para 2+0 and her family is complete. Her husband has had a vasectomy. Six
months earlier, she had an LNG-­IUS inserted for heavy menstrual bleeding, but
t

t
ne

ne

ne

this has recently been removed by the GP due to persistent vaginal bleeding. Prior
to this she had already tried tranexamic acid and mefenamic acid without effect. A
e.

e.

e.

previous pelvic examination suggested an enlarged uterus (equivalent to 9 weeks


re

re

fre

of gestation), and a subsequent ultrasound scan demonstrated multiple small


sf

f
ks

ks

(<2 cm) intramural fibroids.


k
oo

oo

oo
eb

eb

eb
m

m
t

et

t
ne

ne
n
e.

e.

e.
t

t
Gynaecological problems 165

ne

ne

ne
e.

e.

e.
re

fre

fre
Options for questions 325–327

f
ks

ks

ks
A Cervical smear
oo

oo

oo
B Coagulation tests
C CT scan
eb

eb

eb
D Full blood count (FBC)
m

m
E Hysteroscopy
F MRI scan
G No investigation required
t

t
e

ne

ne
H Pelvic ultrasound scan
.n

e.

e.
I Pipelle endometrial biopsy
e
re

fre

re
J Serum ferritin
K Serum follicle-­stimulating hormone (FSH)
sf

sf
ks
k

k
L Serum oestradiol
oo

oo

oo
M Three-dimensional ultrasound scan
eb

eb

eb
N Thyroid function tests
m

m
For each of the following clinical scenarios, what would be the next most appropriate
investigation from the options above? Each option may be used once, more than once or
not at all.
t

et
ne

ne

325. A 39-year-old woman with a BMI of 40 kg/m2 attends the gynaecology clinic with

n
e.

e.

e.
heavy menstrual bleeding. Her medical history is unremarkable and an FBC taken
by the referring GP was normal. Examination is difficult due to the woman’s size.
fre

fre

f
326. A 27-year-old woman attends the GP surgery with heavy menstrual bleeding that
re
ks

ks

ks
is affecting her quality of life. Her history and examination are unremarkable.
oo

oo

oo

327. A 44-year-old woman presents to the clinic with heavy menstrual bleeding and
eb

eb

eb

intermittent but ongoing intermenstrual bleeding. Her smear history is normal.


m

She describes recurrent episodes of vaginal thrush but has no other medical
problems.
t

t
ne

ne

ne
e.

e.

e.
re

re

fre
sf

f
ks

ks
k
oo

oo

oo
eb

eb

eb
m

m
t

et

t
ne

ne
n
e.

e.

e.
t

t
166 Module 13

ne

ne

ne
e.

e.

e.
re

fre

fre
Options for questions 328–330

f
ks

ks

ks
A Cognitive behavioural therapy (CBT)
oo

oo

oo
B Continuous combined hormone replacement therapy (HRT)
C Cyclical norethisterone
eb

eb

eb
D Danazol
m

m
E Drospirenone-containing combined oral contraceptive pill (COCP)
F Evening primrose oil
G GnRH analogue
t

t
e

ne

ne
H LNG-IUS
.n

e.

e.
I Micronised progesterone
e
re

fre

re
J Norethisterone
K Norethisterone-containing COCP
sf

sf
ks
k

k
L Selective serotonin reuptake inhibitors (SSRIs)
oo

oo

oo
M Spironolactone
eb

eb

eb
N Tibolone
O Transdermal oestradiol
m

m
P Vitamin B6

For each of the following clinical scenarios, what would be the most appropriate treat-
t

et
ne

ne

ment option from the list above? Each option may be used once, more than once or not at all.

n
e.

e.

e.
328. A 28-year-old woman presents to clinic with symptoms suggestive of
fre

fre

re
premenstrual syndrome (PMS). She subsequently returns for a review
appointment after 2 months of completing a symptom diary, but the completed
f
ks

ks

ks
diary does not give a clear diagnosis. What treatment should be considered for 3
oo

oo

oo

months to facilitate a definitive diagnosis?


eb

eb

eb

329. An otherwise healthy 30-year-old woman with a BMI of 24 kg/m2 has completed
a symptom diary that is strongly suggestive of a diagnosis of PMS. What would be
m

considered the first-­line pharmaceutical treatment option?


330. A 42-year-old woman with severe PMS opts to have a laparoscopic bilateral
oophorectomy following failed medical management. She decided to retain
t

t
ne

ne

ne

her uterus and has commenced transdermal oestrogen therapy. What would be
considered the optimal first-­line therapy for endometrial protection?
e.

e.

e.
re

re

fre
sf

f
ks

ks
k
oo

oo

oo
eb

eb

eb
m

m
t

et

t
ne

ne
n
e.

e.

e.
t

t
Gynaecological problems 167

ne

ne

ne
e.

e.

e.
re

fre

fre
Options for questions 331 and 332

f
ks

ks

ks
A Atrophic vaginitis
oo

oo

oo
B Bacterial vaginosis
C Candidiasis
eb

eb

eb
D Cervical ectropion
m

m
E Cervical polyp
F Chlamydia infection
G Endometrial polyp
t

t
e

ne

ne
H Fallopian tube carcinoma
.n

e.

e.
I Gonorrhoea
e
re

fre

re
J Physiological discharge
K Trichomoniasis
sf

sf
ks
k

k
L Urethral caruncle
oo

oo

oo
M Vaginal carcinoma
eb

eb

eb
For each of the following clinical scenarios, what is the most likely cause for the vagi-
m

m
nal discharge described from the options above? Each option may be used once, more
than once or not at all.

331. A 26-year-old woman presents with a complaint of recurrent vaginal discharge.


t

et
ne

ne

A gynaecological examination is unremarkable. High vaginal and endocervical

n
swabs are taken for microbiological testing but results are awaited.
e.

e.

e.
fre

fre

re
332. A 35-year-old woman presents to the GP practice with an offensive vaginal
discharge. The discharge is associated with dysuria and vulval itching. f
ks

ks

ks
oo

oo

oo
eb

eb

eb
m

m
t

t
ne

ne

ne
e.

e.

e.
re

re

fre
sf

f
ks

ks
k
oo

oo

oo
eb

eb

eb
m

m
t

et

t
ne

ne
n
e.

e.

e.
t

t
168 Module 13

ne

ne

ne
e.

e.

e.
re

fre

fre
Options for questions 333–335

f
ks

ks

ks
A Amitriptyline
oo

oo

oo
B Codeine
C Co-dydramol
eb

eb

eb
D Combined oral contraceptive pill (COCP)
m

m
E Danazol
F Fluoxetine
G Gabapentin
t

t
e

ne

ne
H Glycerin suppositories
.n

e.

e.
I GnRH analogue
e
re

fre

re
J Ibuprofen
K Lactulose
sf

sf
ks
k

k
L Mebeverine
oo

oo

oo
M No treatment required
eb

eb

eb
N Paracetamol
O Paroxetine
m

m
P Peppermint water
Q Senna
R Sertraline
t

et
ne

ne

n
e.

e.

e.
For each of the following clinical scenarios, what would be the most appropriate first-­
line pharmacological treatment from the options above? Each option may be used once,
fre

fre

re
more than once or not at all.
f
ks

ks

ks
333. A 22-year-old woman presents with an 8-­month history of significant intermittent
oo

oo

oo

abdominal pain. Her menstrual cycle is irregular. She has noted an increase in
frequency in passing stools. The pain seems to improve after defecation.
eb

eb

eb
m

334. A 20-year-old woman presents with a 2-­year history of chronic pelvic pain. The
pain is present on most days but is worse around the time of menstruation. She
has noted that her stools are more firm than previously. She does not have urinary
frequency.
t

t
ne

ne

ne

335. A 30-year-old woman with a BMI of 25 kg/m2 presents to the clinic with cyclical
e.

e.

e.

chronic pelvic pain. An ultrasound scan is normal. At her insistence, a diagnostic


re

re

fre

laparoscopy is performed, which shows no obvious pathology. She has completed


sf

symptoms charts, which show a distinct cyclical pattern to her symptoms.


ks

ks
k
oo

oo

oo
eb

eb

eb
m

m
t

et

t
ne

ne
n
e.

e.

e.
t

t
Gynaecological problems 169

ne

ne

ne
e.

e.

e.
re

fre

fre
Options for questions 336–338

f
ks

ks

ks
A Combined oral contraceptive pill (COCP)
oo

oo

oo
B GnRH analogue
C Laparoscopic bilateral oophorectomy
eb

eb

eb
D Laparoscopic ovarian cystectomy
m

m
E Laparoscopic unilateral oophorectomy
F Laparoscopy and ovarian cyst aspiration
G Laparotomy and bilateral oophorectomy
t

t
e

ne

ne
H Laparotomy and ovarian cystectomy
.n

e.

e.
I Laparotomy, bilateral oophorectomy and omental biopsy
e
re

fre

re
J No treatment required
K Total abdominal hysterectomy and bilateral salpingo-oophorectomy
sf

sf
ks
k

k
L Ultrasound-guided ovarian cyst aspiration
oo

oo

oo
For each of the following clinical scenarios, what would be the most appropriate manage-
eb

eb

eb
ment from the options listed? Each option may be used once, more than once or not at all.
m

m
336. A 45-year-old woman has an incidental finding of an ovarian cyst during an MRI
scan for lower back pain. The following month, an ultrasound scan is arranged by
the GP, which reports a simple 4 cm cyst in the left ovary. The remainder of the
t

et
ne

ne

pelvic ultrasound scan is normal.

n
e.

e.

e.
337. A 26-year-old woman with a BMI of 30 kg/m2 who has never been pregnant
presents with an ache in the right side of the pelvis. A transvaginal ultrasound
fre

fre

re
scan reports a 10 cm complex mass in the right adnexa with appearances
f
ks

ks

ks
compatible with a dermoid cyst. Tumour markers are all normal.
oo

oo

oo

338. A 49-year-old woman with a BMI of 25 kg/m2 presents with left-­sided abdominal
pain and bloating. She is having a regular menstrual cycle. A detailed history
eb

eb

eb

reveals that she has urinary frequency. A transvaginal ultrasound scan reports a
m

simple 8 cm cyst in the left ovary. Tumour markers are normal.


t

t
ne

ne

ne
e.

e.

e.
re

re

fre
sf

f
ks

ks
k
oo

oo

oo
eb

eb

eb
m

m
t

et

t
ne

ne
n
e.

e.

e.
t

t
170 Module 13

ne

ne

ne
e.

e.

e.
re

fre

fre
Options for questions 339 and 340

f
ks

ks

ks
A Adrenocortical adenocarcinoma
oo

oo

oo
B Adrenocortical adenoma
C Congenital adrenal hyperplasia
eb

eb

eb
D Cushing’s disease
m

m
E Cushing’s syndrome
F Exogenous androgens
G Gestational hyperandrogenism
t

t
e

ne

ne
H Idiopathic
.n

e.

e.
I Ovarian hyperthecosis
e
re

fre

re
J Polycystic ovarian syndrome (PCOS)
K Sertoli–Leydig cell tumour
sf

sf
ks
k

k
oo

oo

oo
For each of the following clinical scenarios, what is the most likely cause of hyper­
androgenism from the options listed? Each option may be used once, more than once
eb

eb

eb
or not at all.
m

m
339. A 25-year-old woman presents with excessive facial hair and acne vulgaris.

340. A 65-year-old woman presents with persistent coarse facial hair. Blood tests reveal
t

et
she has hyperandrogenaemia.
ne

ne

n
e.

e.

e.
fre

fre

f re
ks

ks

ks
oo

oo

oo
eb

eb

eb
m

m
t

t
ne

ne

ne
e.

e.

e.
re

re

fre
sf

f
ks

ks
k
oo

oo

oo
eb

eb

eb
m

m
t

et

t
ne

ne
n
e.

e.

e.
t

t
Gynaecological problems 171

ne

ne

ne
e.

e.

e.
re

fre

fre
Answers
f
ks

ks

ks
SBAs
oo

oo

oo
291. Answer   C  Endometrial biopsy at 6 and 12 months and discharge if normal
eb

eb

eb
Explanation
Endometrial surveillance should be arranged at a minimum of 6-­monthly
m

m
intervals, although review schedules should be individualised and responsive
to changes in a woman’s clinical condition. At least two consecutive 6-­monthly
negative biopsies should be obtained prior to discharge. In women at higher risk
t

t
e

ne

ne
of relapse, such as women with a BMI of ≥35 kg/m2 or those treated with oral
.n

progestogens, 6-­monthly endometrial biopsies are recommended. Once two

e.

e.
e

consecutive negative endometrial biopsies have been obtained, then long-­term


re

fre

re
follow-­up should be considered with annual endometrial biopsies.
sf

sf
ks
Reference
k

k
oo

oo

oo
RCOG. Management of endometrial hyperplasia. RCOG GTG No. 67. February 2016.
eb

eb

eb
292. Answer   A  Arrange a follow-­up ultrasound scan in 4 months
m

m
Explanation
Simple, unilateral, unilocular ovarian cysts of <5 cm in diameter have a low risk
of malignancy. It is recommended that, in the presence of normal serum CA125
t

et
levels, they be managed conservatively. Numerous studies have looked at the
ne

ne

risk of malignancy in ovarian cysts, comparing ultrasound morphology with

n
e.

e.

e.
either histology at subsequent surgery or by close follow-­up of those women
managed conservatively. The risk of malignancy in these studies of cysts that
fre

fre

re
are <5 cm, unilateral, unilocular and echo-­free with no solid parts or papillary
f
ks

ks

ks
formations is <1%. In addition, >50% of these cysts will resolve spontaneously
within 3 months. Thus, it is reasonable to manage these cysts conservatively, with
oo

oo

oo

a follow-­up ultrasound scan for cysts of 2–5 cm, a reasonable interval being 4
eb

eb

eb

months. This, of course, depends on the views and symptoms of the woman and
on the gynaecologist’s clinical assessment.
m

Reference
RCOG. Ovarian cysts in postmenopausal women. RCOG GTG No. 34. July 2016.
t

t
ne

ne

ne

293. Answer   C  Insertion of an LNG-IUS


e.

e.

e.

Explanation
re

re

fre

Consider an LNG-­IUS as the first treatment for heavy menstrual bleeding in


sf

women with suspected or diagnosed adenomyosis.


ks

ks
k
oo

oo

oo

Reference
NICE. Heavy menstrual bleeding: assessment and management. NICE Guideline (NG88).
eb

eb

eb

March 2018.
m

m
t

et

t
ne

ne
n
e.

e.

e.
t

t
172 Module 13

ne

ne

ne
e.

e.

e.
re

fre

fre
294. Answer   B  Six cycles

f
Explanation
ks

ks

ks
Women offered an LNG-­IUS should be advised of anticipated changes in the
oo

oo

oo
bleeding pattern, particularly in the first few cycles and possibly lasting >6
months. They should therefore be advised to persevere for at least six cycles to see
eb

eb

eb
the benefits of the treatment.
m

m
Reference
NICE. Heavy menstrual bleeding: assessment and management. NICE Guideline (NG88).
March 2018.
t

t
e

ne

ne
.n

295. Answer   B Erosive

e.

e.
e
re

fre

re
Explanation
The anogenital lesions of lichen planus may be divided into three main groups
sf

sf
ks
according to their clinical presentation:
k

k
oo

oo

oo
• Classical
• Hypertrophic
eb

eb

eb
• Erosive.
m

m
Erosive is the most common subtype to cause vulval symptoms.
Reference
Edwards SK, Bates CM, Lewis F, Sethi G, Grover D. 2014 UK national guideline on the
t

et
ne

ne

management of vulval conditions. International Journal of STD & AIDS 2015;26:611–24.

n
e.

e.

e.
296. Answer   B  Chronic pelvic pain, deep dyspareunia and dysmenorrhoea
fre

fre

re
Explanation
f
ks

ks

ks
Dysmenorrhoea was the chief complaint, reported by 62% of women with mainly
peritoneal endometriosis in a Brazilian study by Bellelis et al. (2010; cited in the
oo

oo

oo

reference article). In the same study, the prevalence of chronic pelvic pain was
eb

eb

eb

57%, deep dyspareunia 55%, cyclic intestinal complaints 48%, infertility 40% and
incapacitating dysmenorrhoea 28%.
m

Reference
ESHRE. Management of women with endometriosis. ESHRE Guideline. September 2013.
t

t
ne

ne

ne

297. Answer   C  Further FSH in 4 weeks’ time >25 IU/l


e.

e.

e.

Explanation
re

re

fre

Although proper diagnostic accuracy in POI is lacking, the guideline development


sf

f
ks

ks

group recommends the following two diagnostic criteria:


k

• Oligo/amenorrhoea for at least 4 months, and


oo

oo

oo

• An elevated FSH level >25 IU/l on two occasions >4 weeks apart.
eb

eb

eb

Reference
m

ESHRE. Management of women with premature ovarian insufficiency. ESHRE Guideline.


December 2015.
t

et

t
ne

ne
n
e.

e.

e.
t

t
Gynaecological problems 173

ne

ne

ne
e.

e.

e.
re

fre

fre
298. Answer   D 60%

f Explanation
ks

ks

ks
Ulipristal acetate use has been found to be associated with benign endometrial
oo

oo

oo
changes termed PAEC. These changes were noted in up to two-­thirds of women
during treatment and resolved within 6 months of discontinuation of treatment.
eb

eb

eb
Reference
m

m
Younas K, Hadoura E, Majoko F, Bunkheila A. A review of evidence-­based management of
uterine fibroids. The Obstetrician & Gynaecologist 2016;18:33–42.
t

t
e

ne

ne
299. Answer   E  Upper vagina
.n

e.

e.
Explanation
e
re

fre

re
The incidence of this is unclear but is probably not greater than 1 in 30,000 to 1 in
50,000. The septae may occur anywhere along the length of the vagina, although
sf

sf
ks
they are classified as upper, mid- and lower, with the upper septae accounting for
k

k
oo

oo

oo
46%, the mid-­vagina 30–40% and the lower vagina 15–20%.
eb

eb

eb
Reference
Edmonds DK, Rose GL. Outflow tract disorders of the female genital tract. The
m

m
Obstetrician & Gynaecologist 2013;15:11–17.

300. Answer   D Oestrone


t

et
ne

ne

Explanation

n
In obese women, it is the increased circulating level of oestrone that is likely to be
e.

e.

e.
the cause of polyp development and growth.
fre

fre

re
Reference
f
ks

ks

ks
Otify M, Fuller J, Ross J, Shaikh H, Johns J. Endometrial pathology in the postmenopausal
oo

oo

oo

woman – an evidence based approach to management. The Obstetrician & Gynaecologist


2015;17:29–38.
eb

eb

eb

301. Answer   E
m

Explanation
RMI is calculated as: RMI = U × M × CA125, where U = 0 (for an ultrasound
t

t
score of 0), U = 1 (for an ultrasound score of 1) or U = 3 (for ultrasound score
ne

ne

ne

of 2–5); M = 3 for all postmenopausal women dealt with by this guideline; and
e.

e.

e.

CA125 is the serum CA125 measurement in IU/ml.


Ultrasound scans score 1 point for each of the following characteristics:
re

re

fre

• Multilocular cyst
sf

f
ks

ks

• Evidence of solid areas


k
oo

oo

oo

• Evidence of metastases
• Presence of ascites
eb

eb

eb

• Bilateral lesions.
m

Reference
RCOG. Ovarian cysts in postmenopausal women. RCOG GTG No. 34. July 2016.
t

et

t
ne

ne
n
e.

e.

e.
t

t
174 Module 13

ne

ne

ne
e.

e.

e.
re

fre

fre
302. Answer   C  Assess for other clinical causes of her symptoms

f
Explanation
ks

ks

ks
A woman with normal serum CA125 level (<35 IU/ml), or with CA125 of
oo

oo

oo
≥35 IU/ml but a normal ultrasound should be assessed carefully for other clinical
causes of her symptoms and investigated if appropriate. If no other clinical cause
eb

eb

eb
is apparent, she should be advised to return to her GP if her symptoms become
m

m
more frequent and/or persistent.
Reference
NICE. Ovarian cancer: recognition and initial management. NICE Clinical Guideline
t

t
e

ne

ne
(CG122). April 2011.
.n

e.

e.
e

303. Answer   E 60–75%


re

fre

re
Explanation
sf

sf
ks
Several randomised controlled trials have demonstrated improvement of PMS
k

k
oo

oo

oo
symptoms, with a response rate to GnRH analogue treatment of between 60% and
75%.
eb

eb

eb
Reference
m

m
Walsh S, Ismaili E, Naheed B, O’Brien S. Diagnosis, pathophysiology and management of
premenstrual syndrome. The Obstetrician & Gynaecologist 2015;17:99–104.
t

et
304. Answer   E  Uterine leiomyoma
ne

ne

n
Explanation
e.

e.

e.
Uterine fibroids (leiomyomata) are the most type of common benign tumour in
fre

fre

re
women, with a lifetime prevalence of around 30%.
f
ks

ks

ks
Reference
oo

oo

oo

Younas K, Hadoura E, Majoko F, Bunkheila A. A review of evidence-­based management of


uterine fibroids. The Obstetrician & Gynaecologist 2016;18:33–42.
eb

eb

eb

305. Answer   B  Combined oral contraceptive pill (COCP)


m

Explanation
Commonly used hormonal therapies include the combined contraceptive pill,
t

t
GnRH analogues, progestogens (via different routes, including an LNG-­IUS and
ne

ne

ne

injectable progestogens) and, less commonly, aromatase inhibitors. Combined


e.

e.

e.

oral contraceptives have been shown to produce similar results to GnRH


re

re

fre

analogues in the management of chronic pelvic pain in these women, and thus,
with their lower adverse effect profile, are more frequently used as a first-­line
sf

f
ks

ks

therapy.
k
oo

oo

oo

Reference
eb

eb

eb

Issa B, Ormesher L, Whorwell PJ, Shah M, Hamdy S. Endometriosis and irritable bowel
syndrome: a dilemma for the gynaecologist and gastroenterologist. The Obstetrician &
m

Gynaecologist 2016;18:9–16.
t

et

t
ne

ne
n
e.

e.

e.
t

t
Gynaecological problems 175

ne

ne

ne
e.

e.

e.
re

fre

fre
306. Answer   C  No progestogen required

f Explanation
ks

ks

ks
Systemic absorption is insignificant with low-­dose topical oestrogen. Additional
oo

oo

oo
systemic progestogen is not required.
eb

eb

eb
Reference
m

m
Bakour SH, Williamson J. Latest evidence on using hormone replacement therapy in the
menopause. The Obstetrician & Gynaecologist 2015;17:20–8.

307. Answer   C 24–27%


t

t
e

ne

ne
.n

Explanation

e.

e.
Co-existing cervical and endometrial polyps will be present in 24–27% of women
e
re

fre

re
at presentation.
sf

sf
ks
Reference
k

k
Otify M, Fuller J, Ross J, Shaikh H, Johns J. Endometrial pathology in the postmenopausal
oo

oo

oo
woman – an evidence based approach to management. The Obstetrician & Gynaecologist
2015;17:29–38.
eb

eb

eb
m

m
308. Answer   D  Cyclical pelvic pain, and vaginal spotting
Explanation
PATSS is a recognised complication known to occur in cases where sterilisation
t

et
ne

ne

is combined with ablation. The main symptoms are cyclical unilateral/bilateral

n
pelvic pain with vaginal spotting. The mechanism of pain is thought to be
e.

e.

e.
retrograde menstruation into the obstructed fallopian tube resulting in distension
fre

fre

re
of the fallopian tubes. The incidence varies from 6% to 8%.
f
ks

ks

ks
Reference
oo

oo

oo

Saraswat L, Cooper K. Surgical management of heavy menstrual bleeding: part 1. The


Obstetrician & Gynaecologist 2017;19:37–45.
eb

eb

eb

309. Answer   E  5 mm
m

Explanation
The endometrium is known for its remarkable ability to regenerate. Full-­thickness
t

t
destruction of the endometrium along with superficial myometrium to 5 mm,
ne

ne

ne

which contains basal endometrial glands, is required to stop menstrual flow.


e.

e.

e.

Regeneration from these basal glands is often a reason for treatment failure.
re

re

fre

Reference
sf

f
ks

ks

Saraswat L, Cooper K. Surgical management of heavy menstrual bleeding: part 1. The


k

Obstetrician & Gynaecologist 2017;19:37–45.


oo

oo

oo

310. Answer   D  Uterine septum


eb

eb

eb

Explanation
m

The incidence of congenital anomalies of the genital tract is difficult to determine,


as many women with anomalies are not diagnosed if they are not symptomatic.
t

et

t
ne

ne
n
e.

e.

e.
t

t
176 Module 13

ne

ne

ne
e.

e.

e.
re

fre

fre
These abnormalities are primarily abnormalities of uterine fusion with septate
uteri constituting 90% of cases, bicornuate uteri 5%, and the didelphic uterus 5%,
f
ks

ks

ks
when the abnormality is detected.
oo

oo

oo
Reference
eb

eb

eb
Edmonds DK, Rose GL. Outflow tract disorders of the female genital tract. The
Obstetrician & Gynaecologist 2013;15:11–7.
m

m
311. Answer   B  Offer hysterectomy
Explanation
t

t
e

ne

ne
Hysterectomy is indicated in women not wanting to preserve their fertility when
.n

there is any one of the following:

e.

e.
e

• Progression to atypical hyperplasia occurs during follow-up


re

fre

re
• There is no histological regression of hyperplasia, despite 12 months of treatment
sf

sf
ks
• There is relapse of endometrial hyperplasia after completing progestogen
k

k
treatment
oo

oo

oo
• There is persistence of bleeding symptoms
eb

eb

eb
• The woman declines to undergo endometrial surveillance or comply with
medical treatment.
m

m
Reference
RCOG. Management of endometrial hyperplasia. RCOG GTG No. 67. February 2016.
t

et
ne

ne

312. Answer   C  To improve the woman’s quality of life

n
e.

e.

e.
Explanation
fre

fre

re
It should be recognised that heavy menstrual bleeding has a major impact on a
woman’s quality of life, and any intervention should aim to improve this rather
f
ks

ks

ks
than focusing on blood loss.
oo

oo

oo

Reference
eb

eb

eb

NICE. Heavy menstrual bleeding: assessment and management. NICE Guideline (NG88).
March 2018.
m

313. Answer   E  Exogenous oestrogens


Pathological findings in women with postmenopausal bleeding are:
t

t
ne

ne

ne

Endometrial/cervical polyps 2–12%


e.

e.

e.

Endometrial hyperplasia 5–10%


re

re

fre

Endometrial carcinoma 10%


sf

f
ks

ks

Exogenous oestrogens 15–25%


k
oo

oo

oo

Atrophic vaginitis/endometritis 60–80%


eb

eb

eb

Reference
m

Otify M, Fuller J, Ross J, Shaikh H, Johns J. Endometrial pathology in the postmenopausal


woman – an evidence based approach to management. The Obstetrician & Gynaecologist
2015;17:29–38.
t

et

t
ne

ne
n
e.

e.

e.
t

t
Gynaecological problems 177

ne

ne

ne
e.

e.

e.
re

fre

fre
314. Answer   E  Insertion of an LNG-IUS

f Explanation
ks

ks

ks
A recent multicentre randomised trial compared the treatment of endometrial
oo

oo

oo
hyperplasia with LNG-­IUS, oral MPA 10 mg administered for 10 days per cycle
or continuous oral MPA 10 mg daily, for 6 months. The LNG-IUS-treated group
eb

eb

eb
achieved a 100% histologically normal endometrium after 6 months of therapy,
m

m
while this value was 96% and 69% for women in the continuous oral group and
cyclical progestogen group, respectively.
Reference
t

t
e

ne

ne
Otify M, Fuller J, Ross J, Shaikh H, Johns J. Endometrial pathology in the postmenopausal
.n

woman – an evidence based approach to management. The Obstetrician & Gynaecologist

e.

e.
e

2015;17:29–38.
re

fre

re
sf

sf
315. Answer   E 25%
ks
k

k
Explanation
oo

oo

oo
The ovary also produces and releases androgens, including 20% of dehydroepian-
eb

eb

eb
drosterone, 50% of androstenedione and 25% of circulating testosterone.
m

m
Reference
Meek CL, Bravis V, Don A, Kaplan F. Polycystic ovary syndrome and the differential
diagnosis of hyperandrogenism. The Obstetrician & Gynaecologist 2013;15:171–6.
t

et
ne

ne

EMQs

n
e.

e.

e.
316. Answer   E  Mayer–Rokitansky–Küster–Hauser (MRKH) syndrome
fre

fre

re
Explanation
f
ks

ks

ks
MRKH syndrome (Müllerian agenesis) is a malformation complex characterised
by congenital absence of the upper two-­thirds of the vagina and an absent or
oo

oo

oo

rudimentary uterus in women who have normal development of secondary sexual


eb

eb

eb

characteristics and a 46,XX karyotype.


m

Reference
Valappil S, Chetan U, Wood N, Garden A. Mayer–Rokitansky–Küster–Hauser syndrome:
diagnosis and management. The Obstetrician & Gynaecologist 2012;14:93–8.
t

t
ne

ne

ne

317. Answer   H  Transverse vaginal septum


e.

e.

e.

Explanation
re

re

fre

In an imperforate hymen, inspection of the vulva reveals a membrane that is


sf

blue in appearance with the darkened blood transilluminating through the thin
ks

ks

membrane. A differential diagnosis of a transverse vaginal septum must always be


k
oo

oo

oo

considered, but the appearances here are totally different, with the septum being
pink, although bulging, because the septum is so much thicker.
eb

eb

eb

Reference
m

Edmonds DK, Rose GL. Outflow tract disorders of the female genital tract. The
Obstetrician & Gynaecologist 2013;15:11–7.
t

et

t
ne

ne
n
e.

e.

e.
t

t
178 Module 13

ne

ne

ne
e.

e.

e.
re

fre

fre
318. Answer   G  Swyer syndrome

f
Explanation
ks

ks

ks
Swyer syndrome is a form of gonadal dysgenesis where the karyotype is 46,XY.
oo

oo

oo
The streak gonads do not make androgens so the person is phenotypically female.
As there is no hormone feedback to the hypothalamus/pituitary, FSH is elevated.
eb

eb

eb
The streak gonads do not make Müllerian inhibitory factor so the uterus is present
m

m
(it would be absent in androgen insensitivity syndrome).
Reference
Garden A, Hernon M, Topping J (eds.). Paediatric and Adolescent Gynaecology for the
t

t
e

ne

ne
MRCOG and Beyond, 2nd edn. London: RCOG Press, 2012, p. 57.
.n

e.

e.
e

319. Answer   E  Lichen sclerosus


re

fre

re
Explanation
sf

sf
ks
The histology of a vulval biopsy for vulval lichen sclerosus is a thinned epidermis
k

k
oo

oo

oo
with subepidermal hyalinisation and deeper inflammatory infiltrate. Although the
diagnosis of lichen sclerosus is usually made on clinical findings, this is a textbook
eb

eb

eb
definition of histopathological findings from a vulval biopsy.
m

m
320. Answer   F  Lichen simplex
Explanation
t

et
Lichen simplex is categorised into four main categories, of which psychiatric
ne

ne

disorders is one. There may be a chronic itch–scratch cycle, which results in

n
e.

e.

e.
lichenification and often loss of pubic hair.
fre

fre

re
321. Answer   D  Lichen planus
f
ks

ks

ks
Explanation
oo

oo

oo

The anogenital lesions of lichen planus may be divided into three main groups
according to their clinical presentation: classical, hypertrophic and erosive.
eb

eb

eb

Erosive is the most common subtype to cause vulval symptoms. The mucosal
m

surfaces are eroded, and at the edges of the erosions, the epithelium is mauve and
a pale network (Wickham’s striae) is sometimes seen.
Reference
t

t
ne

ne

ne

Edwards SK, Bates CM, Lewis F, Sethi G, Grover D. 2014 UK national guideline on the
management of vulval conditions. International Journal of STD & AIDS 2015;26:611–24.
e.

e.

e.
re

re

fre

322. Answer   O  Tranexamic acid


sf

f
ks

ks

Explanation
k

If a woman with heavy menstrual bleeding declines an LNG-­IUS or it is not


oo

oo

oo

suitable, consider the following pharmacological treatments:


eb

eb

eb

Non-hormonal:
m

• Tranexamic acid
• Non-steroidal anti-­inflammatory drugs (NSAIDs).
t

et

t
ne

ne
n
e.

e.

e.
t

t
Gynaecological problems 179

ne

ne

ne
e.

e.

e.
re

fre

fre
Hormonal:

f • Combined hormonal contraception


ks

ks

ks
• Cyclical oral progestogens.
oo

oo

oo
Her history and examination suggest that pharmaceutical options are
appropriate; however, most treatment options are hormonal. NSAIDs should be
eb

eb

eb
avoided with a history of renal impairment.
m

m
323. Answer   I  Mefenamic acid
Explanation
t

t
e

ne

ne
If pharmaceutical treatment is required while investigations and definitive
.n

treatment are being organised, either tranexamic acid or NSAIDs (such as

e.

e.
e

mefenamic acid) should be used. When heavy menstrual bleeding co-­exists with
re

fre

re
dysmenorrhoea, NSAIDs may be more effective than tranexamic acid.
sf

sf
ks
k

k
324. Answer   L  Second-­generation endometrial ablation
oo

oo

oo
Explanation
eb

eb

eb
If treatment for heavy menstrual bleeding is unsuccessful, the woman declines
pharmacological treatment or symptoms are severe, consider referral to
m

m
specialist care for investigations to diagnose the cause of the heavy bleeding, if
needed taking into account any investigations the woman has already had and
alternative treatment choices, including pharmacological options not already
t

et
ne

ne

tried, and surgical options, such as second-­generation endometrial ablation or a

n
hysterectomy.
e.

e.

e.
In view of the woman’s raised BMI, it would be preferable to try endometrial
fre

fre

re
ablation in the first instance, rather than a hysterectomy.
f
ks

ks

ks
Reference
oo

oo

oo

NICE. Heavy menstrual bleeding: assessment and management. NICE Guideline (NG88).
March 2018.
eb

eb

eb

325. Answer   H  Pelvic ultrasound scan


m

Explanation
Offer a pelvic ultrasound to women with heavy menstrual bleeding if any of the
t

t
following apply:
ne

ne

ne

• Their uterus is palpable abdominally


e.

e.

e.

• Their history or examination suggests a pelvic mass


re

re

fre

• Examination is inconclusive or difficult, for example in women who are obese.


sf

f
ks

ks
k

326. Answer   D  Full blood count (FBC)


oo

oo

oo

An FBC should be carried out on all women with heavy menstrual bleeding. This
eb

eb

eb

should be done in parallel with any heavy menstrual bleeding treatment offered.
m

m
t

et

t
ne

ne
n
e.

e.

e.
t

t
180 Module 13

ne

ne

ne
e.

e.

e.
re

fre

fre
327. Answer   E Hysteroscopy

f
Offer an outpatient hysteroscopy to women with heavy menstrual bleeding if their
ks

ks

ks
history suggests submucosal fibroids, polyps or endometrial pathology because:
oo

oo

oo
• They have symptoms such as persistent intermenstrual bleeding or
eb

eb

eb
• They have risk factors for endometrial pathology.
m

m
Reference
NICE. Heavy menstrual bleeding: assessment and management. NICE Guideline (NG88).
March 2018.
t

t
e

ne

ne
.n

328. Answer   G  GnRH analogue

e.

e.
e

Explanation
re

fre

re
GnRH analogues may be used for 3 months for a definitive diagnosis if the
sf

sf
completed symptom diary alone is inconclusive.
ks
k

k
oo

oo

oo
329. Answer   E  Drospirenone-­containing combined oral contraceptive pill (COCP)
eb

eb

eb
Explanation
When treating women with PMS, the drospirenone-­containing COCP
m

m
may represent effective treatment and should be considered as a first-­line
pharmaceutical intervention.
t

et
ne

ne

330. Answer   I  Micronised progesterone

n
e.

e.

e.
Explanation
fre

fre

re
Micronised progesterone is theoretically less likely to reintroduce PMS-­like
symptoms and should therefore be considered as a first-­line treatment for
f
ks

ks

ks
progestogenic opposition rather than progestogens.
oo

oo

oo

Reference
eb

eb

eb

RCOG. Management of premenstrual syndrome. RCOG GTG No. 48. December 2016.
m

331. Answer   B  Bacterial vaginosis


Bacterial vaginosis is the commonest cause of abnormal vaginal discharge.
t

t
ne

ne

ne

332. Answer   K Trichomoniasis


e.

e.

e.

Explanation
re

re

fre

The symptoms are suggestive of trichomoniasis. Chlamydia infection and


gonorrhoea are less likely due to the patient’s age and are not usually associated
sf

f
ks

ks

with vaginal itching.


k
oo

oo

oo

Reference
eb

eb

eb

Lazaro N. Sexually transmitted infections in primary care. RCGP/BASHH. 2013.


m

m
t

et

t
ne

ne
n
e.

e.

e.
t

t
Gynaecological problems 181

ne

ne

ne
e.

e.

e.
re

fre

fre
333. Answer   L Mebeverine

f Explanation
ks

ks

ks
This woman has irritable bowel syndrome according to the Rome III criteria.
oo

oo

oo
A systematic review concluded that smooth muscle relaxants such as
mebeverine hydrochloride are beneficial in the treatment of irritable bowel
eb

eb

eb
syndrome where abdominal pain is a prominent feature. The efficacy of bulking
m

m
agents has not been established, but they are commonly used.

334. Answer   D  Combined oral contraceptive pill (COCP)


t

t
e

ne

ne
Explanation
.n

This woman’s symptoms are suggestive of endometriosis. She does not meet the

e.

e.
e

criteria for irritable bowel syndrome. A therapeutic trial of hormonal treatment


re

fre

re
should be offered. The COCP is the most appropriate option. GnRH agonists will
sf

sf
render her hypo-­oestrogenic, which is not appropriate at the age of 20. Danazol is
ks
k

k
associated with masculinising side effects.
oo

oo

oo
335. Answer   D  Combined oral contraceptive pill (COCP)
eb

eb

eb
Explanation
m

m
Ovarian suppression can be an effective treatment for cyclical pain associated with
endometriosis. The effect can be achieved with the COCP, progestogens, danazol
or GnRH analogues, all of which are equally effective but have differing adverse
t

et
ne

ne

effect profiles.

n
Non-endometriosis-related cyclical pain also appears to be well controlled by
e.

e.

e.
these treatments.
fre

fre

re
This woman is young with a normal BMI. The COCP will be associated with
the fewest side effects for her. f
ks

ks

ks
Reference
oo

oo

oo

RCOG. The initial management of chronic pelvic pain. RCOG GTG No. 41. May 2012.
eb

eb

eb

336. Answer   J  No treatment required


m

Explanation
Women with small (<50 mm diameter) simple ovarian cysts generally do not
t

t
require follow-­up as these cysts are very likely to be physiological and almost
ne

ne

ne

always resolve within three menstrual cycles.


e.

e.

e.

337. Answer   H  Laparotomy and ovarian cystectomy


re

re

fre
sf

Explanation
ks

ks
k

In the presence of large masses with solid components (e.g. large dermoid
oo

oo

oo

cysts), laparotomy may be appropriate. This is a young woman who has not been
pregnant, so it would be preferable to conserve any normal ovarian tissue if
eb

eb

eb

possible.
m

m
t

et

t
ne

ne
n
e.

e.

e.
t

t
182 Module 13

ne

ne

ne
e.

e.

e.
re

fre

fre
338. Answer   E  Laparoscopic unilateral oophorectomy

f
Explanation
ks

ks

ks
Cysts >70 mm in diameter should be considered for either further imaging (MRI)
oo

oo

oo
or surgical intervention due to difficulties in examining the entire cyst adequately
at the time of ultrasound.
eb

eb

eb
The laparoscopic approach for elective surgical management of ovarian masses
m

m
presumed to be benign is associated with lower postoperative morbidity and
shorter recovery time, and is preferred to laparotomy in suitable patients.
This woman is symptomatic, and her urinary frequency may be related to
t

t
pressure effects of the cyst. Surgery is therefore preferable to further imaging. An
e

ne

ne
.n

oophorectomy will be a simpler operation with less chance of spillage of the cyst

e.

e.
contents.
e
re

fre

re
An argument could be made for a bilateral oophorectomy as the women is
close to menopausal age, but if the remaining ovary is normal, there is no clear
sf

sf
ks
justification for removing it.
k

k
oo

oo

oo
Reference
eb

eb

eb
RCOG. Management of suspected ovarian masses in premenopausal women. RCOG GTG
No. 62. December 2011.
m

m
339. Answer   J  Polycystic ovarian syndrome (PCOS)
Explanation
t

et
ne

ne

PCOS is the most common cause of hyperandrogenism in women of reproductive

n
age.
e.

e.

e.
fre

fre

re
340. Answer   I  Ovarian hyperthecosis
f
ks

ks

ks
Explanation
Ovarian hyperthecosis accounts for most of the cases of hyperandrogenaemia in
oo

oo

oo

postmenopausal women.
eb

eb

eb

Reference
m

Meek CL, Bravis V, Don A, Kaplan F. Polycystic ovary syndrome and the differential
diagnosis of hyperandrogenism. The Obstetrician & Gynaecologist 2013;15:171–6.
t

t
ne

ne

ne
e.

e.

e.
re

re

fre
sf

f
ks

ks
k
oo

oo

oo
eb

eb

eb
m

m
t

et

t
ne

ne
n
e.

e.

e.
t

t
ne

ne

ne
e.

e.

e.
re

fre

fre
Subfertility
Module
f
ks

ks

ks
14
oo

oo

oo
eb

eb

eb
m

m
t

t
e

ne

ne
SBAs
.n

e.

e.
e

341. A woman is diagnosed with hyperthyroidism as part of fertility investigations and


re

fre

re
opts to have treatment with radioactive iodine.
sf

sf
What is the minimum time she should delay conception after radioactive iodine
ks
k

k
treatment?
oo

oo

oo
A. 2 weeks
eb

eb

eb
B. 1 month
C. 3 months
m

m
D. 6 months
E. 12 months
t

et
342. What is the most common cause of testicular failure?
ne

ne

n
A. Idiopathic
e.

e.

e.
B. Klinefelter’s syndrome
fre

fre

re
C. Radiotherapy
D. Varicocele
f
ks

ks

ks
E. Y chromosome microdeletions
oo

oo

oo

343. One hundred couples are investigated for failure to conceive after 1 year of regular
eb

eb

eb

unprotected intercourse.
m

What proportion of these couples will have unexplained infertility?


A. 15%
B. 25%
t

t
C. 35%
ne

ne

ne

D. 45%
e.

e.

e.

E. 55%
re

re

fre

344. What is the single most important factor in determining female reproductive
sf

f
ks

ks

outcome?
k
oo

oo

oo

A. Female age
B. Ovarian antral follicle count
eb

eb

eb

C. Previous pregnancy
D. Serum anti-­Müllerian hormone (AMH)
m

E. Serum follicle-­stimulating hormone (FSH) in the early follicular phase

183
t

et

t
ne

ne
n
e.

e.

e.
t

t
184 Module 14

ne

ne

ne
e.

e.

e.
re

fre

fre
345. What is the most frequently used test of ovarian reserve?

f
A. Basal FSH in the early follicular phase
ks

ks

ks
B. Clomifene citrate challenge test
oo

oo

oo
C. Ovarian antral follicle count
D. Serum AMH
eb

eb

eb
E. Serum inhibin B
m

m
346. During the course of fertility investigations, a 34-year-old woman has
an ultrasound scan that suggests the presence of adenomyosis. All other
investigations are normal, and the partner’s semen analysis is satisfactory. The
t

t
e

ne

ne
couple have been trying to conceive for 1 year.
.n

What is the optimal management?

e.

e.
e

A. Diagnostic laparoscopy to confirm the diagnosis


re

fre

re
B. Explain that there is no known association between adenomyosis and
sf

sf
ks
subfertility
k

k
C. Hysteroscopic resection of the adenomyosis
oo

oo

oo
D. Offer in vitro fertilisation (IVF) treatment
eb

eb

eb
E. Treat with gonadotropin-­releasing hormone (GnRH) analogues
m

m
347. During the course of fertility investigations, a 35-year-old woman has an
ultrasound scan that shows the presence of three intramural fibroids, each
measuring 2 cm in diameter.
t

et
ne

ne

What is the optimal management?

n
A. Conservative management
e.

e.

e.
B. GnRH analogues for 6 months
fre

fre

re
C. Hysteroscopic myomectomy
D. Laparoscopic myomectomy f
ks

ks

ks
E. Open myomectomy
oo

oo

oo

348. What is the best test of fallopian tube function?


eb

eb

eb

A. Hystero-contrast-salpingography (HyCoSy)
m

B. Hysterosalpingogram (HSG)
C. Laparoscopy and dye test
D. Rubin’s test
t

t
E. Spontaneous intrauterine pregnancy
ne

ne

ne
e.

e.

e.

349. Which cell type is the end product of meiosis in the testes?
re

re

fre

A. Leydig cell
sf

B. Primary spermatocyte
ks

ks

C. Secondary spermatocyte
k
oo

oo

oo

D. Spermatid
E. Spermatogonium
eb

eb

eb
m

m
t

et

t
ne

ne
n
e.

e.

e.
t

t
Subfertility 185

ne

ne

ne
e.

e.

e.
re

fre

fre
350. What proportion of infertility is thought to be attributable to male factors?

f A. 10%
ks

ks

ks
B. 30%
oo

oo

oo
C. 50%
D. 70%
eb

eb

eb
E. 90%
m

m
351. Which cases of ovarian hyperstimulation syndrome (OHSS) should be reported
to the HFEA?
t

t
A. All cases of OHSS
e

ne

ne
B. Only cases of critical OHSS
.n

e.

e.
C. Only cases of moderate, severe and critical OHSS
e

D. Only cases of severe and critical OHSS


re

fre

re
E. Only cases of severe OHSS
sf

sf
ks
k

k
352. Which analgesic should be avoided in women presenting with OHSS?
oo

oo

oo
A. Codeine
eb

eb

eb
B. Ibuprofen
C. Morphine
m

m
D. Paracetamol
E. Pethidine
t

et
ne

ne

353. Following IVF treatment in which 15 oocytes were collected, a woman attends the

n
gynaecology emergency unit with mild lower abdominal pain and bloating. Her
e.

e.

e.
GP suspects the presence of OHSS. An ultrasound scan is performed. The mean
fre

fre

re
ovarian diameter is 6 cm. There is no evidence of ascites.
What type of OHSS is this? f
ks

ks

ks
A. Critical OHSS
oo

oo

oo

B. Mild OHSS
eb

eb

eb

C. Moderate OHSS
D. Severe OHSS
m

E. This is not OHSS

354. What is the most appropriate management of hydrosalpinges prior to IVF


t

t
treatment?
ne

ne

ne

A. Conservative management
e.

e.

e.

B. Laparoscopic salpingectomy
re

re

fre

C. Laparoscopic salpingostomy
sf

D. Laparoscopic tubal clipping


ks

ks

E. Ultrasound-guided drainage
k
oo

oo

oo
eb

eb

eb
m

m
t

et

t
ne

ne
n
e.

e.

e.
t

t
186 Module 14

ne

ne

ne
e.

e.

e.
re

fre

fre
355. A woman who has had three previous IVF treatments is diagnosed with recurrent
implantation failure and opts to have an endometrial scratch (endometrial injury)
f
ks

ks

ks
as part of her upcoming treatment?
oo

oo

oo
When should this be performed in relation to the IVF treatment cycle?
A. At the onset of menstruation, immediately before the start of ovarian
eb

eb

eb
stimulation
m

m
B. 7 days prior to the onset of menstruation, immediately before the start
of ovarian stimulation
C. 14 days prior to the onset of menstruation, immediately before the start
of ovarian stimulation
t

t
e

ne

ne
D. 21 days prior to the onset of menstruation, immediately before the start
.n

of ovarian stimulation

e.

e.
e

E. 7 days after the onset of menstruation, during ovarian stimulation


re

fre

re
sf

sf
ks
k

k
oo

oo

oo
eb

eb

eb
m

m
t

et
ne

ne

n
e.

e.

e.
fre

fre

f re
ks

ks

ks
oo

oo

oo
eb

eb

eb
m

m
t

t
ne

ne

ne
e.

e.

e.
re

re

fre
sf

f
ks

ks
k
oo

oo

oo
eb

eb

eb
m

m
t

et

t
ne

ne
n
e.

e.

e.
t

t
Subfertility 187

ne

ne

ne
e.

e.

e.
re

fre

fre
EMQs
f
ks

ks

ks
Options for questions 356–358
oo

oo

oo
A Daily follicle-­stimulating hormone (FSH) injections
eb

eb

eb
B Donated embryos
C Donor insemination
m

m
D Intracytoplasmic sperm injection (ICSI)
E ICSI with donated eggs
t

t
F ICSI with donor eggs and donor sperm
e

ne

ne
.n

G Intrauterine insemination with donor sperm

e.

e.
e

H Intrauterine insemination with partner sperm


re

fre

re
I In vitro fertilisation (IVF)
sf

sf
J IVF with donor eggs and donor sperm
ks
k

k
K No treatment required
oo

oo

oo
L Ovulation induction and donor insemination
eb

eb

eb
M Ovulation induction and donor intrauterine insemination
m

m
N Ovualtion induction with clomifene
O Surgical sperm retrieval followed by ICSI
P Surgical sperm retrieval followed by intrauterine insemination
t

et
Q Surgical sperm retrieval followed by IVF
ne

ne

n
e.

e.

e.
For each of the following clinical scenarios, what is the most appropriate treatment
fre

fre

re
option from the list above? Each option may be used once, more than once or not at all.
f
ks

ks

ks
356. A 46-year-old woman presents to the IVF clinic with her new partner. She wishes
to become pregnant. Her menstrual cycle is irregular with a duration of 28–45
oo

oo

oo

days, with periods lasting 1–2 days. Her serum FSH is 18.7 IU/l and her AMH
eb

eb

eb

level is <1.1 pmol/l. A pelvic ultrasound scan is normal.


Her partner’s semen analysis results are:
m

Semen count analysis: 8.3 × 106/ml


Progressive motility 11%
t

t
ne

ne

ne

Morphology 2%
e.

e.

e.

357. A couple attend the fertility clinic. The woman has irregular periods and clinical
re

re

fre

evidence of hyperandrogenism. Her AMH level is 67 pmol/l. An HSG confirms


sf

f
ks

ks

patent fallopian tubes. The man has azoospermia. In the past, he has been
k

investigated and a testicular biopsy showed maturation arrest.


oo

oo

oo
eb

eb

eb
m

m
t

et

t
ne

ne
n
e.

e.

e.
t

t
188 Module 14

ne

ne

ne
e.

e.

e.
re

fre

fre
358. A couple are referred to the fertility clinic for investigation. The woman is 32 years
of age and has no significant past medical history. She has a regular 28-­day cycle.
f
ks

ks

ks
The man is 34 years of age and has a history of orchidopexy as a child for a left
oo

oo

oo
undescended testis. He has two semen analyses, both of which are normal.
The woman’s results are:
eb

eb

eb
Day 2 FSH 4.5 IU/l
m

m
Day 21 progesterone 35 nmol/l
Pelvic ultrasound scan Normal
t

t
HSG Normal cavity, bilateral patent tubes
e

ne

ne
.n

e.

e.
Options for questions 359–361
e
re

fre

re
A Anastrozole
sf

sf
ks
k

k
B Bromocriptine
oo

oo

oo
C Cabergoline
eb

eb

eb
D Clomifene citrate
E Follicle-stimulating hormone (FSH) injections
m

m
F Human chorionic gonadotropin (hCG) injections
G Laparoscopic ovarian drilling
H Letrozole
t

et
ne

ne

I Metformin

n
e.

e.

e.
J No treatment required
fre

fre

re
K Oral cetrorelix
L Oral progestogen for 10 days
f
ks

ks

ks
M Ovarian wedge resection
oo

oo

oo

N Tamoxifen
eb

eb

eb

O Weight gain
P Weight loss
m

For each of the following clinical scenarios, what is the most appropriate treatment
option from the list above? Each option may be used once, more than once or not at all.
t

t
ne

ne

ne

359. During the course of fertility investigations, a 28-year-old woman with a BMI
e.

e.

e.

of 35 kg/m2 is found to have polycystic ovaries on ultrasound scanning. Her


re

re

fre

menstrual cycle has a length of 35–70 days. Her partner’s semen analysis is
sf

normal.
ks

ks
k

360. A 32-year-old woman and her husband present to the clinic with 1 year of
oo

oo

oo

subfertility. Investigations show them to have unexplained infertility. The woman


eb

eb

eb

requests treatment as she has read on the internet that there are drugs that can be
used to ‘boost fertility’.
m

m
t

et

t
ne

ne
n
e.

e.

e.
t

t
Subfertility 189

ne

ne

ne
e.

e.

e.
re

fre

fre
361. A woman with a BMI of 28 kg/m2 and polycystic ovarian syndrome (PCOS)
has attempted ovulation induction with clomifene citrate without success.
f
ks

ks

ks
She now returns to the clinic with pelvic pain that is worse around the time of
oo

oo

oo
menstruation.
eb

eb

eb
Options for questions 362–364
m

m
A Serum androstenedione
B Serum anti-­Müllerian hormone (AMH)
C Serum CA125
t

t
e

ne

ne
D Serum follicle-­stimulating hormone (FSH) on days 10–12 of cycle
.n

e.

e.
E Serum FSH on day 21 of cycle
e
re

fre

re
F Serum FSH on days 2–5 of cycle
G Serum luteinising hormone (LH) on day 5 of cycle
sf

sf
ks
k

k
H Serum LH on day 14 of cycle
oo

oo

oo
I Serum LH on day 21 of cycle
eb

eb

eb
J Serum progesterone on day 14 of cycle
K Serum progesterone on day 21 of cycle
m

m
L Serum progesterone on day 28 of cycle
M Serum prolactin
N Serum testosterone
t

et
ne

ne

O Thyroid function tests

n
e.

e.

e.
fre

fre

re
For each of the following clinical scenarios, what is the most appropriate investigation
option from the list above? Each option may be used once, more than once or not at all.
f
ks

ks

ks
362. A woman with a regular 35-­day cycle attends the fertility clinic. Investigations are
oo

oo

oo

instigated. Which test should be arranged to check for ovulation?


eb

eb

eb

363. Which test is probably the best biochemical marker of polycystic ovaries?
m

364. A 28-year-old woman with a BMI of 21 kg/m2 attends the fertility clinic with her
partner. She has very infrequent periods. She is treated with clomifene therapy to
induce ovulation, but the treatment fails. Which test will indicate if this woman
t

t
ne

ne

ne

is likely to have a good clinical and endocrine response to laparoscopic ovarian


drilling?
e.

e.

e.
re

re

fre
sf

f
ks

ks
k
oo

oo

oo
eb

eb

eb
m

m
t

et

t
ne

ne
n
e.

e.

e.
t

t
190 Module 14

ne

ne

ne
e.

e.

e.
re

fre

fre
Options for questions 365–367

f
ks

ks

ks
A Abdominal examination
oo

oo

oo
B Abdominal ultrasound scan
C Barium enema
eb

eb

eb
D Colonoscopy
m

m
E CT scan of pelvis
F Diagnostic laparoscopy
G Diagnostic laparoscopy and peritoneal biopsy
t

t
e

ne

ne
H Endoanal ultrasound
.n

e.

e.
I Endometrial biopsy
e
re

fre

re
J Endometrial cytokine levels
K MRI of pelvis
sf

sf
ks
k

k
L Rectal examination
oo

oo

oo
M Serum CA125
eb

eb

eb
N Transperineal ultrasound scan
O Transvaginal ultrasound scan
m

m
P Vaginal examination

For each of the following clinical scenarios, what is the most appropriate management
t

et
ne

ne

option from the list above? Each option may be used once, more than once or not at all.

n
e.

e.

e.
365. A 35-year-old woman presents to the gynaecology clinic with pelvic pain and
fre

fre

re
dysmenorrhoea. A pelvic examination demonstrates tenderness and fullness in
f
the right iliac fossa. Which test should be used to diagnose or exclude an ovarian
ks

ks

ks
endometrioma?
oo

oo

oo

366. A 24-year-old woman, virgo intacta, presents to the gynaecology clinic with
eb

eb

eb

abdominal and pelvic pain, dysmenorrhoea and dyschezia. What is the most
appropriate initial assessment for the diagnosis of endometriosis?
m

367. A woman with a previous diagnosis of pelvic endometriosis presents to the clinic
with cylical rectal bleeding. Vaginal and rectal examinations are inconclusive.
t

t
What is the most appropriate initial assessment to identify or exclude rectal
ne

ne

ne

endometriosis?
e.

e.

e.
re

re

fre
sf

f
ks

ks
k
oo

oo

oo
eb

eb

eb
m

m
t

et

t
ne

ne
n
e.

e.

e.
t

t
Subfertility 191

ne

ne

ne
e.

e.

e.
re

fre

fre
Options for questions 368–370

f
ks

ks

ks
A Anabolic steroid abuse
oo

oo

oo
B Congenital bilateral absence of vas deferens (CBAVD)
C Idiopathic
eb

eb

eb
D Kallmann syndrome
m

m
E Kartagener’s syndrome
F Klinefelter syndrome
G Maturation arrest
t

t
e

ne

ne
H Noonan syndrome
.n

e.

e.
I Pituitary adenoma
e
re

fre

re
J Primary testicular failure
K Retrograde ejaculation
sf

sf
ks
k

k
L Seminoma
oo

oo

oo
M Sertoli cell-­only syndrome
eb

eb

eb
N Varicocele
O Vasectomy
m

m
P Y chromosome microdeletion

For each of the following clinical scenarios, what is the most likely diagnosis from the
t

et
ne

ne

options listed? Each option may be used once, more than once or not at all.

n
e.

e.

e.
368. A man is found to have azoospermia during the course of fertility investigations.
fre

fre

re
He has two children from a previous relationship. He denies any significant
medical history. On examination, he is muscular and both testes are small and
f
ks

ks

ks
soft.
oo

oo

oo

369. A man is found to have azoospermia. His serum FSH and testosterone levels are
eb

eb

eb

normal. On examination, he has normal secondary sexual characteristics and


both testes are of normal size. His karyotype is normal and a cystic fibrosis screen
m

is negative. A testicular biopsy shows germ cells to be present.

370. A man is found to have azoospermia. His serum FSH level is elevated but his
t

t
testosterone is normal. On examination, the man is tall and has gynaecomastia.
ne

ne

ne

Both testes are in the scrotum but are small and soft.
e.

e.

e.
re

re

fre
sf

f
ks

ks
k
oo

oo

oo
eb

eb

eb
m

m
t

et

t
ne

ne
n
e.

e.

e.
t

t
192 Module 14

ne

ne

ne
e.

e.

e.
re

fre

fre
Answers
f
ks

ks

ks
SBAs
oo

oo

oo
341. Answer   D  6 months
eb

eb

eb
Explanation
Conception should be delayed for 6 months after radioactive iodine therapy.
m

m
Reference
Jefferys A, Vanderpump M, Yasmin E. Thyroid dysfunction and reproductive health. The
t

t
Obstetrician & Gynaecologist 2015;17:39–45.
e

ne

ne
.n

e.

e.
342. Answer   A Idiopathic
e
re

fre

re
Explanation
sf

sf
Causes of testicular failure include bilateral cryptorchidism, genetic disorders,
ks
k

k
systemic disease, radiotherapy and chemotherapy. However, in the majority of
oo

oo

oo
cases (66%), the cause is unknown.
eb

eb

eb
Reference
Karavolos S, Stewart J, Evbuomwan I, McEleny K, Aird I. Assessment of the infertile male.
m

m
The Obstetrician & Gynaecologist 2013;15:1–9.

343. Answer   B 25%


t

et
ne

ne

Explanation

n
e.

e.

e.
The main causes of infertility in the UK are (percentage figures indicate
approximate prevalence):
fre

fre

re
• Unexplained infertility (no identified male or female cause): 25%
f
ks

ks

ks
• Ovulatory disorders: 25%
oo

oo

oo

• Tubal damage: 20%


• Factors in the male causing infertility: 30%
eb

eb

eb

• Uterine or peritoneal disorders: 10%.


m

Reference
NICE. Fertility problems: assessment and treatment. NICE Guidance (CG 156). Updated
September 2017.
t

t
ne

ne

ne

344. Answer   A  Female age


e.

e.

e.

Explanation
re

re

fre

A woman’s age remains the single most important factor in determining


sf

f
ks

ks

reproductive outcome; ovarian reserve can only predict ovarian response in an


k

assisted reproductive technology cycle.


oo

oo

oo

Reference
eb

eb

eb

Nandi A, Homburg R. Unexplained subfertility: diagnosis and management. The


m

Obstetrician & Gynaecologist 2016;18:107–15.


t

et

t
ne

ne
n
e.

e.

e.
t

t
Subfertility 193

ne

ne

ne
e.

e.

e.
re

fre

fre
345. Answer   A  Basal FSH in the early follicular phase

f Explanation
ks

ks

ks
Although the basal FSH test is the most frequently used, it has significant intra-
oo

oo

oo
and intercycle variability, which limits its reliability.
eb

eb

eb
Reference
m

m
Nandi A, Homburg R. Unexplained subfertility: diagnosis and management. The
Obstetrician & Gynaecologist 2016;18:107–15.

346. Answer   B  Explain that there is no known association between adenomyosis


t

t
e

ne

ne
and subfertility
.n

e.

e.
Explanation
e
re

fre

re
The impact of adenomyosis or its treatment on fertility remains unsubstantiated
because of a paucity of data. Therefore, subfertility in women with adenomyosis
sf

sf
ks
currently remains unexplained.
k

k
oo

oo

oo
Reference
eb

eb

eb
Nandi A, Homburg R. Unexplained subfertility: diagnosis and management. The
Obstetrician & Gynaecologist 2016;18:107–15.
m

347. Answer  
Explanation
A  Conservative management
m
t

et
ne

ne

There is insufficient evidence that myomectomy for intramural or subserous

n
fibroids improves pregnancy rates.
e.

e.

e.
fre

fre

re
Reference

f
Nandi A, Homburg R. Unexplained subfertility: diagnosis and management. The
ks

ks

ks
Obstetrician & Gynaecologist 2016;18:107–15.
oo

oo

oo

348. Answer   E  Spontaneous intrauterine pregnancy


eb

eb

eb

Explanation
m

Assessment of tubal patency can be achieved by various methods, such as HSG,


HyCoSy, and laparoscopy and dye tests. None of these methods, however, can
detect tubal function defects, which can potentially contribute to a couple’s
t

t
subfertility.
ne

ne

ne

If a couple conceive spontaneously and the pregnancy is intrauterine, then the


e.

e.

e.

fallopian tube must be functioning correctly.


re

re

fre

Reference
sf

f
ks

ks

Nandi A, Homburg R. Unexplained subfertility: diagnosis and management. The


k

Obstetrician & Gynaecologist 2016;18:107–15.


oo

oo

oo
eb

eb

eb
m

m
t

et

t
ne

ne
n
e.

e.

e.
t

t
194 Module 14

ne

ne

ne
e.

e.

e.
re

fre

fre
349. Answer   D Spermatid

f
Explanation
ks

ks

ks
Sperm are formed in the seminiferous tubules, from germinal cells called
oo

oo

oo
spermatogonia. Spermatogonia divide by mitosis into primary spermatocytes,
which in turn undergo two reduction divisions (meiosis I and II) to form
eb

eb

eb
spermatids.
m

m
Reference
Karavolos S, Stewart J, Evbuomwan I, McEleny K, Aird I. Assessment of the infertile male.
The Obstetrician & Gynaecologist 2013;15:1–9.
t

t
e

ne

ne
.n

350. Answer   B 30%

e.

e.
e
re

fre

re
Explanation
Long-term data from the Human Fertilisation and Embryology Authority (HFEA)
sf

sf
ks
analysing male factors as a cause for referral to fertility centres have shown that the
k

k
oo

oo

oo
percentage of infertility attributable to male factors appears to have increased from
27.6% in 2000 to 32.5% in 2006, but since then has averaged approximately 30%.
eb

eb

eb
Reference
m

m
Karavolos S, Stewart J, Evbuomwan I, McEleny K, Aird I. Assessment of the infertile male.
The Obstetrician & Gynaecologist 2013;15:1–9.
t

et
351. Answer   D  Only cases of severe and critical OHSS
ne

ne

n
Explanation
e.

e.

e.
Licensed centres should comply with HFEA regulations in reporting cases of
fre

fre

re
severe or critical OHSS among their patients.
f
ks

ks

ks
Reference
oo

oo

oo

RCOG. The management of ovarian hyperstimulation syndrome. RCOG GTG No. 5.


February 2016.
eb

eb

eb

352. Answer   B Ibuprofen


m

Explanation
Relief of abdominal pain and nausea forms an important part of the supportive
t

t
care of women with OHSS. Analgesia with paracetamol and opiates, if required,
ne

ne

ne

is appropriate, while non-­steroidal anti-­inflammatory drugs (NSAIDs) should be


e.

e.

e.

avoided as they may compromise renal function.


re

re

fre

Reference
sf

f
ks

ks

RCOG. The management of ovarian hyperstimulation syndrome. RCOG GTG No. 5.


k

February 2016.
oo

oo

oo
eb

eb

eb
m

m
t

et

t
ne

ne
n
e.

e.

e.
t

t
Subfertility 195

ne

ne

ne
e.

e.

e.
re

fre

fre
353. Answer   B  Mild OHSS

f Explanation
ks

ks

ks
See Table 3 in the reference article.
oo

oo

oo
Reference
eb

eb

eb
RCOG. The management of ovarian hyperstimulation syndrome. RCOG GTG No. 5.
m

m
February 2016.

354. Answer   B  Laparoscopic salpingectomy


t

t
Explanation
e

ne

ne
.n

Women with hydrosalpinges should be offered salpingectomy, preferably by

e.

e.
laparoscopy, before IVF treatment because this improves the chance of a live birth.
e
re

fre

re
Reference
sf

sf
ks
NICE. Fertility problems: assessment and treatment. NICE Guidance (CG 156). Updated
k

k
September 2017.
oo

oo

oo
355. Answer   C  7 days prior to the onset of menstruation, immediately before the
eb

eb

eb
start of ovarian stimulation
m

m
Explanation
The conclusions of studies to date suggest that an endometrial scratch should be
carried out approximately 7 days prior to the onset of menstruation, immediately
t

et
ne

ne

before the start of ovarian stimulation for IVF treatment.

n
e.

e.

e.
Reference
fre

fre

re
RCOG. Local endometrial trauma (endometrial scratch): a treatment strategy to improve
implantation rates. RCOG Scientific Impact Paper No. 54. November 2016.
f
ks

ks

ks
EMQs
oo

oo

oo

356. Answer   E  ICSI with donated eggs


eb

eb

eb

Explanation
m

The woman is >45 years and appears to be perimenopausal. It is very unlikely that
she would respond to ovarian stimulation. Even if eggs were retrieved, her chance
of a successful pregnancy would be around 3% (HFEA data). With donated
t

t
ne

ne

ne

eggs, her chance of pregnancy would be around 50%. The semen analysis is very
suboptimal, so ICSI is almost certainly required.
e.

e.

e.
re

re

fre

357. Answer   M  Ovulation induction and donor intrauterine insemination


sf

f
ks

ks

Explanation
k

The female clinical picture is that of polycystic ovarian syndrome (PCOS). With
oo

oo

oo

an irregular cycle, she is unlikely to be ovulating consistently and therefore


eb

eb

eb

ovulation induction is required.


The male has maturation arrest, and therefore donor sperm will be required as
m

it is not possible to use immature gametes for treatment in the UK.


t

et

t
ne

ne
n
e.

e.

e.
t

t
196 Module 14

ne

ne

ne
e.

e.

e.
re

fre

fre
358. Answer   K  No treatment required

f
Explanation
ks

ks

ks
This couple have unexplained infertility, despite the history of orchidopexy. As
oo

oo

oo
the woman is <35 years and they have been trying to conceive for <2 years, no
treatment is required at this moment in time. If they have still not conceived after
eb

eb

eb
2 years, then the recommended treatment would be IVF.
m

m
Reference
NICE. Fertility problems: assessment and treatment. NICE Guideline (CG156). Updated
September 2017.
t

t
e

ne

ne
.n

359. Answer   P  Weight loss

e.

e.
e

Explanation
re

fre

re
Advise women with World Health Organization (WHO) group II anovulatory
sf

sf
ks
infertility who have a BMI of ≥30 kg/m2 to lose weight. Inform them that this
k

k
alone may restore ovulation, improve their response to ovulation induction agents
oo

oo

oo
and have a positive impact on pregnancy outcomes.
eb

eb

eb
360. Answer   J  No treatment required
m

m
Explanation
Do not offer oral ovarian stimulation agents (such as clomifene citrate,
anastrozole or letrozole) to women with unexplained infertility.
t

et
ne

ne

n
361. Answer   G  Laparoscopic ovarian drilling
e.

e.

e.
Explanation
fre

fre

re
For women with WHO group II ovulation disorders who are known to be
f
ks

ks

ks
resistant to clomifene citrate, consider one of the following second-­line
treatments, depending on clinical circumstances and the woman’s preference:
oo

oo

oo

• Laparoscopic ovarian drilling


eb

eb

eb

• Combined treatment with clomifene citrate and metformin if not already


offered as a first-­line treatment
m

• Gonadotropins.
In this case, the woman’s symptoms could be suggestive of endometriosis, so
laparoscopic ovarian drilling would allow inspection of the peritoneal cavity at
t

t
ne

ne

ne

the same time.


e.

e.

e.

Reference
re

re

fre

NICE. Fertility problems: assessment and treatment. NICE Guideline (CG156). Updated
sf

September 2017.
ks

ks
k
oo

oo

oo

362. Answer   L  Serum progesterone on day 28 of her cycle


eb

eb

eb

Explanation
Serum progesterone should be measured 7 days before the expected day of
m

menstruation.
t

et

t
ne

ne
n
e.

e.

e.
t

t
Subfertility 197

ne

ne

ne
e.

e.

e.
re

fre

fre
363. Answer   B  Serum AMH

f Explanation
ks

ks

ks
It has been suggested recently that the threshold number of follicles to define a
oo

oo

oo
polycystic ovary should be 25, and that the biochemical marker of AMH may be
even more precise than ultrasound, with a threshold serum concentration of
eb

eb

eb
>35 pmol/l.
m

m
364. Answer   G  Serum LH on day 5 of her cycle
Explanation
t

t
e

ne

ne
After laparoscopic ovarian drilling, with restoration of ovarian activity, serum
.n

concentrations of LH and testosterone fall. The response depends on pretreatment

e.

e.
e

characteristics, with those who are slim and with high basal LH concentrations
re

fre

re
having a better clinical and endocrine response.
sf

sf
ks
Reference
k

k
oo

oo

oo
Balen AH. Polycystic ovary syndrome (PCOS). The Obstetrician & Gynaecologist
2017;19:119–29.
eb

eb

eb
365. Answer   O  Transvaginal ultrasound scan
m

m
Explanation
Clinicians are recommended to perform transvaginal sonography to diagnose or
t

et
to exclude ovarian endometrioma.
ne

ne

n
e.

e.

e.
366. Answer   L  Rectal examination
fre

fre

re
Explanation
f
The guideline development group recommends that clinicians perform a clinical
ks

ks

ks
examination in all women with suspected endometriosis, although vaginal
oo

oo

oo

examination may be inappropriate for adolescents and/or women without


previous sexual intercourse. In such cases, a rectal examination can be helpful for
eb

eb

eb

the diagnosis of endometriosis.


m

367. Answer   O  Transvaginal ultrasound scan


Explanation
t

t
ne

ne

ne

In women with symptoms and signs of rectal endometriosis, transvaginal


sonography is useful for identifying or ruling out rectal endometriosis.
e.

e.

e.
re

re

fre

Reference
ESHRE. Management of women with endometriosis. ESHRE Guideline. September 2013.
sf

f
ks

ks
k
oo

oo

oo

368 Answer   A  Anabolic steroid abuse


Explanation
eb

eb

eb

The man already has children so has proven fertility. This rules out many of the
m

causes of azoospermia above. Anabolic steroid abuse is common in gym users and
results in pituitary suppression. This is the most likely cause of his azoospermia
and small testes.
t

et

t
ne

ne
n
e.

e.

e.
t

t
198 Module 14

ne

ne

ne
e.

e.

e.
re

fre

fre
369. Answer   G  Maturation arrest

f
Explanation
ks

ks

ks
With maturation arrest, germ cells are present in the testes, but the process of
oo

oo

oo
spermatogenesis is arrested, at either the spermatocyte or spermatid stage. This
will result in azoospermia, but a testicular biopsy will confirm germ cells to be
eb

eb

eb
present. Leydig and Sertoli cells are present so there will be normal secondary
m

m
sexual characteristics and normal feedback to the pituitary, resulting in a normal
serum FSH.
t

t
370. Answer   F  Klinefelter syndrome
e

ne

ne
.n

Explanation

e.

e.
e

From the clinical description, this is most likely to be Klinefelter’s syndrome


re

fre

re
(karyotype 47,XXY), which can be confirmed by karyotyping. Primary testicular
sf

sf
failure is also possible, but the features of a tall man with gynaecomastia make it
ks
k

k
much more likely that this is a case of Klinefelter syndrome.
oo

oo

oo
Reference
eb

eb

eb
Karavolos S, Stewart J, Evbuomwan I, McEleny K, Aird I. Assessment of the infertile male.
The Obstetrician & Gynaecologist 2013;15:1–9.
m

m
t

et
ne

ne

n
e.

e.

e.
fre

fre

f re
ks

ks

ks
oo

oo

oo
eb

eb

eb
m

m
t

t
ne

ne

ne
e.

e.

e.
re

re

fre
sf

f
ks

ks
k
oo

oo

oo
eb

eb

eb
m

m
t

et

t
ne

ne
n
e.

e.

e.
t

t
ne

ne

ne
e.

e.

e.
re

fre

fre
Sexual and reproductive health
Module
f
ks

ks

ks
15
oo

oo

oo
eb

eb

eb
m

m
t

t
e

ne

ne
SBAs
.n

e.

e.
e

371. Spasm of which muscle of the pelvic floor is most commonly implicated in
re

fre

re
vaginismus?
sf

sf
ks
A. Coccygeus
k

k
B. Iliococcygeus
oo

oo

oo
C. Obturator internus
eb

eb

eb
D. Piriformis
E. Pubococcygeus
m

m
372. What is the most common cause of hypoactive sexual desire disorder (HSDD)?
A. Depression
t

et
ne

ne

B. Drug-induced

n
C. Menopause
e.

e.

e.
D. Psychosexual
fre

fre

re
E. Surgically induced
f
ks

ks

ks
373. Which method of emergency contraception works primarily by inhibiting
oo

oo

oo

fertilisation?
A. Copper intrauterine contraceptive device (IUCD)
eb

eb

eb

B. Levonorgestrel-releasing intrauterine system (LNG-IUS)


m

C. Oral levonorgestrel
D. Ulipristal acetate
E. Yuzpe regimem
t

t
ne

ne

ne

374. According to the UK Medical Eligibility Criteria for Contraceptive Use (UKMEC),
e.

e.

e.

what is the medical contraindication to the use of levonorgestrel for emergency


contraception?
re

re

fre

A. Breastfeeding
sf

f
ks

ks

B. Female age >35 years


k
oo

oo

oo

C. No medical contraindications
D. Nulliparity
eb

eb

eb

E. Previous ectopic pregnancy


m

199
t

et

t
ne

ne
n
e.

e.

e.
t

t
200 Module 15

ne

ne

ne
e.

e.

e.
re

fre

fre
375. What is the failure rate of the lactational amenorrhoea method (LAM) of
postpartum contraception?
f
ks

ks

ks
A. 2 in 100 women
oo

oo

oo
B. 2 in 500 women
C. 2 in 1000 women
eb

eb

eb
D. 2 in 5000 women
m

m
E. 2 in 10,000 women

376. Once a menopausal woman is established on hormone replacement therapy


(HRT), how often should she be reviewed?
t

t
e

ne

ne
A. 3-monthly
.n

e.

e.
B. 6-monthly
e

C. Annually
re

fre

re
D. Every 2 years
sf

sf
ks
E. Every 5 years
k

k
oo

oo

oo
377. What is the most sensitive test for detection of Trichomonas vaginalis infection?
eb

eb

eb
A. Immunochromatographic capillary flow (dipstick) assay
B. Laboratory culture
m

m
C. Light-field microscopy
D. Nucleic acid amplification test (NAAT)
E. Trichomonas serum antibody test
t

et
ne

ne

n
378. A 47-year-old woman who has been using condoms for contraception with her
e.

e.

e.
partner attends for contraceptive advice. Her last menstrual period was 8 months
fre

fre

re
ago and she has no overt menopausal symptoms.
f
Assuming she has no further periods, for how long should she continue to use
ks

ks

ks
contraception?
oo

oo

oo

A. She can stop now as the chances of conception are so low


eb

eb

eb

B. 4 months
C. 12 months
m

D. 16 months
E. 24 months
t

t
379. Which two hormonal methods of contraception are associated with the highest
ne

ne

ne

failure rate in perimenopausal women in the first year of typical (non-­perfect) use?
e.

e.

e.
re

re

fre

Combined LNG-IUS Progestogen- Progestogen- Progestogen-


hormonal only implant only only
sf

f
ks

ks

contraception injectable pill


k
oo

oo

oo

A ✓ ✓
B ✓ ✓
eb

eb

eb

C ✓ ✓
m

D ✓ ✓
E ✓ ✓
t

et

t
ne

ne
n
e.

e.

e.
t

t
Sexual and reproductive health 201

ne

ne

ne
e.

e.

e.
re

fre

fre
380. Chlamydia trachomatis infection is known to have a high frequency of
transmission.
f
ks

ks

ks
If one partner is infected, what is the likelihood that the other partner will also
oo

oo

oo
be infected?
A. 10%
eb

eb

eb
B. 25%
m

m
C. 50%
D. 75%
E. 100%
t

t
e

ne

ne
.n

e.

e.
e
re

fre

re
sf

sf
ks
k

k
oo

oo

oo
eb

eb

eb
m

m
t

et
ne

ne

n
e.

e.

e.
fre

fre

f re
ks

ks

ks
oo

oo

oo
eb

eb

eb
m

m
t

t
ne

ne

ne
e.

e.

e.
re

re

fre
sf

f
ks

ks
k
oo

oo

oo
eb

eb

eb
m

m
t

et

t
ne

ne
n
e.

e.

e.
t

t
202 Module 15

ne

ne

ne
e.

e.

e.
re

fre

fre
EMQs
f
ks

ks

ks
Options for questions 381–384
oo

oo

oo
A Aciclovir 400 mg orally three times per day for 21 days
eb

eb

eb
B Azithromycin 1 g orally as a single dose
C Ceftriaxone 500 mg intramuscularly plus azithromycin 1 g orally
m

m
D Clindamycin orally 300 mg twice daily for 7 days
E Clotrimazole vaginal pessary 1 g for 5 nights
t

t
F Fluconazole 1 g orally
e

ne

ne
.n

G Fluconazole 150 mg every 72 hours for three doses

e.

e.
e

H Metronidazole 2 g orally as a single dose


re

fre

re
I Metronidazole 500 mg orally twice daily for 14 days
sf

sf
J Nevirapine intravenous stat dose
ks
k

k
K Ofloxacin 400 mg twice daily orally plus oral metronidazole 400 mg twice daily for 14 days
oo

oo

oo
L Valaciclovir 500 mg twice daily for 5 days
eb

eb

eb
M Zidovudine intravenous infusion
m

m
Each of the following clinical scenarios relates to a woman with a pelvic infection. For
each woman, select the single most appropriate initial management from the list above.
Each option may be used once, more than once or not at all.
t

et
ne

ne

n
381. A 26-year-old woman attends the gynaecology emergency services complaining
e.

e.

e.
of painful blisters and ulceration on her vulva for last 2 days. She has dysuria
fre

fre

re
and vaginal discharge. On examination, there is bilateral tender inguinal
lymphadenitis.
f
ks

ks

ks
382. A 21-year-old woman who suffers with alcohol dependence attends the
oo

oo

oo

gynaecology clinic complaining of a persistent watery vaginal discharge. There


eb

eb

eb

is no history of irritation or pruritus. She smokes ten cigarettes a day. On vaginal


examination, there is an offensive fishy-­smelling vaginal discharge.
m

383. A 23-year-old university student complains of persistent purulent vaginal


discharge. On vaginal examination, there is contact bleeding from the
t

t
cervix. Microscopy of a Gram-­stained endocervical swab specimen showed
ne

ne

ne

monomorphic Gram-­negative diplococci within polymorphonuclear leucocytes.


e.

e.

e.

384. A 49-year-old menopausal diabetic woman presents with vulval soreness and
re

re

fre

pruritus, and a non-­offensive thick white vaginal discharge. She gives a history
sf

f
ks

ks

of at least three past similar episodes over the last 12 months and suffers from
k

superficial dyspareunia. On examination, there is erythema and some fissuring


oo

oo

oo

of the vulval skin.


eb

eb

eb
m

m
t

et

t
ne

ne
n
e.

e.

e.
t

t
Sexual and reproductive health 203

ne

ne

ne
e.

e.

e.
re

fre

fre
Options for questions 385–387

f
ks

ks

ks
A Black cohosh
oo

oo

oo
B Clonidine
C Cognitive behavioural therapy (CBT)
eb

eb

eb
D Oestradiol implant
m

m
E Oestriol cream vaginally
F Oral conjugated equine oestrogens
G Oral continuous combined HRT
t

t
e

ne

ne
H Oral cyclical HRT
.n

e.

e.
I Oral oestradiol
e
re

fre

re
J Psychosexual counselling
K St John’s wort
sf

sf
ks
k

k
L Testosterone gel
oo

oo

oo
M Testosterone implant
eb

eb

eb
N Tibolone
O Transdermal continuous combined HRT
m

m
P Transdermal cyclical HRT
Q Transdermal oestrogen
t

et
ne

ne

For each of the following clinical scenarios, choose the single most appropriate treat-

n
ment from the list of options above. Each option may be used once, more than once or
e.

e.

e.
not at all.
fre

fre

385. A 52-year-old woman with a BMI of 31 kg/m2 presents with vasomotor symptoms f re
ks

ks

ks
and vaginal dryness. She has no significant past medical history. Her last
oo

oo

oo

menstrual period was 12 months ago.


eb

eb

eb

386. A 50-year-old woman who previously had a hysterectomy for uterine fibroids
presents with severe menopausal symptoms. She is concerned about taking HRT
m

as her elder sister recently had a stroke.

387. A 54-year-old woman who is taking continuous combined HRT presents with a
t

t
lack of libido.
ne

ne

ne
e.

e.

e.
re

re

fre
sf

f
ks

ks
k
oo

oo

oo
eb

eb

eb
m

m
t

et

t
ne

ne
n
e.

e.

e.
t

t
204 Module 15

ne

ne

ne
e.

e.

e.
re

fre

fre
Options for questions 388–390

f
ks

ks

ks
A Candida albicans
oo

oo

oo
B Chlamydia trachomatis
C Gardnerella vaginalis
eb

eb

eb
D Haemophilus ducreyi
m

m
E Herpes simplex virus
F Herpes zoster virus
G Human papillomavirus
t

t
e

ne

ne
H Molluscum contagiosum virus
.n

e.

e.
I Mycoplasma genitalium
e
re

fre

re
J Neisseria gonorrhoeae
K Phthirus pubis
sf

sf
ks
k

k
L Sarcoptes scabiei
oo

oo

oo
M Treponema pallidum
eb

eb

eb
N Trichomonas vaginalis
m

m
In each of the following scenarios, what is the most likely organism that is being
described? Each option may be used once, more than once or not at all.
t

et
388. A large DNA pox virus that causes a benign epidermal eruption of the skin. The
ne

ne

lesions are usually characteristic, presenting as smooth-­surfaced, firm, dome-­

n
e.

e.

e.
shaped papules with central umbilication.
fre

fre

re
389. A sexually transmitted infection commonly presenting with vulval discharge
and itching, dysuria and offensive odour. Overall, 10–50% of women are f
ks

ks

ks
asymptomatic. Of women that are infected, the urethra is colonised in 90%, and
oo

oo

oo

2% of women will have a ‘strawberry cervix’.


eb

eb

eb

390. Permethrin is the first-­line treatment for infection with this organism. Classic
m

sites of infection include the interdigital folds, the wrists and elbows, and around
the nipples in women.
t

t
ne

ne

ne
e.

e.

e.
re

re

fre
sf

f
ks

ks
k
oo

oo

oo
eb

eb

eb
m

m
t

et

t
ne

ne
n
e.

e.

e.
t

t
Sexual and reproductive health 205

ne

ne

ne
e.

e.

e.
re

fre

fre
Answers
f
ks

ks

ks
SBAs
oo

oo

oo
371. Answer   E Pubococcygeus
eb

eb

eb
Explanation
Vaginismus is the involuntary spasm of the pubococcygeal and associated muscles
m

m
causing painful and difficult penetration of the vagina during intercourse, tampon
insertion or clinical examination.
t

t
Reference
e

ne

ne
.n

Cowan F, Frodsham L. Management of common disorders in psychosexual medicine. The

e.

e.
Obstetrician & Gynaecologist 2015;17:47–53.
e
re

fre

re
372. Answer   D Psychosexual
sf

sf
ks
Explanation
k

k
oo

oo

oo
Most patients with HSDD will have a psychosexual cause but organic causes can
include menopause, depression, drug therapy such as selective serotonin reuptake
eb

eb

eb
inhibitors and tricyclic antidepressants, or it may be acquired, for example after
m

m
bilateral oophrectomy, chemotherapy and irradiation.
Reference
Cowan F, Frodsham L. Management of common disorders in psychosexual medicine. The
t

et
ne

ne

Obstetrician & Gynaecologist 2015;17:47–53.

n
e.

e.

e.
373. Answer   A  Copper intrauterine contraceptive device (IUCD)
fre

fre

re
Explanation
f
Copper is toxic to the ovum and sperm, and thus a copper-­bearing IUCD is effective
ks

ks

ks
immediately after insertion and works primarily by inhibiting fertilisation.
oo

oo

oo

Reference
eb

eb

eb

FSRH. Emergency contraception. FSRH Guideline. Updated December 2017.


m

374. Answer   C  No medical contraindications


Explanation
t

t
UKMEC advises that there are no medical contraindications to levonorgestrel,
ne

ne

ne

including breastfeeding.
e.

e.

e.

Reference
re

re

fre

FSRH. Emergency contraception. FSRH Guideline. Updated December 2017.


sf

f
ks

ks
k

375. Answer   A  2 in 100 women


oo

oo

oo

Explanation
eb

eb

eb

LAM is a temporary contraceptive method that relies on exclusive breastfeeding.


It has a failure rate of around 2 in 100 women.
m

Reference
RCOG. Postpartum family planning. RCOG Best Practice Paper No. 1. June 2015.
t

et

t
ne

ne
n
e.

e.

e.
t

t
206 Module 15

ne

ne

ne
e.

e.

e.
re

fre

fre
376. Answer   C Annually

f
Explanation
ks

ks

ks
Once established on HRT, an annual review is all that is necessary. HRT does not
oo

oo

oo
increase blood pressure and there is no indication to monitor more frequently.
eb

eb

eb
Reference
m

m
Bakour SH, Williamson J. Latest evidence on using hormone replacement therapy in the
menopause. The Obstetrician & Gynaecologist 2015;17:20–8.

377. Answer   D  Nucleic acid amplification test (NAAT)


t

t
e

ne

ne
.n

Explanation

e.

e.
NAATs offer the highest sensitivity for the detection of T. vaginalis. They should
e
re

fre

re
be the test of choice where resources allow and are becoming the current ‘gold
standard’. In-­house polymerase chain reaction (PCR) tests have shown increased
sf

sf
ks
sensitivity in comparison with both microscopy and culture.
k

k
oo

oo

oo
Reference
eb

eb

eb
Sherrard J, Ison C, Moody J, et al. United Kingdom national guideline on the management
of Trichomonas vaginalis. International Journal of STD & AIDS 2014;25:541–9.
m

378. Answer  
Explanation
D  16 months
m
t

et
ne

ne

For women over the age of 50 who do not use hormonal methods, contraception

n
can be stopped after 1 year of amenorrhoea as fertility is unlikely to return. In
e.

e.

e.
women <50 years, contraception should be continued for 2 years, as the return of
fre

fre

re
fertile ovulation is more likely to occur.
f
ks

ks

ks
Reference
oo

oo

oo

Bakour SH, Hatti A, Whalen S. Contraceptive methods and issues around the menopause:
an evidence update. The Obstetrician & Gynaecologist 2017;19:289–97.
eb

eb

eb

379. Answer   E  Combined hormonal contraception and progestogen-­only pill


m

Explanation
See Table 1 in the reference article.
t

t
ne

ne

ne

Reference
Bakour SH, Hatti A, Whalen S. Contraceptive methods and issues around the menopause:
e.

e.

e.

an evidence update. The Obstetrician & Gynaecologist 2017;19:289–97.


re

re

fre
sf

380. Answer   D 75%


ks

ks
k
oo

oo

oo

Explanation
Chlamydia infection has a high frequency of transmission, with concordance rates
eb

eb

eb

of up to 75% of partners being reported.


m

Reference
Nwokolo NC, Dragovic B, Patel S, et al. 2015 UK national guideline for the management of
infection with Chlamydia trachomatis. International Journal of STD & AIDS 2016;27:251–67.
t

et

t
ne

ne
n
e.

e.

e.
t

t
Sexual and reproductive health 207

ne

ne

ne
e.

e.

e.
re

fre

fre
EMQs
f
ks

ks

ks
381. Answer   L  Valaciclovir 500 mg twice daily for 5 days
oo

oo

oo
Explanation
The woman has genital herpes. The other possible medication used to treat
eb

eb

eb
anogenital herpes is aciclovir, but the doses described in the list of options are
incorrect.
m

m
Reference
Patel R, Green J, Clarke E, et al. 2014 UK national guideline for the management of
t

t
anogenital herpes. International Journal of STD & AIDS 2015;26:763–76.
e

ne

ne
.n

e.

e.
382. Answer   D  Clindamycin orally 300 mg twice daily for 7 days
e
re

fre

re
Explanation
sf

sf
ks
The clinical features are typical of bacterial vaginosis and the other treatments
k

k
mentioned in the list are not applicable for bacterial vaginosis. The doses of
oo

oo

oo
metronidazole are incorrect. Both oral and topical metronidazole should not be
eb

eb

eb
used if alcohol cannot be avoided.
m

m
Reference
BASHH. UK national guideline for the management of bacterial vaginosis. Clinical
Effectiveness Group, British Association for Sexual Health and HIV. Updated 2012.
t

et
ne

ne

383. Answer   C  Ceftriaxone 500 mg intramuscularly plus azithromycin 1 g orally

n
e.

e.

e.
Explanation
fre

fre

re
The clinical features and laboratory findings are typical of gonococcal infection.
Ofloxacin and moxifloxacin should be avoided in patients who are at high risk
f
ks

ks

ks
of gonococcal pelvic inflammatory disease (PID) (e.g. when the patient’s partner
oo

oo

oo

has gonorrhoea, in clinically severe disease or following sexual contact abroad),


because of increasing quinolone resistance in the UK. Quinolones should also be
eb

eb

eb

avoided as the first-­line empirical treatment for PID in areas where >5% of PID is
caused by quinolone-­resistant Neisseria gonorrhoeae.
m

Reference
Bignell C, Fitzgerald M on behalf of BASHH. UK national guideline for the management of
t

t
gonorrhoea in adults, 2011. International Journal of STD & AIDS 2011;22:541–7.
ne

ne

ne
e.

e.

e.

384. Answer   G  Fluconazole 150 mg every 72 hours for three doses


re

re

fre

Explanation
sf

This woman has typical history and features of recurrent vulvovaginal candidiasis.
ks

ks
k

The other treatments mentioned are not applicable for recurrent candida
oo

oo

oo

infection.
eb

eb

eb

Reference
m

BASHH. United Kingdom national guideline on the management of vulvovaginal


candidiasis. BASHH Guideline. June 2007.
t

et

t
ne

ne
n
e.

e.

e.
t

t
208 Module 15

ne

ne

ne
e.

e.

e.
re

fre

fre
385. Answer   O  Transdermal continuous combined HRT

f
Explanation
ks

ks

ks
Consider transdermal rather than oral HRT for menopausal women who are at
oo

oo

oo
increased risk of previous venous thromboembolism (VTE), including those with
a BMI >30 kg/m2.
eb

eb

eb
m

m
386. Answer   Q  Transdermal oestrogen
Explanation
Explain to women that taking oral (but not transdermal) oestrogen is associated
t

t
e

ne

ne
with a small increase in the risk of stroke. Also explain that the baseline
.n

population risk of stroke in women aged <60 years is very low.

e.

e.
e
re

fre

re
387. Answer   L  Testosterone gel
sf

sf
ks
Explanation
k

k
Consider testosterone supplementation for menopausal women with low sexual
oo

oo

oo
desire if HRT alone is not effective.
eb

eb

eb
Reference
m

m
NICE. Menopause: diagnosis and management. NICE Guideline (NG23). November 2015.

388. Answer   H  Molluscum contagiosum virus


t

et
ne

ne

n
Reference
e.

e.

e.
Fernando I, Pritchard J, Edwards SK, Grover D. UK national guideline for the management
fre

fre

re
of genital molluscum in adults. International Journal of STD & AIDS 2015;26:687–95.

f
ks

ks

ks
389. Answer   N  Trichomonas vaginalis
oo

oo

oo

Reference
eb

eb

eb

Sherrard J, Ison C, Moody J, et al. United Kingdom national guideline on the management
m

of Trichomonas vaginalis. International Journal of STD & AIDS 2014;25:541–9.

390. Answer   L  Sarcoptes scabiei


t

t
ne

ne

ne

Reference
e.

e.

e.

BASHH. 2016 UK national guideline on the management of scabies. BASHH Guideline.


March 2016.
re

re

fre
sf

f
ks

ks
k
oo

oo

oo
eb

eb

eb
m

m
t

et

t
ne

ne
n
e.

e.

e.
t

t
ne

ne

ne
e.

e.

e.
re

fre

fre
Early pregnancy care
Module
f
ks

ks

ks
16
oo

oo

oo
eb

eb

eb
m

m
t

t
e

ne

ne
SBAs
.n

e.

e.
e

391. What is the diagnostic tool of choice for tubal ectopic pregnancy?
re

fre

re
A. Laparoscopy
sf

sf
ks
B. MRI of pelvis
k

k
C. Serial β-­human chorionic gonadotropin (β-­hCG) measurements
oo

oo

oo
D. Transabdominal ultrasound
eb

eb

eb
E. Transvaginal ultrasound
m

m
392. What proportion of ectopic pregnancies occur in the interstitial portion of the
fallopian tube?
A. 1–6%
t

et
B. 7–10%
ne

ne

n
C. 11–16%
e.

e.

e.
D. 17–20%
fre

fre

re
E. 21–26%
f
ks

ks

ks
393. A woman attends the early pregnancy unit having experienced some discomfort
and vaginal bleeding in early pregnancy. It is 6 weeks since her last menstrual
oo

oo

oo

period. She is actively trying to conceive. A transvaginal ultrasound scan is


eb

eb

eb

performed, but there is no evidence of an intrauterine gestation sac. Her serum


β-­hCG level is 1200 IU/l. The β-­hCG is repeated after 48 hours and is 950 IU/l.
m

What is the most appropriate management?


A. Diagnostic laparoscopy
B. Discharge the patient
t

t
ne

ne

ne

C. Medical therapy with methotrexate


D. Perform a urinary pregnancy test in 2–3 weeks
e.

e.

e.

E. Repeat a further β-­hCG in 48 hours


re

re

fre
sf

f
ks

ks
k
oo

oo

oo
eb

eb

eb
m

209
t

et

t
ne

ne
n
e.

e.

e.
t

t
210 Module 16

ne

ne

ne
e.

e.

e.
re

fre

fre
394. A woman attends the early pregnancy unit with a small amount of per vaginam
spotting. It is 7 weeks since her last menstrual period, but her cycle is irregular.
f
ks

ks

ks
A transvaginal scan is performed, which shows an intrauterine gestation sac
oo

oo

oo
measuring 15 mm in diameter. A follow-­up scan is arranged 14 days later, which
shows a gestation sac with a diameter of 24 mm. There is now a yolk sac visible
eb

eb

eb
with a fetal pole with a crown–rump length of 7 mm, but no fetal heartbeat is
visible.
m

m
What is the diagnosis?
A. Incomplete miscarriage
B. Missed miscarriage
t

t
e

ne

ne
C. Pregnancy of unknown location
.n

D. Pregnancy of unknown viability

e.

e.
e

E. Viable intrauterine pregnancy


re

fre

re
sf

sf
ks
395. What proportion of miscarriages occur after the identification of fetal heart
k

k
activity?
oo

oo

oo
A. <5%
eb

eb

eb
B. 5–8%
C. 9–12%
m

m
D. 13–17%
E. 18–24%
t

et
ne

ne

396. Which objective and validated index is typically used to classify the severity

n
of nausea and vomiting in pregnancy (NVP)?
e.

e.

e.
A. Morning sickness evaluation score
fre

fre

re
B. Korttila scale
C. Pregnancy-unique quantification of emesis (PUQE) f
ks

ks

ks
D. Rhodes index of nausea, vomiting and retching
oo

oo

oo

E. Value of eMesis score (VOMS)


eb

eb

eb

397. At what gestation do the symptoms of NVP typically peak?


m

A. 4 weeks
B. 7 weeks
C. 9 weeks
t

t
D. 12 weeks
ne

ne

ne

E. 15 weeks
e.

e.

e.
re

re

fre

398. Which abnormality of acid–base balance is most commonly seen in women


with NVP?
sf

f
ks

ks

A. Metabolic acidosis
k
oo

oo

oo

B. Metabolic acidosis with respiratory compensation


C. Metabolic alkalosis
eb

eb

eb

D. Respiratory acidosis
m

E. Respiratory alkalosis
t

et

t
ne

ne
n
e.

e.

e.
t

t
Early pregnancy care 211

ne

ne

ne
e.

e.

e.
re

fre

fre
399. What is the most appropriate solution for intravenous rehydration in women
with NVP?
f
ks

ks

ks
A. 5% dextrose
oo

oo

oo
B. Hartmann’s solution
C. Human albumin solution 4%
eb

eb

eb
D. Normal saline 0.9%
m

m
E. Normal saline 0.9% with potassium chloride

400. What triad of findings defines hyperemesis gravidarum in women with severe
protracted NVP?
t

t
e

ne

ne
.n

Electrolyte 2% weight 5% weight Dehydration Ketonuria

e.

e.
e

imbalance loss from loss from


re

fre

re
prepregnancy prepregnancy
sf

sf
ks
A ✓ ✓ ✓
k

k
✓ ✓ ✓
oo

oo

oo
B
C ✓ ✓ ✓
eb

eb

eb
D ✓ ✓ ✓
m

m
E ✓ ✓ ✓
t

et
401. What proportion of women with hyperemesis gravidarum have abnormal thyroid
ne

ne

function tests?

n
e.

e.

e.
A. 10%
fre

fre

re
B. 33%
C. 50%
f
ks

ks

ks
D. 66%
oo

oo

oo

E. 90%
eb

eb

eb

402. A 25-year-old woman with no live births attends the recurrent miscarriage clinic. She
has had three first-­trimester miscarriages in the past. During consultation, she enquires
m

about her chances of a successful pregnancy when she next becomes pregnant.
Which figure should be quoted?
A. 20%
t

t
ne

ne

ne

B. 40%
C. 60%
e.

e.

e.

D. 80%
re

re

fre

E. 100%
sf

f
ks

ks
k
oo

oo

oo
eb

eb

eb
m

m
t

et

t
ne

ne
n
e.

e.

e.
t

t
212 Module 16

ne

ne

ne
e.

e.

e.
re

fre

fre
403. What proportion of parents with recurrent miscarriage will be found to have
a balanced chromosomal translocation if peripheral blood karyotyping is
f
ks

ks

ks
performed?
oo

oo

oo
A. 2%
B. 12%
eb

eb

eb
C. 22%
m

m
D. 32%
E. 42%

404. What is the most common symptom in women who develop gestational
t

t
e

ne

ne
trophoblastic disease after a miscarriage?
.n

e.

e.
A. Breast engorgement
e

B. Nausea and vomiting


re

fre

re
C. Passing tissue vaginally
sf

sf
ks
D. Symptoms of hyperthyroidism
k

k
E. Vaginal bleeding
oo

oo

oo
eb

eb

eb
405. A woman who is rhesus negative is diagnosed with a molar pregnancy and
undergoes surgical evacuation of the uterus. Histological analysis confirms this to
m

m
be a complete molar pregnancy.
In terms of anti-­D prophylaxis, what is the most appropriate management?
A. Administer 250 IU anti-D
t

et
ne

ne

B. Administer 1500 IU anti-D

n
C. Anti-D is not required
e.

e.

e.
D. Arrange paternal genetic testing
fre

fre

re
E. Perform a Kleihauer test
f
ks

ks

ks
oo

oo

oo
eb

eb

eb
m

m
t

t
ne

ne

ne
e.

e.

e.
re

re

fre
sf

f
ks

ks
k
oo

oo

oo
eb

eb

eb
m

m
t

et

t
ne

ne
n
e.

e.

e.
t

t
Early pregnancy care 213

ne

ne

ne
e.

e.

e.
re

fre

fre
EMQs
f
ks

ks

ks
Options for questions 406–408
oo

oo

oo
A Dilute Russell viper venom test
eb

eb

eb
B Glucose tolerance test
C Karyotyping of products of conception
m

m
D No test required
E Ovarian autoantibodies
t

t
F Parental peripheral blood karyotyping
e

ne

ne
.n

G Pelvic ultrasound scan

e.

e.
e

H Peripheral blood natural killer cell testing


re

fre

re
I Serum cytokine levels
sf

sf
J Thrombophilia screen
ks
k

k
K Thyroid function tests
oo

oo

oo
L Thyroid peroxidase antibodies
eb

eb

eb
M TORCH (toxoplasmosis, rubella virus, cytomegalovirus, herpes simplex virus and HIV) screen
m

m
N Uterine natural killer cell testing

For each of the following clinical scenarios, choose the single next most appropriate
t

et
test from the options listed. Each option may be used once, more than once or not at all.
ne

ne

n
406. A 32-year-old woman attends the miscarriage clinic for a follow-­up appointment.
e.

e.

e.
She has previously been investigated for three recurrent first-­trimester losses (all
fre

fre

re
tests according to RCOG guidance), but now presents with a further pregnancy
loss at 16 weeks of gestation. f
ks

ks

ks
oo

oo

oo

407. A woman attends the miscarriage clinic after three consecutive miscarriages. All
investigations are normal. She subsequently has a fourth miscarriage, which is
eb

eb

eb

managed surgically.
m

408. A 38-year-old woman is referred by her GP to the miscarriage clinic after two
consecutive miscarriages.
t

t
ne

ne

ne
e.

e.

e.
re

re

fre
sf

f
ks

ks
k
oo

oo

oo
eb

eb

eb
m

m
t

et

t
ne

ne
n
e.

e.

e.
t

t
214 Module 16

ne

ne

ne
e.

e.

e.
re

fre

fre
Options for questions 409–411

f
ks

ks

ks
A Clinical examination
oo

oo

oo
B CT scan
C Hysteroscopy
eb

eb

eb
D Laparoscopy
m

m
E Laparotomy
F MRI scan
G No diagnostic test required
t

t
e

ne

ne
H Serum α-­fetoprotein (AFP)
.n

Serum β-­human chorionic gonadotropin (β-hCG)

e.

e.
I
e
re

fre

re
J Serum progesterone
K Transabdominal ultrasound scan
sf

sf
ks
k

k
L Transperineal ultrasound
oo

oo

oo
M Transvaginal ultrasound scan
eb

eb

eb
For each of the following scenarios, what is the most appropriate investigation from
m

m
the options listed? Each option may be used once, more than once or not at all.

409. A 35-year-old woman who has had three previous caesarean sections presents
with scar pain and vaginal bleeding at 7 weeks of gestation. What is the primary
t

et
ne

ne

diagnostic modality for diagnosing caesarean scar pregnancies?

n
e.

e.

e.
410. A 33-year-old woman in her fourth pregnancy is referred to an obstetric
fre

fre

re
consultant following a late booking scan. It is 16 weeks since her last menstrual
period. An ultrasound scan had shown an empty uterus but a gestation sac f
ks

ks

ks
outside the uterus surrounded by loops of bowel. What diagnostic modality can
oo

oo

oo

be a useful adjunct in advanced abdominal pregnancy?


eb

eb

eb

411. A 29-year-old woman attends the early pregnancy unit following in vitro
fertilisation (IVF) treatment. She has mild abdominal tenderness and a small
m

amount of vaginal bleeding. Her embryo transfer was 5 weeks previously and
her pregnancy test is positive. The patient underwent IVF treatment due to tubal
disease secondary to Chlamydia infection. What would be the diagnostic tool of
t

t
ne

ne

ne

choice for ectopic pregnancy in this situation?


e.

e.

e.
re

re

fre
sf

f
ks

ks
k
oo

oo

oo
eb

eb

eb
m

m
t

et

t
ne

ne
n
e.

e.

e.
t

t
Early pregnancy care 215

ne

ne

ne
e.

e.

e.
re

fre

fre
Options for questions 412–414

f
ks

ks

ks
A Cornual resection
oo

oo

oo
B Dilation and curettage
C Evacuation of retained products of conception
eb

eb

eb
D Expectant management
m

m
E Hysteroscopic resection
F Intramuscular methotrexate
G Laparoscopic salpingectomy
t

t
e

ne

ne
H Laparoscopic salpingotomy
.n

e.

e.
I Laparoscopy and proceed
e
re

fre

re
J Laparotomy and proceed
K Manual vacuum aspiration
sf

sf
ks
k

k
L No treatment required
oo

oo

oo
M Potassium injected into the pregnancy under ultrasound guidance
eb

eb

eb
N Uterine artery embolisation
m

m
For each of the following scenarios, what is the most appropriate management from
the options listed? Each option may be used once, more than once or not at all.
t

et
412. A woman presents to the early pregnancy unit with pain and bleeding at 6 weeks
ne

ne

of gestation. An ultrasound scan is performed, which shows an empty uterus with

n
e.

e.

e.
a barrel-­shaped cervix. There is a gestation sac below the level of the internal os
with blood flow around the sac visible using colour Doppler.
fre

fre

413. A woman with a history of left ectopic pregnancy managed by salpingotomy f re


ks

ks

ks
attends the early pregnancy unit with right-­sided pain and vaginal bleeding.
oo

oo

oo

It is 7 weeks since her last menstrual period. A transvaginal ultrasound scan


demonstrates a mass in the right adnexa, which is tender. Her serum β-­hCG level
eb

eb

eb

is 7800 IU/l.
m

414. A woman with a previous history of ectopic pregnancy attends the early
pregnancy unit for a reassurance scan at 7 weeks of gestation. She has no
symptoms. A transvaginal scan shows an empty uterus but a 32 mm left adnexal
t

t
ne

ne

ne

cystic mass separate from the ovary. A yolk sac and fetal pole are seen, but there
is no fetal heartbeat. There is no free fluid. Her serum β-­hCG is 3500 IU/l. This is
e.

e.

e.

repeated 48 hours later and is 3800 IU/l.


re

re

fre
sf

f
ks

ks
k
oo

oo

oo
eb

eb

eb
m

m
t

et

t
ne

ne
n
e.

e.

e.
t

t
216 Module 16

ne

ne

ne
e.

e.

e.
re

fre

fre
Options for questions 415–417

f
ks

ks

ks
A 23,X
oo

oo

oo
B 23,Y
C 45,XO
eb

eb

eb
D 46,XX
m

m
E 46,XY
F 47,XXO
G 47,XXX
t

t
e

ne

ne
H 47,XXY
.n

e.

e.
I 47,XYY
e
re

fre

re
J 69,XXX
K 69,XXY
sf

sf
ks
k

k
L 69,XYY
oo

oo

oo
M 69,YYY
eb

eb

eb
For each of the following clinical scenarios, what is the most likely karyotype from the
m

m
options listed? Each option may be used once, more than once or not at all.

415. A woman presents with hyperemesis and bleeding to the gynaecology assessment
unit. An ultrasound scan suggests a complete molar pregnancy and this is
t

et
ne

ne

confirmed following histopathological analysis of the evacuated products of

n
conception.
e.

e.

e.
What is the karyotype of the sperm that fertilised the oocyte?
fre

fre

f
416. A woman attends for a routine dating scan at 12 weeks of gestation. This shows
re
ks

ks

ks
an enlarged placenta with cystic spaces and a small fetus. A surgical evacuation is
oo

oo

oo

performed, which confirms a partial molar pregnancy.


What is the most likely karyotype?
eb

eb

eb

417. A recent immigrant to the UK presents with shortness of breath, haemoptysis and
m

vaginal bleeding. She describes a ‘miscarriage’ a few months earlier. A pregnancy


test is strongly positive and an ultrasound scan shows a haemorrhagic cystic mass
in the uterus. A chest X-­ray shows multiple nodules in both lungs.
t

t
ne

ne

ne

What is the most likely karyotype of the tumour?


e.

e.

e.
re

re

fre
sf

f
ks

ks
k
oo

oo

oo
eb

eb

eb
m

m
t

et

t
ne

ne
n
e.

e.

e.
t

t
Early pregnancy care 217

ne

ne

ne
e.

e.

e.
re

fre

fre
Options for questions 418–420

f
ks

ks

ks
A <1%
B 10%
oo

oo

oo
C 25%
eb

eb

eb
D 33%
m

m
E 40%
F 50%
G 66%
t

t
e

ne

ne
H 75%
.n

I 80%

e.

e.
e

J 90%
re

fre

re
sf

sf
For each of the following clinical scenarios, what is the closest risk of each event
ks
k

k
occurring from the options listed? Each option may be used once, more than once or
oo

oo

oo
not at all.
eb

eb

eb
418. The proportion of partial moles that are triploid in origin.
m

m
419. The proportion of ectopic pregnancies that are cervical pregnancies.

420. The risk of miscarriage in a recognised pregnancy in a woman aged 35–39 years.
t

et
ne

ne

n
e.

e.

e.
fre

fre

f re
ks

ks

ks
oo

oo

oo
eb

eb

eb
m

m
t

t
ne

ne

ne
e.

e.

e.
re

re

fre
sf

f
ks

ks
k
oo

oo

oo
eb

eb

eb
m

m
t

et

t
ne

ne
n
e.

e.

e.
t

t
218 Module 16

ne

ne

ne
e.

e.

e.
re

fre

fre
Answers
f
ks

ks

ks
SBAs
oo

oo

oo
391. Answer  E  Transvaginal ultrasound
eb

eb

eb
Explanation
Transvaginal ultrasound is the diagnostic tool of choice for tubal ectopic
m

m
pregnancy.
Reference
t

t
RCOG. Diagnosis and management of ectopic pregnancy. RCOG GTG No. 21. November 2016.
e

ne

ne
.n

e.

e.
392. Answer  A 1–6%
e
re

fre

re
Explanation
sf

sf
Interstitial ectopic pregnancy is when implantation occurs in the interstitial
ks
k

k
portion of the fallopian tube and occurs in 1–6% of ectopic pregnancies.
oo

oo

oo
Reference
eb

eb

eb
Al-Memar M, Kirk E, Bourne T. The role of ultrasonography in the diagnosis and
management of early pregnancy complications. The Obstetrician & Gynaecologist
m

m
2015;17:173–81.

393. Answer  D  Perform a urinary pregnancy test in 2–3 weeks


t

et
ne

ne

Explanation

n
e.

e.

e.
See Figure 4 in the reference article.
fre

fre

re
Reference
Al-Memar M, Kirk E, Bourne T. The role of ultrasonography in the diagnosis and f
ks

ks

ks
management of early pregnancy complications. The Obstetrician & Gynaecologist
oo

oo

oo

2015;17:173–81.
eb

eb

eb

394. Answer  B  Missed miscarriage


m

A common-­sense approach to the diagnosis of miscarriage was proposed


following a consensus conference of the US Society for Radiologists in Ultrasound
in 2012. The authors proposed that on a follow-­up transvaginal ultrasound scan, a
t

t
diagnosis of pregnancy failure may be made on the basis of the following findings:
ne

ne

ne

• An embryo with a crown–rump length of >7 mm with no heartbeat


e.

e.

e.

• A mean gestational sac diameter of >25 mm with no embryo


re

re

fre

• Absence of an embryo with a heartbeat if >2 weeks has elapsed following a


sf

scan that showed a gestational sac without a yolk sac


ks

ks
k

• Absence of an embryo with a heartbeat >11 days after a scan that showed a
oo

oo

oo

gestational sac and yolk sac.


eb

eb

eb

Reference
m

Al-Memar M, Kirk E, Bourne T. The role of ultrasonography in the diagnosis and


management of early pregnancy complications. The Obstetrician & Gynaecologist
2015;17:173–81.
t

et

t
ne

ne
n
e.

e.

e.
t

t
Early pregnancy care 219

ne

ne

ne
e.

e.

e.
re

fre

fre
395. Answer  A  <5%
f Explanation
ks

ks

ks
Less than 5% of miscarriages occur after the identification of fetal heart activity.
oo

oo

oo
Reference
eb

eb

eb
Saraswat L, Ashok PW, Mathur M. Medical management of miscarriage. The Obstetrician &
m

m
Gynaecologist 2014;16:79–85.

396. Answer  C  Pregnancy-­unique quantification of emesis (PUQE)


t

t
Explanation
e

ne

ne
.n

An objective and validated index of nausea and vomiting such as the PUQE score

e.

e.
can be used to classify the severity of NVP.
e
re

fre

re
The Rhodes index was originally validated to measure nausea and vomiting
in chemotherapy patients, including assessment of physical symptoms and the
sf

sf
ks
resulting stress, but has subsequently been used for NVP.
k

k
oo

oo

oo
Reference
eb

eb

eb
RCOG. The management of nausea and vomiting of pregnancy and hyperemesis
gravidarum. RCOG GTG No. 69. June 2016.
m

397. Answer 
Explanation
C  9 weeks
m
t

et
ne

ne

NVP typically starts between 4 and 7 weeks of gestation, peaking at approximately

n
week 9 and resolving by week 20 in 90% of women.
e.

e.

e.
fre

fre

re
Reference
RCOG. The management of nausea and vomiting of pregnancy and hyperemesis
f
ks

ks

ks
gravidarum. RCOG GTG No. 69. June 2016.
oo

oo

oo

398. Answer  C  Metabolic alkalosis


eb

eb

eb

Explanation
m

NVP and hyperemesis gravidarum are associated with hyponatraemia,


hypokalaemia, low serum urea, a raised haematocrit and ketonuria with a
metabolic hypochloraemic alkalosis.
t

t
ne

ne

ne

Reference
RCOG. The management of nausea and vomiting of pregnancy and hyperemesis
e.

e.

e.

gravidarum. RCOG GTG No. 69. June 2016.


re

re

fre
sf

399. Answer  E  Normal saline 0.9% with potassium chloride


ks

ks
k
oo

oo

oo

Explanation
Normal saline with additional potassium chloride in each bag, with
eb

eb

eb

administration guided by daily monitoring of electrolytes, is the most appropriate


method of intravenous hydration for NVP.
m

Reference
RCOG. The management of nausea and vomiting of pregnancy and hyperemesis
t

et

gravidarum. RCOG GTG No. 69. June 2016.


ne

ne
n
e.

e.

e.
t

t
220 Module 16

ne

ne

ne
e.

e.

e.
re

fre

fre
400. Answer  B  Electrolyte imbalance, 5% weight loss from prepregnancy weight
and dehydration
f
ks

ks

ks
Explanation
oo

oo

oo
Hyperemesis gravidarum is characterised by severe, protracted nausea and
vomiting associated with weight loss of >5% of prepregnancy weight, dehydration
eb

eb

eb
and electrolyte imbalances.
m

m
Reference
RCOG. The management of nausea and vomiting of pregnancy and hyperemesis
gravidarum. RCOG GTG No. 69. June 2016.
t

t
e

ne

ne
.n

401. Answer  D 66%

e.

e.
e
re

fre

re
Explanation
In two-­thirds of patients with hyperemesis gravidarum, there may be abnormal
sf

sf
ks
thyroid function tests (based on a structural similarity between thyroid-­
k

k
oo

oo

oo
stimulating hormone (TSH) and β-­hCG) with a biochemical thyrotoxicosis and
raised free thyroxine levels with or without a suppressed TSH level. These patients
eb

eb

eb
rarely have thyroid antibodies and are euthyroid clinically. The biochemical
thyrotoxicosis resolves as the hyperemesis gravidarum improves, and treatment
m

m
with antithyroid drugs is inappropriate.
Reference
t

et
ne

ne

RCOG. The management of nausea and vomiting of pregnancy and hyperemesis

n
gravidarum. RCOG GTG No. 69. June 2016.
e.

e.

e.
fre

fre

re
402. Answer  C 60%
Explanation f
ks

ks

ks
Previous reproductive history is an independent predictor of future pregnancy
oo

oo

oo

outcome. The risk of a further miscarriage increases after each successive


pregnancy loss, reaching approximately 40% after three consecutive pregnancy
eb

eb

eb

losses, and the prognosis worsens with increasing maternal age.


m

Reference
RCOG. The investigation and treatment of couples with recurrent first trimester and
second-­trimester miscarriage. RCOG GTG No. 17. April 2011.
t

t
ne

ne

ne

403. Answer  A 2%


e.

e.

e.
re

re

fre

Explanation
A recent retrospective UK audit of four UK centres over periods of 5–30 years
sf

f
ks

ks

reported that balanced translocations were found in 1.99% (406 out of 20,432) of
k

parents with recurrent miscarriage.


oo

oo

oo

Reference
eb

eb

eb

RCOG. The investigation and treatment of couples with recurrent first trimester and
m

second-­trimester miscarriage. RCOG GTG No. 17. April 2011.


t

et

t
ne

ne
n
e.

e.

e.
t

t
Early pregnancy care 221

ne

ne

ne
e.

e.

e.
re

fre

fre
404. Answer  E  Vaginal bleeding

f Explanation
ks

ks

ks
Several case series have shown that vaginal bleeding is the most common
oo

oo

oo
presenting symptom of gestational trophoblastic disease diagnosed after
miscarriage, therapeutic termination of pregnancy or postpartum.
eb

eb

eb
Reference
m

m
RCOG. The management of gestational trophoblastic disease. RCOG GTG No. 38. March 2010.

405. Answer  C  Anti-­D is not required


t

t
e

ne

ne
.n

Explanation

e.

e.
e

Because of poor vascularisation of the chorionic villi and absence of the anti-­D
re

fre

re
antigen in complete moles, anti-­D prophylaxis is not required. It is, however,
required for partial moles. Confirmation of the diagnosis of a complete molar
sf

sf
ks
pregnancy may not occur for some time after evacuation and so administration of
k

k
oo

oo

oo
anti-­D could be delayed when required, within an appropriate timeframe.
eb

eb

eb
Reference
RCOG. The management of gestational trophoblastic disease. RCOG GTG No. 38. March
m

m
2010.

EMQs
t

et
ne

ne

406. Answer  J  Thrombophilia screen

n
e.

e.

e.
Explanation
fre

fre

re
Women with a second-­trimester miscarriage should be screened for inherited
thrombophilias including factor V Leiden, factor II (prothrombin) gene mutation
f
ks

ks

ks
and protein S.
oo

oo

oo

407. Answer  C  Karyotyping of products of conception


eb

eb

eb

Explanation
m

Cytogenetic analysis should be performed on the products of conception of the


third and subsequent consecutive miscarriage(s).
t

t
408. Answer  D  No test required
ne

ne

ne

Explanation
e.

e.

e.

Recurrent miscarriage, defined as the loss of three or more consecutive


re

re

fre

pregnancies, affects 1% of couples trying to conceive. There is no need to


sf

investigate after two miscarriages.


ks

ks
k
oo

oo

oo

Reference
RCOG. The investigation and treatment of couples with recurrent first trimester and
eb

eb

eb

second-­trimester miscarriage. RCOG GTG No. 17. April 2011.


m

m
t

et

t
ne

ne
n
e.

e.

e.
t

t
222 Module 16

ne

ne

ne
e.

e.

e.
re

fre

fre
409. Answer  M  Transvaginal ultrasound scan

f
Explanation
ks

ks

ks
Clinicians should be aware that ultrasound is the primary diagnostic modality,
oo

oo

oo
using a transvaginal approach, supplemented by transabdominal imaging if
required.
eb

eb

eb
m

m
410. Answer  F  MRI scan
Explanation
MRI can be a useful diagnostic adjunct in advanced abdominal pregnancy and
t

t
e

ne

ne
can help to plan the surgical approach.
.n

e.

e.
e

411. Answer  M  Transvaginal ultrasound scan


re

fre

re
Explanation
sf

sf
ks
A transvaginal ultrasound scan is the diagnostic tool of choice for tubal ectopic
k

k
pregnancies.
oo

oo

oo
Reference
eb

eb

eb
RCOG. Diagnosis and management of ectopic pregnancy. RCOG GTG No. 21. November
m

m
2016.

412. Answer  F  Intramuscular methotrexate


t

et
ne

ne

Explanation

n
Medical management with methotrexate can be considered for cervical
e.

e.

e.
pregnancy. Surgical methods of management are associated with a high
fre

fre

re
failure rate and should be reserved for those women suffering life-­threatening
bleeding. f
ks

ks

ks
A retrospective review of 62 cases of cervical ectopic pregnancy estimated
oo

oo

oo

the efficacy of systemic methotrexate administration in the treatment of cervical


ectopic pregnancy to be approximately 91%.
eb

eb

eb
m

413. Answer  H  Laparoscopic salpingotomy


Explanation
A laparoscopic surgical approach is preferable to an open approach. In women
t

t
ne

ne

ne

with a history of fertility-­reducing factors (previous ectopic pregnancy,


contralateral tubal damage, previous abdominal surgery, previous pelvic
e.

e.

e.

inflammatory disease), salpingotomy should be considered.


re

re

fre

Success rates for methotrexate are only 38% if β-­hCG levels are >5000 IU/l.
sf

f
ks

ks
k

414. Answer  F  Intramuscular methotrexate


oo

oo

oo

Explanation
eb

eb

eb

NICE recommends that methotrexate should be the first-­line management for


women who are able to return for follow-­up and who have:
m

• No significant pain
• An unruptured ectopic pregnancy with a mass smaller than 35 mm with no
visible heartbeat
t

et

t
ne

ne
n
e.

e.

e.
t

t
Early pregnancy care 223

ne

ne

ne
e.

e.

e.
re

fre

fre
• A serum β-­hCG between 1500 and 5000 IU/l
• No intrauterine pregnancy (as confirmed by an ultrasound scan).
f
ks

ks

ks
Expectant management is not appropriate as the serum β-­hCG level is rising.
oo

oo

oo
Reference
eb

eb

eb
RCOG. Diagnosis and management of ectopic pregnancy. RCOG GTG No. 21. November
2016.
m

m
415. Answer  A 23,X
Explanation
t

t
e

ne

ne
Complete moles usually (75–80%) arise as a consequence of duplication of a
.n

e.

e.
single sperm following fertilisation of an ‘empty’ ovum.
e

Two Y chromosomes are not compatible with development, so as duplication


re

fre

re
is the most likely cause, the sperm must have been 23X.
sf

sf
ks
k

k
Reference
oo

oo

oo
RCOG. The management of gestational trophoblastic disease. RCOG GTG No. 38. March 2010.
eb

eb

eb
416. Answer  K 69,XXY
m

m
Explanation
The frequencies of the karyotypes of partial moles are:
t

et
ne

ne

69,XXY 70%

n
69,XXX 27%
e.

e.

e.
69,XYY 3%
fre

fre

f re
ks

ks

ks
Reference
oo

oo

oo

Nyberg DA, McGahan JP, Pretorius DH, Pilu G (eds.). Diagnostic Imaging of Fetal
Anomalies. Philadelphia: Lippincott Williams & Wilkins, 2003.
eb

eb

eb

417. Answer  D 46,XX


m

This is likely to be a choriocarcinoma, and 50% of these tumours arise from a


hydatidiform mole. The majority (90%) of hydatidiform moles have the karyotype
t

t
46,XX.
ne

ne

ne

Reference
e.

e.

e.

Fox H, Buckley CH. Pathology for Gynaecologists. 2nd edn. London: Hodder Arnold, 1991.
re

re

fre
sf

418. Answer  J 90%


ks

ks
k
oo

oo

oo

Explanation
Partial moles are usually (90%) triploid in origin, with two sets of paternal haploid
eb

eb

eb

genes and one set of maternal haploid genes.


m

Reference
RCOG. The management of gestational trophoblastic disease. RCOG GTG No. 38. March 2010.
t

et

t
ne

ne
n
e.

e.

e.
t

t
224 Module 16

ne

ne

ne
e.

e.

e.
re

fre

fre
419. Answer  A  <1%
f
Explanation
ks

ks

ks
Cervical pregnancies are rare, accounting for <1% of all ectopic gestations.
oo

oo

oo
Reference
eb

eb

eb
RCOG. Diagnosis and management of ectopic pregnancy. RCOG GTG No. 21. November
m

m
2016.

420. Answer  C 25%


t

t
Explanation
e

ne

ne
.n

The age-­related risks of miscarriage in recognised pregnancies are:

e.

e.
e
re

fre

re
12–19 years 13%
sf

sf
20–24 years 11%
ks
k

k
25–29 years 12%
oo

oo

oo
30–34 years 15%
eb

eb

eb
35–39 years 25%
40–44 years 51%
m

m
≥45 years 93%
t

et
ne

ne

Reference

n
RCOG. The investigation and treatment of couples with recurrent first trimester and
e.

e.

e.
second-­trimester miscarriage. RCOG GTG No. 17. April 2011.
fre

fre

f re
ks

ks

ks
oo

oo

oo
eb

eb

eb
m

m
t

t
ne

ne

ne
e.

e.

e.
re

re

fre
sf

f
ks

ks
k
oo

oo

oo
eb

eb

eb
m

m
t

et

t
ne

ne
n
e.

e.

e.
t

t
ne

ne

ne
e.

e.

e.
re

fre

fre
Gynaecological oncology
Module
f
ks

ks

ks
17
oo

oo

oo
eb

eb

eb
m

m
t

t
e

ne

ne
SBAs
.n

e.

e.
e

421. To which group of lymph nodes does a squamous cell carcinoma of the vulva
re

fre

re
initially spread?
sf

sf
ks
A. External iliac
k

k
B. Femoral
oo

oo

oo
C. Inguinal
eb

eb

eb
D. Internal iliac
E. Para-aortic
m

m
422. A 70-year-old woman presents with vulval itching and bleeding. On examination,
a suspicious lesion is identified and an excision biopsy is performed. The lesion is
t

et
confirmed as a squamous cell carcinoma with depth of invasion of <1 mm.
ne

ne

n
What is the likelihood of lymph node involvement?
e.

e.

e.
A. <1%
fre

fre

re
B. 5%
C. 10%
f
ks

ks

ks
D. 15%
oo

oo

oo

E. 20%
eb

eb

eb

423. Which two chemotherapeutic agents are most commonly used in the management
m

of ovarian cancer?

Carboplatin Chlorambucil Cisplatin Cyclophosphamide Paclitaxel


t

t
A ✓ ✓
ne

ne

ne

B ✓ ✓
e.

e.

e.

C ✓ ✓
re

re

fre

D ✓ ✓
sf

f
ks

ks

E ✓ ✓
k
oo

oo

oo
eb

eb

eb
m

225
t

et

t
ne

ne
n
e.

e.

e.
t

t
226 Module 17

ne

ne

ne
e.

e.

e.
re

fre

fre
424. From which type of tumour does a secondary tumour of the ovary most
commonly originate?
f
ks

ks

ks
A. Breast
oo

oo

oo
B. Colon
C. Endometrium
eb

eb

eb
D. Lung
m

m
E. Pancreas

425. Which tumour marker(s) should be used to assess the risk of malignancy in
postmenopausal ovarian cysts?
t

t
e

ne

ne
.n

α-Fetoprotein Cancer Cancer Carcinoembryonc Lactate

e.

e.
e

(AFP) antigen antigen antigen (CEA) dehydrogenase


re

fre

re
125 19-­9 (LDH)
sf

sf
(CA125) (CA19-9)
ks
k

k
A ✓ ✓ ✓
oo

oo

oo
B ✓
eb

eb

eb
C ✓ ✓ ✓
m

m
D ✓ ✓ ✓
E ✓ ✓ ✓
t

et
426. A 65-year-old woman presented with lower abdominal pain and bloating. A pelvic
ne

ne

n
ultrasound scan demonstrated a complex ovarian cyst and this, with a serum
e.

e.

e.
CA125, gave a risk of malignancy index (RMI) score of 280.
fre

fre

re
What further imaging is now required?
A. CT of abdomen and pelvis f
ks

ks

ks
B. CT of pelvis
oo

oo

oo

C. MRI of abdomen and pelvis


D. MRI of pelvis
eb

eb

eb

E. Positron emission tomography (PET)-CT of thorax, abdomen and pelvis


m

427. For an ovarian mass to be defined as complex, one or more features need to be
present.
What are these features?
t

t
ne

ne

ne

Anechoic Papillary Posterior Complete Solid


e.

e.

e.

fluid projections attenuation septation/ nodules


re

re

fre

multilocular
sf

f
ks

ks

A ✓ ✓ ✓
k

✓ ✓ ✓
oo

oo

oo

B
C ✓ ✓ ✓
eb

eb

eb

D ✓ ✓ ✓
m

E ✓ ✓ ✓
t

et

t
ne

ne
n
e.

e.

e.
t

t
Gynaecological oncology 227

ne

ne

ne
e.

e.

e.
re

fre

fre
428. Which type of vulval intraepithelial neoplasia (VIN) is associated with the greatest
risk of progression to vulval squamous cell carcinoma?
f
ks

ks

ks
A. Differentiated type
oo

oo

oo
B. Non-classical type
C. Undifferentiated type
eb

eb

eb
D. Usual type (HPV-16)
m

m
E. Warty type (HPV-2)

429. A 62-year-old woman is taking tamoxifen following surgery for breast cancer. She
has experienced some vaginal bleeding. A transvaginal ultrasound scan measures
t

t
e

ne

ne
her endometrial thickness as 7 mm.
.n

How should this be investigated?

e.

e.
e

A. Dilation and curettage


re

fre

re
B. Hysteroscopy and targeted biopsy
sf

sf
ks
C. MRI scan
k

k
D. No investigation required
oo

oo

oo
E. Pipelle endometrial biopsy
eb

eb

eb
430. What is the estimated reduction in risk of ovarian cancer for each year of use of
m

m
the combined oral contraceptive pill (COCP)?
A. 2%
B. 7%
t

et
ne

ne

C. 12%

n
D. 17%
e.

e.

e.
E. 22%
fre

fre

431. In which age group do malignant ovarian germ cell tumours (MOGCTs) most f re
ks

ks

ks
commonly occur?
oo

oo

oo

A. 0–20 years
eb

eb

eb

B. 21–40 years
C. 41–60 years
m

D. 61–80 years
E. ≥81 years
t

t
ne

ne

ne
e.

e.

e.
re

re

fre
sf

f
ks

ks
k
oo

oo

oo
eb

eb

eb
m

m
t

et

t
ne

ne
n
e.

e.

e.
t

t
228 Module 17

ne

ne

ne
e.

e.

e.
re

fre

fre
432. A 20-year-old woman is found to have a solid ovarian mass, and tests for serum
tumour markers reveal an elevated LDH. Following a multidisciplinary team
f
ks

ks

ks
discussion, there is the suspicion of a germ cell tumour.
oo

oo

oo
What is the most appropriate surgical management?
eb

eb

eb
Laparo- Laparos- Unilateral Bilateral Omental Selective Systematic
tomy copy oophorec- oophorec- biopsy removal removal
m

m
tomy tomy of lymph of lymph
nodes nodes
A ✓ ✓ ✓ ✓
t

t
e

ne

ne
B ✓ ✓ ✓ ✓
.n

e.

e.
C ✓ ✓ ✓ ✓
e
re

fre

re
D ✓ ✓ ✓
sf

sf
E ✓ ✓ ✓
ks
k

k
oo

oo

oo
433. What are the two most common types of malignant ovarian germ cell tumours?
eb

eb

eb
m

m
Choriocarcinoma Dysgerminoma Embryonal Endodermal Immature
tumour sinus teratoma
tumour
✓ ✓
t

et
A
ne

ne

✓ ✓

n
B
e.

e.

e.
C ✓ ✓
fre

fre

re
D ✓ ✓
E ✓ ✓
f
ks

ks

ks
oo

oo

oo

434. What is the strongest known risk factor for ovarian cancer?
eb

eb

eb

A. Family history of breast cancer


m

B. Family history of ovarian cancer


C. Fertility treatment
D. Nulliparity
t

t
E. Smoking
ne

ne

ne
e.

e.

e.

435. What is the current recommendation regarding screening for ovarian cancer in
women with no family history of the disease?
re

re

fre

A. Annual calculation of RMI


sf

f
ks

ks

B. Annual pelvic ultrasound scan


k
oo

oo

oo

C. Annual serum CA125


D. Annual serum CA125 and pelvic ultrasound scans on alternate years
eb

eb

eb

E. No screening recommended
m

m
t

et

t
ne

ne
n
e.

e.

e.
t

t
Gynaecological oncology 229

ne

ne

ne
e.

e.

e.
re

fre

fre
436. For women who are carriers of a BRCA gene mutation and go on to have risk-­
reducing surgery (bilateral salpingo-­oophorectomy), what is the likelihood of the
f
ks

ks

ks
subsequent development of a primary peritoneal cancer?
oo

oo

oo
A. 0.5%
B. 2%
eb

eb

eb
C. 7%
m

m
D. 12%
E. 15%

437. With regard to squamous cell carcinoma of the cervix, what is believed to be
t

t
e

ne

ne
the minimum time span between infection by HPV and the development of a
.n

premalignant lesion with true malignant potential?

e.

e.
e

A. 1 year
re

fre

re
B. 3 years
sf

sf
ks
C. 5 years
k

k
D. 7 years
oo

oo

oo
E. 9 years
eb

eb

eb
438. What is the principle role of radiotherapy in the treatment of endometrial cancer?
m

m
A. It is the preferred treatment option in early stage disease
B. Postoperatively as an adjuvant therapy
C. Postoperatively to sensitise any remaining disease to chemotherapy
t

et
ne

ne

D. Preoperatively to reduce the size of large tumours

n
E. There is no place for radiotherapy in the treatment of endometrial cancer
e.

e.

e.
fre

fre

re
439. Which isotopes are used in modern-­day brachytherapy for the treatment of
gynaecological cancers? f
ks

ks

ks
oo

oo

oo

Caesium Cobalt Iridium Radium


eb

eb

eb

A ✓ ✓
B ✓ ✓
m

C ✓ ✓
D ✓ ✓
t

t
E ✓ ✓
ne

ne

ne
e.

e.

e.

440. Which subtype of ovarian cancer is most commonly associated with bowel
re

re

fre

obstruction?
sf

f
ks

ks

A. Brenner tumour
k

B. Dysgerminoma
oo

oo

oo

C. Epithelial cancer
eb

eb

eb

D. Sex-cord stromal tumour


E. Teratoma
m

m
t

et

t
ne

ne
n
e.

e.

e.
t

t
230 Module 17

ne

ne

ne
e.

e.

e.
re

fre

fre
EMQs
f
ks

ks

ks
Options for questions 441–443
oo

oo

oo
A IA
eb

eb

eb
B IB
C I C1
m

m
D I C2
E I C3
t

t
F II
e

ne

ne
.n

G III A1

e.

e.
e

H III A2
re

fre

re
I III B
sf

sf
J III C1
ks
k

k
K III C2
oo

oo

oo
L IV A
eb

eb

eb
M IV B
m

m
For each of the following clinical scenarios, choose the single most appropriate stage
of disease from the list of options given above. Each option may be used once, more than
once or not at all.
t

et
ne

ne

n
441. A 60-year-old woman has a laparoscopic bilateral oophorectomy for a persistent
e.

e.

e.
ovarian cyst. Her serum CA125 is 50 IU/ml. The ovarian capsule is noted to
fre

fre

re
be intact, but during the course of the operation, the cyst ruptures. Peritoneal
washings prior to the rupture are clear. The contralateral ovary is normal.
f
ks

ks

ks
Histology confirms an ovarian carcinoma.
oo

oo

oo

442. A woman is diagnosed with cervical cancer. The carcinoma has extended into the
eb

eb

eb

pelvic sidewall. On rectal examination, there is no cancer-­free space between the


tumour and the pelvic sidewall. The tumour involves the lower third of the vagina.
m

There is unilateral hydronephrosis.

443. A 90-year-old woman is diagnosed with vulval cancer. Imaging suggests


t

t
metastases to both inguinofemoral and pelvic lymph nodes.
ne

ne

ne
e.

e.

e.
re

re

fre
sf

f
ks

ks
k
oo

oo

oo
eb

eb

eb
m

m
t

et

t
ne

ne
n
e.

e.

e.
t

t
Gynaecological oncology 231

ne

ne

ne
e.

e.

e.
re

fre

fre
Options for questions 444–446

f
ks

ks

ks
A Cancer antigen 19-­9 (CA19-9)
oo

oo

oo
B Cancer antigen 72-­4 (CA72-4)
C Carcinoembryonic antigen (CEA)
eb

eb

eb
D Epithelial tumour antigen (ETA)
m

m
E α-Fetoprotein (AFP)
F Homeobox protein CDX2
G Human chorionic gonadotropin (hCG)
t

t
e

ne

ne
H Human epididymis protein 4 (HE4)
.n

e.

e.
I Inhibin
e
re

fre

re
J Lactate dehydrogenase (LDH)
K Oestradiol
sf

sf
ks
k

k
L Oestriol
oo

oo

oo
For each of the following situations, choose the single most appropriate tumour
eb

eb

eb
marker from the list of options above. Each option may be chosen once, more than once
m

m
or not at all.

444. A compound belonging to the transforming growth factor-­β (TGF-­β) superfamily,


produced by both the Sertoli cells of the testis and granulosa cell tumours of the
t

et
ne

ne

ovary.

n
e.

e.

e.
445. A steroid that is almost undetectable in the non-­pregnant female, but whose levels
fre

fre

re
rise >1000-­fold during pregnancy.
f
ks

ks

ks
446. A compound that is elevated in women with an ovarian dysgerminoma.
oo

oo

oo
eb

eb

eb
m

m
t

t
ne

ne

ne
e.

e.

e.
re

re

fre
sf

f
ks

ks
k
oo

oo

oo
eb

eb

eb
m

m
t

et

t
ne

ne
n
e.

e.

e.
t

t
232 Module 17

ne

ne

ne
e.

e.

e.
re

fre

fre
Options for questions 447–450

f
ks

ks

ks
A No action necessary
oo

oo

oo
B Perform cervical cytology in 1 year’s time
C Perform cervical cytology in 3 years’ time
eb

eb

eb
D Perform colposcopic treatment
m

m
E Perform human papillomavirus (HPV) test of cure
F Perform HPV triage testing
G Refer to colposcopy
t

t
e

ne

ne
H Repeat cervical cytology in 3 months’ time
.n

e.

e.
I Repeat cervical cytology in 6 months’ time
e
re

fre

re
J Return to routine recall
sf

sf
ks
For each of the following clinical scenarios, what is the most appropriate management
k

k
oo

oo

oo
from the list above? Each option may be used once, more than once or not at all.
eb

eb

eb
447. A 28-year-old woman attends her GP surgery for a routine cervical smear. The
examination is difficult and an ‘inadequate’ result is returned.
m

m
448. A 35-year-old woman attends her GP surgery for a cervical smear having never
had a smear in the past. A result of high-­grade dyskaryosis is returned.
t

et
ne

ne

449. A 31-year-old woman attends her GP surgery for a cervical smear. This is reported

n
as low-­grade dyskaryosis and an HPV test is organised. Unfortunately, the HPV
e.

e.

e.
test is inadequate.
fre

fre

f re
450. A 30-year-old woman undergoes colposcopic treatment for cervical intraepithelial
ks

ks

ks
neoplasia (CIN) 3. Histology shows incomplete excision at the ectocervical
oo

oo

oo

margin but complete excision at the endocervical margin.


eb

eb

eb
m

m
t

t
ne

ne

ne
e.

e.

e.
re

re

fre
sf

f
ks

ks
k
oo

oo

oo
eb

eb

eb
m

m
t

et

t
ne

ne
n
e.

e.

e.
t

t
Gynaecological oncology 233

ne

ne

ne
e.

e.

e.
re

fre

fre
Options for questions 451–453

f
ks

ks

ks
A 15–19 years
oo

oo

oo
B 20–24 years
C 25–29 years
eb

eb

eb
D 30–34 years
m

m
E 35–39 years
F 40–44 years
G 45–49 years
t

t
e

ne

ne
H 50–54 years
.n

e.

e.
I 55–59 years
e
re

fre

re
J 60–64 years
K 65–69 years
sf

sf
ks
k

k
L 70–74 years
oo

oo

oo
M 75–79 years
eb

eb

eb
N 80–84 years
O 85–89 years
m

m
For each of the following scenarios, choose the single most appropriate age range
from the list of options above. Each option may be chosen once, more than once or not
t

et
ne

ne

at all.

n
e.

e.

e.
451. The age group with the highest rate of cervical cancer in the UK.
fre

fre

re
452. The age group with the highest incidence of ovarian cancer per year in the UK.
f
ks

ks

ks
453. The age group with the highest incidence of uterine cancer per year in the UK.
oo

oo

oo
eb

eb

eb
m

m
t

t
ne

ne

ne
e.

e.

e.
re

re

fre
sf

f
ks

ks
k
oo

oo

oo
eb

eb

eb
m

m
t

et

t
ne

ne
n
e.

e.

e.
t

t
234 Module 17

ne

ne

ne
e.

e.

e.
re

fre

fre
Options for questions 454–456

f
ks

ks

ks
A 1%
oo

oo

oo
B 2%
C 5%
eb

eb

eb
D 10%
m

m
E 20%
F 40%
G 55%
t

t
e

ne

ne
H 60%
.n

e.

e.
I 70%
e
re

fre

re
J 80%
K 90%
sf

sf
ks
k

k
L 95%
oo

oo

oo
M 100%
eb

eb

eb
For each of the following cancers, choose the single most appropriate figure for the
m

m
5-­year survival rate from the list of options above. Each option may be chosen once, more
than once or not at all.

454. Stage 2 carcinoma of the cervix.


t

et
ne

ne

n
455. Stage 1 uterine (endometrial) cancer.
e.

e.

e.
fre

fre

re
456. Stage 3 ovarian cancer.

f
ks

ks

ks
oo

oo

oo
eb

eb

eb
m

m
t

t
ne

ne

ne
e.

e.

e.
re

re

fre
sf

f
ks

ks
k
oo

oo

oo
eb

eb

eb
m

m
t

et

t
ne

ne
n
e.

e.

e.
t

t
Gynaecological oncology 235

ne

ne

ne
e.

e.

e.
re

fre

fre
Options for questions 457–460

f
ks

ks

ks
A Arrange follow-­up scan in 6 months’ time
oo

oo

oo
B Arrange follow-­up scan in 12 months’ time
C Chemotherapy
eb

eb

eb
D Combined oral contraceptive pill (COCP) for six cycles
m

m
E Full staging laparotomy
F Gonadotropin-releasing hormone (GnRH) analogue for 6 months
G Laparoscopic cyst drainage
t

t
e

ne

ne
H Laparoscopic bilateral salpingo-oophorectomy
.n

e.

e.
I Laparoscopic ovarian cystectomy
e
re

fre

re
J Laparoscopic unilateral oophorectomy
K Laparotomy, bilateral salpingo-oophorectomy and omental biopsy
sf

sf
ks
k

k
L Measure serum CA125 and calculate risk of malignancy index (RMI)
oo

oo

oo
M No treatment required
eb

eb

eb
N Radical hysterectomy including lymph node dissection
O Radiotherapy
m

m
P Sentinel node biopsy
Q Total abdominal hysterectomy, bilateral salpingo-­oophorectomy
and omental biopsy
t

et
ne

ne

R Ultrasound-guided cyst drainage

n
e.

e.

e.
For each of the following clinical scenarios, choose the single most appropriate man-
fre

fre

re
agement option from the list above. Each option may be chosen once, more than once or
f
ks

ks

ks
not at all.
oo

oo

oo

457. A 65-year-old woman has an incidental finding of an ovarian cyst during an MRI
scan to evaluate her spine. A follow-­up ultrasound confirms this to be a simple
eb

eb

eb

cyst with a maximum diameter of 4.5 cm. Her serum CA125 is 5 IU/l. The woman
m

is asymptomatic.

458. A 65-year-old woman is referred urgently with weight loss, abdominal bloating
and urinary urgency. An ultrasound scan demonstrates a multicystic mass
t

t
ne

ne

ne

with solid areas and ascites. Her serum CA125 is 100 IU/l. A CT scan confirms
a suspicious mass. Which management should be recommended by the
e.

e.

e.

multidisciplinary team?
re

re

fre

459. A healthy 60-year-old woman with a BMI of 28 kg/m2 presents with lower
sf

f
ks

ks

abdominal pain and bloating. An ultrasound scan demonstrates a unilocular cyst


k
oo

oo

oo

of 4 cm diameter with a solid component. There is no ascites. Her serum CA125 is


40 IU/l.
eb

eb

eb

460. A frail 90-year-old woman presents with abdominal discomfort and bloating. She
m

has mitral stenosis and atrial fibrillation. She has urinary frequency and nocturia.
An ultrasound scan reveals a 10 cm simple ovarian cyst. Her serum CA125 is
2 IU/l.
t

et

t
ne

ne
n
e.

e.

e.
t

t
236 Module 17

ne

ne

ne
e.

e.

e.
re

fre

fre
Answers
f
ks

ks

ks
SBAs
oo

oo

oo
421. Answer  C Inguinal
eb

eb

eb
Explanation
The natural history of vulval cancer is to grow by direct extension followed by
m

m
lymphatic embolisation. Initially, this is to local inguinal lymph nodes and later to
femoral and the external iliac chain.
t

t
Reference
e

ne

ne
.n

Bailey C, Luesley D. Squamous vulval cancer – an update. The Obstetrician & Gynaecologist

e.

e.
2013;15:227–31.
e
re

fre

re
422. Answer  A  <1%
sf

sf
ks
k

k
Explanation
oo

oo

oo
See Table 1 in the reference article.
eb

eb

eb
Reference
Bailey C, Luesley D. Squamous vulval cancer – an update. The Obstetrician & Gynaecologist
m

m
2013;15:227–31.

423. Answer  E  Carboplatin and paclitaxel


t

et
ne

ne

Explanation

n
e.

e.

e.
Surgery remains at the forefront of treatment, either as the initial therapy or as a
delayed primary procedure. However, the conventional agents that continue to be
fre

fre

re
used most frequently are carboplatin and paclitaxel.
f
ks

ks

ks
Reference
oo

oo

oo

Reed NS, Sadozye AH. Update on chemotherapy in gynaecological cancers. The


Obstetrician & Gynaecologist 2016;18:182–8.
eb

eb

eb
m

424. Answer  C Endometrium


Explanation
Approximately 10% of ovarian tumours are of secondary origin. The most
t

t
ne

ne

ne

common metastasis is from endometrium. Krukenberg tumours commonly


metastasise from the stomach or colon, although metastasis may also arise from
e.

e.

e.

the breast, lung and pancreas.


re

re

fre

Reference
sf

f
ks

ks

Sanusi FA, Carter P, Barton DPJ. Non-epithelial ovarian cancers. The Obstetrician &
k

Gynaecologist 2000;2:37–9.
oo

oo

oo
eb

eb

eb

425. Answer  B CA125


m

Explanation
CA125 should be the only serum tumour marker used for primary evaluation.
t

et

t
ne

ne
n
e.

e.

e.
t

t
Gynaecological oncology 237

ne

ne

ne
e.

e.

e.
re

fre

fre
There is currently not enough evidence to support the routine clinical use
of other tumour markers, such as human epididymis protein 4 (HE4), CEA,
f
ks

ks

ks
homeobox protein CDX2, cancer antigen 72-­4 (CA72-­4), CA19-­9, AFP, LDH
oo

oo

oo
or human chorionic gonadotropin (hCG), to assess the risk of malignancy in
postmenopausal ovarian cysts.
eb

eb

eb
Reference
m

m
RCOG. The management of ovarian cysts in postmenopausal women. RCOG GTG No. 34,
July 2016.
t

t
426. Answer  A  CT of abdomen and pelvis
e

ne

ne
.n

Explanation

e.

e.
e

CT of the abdomen and pelvis should be performed for all postmenopausal


re

fre

re
women with ovarian cysts who have an RMI score of ≥200, with onward referral
sf

sf
to a gynaecological oncology multidisciplinary team.
ks
k

k
oo

oo

oo
Reference
RCOG. The management of ovarian cysts in postmenopausal women. RCOG GTG No. 34,
eb

eb

eb
July 2016.
m

m
427. Answer  E  Papillary projections, complete septation/multilocular and/or
solid nodules
t

et
Explanation
ne

ne

n
An ovarian cyst is defined as complex in the presence of one or more of the
e.

e.

e.
following features:
fre

fre

re
• Complete septation (i.e. multilocular cyst)
• Solid nodules
f
ks

ks

ks
• Papillary projections.
oo

oo

oo

Reference
eb

eb

eb

RCOG. The management of ovarian cysts in postmenopausal women. RCOG GTG No. 34,
July 2016.
m

428. Answer  A  Differentiated type


t

t
Explanation
ne

ne

ne

In genitourinary medicine clinics, the commonest aetiological agent is human


e.

e.

e.

papillomavirus (HPV). This type of VIN is known as the usual type and is
re

re

fre

associated mainly with HPV-­16. A second type, generally not HPV-­related, occurs
in conjunction with lichen sclerosus or lichen planus (known as differentiated
sf

f
ks

ks

type). The risk of progression to squamous cell carcinoma is much greater with
k

the differentiated type of VIN and this needs specialised management.


oo

oo

oo

Reference
eb

eb

eb

Edwards SK, Bates CM, Lewis F, Sethi G, Grover D. 2014 UK national guideline on the
m

management of vulval conditions. International Journal of STD & AIDS 2015;26:611–24.


t

et

t
ne

ne
n
e.

e.

e.
t

t
238 Module 17

ne

ne

ne
e.

e.

e.
re

fre

fre
429. Answer  B  Hysteroscopy and targeted biopsy

f
Explanation
ks

ks

ks
All abnormal bleeding or spotting should be investigated, but pipelle endometrial
oo

oo

oo
biopsy rarely provides useful diagnostic information in women treated with
tamoxifen; therefore, symptomatic women with a thickened endometrium should
eb

eb

eb
be investigated with a hysteroscopy and targeted biopsy. This is primarily because
m

m
of tamoxifen-­induced subepithelial stromal hypertrophy.
Reference
Otify M, Fuller J, Ross J, Shaikh H, Johns J. Endometrial pathology in the postmenopausal
t

t
e

ne

ne
woman – an evidence based approach to management. The Obstetrician & Gynaecologist
.n

2015;17:29–38.

e.

e.
e
re

fre

re
430. Answer  B 7%
sf

sf
ks
Explanation
k

k
It is estimated that each year of using the COCP brings an approximate 7%
oo

oo

oo
reduction in risk of ovarian cancer.
eb

eb

eb
Reference
m

m
Louis LS, Saso S, Ghaem-Maghami S, Abdalla H, Smith JR. The relationship between
infertility treatment and cancer including gynaecological cancers. The Obstetrician &
Gynaecologist 2013;15:177–83.
t

et
ne

ne

431. Answer  A  0–20 years

n
e.

e.

e.
Explanation
fre

fre

re
MOGCTs occur most commonly in the first two decades of life, but can appear
at any age, with 82.3% of all MOGCTs occurring between the ages of 14 and 54
f
ks

ks

ks
years.
oo

oo

oo

Reference
eb

eb

eb

RCOG. Management of female malignant ovarian germ cell tumours. RCOG Scientific
Impact Paper No. 52. November 2016.
m

432. Answer  A  Laparotomy, unilateral oophorectomy, omental biopsy and selective


removal of lymph nodes
t

t
ne

ne

ne

Explanation
Surgery, when appropriate, should comprise unilateral oophorectomy, peritoneal
e.

e.

e.

washing, omental biopsy and selective removal of enlarged lymph nodes. Biopsy
re

re

fre

of a normal contralateral ovary is not indicated. Surgery should be by an open


sf

f
ks

ks

procedure to enable removal of the affected ovary with its tumour intact rather
k

than broken or ruptured.


oo

oo

oo

Reference
eb

eb

eb

RCOG. Management of female malignant ovarian germ cell tumours. RCOG Scientific
m

Impact Paper No. 52. November 2016.


t

et

t
ne

ne
n
e.

e.

e.
t

t
Gynaecological oncology 239

ne

ne

ne
e.

e.

e.
re

fre

fre
433. Answer  E  Dysgerminoma and immature teratoma

f Explanation
ks

ks

ks
Approximately one-­third of such cases are dysgerminomas, one-­third are
oo

oo

oo
immature teratomas, and one-­third include embryonal tumours, endodermal
sinus tumours, choriocarcinoma and mixed-­cell types.
eb

eb

eb
Reference
m

m
RCOG. Management of female malignant ovarian germ cell tumours. RCOG Scientific
Impact Paper No. 52. November 2016.
t

t
e

ne

ne
434. Answer  B  Family history of ovarian cancer
.n

e.

e.
Explanation
e
re

fre

re
The strongest known risk factor is a family history of the disease, which is present
in about 10–15% of women with ovarian cancer.
sf

sf
ks
k

k
Reference
oo

oo

oo
Gaughan EMG, Walsh TA. Risk-reducing surgery for women at high risk of epithelial
eb

eb

eb
ovarian cancer. The Obstetrician & Gynaecologist 2014;16:185–91.
m

m
435. Answer  E  No screening recommended
Explanation
Screening for ovarian cancer with CA125 or ultrasound is not recommended for
t

et
ne

ne

premenopausal and postmenopausal women without a family history of ovarian

n
cancer. The predictive value of either test alone (<3%) yields an unacceptably high
e.

e.

e.
rate of false-­positive results and attendant morbidity and costs.
fre

fre

re
Reference
f
ks

ks

ks
Gaughan EMG, Walsh TA. Risk-reducing surgery for women at high risk of epithelial
oo

oo

oo

ovarian cancer. The Obstetrician & Gynaecologist 2014;16:185–91.


eb

eb

eb

436. Answer  B 2%


m

Explanation
It is important to remember that risk-­reducing bilateral salpingo-­oophorectomy
is not completely protective and BRCA carriers still have a risk of developing
t

t
primary peritoneal cancer (approximately 2% risk).
ne

ne

ne

Reference
e.

e.

e.

Gaughan EMG, Walsh TA. Risk-reducing surgery for women at high risk of epithelial
re

re

fre

ovarian cancer. The Obstetrician & Gynaecologist 2014;16:185–91.


sf

f
ks

ks
k

437. Answer  D  7 years


oo

oo

oo

Explanation
eb

eb

eb

Seven years is believed to be the minimum time span between infection by HPV
and the development of a premalignant lesion with true malignant potential.
m

Reference
Aref-Adib M, Freeman-Wang T. Cervical cancer prevention and screening: the role of
t

et

human papillomavirus testing. The Obstetrician & Gynaecologist 2016;18:251–63.


ne

ne
n
e.

e.

e.
t

t
240 Module 17

ne

ne

ne
e.

e.

e.
re

fre

fre
438. Answer  B  Postoperatively as an adjuvant therapy

f
Explanation
ks

ks

ks
In endometrial cancer, the principal role of radiotherapy is as an adjuvant
oo

oo

oo
treatment postoperatively, and the past decade has seen this refined with far
greater use of vagina brachytherapy and a marked reduction in the use of external
eb

eb

eb
beam radiotherapy.
m

m
Reference
Reed NS, Sadozye AH. Update on radiotherapy in gynaecological malignancies. The
Obstetrician & Gynaecologist 2017;19:29–36.
t

t
e

ne

ne
.n

439. Answer  D  Cobalt and iridium

e.

e.
e
re

fre

re
Explanation
Over the decades, gynaecological brachytherapy has evolved from radium to
sf

sf
ks
caesium to modern-­day cobalt and iridium sources.
k

k
oo

oo

oo
Reference
eb

eb

eb
Reed NS, Sadozye AH. Update on radiotherapy in gynaecological malignancies. The
Obstetrician & Gynaecologist 2017;19:29–36.
m

440. Answer  C  Epithelial cancer


Explanation
m
t

et
ne

ne

While bowel obstruction is a rare presentation in women with gynaecological

n
cancers, it is most commonly associated with ovarian cancer, the main subtype of
e.

e.

e.
which is epithelial ovarian cancer.
fre

fre

re
Reference
f
ks

ks

ks
Kolomainen DF, Riley J, Wood J, Barton DPJ. Surgical management of bowel obstruction in
oo

oo

oo

gynaecological cancer. The Obstetrician & Gynaecologist 2017;19:63–70.


eb

eb

eb

EMQs
m

441. Answer  C  I C1
This is stage I C1 according to the International Federation of Gynecology and
Obstetrics (FIGO) ovarian cancer staging classification (2014).
t

t
ne

ne

ne

442. Answer  I  III B


e.

e.

e.

This is stage III B according to the FIGO staging of cervical carcinomas


re

re

fre

classification (2006).
sf

f
ks

ks
k

443. Answer  M IVB


oo

oo

oo

This is stage IV B according to the FIGO vulval cancer staging classification (2014).
eb

eb

eb
m

m
t

et

t
ne

ne
n
e.

e.

e.
t

t
Gynaecological oncology 241

ne

ne

ne
e.

e.

e.
re

fre

fre
444. Answer  I Inhibin

f Explanation
ks

ks

ks
Inhibin supresses follicle-­stimulating hormone (FSH) production and secretion by
oo

oo

oo
the anterior pituitary. Inhibin has been used as a tumour marker for granulosa cell
tumours.
eb

eb

eb
m

m
445. Answer  L Oestriol
Explanation
The key here is to read the question properly. Although most people will
t

t
e

ne

ne
answer    G, hCG is not a steroid – it is a glycoprotein. Oestriol is a steroid that
.n

is produced by the placenta, and is one of the components of serum screening.

e.

e.
e
re

fre

re
446. Answer  J  Lactate dehydrogenase (LDH)
sf

sf
ks
Explanation
k

k
LDH is found throughout the body, and levels are elevated in a number of cancers.
oo

oo

oo
From a gynaecological perspective, it has been used as a tumour marker in
eb

eb

eb
ovarian dysgerminoma.
m

m
Reference
Sanusi FA, Carter P, Barton DPJ. Non-epithelial ovarian cancers. The Obstetrician &
Gynaecologist 2000;2:37–9.
t

et
ne

ne

447. Answer  H  Repeat cervical cytology in 3 months’ time

n
e.

e.

e.
fre

fre

re
448. Answer  G  Refer to colposcopy
f
ks

ks

ks
449. Answer  G  Refer to colposcopy
oo

oo

oo
eb

eb

eb

450. Answer  E  Perform human papillomavirus (HPV) test of cure


m

Explanation
See Figure 5 in the reference article.
t

t
Reference
ne

ne

ne

Aref-Adib M, Freeman-Wang T. Cervical cancer prevention and screening: the role of


e.

e.

e.

human papillomavirus testing. The Obstetrician & Gynaecologist 2016;18:251–63.


re

re

fre

451. Answer  C  25–29 years


sf

f
ks

ks
k

The incidence rates for cervical cancer in the UK for 2012–14 were highest in
oo

oo

oo

people aged 25–29 years.


eb

eb

eb

452. Answer  M  75–79 years


m

The incidence rates for ovarian cancer in the UK for 2012–14 were highest in
females aged 75–79 years.
t

et

t
ne

ne
n
e.

e.

e.
t

t
242 Module 17

ne

ne

ne
e.

e.

e.
re

fre

fre
453. Answer  L  70–74 years

f
The incidence rates for uterine cancer in the UK for 2012–14 were highest in
ks

ks

ks
females aged 70–74 years.
oo

oo

oo
Reference
eb

eb

eb
See the Cancer Research UK website at www.cancerresearchuk.org (accessed 25 July 2018).
m

m
454. Answer  G 55%
The 5-­year survival rates for stage 1 to stage 4 cervical cancer are 95.9%, 54.4%,
t

t
37.9% and 5.3%, respectively. For all stages taken together, the overall 5-­year
e

ne

ne
.n

survival rate is 69.9%. Where the stage is not known, the 5-­year survival rate is

e.

e.
e

31.6%.
re

fre

re
455. Answer  L 95%
sf

sf
ks
k

k
The 5-­year survival rates for stage 1 to stage 4 uterine (endometrial ) cancer are
oo

oo

oo
95.3%, 77%, 39% and 13.6%,respectively. For all stages taken together, the overall
eb

eb

eb
5-­year survival rate is 84.4%. Where the stage is not known, the 5-­year survival
rate is 54.4%.
m

456. Answer  E 20%


m
The 5-­year survival rates for stage 1 to stage 4 ovarian cancer are 90%, 42.8%, 18.6%
t

et
ne

ne

and 3.5%, respectively. For all stages taken together, the overall 5-­year survival rate

n
is 39.3%. Where the stage is not known, the 5-­year survival rate is 12.5%.
e.

e.

e.
fre

fre

re
Reference
f
See the Cancer Research UK website at www.cancerresearchuk.org (accessed 25 July 2018).
ks

ks

ks
oo

oo

oo

457. Answer  A  Arrange follow-­up scan in 6 months’ time


eb

eb

eb

Explanation
Asymptomatic, simple, unilateral, unilocular ovarian cysts of <5 cm in diameter
m

have a low risk of malignancy. In the presence of normal serum CA125 levels,
these cysts can be managed conservatively, with a repeat evaluation in 4–6
months. It is reasonable to discharge these women from follow-­up after 1 year if
t

t
the cyst remains unchanged or reduces in size, with normal serum CA125, taking
ne

ne

ne

into consideration the woman’s wishes and surgical fitness.


e.

e.

e.
re

re

fre

458. Answer  E  Full staging laparotomy


sf

f
ks

ks

Explanation
k

RMI is calculated as: U × M × CA125 (see question 301 for details). This woman’s
oo

oo

oo

RMI is 3 × 3 × 100 = 900.


eb

eb

eb

All ovarian cysts that are suspicious of malignancy in a postmenopausal


woman, as indicated by an RMI of ≥200, CT findings, clinical assessment or
m

findings at laparoscopy, require a full laparotomy and staging procedure.


t

et

t
ne

ne
n
e.

e.

e.
t

t
Gynaecological oncology 243

ne

ne

ne
e.

e.

e.
re

fre

fre
459. Answer  H  Laparoscopic bilateral salpingo-oophorectomy

f Explanation
ks

ks

ks
This woman’s RMI is 1 × 3 × 40 = 120 (see question 301 for details).
oo

oo

oo
Women with an RMI of <200 (i.e. at low risk of malignancy) are suitable
for laparoscopic management. Laparoscopic management of ovarian cysts in
eb

eb

eb
postmenopausal women should comprise bilateral salpingo-­oophorectomy rather
m

m
than cystectomy.

460. Answer  R  Ultrasound-­guided cyst drainage


t

t
e

ne

ne
Explanation
.n

This woman’s RMI is 1 × 3 × 2 = 6 (see question 301 for details).

e.

e.
e

This is very unlikely to be a malignant cyst, but the woman is symptomatic.


re

fre

re
She is frail with co-morbidities.
sf

sf
Aspiration has no role in the management of asymptomatic ovarian cysts in
ks
k

k
postmenopausal women. An exception exists for those symptomatic women who
oo

oo

oo
are medically unfit to undergo surgery or further intervention. In these women,
aspiration will provide relief of their symptoms, albeit temporarily.
eb

eb

eb
Reference
m

m
RCOG. The management of ovarian cysts in postmenopausal women. RCOG GTG No. 34.
July 2016.
t

et
ne

ne

n
e.

e.

e.
fre

fre

f re
ks

ks

ks
oo

oo

oo
eb

eb

eb
m

m
t

t
ne

ne

ne
e.

e.

e.
re

re

fre
sf

f
ks

ks
k
oo

oo

oo
eb

eb

eb
m

m
t

et

t
ne

ne
n
e.

e.

e.
t

t
ne

ne

ne
e.

e.

e.
re

fre

fre
Module
f Urogynaecology and pelvic
ks

ks

ks
18 floor problems
oo

oo

oo
eb

eb

eb
m

m
t

t
e

ne

ne
SBAs
.n

e.

e.
e

461. A 55-year-old woman attends the urogynaecology clinic with urinary


re

fre

re
incontinence associated with urgency. A urine dipstick is negative. A diagnosis of
sf

sf
urgency urinary incontinence is made.
ks
k

k
What would be considered the first-­line treatment?
oo

oo

oo
A. Bladder training
eb

eb

eb
B. Botulinum toxin A
C. Mirabegron
m

m
D. Oxybutynin
E. Pelvic floor muscle training
t

et
462. A 60-year-old woman attends the urogynaecology clinic with urinary
ne

ne

n
incontinence associated with coughing and sneezing. A urine dipstick is negative.
e.

e.

e.
A diagnosis of stress urinary incontinence is made.
fre

fre

re
What would be considered the first-­line treatment?
A. Bladder training
f
ks

ks

ks
B. Desmopressin
oo

oo

oo

C. Duloxetine
D. Systemic hormone replacement therapy (HRT)
eb

eb

eb

E. Supervised pelvic floor muscle training


m

463. What proportion of women who use vaginal pessaries to manage pelvic organ
prolapse will report satisfaction in symptom relief?
t

t
A. 47%
ne

ne

ne

B. 58%
e.

e.

e.

C. 69%
D. 80%
re

re

fre

E. 92%
sf

f
ks

ks
k
oo

oo

oo
eb

eb

eb
m

244
t

et

t
ne

ne
n
e.

e.

e.
t

t
Urogynaecology and pelvic floor problems 245

ne

ne

ne
e.

e.

e.
re

fre

fre
464. The incidence of asymptomatic bacteriuria in pregnancy is 2–5%.
If untreated, what proportion of cases will proceed to lower urinary tract
f
ks

ks

ks
infection (UTI)?
oo

oo

oo
A. 20%
B. 40%
eb

eb

eb
C. 60%
m

m
D. 80%
E. 90%
t

t
465. Which aetiological factor is associated with the greatest increase in risk of
e

ne

ne
developing pelvic organ prolapse?
.n

e.

e.
A. Age
e
re

fre

re
B. Connective tissue disorders
C. Menopausal status
sf

sf
ks
D. Parity
k

k
oo

oo

oo
E. Weight
eb

eb

eb
466. By what factor are obese women at increased risk of anal incontinence compared
m

m
with non-­obese women?
A. 2-fold
B. 3-fold
t

et
C. 4-fold
ne

ne

n
D. 6-fold
e.

e.

e.
E. 8-fold
fre

fre

467. Which antimuscarinic drug used in the management of an overactive bladder


f re
ks

ks

ks
should be avoided in frail older women?
oo

oo

oo

A. Darifenacin
B. Desmopressin
eb

eb

eb

C. Mirabegron
m

D. Oxybutynin
E. Tolterodine
t

t
468. What is the mode of action of duloxetine?
ne

ne

ne

A. M3 receptor antagonist
e.

e.

e.

B. Monoamine oxidase inhibitor


re

re

fre

C. α2-Receptor agonist
sf

D. β3-Receptor agonist
ks

ks

E. Serotonin–noradrenaline uptake inhibitor


k
oo

oo

oo
eb

eb

eb
m

m
t

et

t
ne

ne
n
e.

e.

e.
t

t
246 Module 18

ne

ne

ne
e.

e.

e.
re

fre

fre
469. A woman is seen on the postoperative ward round following the
insertion of a transobturator tape for the treatment of stress urinary
f
ks

ks

ks
incontinence.
oo

oo

oo
Within what timeframe should she be offered a follow-­up appointment
with a vaginal examination to exclude erosion?
eb

eb

eb
A. Within 1 month
m

m
B. Within 2 months
C. Within 3 months
D. Within 6 months
E. Within 12 months
t

t
e

ne

ne
.n

470. Which derangement of acid–base balance is associated with augmentation

e.

e.
e

cystoplasty for the treatment of an overactive bladder?


re

fre

re
A. Metabolic acidosis
sf

sf
ks
B. Metabolic alkalosis
k

k
C. No derangement
oo

oo

oo
D. Respiratory acidosis
eb

eb

eb
E. Respiratory alkalosis
m

m
471. What is the mode of action of darifenacin?
A. M3 receptor antagonist
B. Monoamine oxidase inhibitor
t

et
C. α2-Receptor agonist
ne

ne

n
D. β3-Receptor agonist
e.

e.

e.
E. Serotonin–noradrenaline uptake inhibitor
fre

fre

re
472. Following a vaginal hysterectomy, it is noted that the vaginal vault is at the level
f
ks

ks

ks
of the introitus.
What further surgical procedure would be advised?
oo

oo

oo

A. Anterior repair
eb

eb

eb

B. McCall culdoplasty
C. Moschcowitz procedure
m

D. Sacrocolpopexy
E. Sacrospinous fixation
t

t
ne

ne

ne

473. Which invasive treatment for an overactive bladder is suitable for women who are
unable to perform clean intermittent catheterisation?
e.

e.

e.

A. Augmentation cystoplasty
re

re

fre

B. Botulinum toxin A
sf

C. Detrusor myomectomy
ks

ks
k

D. Percutaneous sacral nerve stimulation


oo

oo

oo

E. Transobturator tape
eb

eb

eb
m

m
t

et

t
ne

ne
n
e.

e.

e.
t

t
Urogynaecology and pelvic floor problems 247

ne

ne

ne
e.

e.

e.
re

fre

fre
474. In the pelvic organs prolapse quantification (POP-­Q) examination, what is the
description of Aa?
f
ks

ks

ks
A. Anterior vaginal wall at the level of the hymen
oo

oo

oo
B. Anterior vaginal wall 1 cm proximal to the hymen
C. Anterior vaginal wall 2 cm proximal to the hymen
eb

eb

eb
D. Anterior vaginal wall 3 cm proximal to the hymen
m

m
E. Anterior wall 4 cm proximal to the hymen

475. Colpocleisis is a safe and effective procedure that can be considered for frail
women and/or women who do not wish to retain sexual function.
t

t
e

ne

ne
What proportion of cases of colpocleisis that are performed would be
.n

considered successful?

e.

e.
e

A. 17%
re

fre

re
B. 37%
sf

sf
ks
C. 57%
k

k
D. 77%
oo

oo

oo
E. 97%
eb

eb

eb
476. What factor is most likely to aggravate the pain in patients with bladder pain
m

m
syndrome?
A. Coffee
B. Constrictive clothing
t

et
ne

ne

C. Sexual intercourse

n
D. Spicy foods
e.

e.

e.
E. Stress
fre

fre

477. What proportion of patients with bladder pain syndrome will get relief of
f re
ks

ks

ks
symptoms from voiding?
oo

oo

oo

A. 17–28%
B. 31–42%
eb

eb

eb

C. 44–56%
m

D. 57–73%
E. 78–92%
t

t
478. What is the most common problem for a woman following cystoscopy?
ne

ne

ne

A. Bladder perforation requiring catheterisation


e.

e.

e.

B. Mild burning or bleeding during micturition


re

re

fre

C. Significant haematuria requiring clot evacuation


D. UTI
sf

f
ks

ks

E. Urinary retention requiring catheterisation


k
oo

oo

oo
eb

eb

eb
m

m
t

et

t
ne

ne
n
e.

e.

e.
t

t
248 Module 18

ne

ne

ne
e.

e.

e.
re

fre

fre
479. What is the only true contraindication to cystoscopy?

f
A. Congenital urinary tract anomalies
ks

ks

ks
B. Untreated UTI
oo

oo

oo
C. Urethral stricture
D. Vesicovaginal fistula
eb

eb

eb
E. Visible haematuria
m

m
480. What is the risk of developing a pelvic abscess following a vaginal hysterectomy
for uterovaginal prolapse?
t

t
A. 1 in 1000 women
e

ne

ne
B. 2 in 1000 women
.n

e.

e.
C. 3 in 1000 women
e

D. 4 in 1000 women
re

fre

re
E. 5 in 1000 women
sf

sf
ks
k

k
oo

oo

oo
eb

eb

eb
m

m
t

et
ne

ne

n
e.

e.

e.
fre

fre

f re
ks

ks

ks
oo

oo

oo
eb

eb

eb
m

m
t

t
ne

ne

ne
e.

e.

e.
re

re

fre
sf

f
ks

ks
k
oo

oo

oo
eb

eb

eb
m

m
t

et

t
ne

ne
n
e.

e.

e.
t

t
Urogynaecology and pelvic floor problems 249

ne

ne

ne
e.

e.

e.
re

fre

fre
EMQs
f
ks

ks

ks
Options for questions 481–483
oo

oo

oo
A Abdominal X-ray
eb

eb

eb
B Bladder scan
C Complete a bladder diary
m

m
D CT of pelvis
E Cystoscopy
t

t
F Digital assessment of pelvic floor contraction
e

ne

ne
.n

G Filling and voiding cystometry

e.

e.
e

H MRI of pelvis
re

fre

re
I No investigation required
sf

sf
J Pad testing
ks
k

k
K Pelvic ultrasound scan
oo

oo

oo
L Perform a quality-of-life assessment
eb

eb

eb
M Repeat midstream urine
m

m
N Ultrasound scan of renal tract
O Urinary catheterisation
P Urine dipstick test
t

et
Q Video urodynamics
ne

ne

n
e.

e.

e.
For each of the following clinical scenarios, what is the most appropriate investigation
fre

fre

re
that needs to be performed before any therapy is commenced? Each option may be used
once, more than once or not at all.
f
ks

ks

ks
481. A 40-year-old woman attends the gynaecology clinic with a history of leakage of
oo

oo

oo

urine on coughing or sneezing following the birth of her last child 2 years ago. A
eb

eb

eb

urine dipstick is negative. A decision is made to commence a course of supervised


pelvic floor muscle training.
m

482. A 55-year-old woman who previously had insertion of a tension-­free vaginal tape
for the treatment of stress incontinence attends with new symptoms of urinary
leakage with physical exertion or coughing. She is requesting further surgery.
t

t
ne

ne

ne

483. A 60-year-old woman has an initial appointment at the gynaecology clinic. Her
e.

e.

e.

presenting symptoms are urinary leakage, urgency and nocturia.


re

re

fre
sf

f
ks

ks
k
oo

oo

oo
eb

eb

eb
m

m
t

et

t
ne

ne
n
e.

e.

e.
t

t
250 Module 18

ne

ne

ne
e.

e.

e.
re

fre

fre
Options for questions 484–486

f
ks

ks

ks
A Botulinum toxin A
oo

oo

oo
B Botulinum toxin B
C Darifenacin
eb

eb

eb
D Desmopressin
m

m
E Dimethyl sulfoxide
F Duloxetine
G Flavoxate
t

t
e

ne

ne
H Imipramine
.n

e.

e.
I Mirabegron
e
re

fre

re
J Oestriol cream
K Oxybutynin
sf

sf
ks
k

k
L Propantheline
oo

oo

oo
M Propiverine
eb

eb

eb
N Solifenacin
O Tibolone
m

m
P Tolterodine
Q Transdermal oestrogen
R Trospium
t

et
ne

ne

n
e.

e.

e.
For each of the following clinical scenarios, choose the single most appropriate phar-
fre

fre

re
macological therapy from the list above. Each option may be used once, more than once
or not at all.
f
ks

ks

ks
484. A 60-year-old woman returns to the urogynaecology clinic for a review. She
oo

oo

oo

initially presented with symptoms of urinary leakage and urgency. She has
eb

eb

eb

completed a course of bladder training with no effect. She has myasthenia gravis
but is otherwise well.
m

485. A 55-year-old woman initially presented with urinary leakage on coughing and
sneezing. She completed a course of pelvic floor muscle training with little effect.
t

t
She wishes to avoid surgical intervention.
ne

ne

ne

486. A 62-year-old woman who is otherwise fit and well presents with urgency, urinary
e.

e.

e.

leakage and nocturia. The urgency and leakage are improved with transdermal
re

re

fre

oxybutynin, but the nocturia remains troublesome with her needing to go to the
sf

toilet four or five times per night.


ks

ks
k
oo

oo

oo
eb

eb

eb
m

m
t

et

t
ne

ne
n
e.

e.

e.
t

t
Urogynaecology and pelvic floor problems 251

ne

ne

ne
e.

e.

e.
re

fre

fre
Options for questions 487–489

f
ks

ks

ks
A Berger’s disease (IgA nephropathy)
oo

oo

oo
B Bladder calculus
C Bladder endometriosis
eb

eb

eb
D Cystocele
m

m
E Foreign body
F Haemophilia A
G Idiopathic haematuria
t

t
e

ne

ne
H Paroxysmal nocturnal haemoglobinuria
.n

e.

e.
I Polycystic kidney disease
e
re

fre

re
J Poststreptococcal glomerulonephritis
K Renal calculus
sf

sf
ks
k

k
L Sickle-cell disease
oo

oo

oo
M Transitional cell carcinoma
eb

eb

eb
N Urethrocele
O Urinary tract infection (UTI)
m

m
P Von Willebrand’s disease

For each of the following clinical scenarios, choose the single most likely cause
t

et
ne

ne

of haematuria. Each option may be used once, more than once or not at all.

n
e.

e.

e.
487. A 25-year-old woman presents to her GP with urinary frequency and dysuria.
fre

fre

re
Her urine dipstick results are:
f
ks

ks

ks
Leucocytes +
oo

oo

oo

Nitrites +
Blood +++
eb

eb

eb

Protein +
m

Ketones −
Glucose −
t

t
ne

ne

ne

488. Two days after an upper respiratory tract infection, a 20-year-old woman presents
to her GP with episodes of frank haematuria. Renal function tests are normal.
e.

e.

e.
re

re

fre

489. A 45-year-old woman presents with acute loin pain and haematuria. She is found
sf

to be hypertensive with abnormal renal function tests.


ks

ks
k
oo

oo

oo
eb

eb

eb
m

m
t

et

t
ne

ne
n
e.

e.

e.
t

t
252 Module 18

ne

ne

ne
e.

e.

e.
re

fre

fre
Options for questions 490–492

f
ks

ks

ks
A Anterior and posterior colporrhaphy
oo

oo

oo
B Anterior colporrhaphy
C Artificial urinary sphincter
eb

eb

eb
D Augmentation cystoplasty
m

m
E Colpocleisis
F Fenton’s procedure
G Intramural bulking agents
t

t
e

ne

ne
H Laparoscopic colposuspension
.n

e.

e.
I Marshall–Marchetti–Krantz procedure
e
re

fre

re
J Open colposuspension
K Posterior colporrhaphy
sf

sf
ks
k

k
L Tension-free vaginal tape
oo

oo

oo
M Transobturator tape
eb

eb

eb
N Urinary diversion
O Vaginal hysterectomy
m

m
For each of the following clinical scenarios, choose the single most appropriate surgi-
cal intervention from the list of options above. Each option may be used once, more than
t

et
ne

ne

once or not at all.

n
e.

e.

e.
490. A woman attends the urogynaecology clinic with symptoms of stress incontinence
fre

fre

re
that have not responded to conservative measures. She is keen for surgical
intervention but wishes to avoid synthetic meshes and tapes as she has read f
ks

ks

ks
adverse reports in the media.
oo

oo

oo

491. A 45-year-old woman presents with urinary frequency and urgency and a
eb

eb

eb

diagnosis of idiopathic detrusor overactivity is made. This has not responded to


m

conservative measures and she is ready to proceed with surgical intervention.

492. A 90-year-old woman with hypertension and type 2 diabetes presents with
t

t
worsening uterovaginal prolapse that is not being controlled with shelf pessaries.
ne

ne

ne
e.

e.

e.
re

re

fre
sf

f
ks

ks
k
oo

oo

oo
eb

eb

eb
m

m
t

et

t
ne

ne
n
e.

e.

e.
t

t
Urogynaecology and pelvic floor problems 253

ne

ne

ne
e.

e.

e.
re

fre

fre
Options for questions 493 and 494

f
ks

ks

ks
A Atrophic vaginitis
oo

oo

oo
B Bladder calculus
C Bladder diverticulum
eb

eb

eb
D Bladder endometriosis
m

m
E Bladder pain syndrome
F Fibromyalgia
G Pelvic inflammatory disease (PID)
t

t
e

ne

ne
H Peritoneal adhesions
.n

e.

e.
I Sjögren’s syndrome
e
re

fre

re
J Systemic lupus erythematosus
K Transitional cell carcinoma
sf

sf
ks
k

k
L Urethral diverticulum
oo

oo

oo
M Urinary tract infection (UTI)
eb

eb

eb
N Vesicovaginal fistula
m

m
For each of the following clinical scenarios, choose the single most likely diagnosis
from the list of options above. Each option may be used once, more than once or not at all.

493. A 40-year-old woman with no significant past medical history presents with a
t

et
ne

ne

10-­month history of pelvic pain mainly located to the suprapubic area. She has

n
urinary urgency and frequency but no leakage. The symptoms persist through her
e.

e.

e.
cycle. She suffers with constipation but not diarrhoea.
fre

fre

f
494. A 48-year-old woman presents to clinic with complex symptoms. She has urinary
re
ks

ks

ks
urgency and frequency but also dysuria and postmicturition dribble. A full
oo

oo

oo

gynaecological history also reveals dyspareunia. An initial pelvic examination is


unremarkable.
eb

eb

eb
m

m
t

t
ne

ne

ne
e.

e.

e.
re

re

fre
sf

f
ks

ks
k
oo

oo

oo
eb

eb

eb
m

m
t

et

t
ne

ne
n
e.

e.

e.
t

t
254 Module 18

ne

ne

ne
e.

e.

e.
re

fre

fre
Options for questions 495–497

f
ks

ks

ks
A Arrange CT of kidneys, ureters and bladder (CT KUB)
oo

oo

oo
B Arrange MRI scan
C Complete bladder diary
eb

eb

eb
D Consider antibiotics while awaiting midstream urine culture results
m

m
E Measure postvoid residual volume by bladder scan
F Measure postvoid residual volume by catheterisation
G Perform cystoscopy
t

t
e

ne

ne
H Perform digital pelvic examination
.n

e.

e.
I Perform pad test
e
re

fre

re
J Perform urodynamic testing
K Prescribe antibiotics while awaiting midstream urine culture results
sf

sf
ks
k

k
L Refer to urologist
oo

oo

oo
M Send urine for culture and sensitivity
eb

eb

eb
From the list of management options above, choose the single most appropriate man-
m

m
agement for each of the following clinical scenarios. Each option may be used once, more
than once or not at all.
495. A woman attends a general gynaecology clinic and has routine urinalysis by
t

et
ne

ne

dipstick. She has no symptoms, but the urine tests positive for both leucocytes and

n
nitrites.
e.

e.

e.
fre

fre

re
496. A woman is referred to a urogynaecologist with recurrent UTIs. A urine dipstick
in the clinic is negative.
f
ks

ks

ks
497. A 53-year-old woman is referred to the urogynaecology clinic with urinary
oo

oo

oo

incontinence. She has routine urine dipstick testing and is found to have
microscopic haematuria.
eb

eb

eb
m

m
t

t
ne

ne

ne
e.

e.

e.
re

re

fre
sf

f
ks

ks
k
oo

oo

oo
eb

eb

eb
m

m
t

et

t
ne

ne
n
e.

e.

e.
t

t
Urogynaecology and pelvic floor problems 255

ne

ne

ne
e.

e.

e.
re

fre

fre
Options for questions 498–500

f
ks

ks

ks
A Bulbospongiosus muscle
oo

oo

oo
B Conjoint longitudinal coat
C External anal sphincter
eb

eb

eb
D Iliococcygeus
m

m
E Internal anal sphincter
F Ischial tuberosities
G Ischiocavernosus muscle
t

t
e

ne

ne
H Ischiococcygeus muscle
.n

e.

e.
I Ischiopubic rami
e
re

fre

re
J Levator ani
K Levator hiatus
sf

sf
ks
k

k
L Puborectalis muscle
oo

oo

oo
M Pubovaginalis muscle
eb

eb

eb
N Sacrotuberous ligaments
O Superficial transverse perineal muscles
m

m
From the list of options above, choose the single most appropriate anatomical struc-
ture from the list of descriptions below. Each option may be used once, more than once
t

et
or not at all.
ne

ne

n
498. The most caudal component of the levator ani complex.
e.

e.

e.
fre

fre

re
499. The structure separating the external and internal anal sphincters.
f
ks

ks

ks
500. The structure accounting for the majority of the resting anal pressure.
oo

oo

oo
eb

eb

eb
m

m
t

t
ne

ne

ne
e.

e.

e.
re

re

fre
sf

f
ks

ks
k
oo

oo

oo
eb

eb

eb
m

m
t

et

t
ne

ne
n
e.

e.

e.
t

t
256 Module 18

ne

ne

ne
e.

e.

e.
re

fre

fre
Answers
f
ks

ks

ks
SBAs
oo

oo

oo
461. Answer  A  Bladder training
eb

eb

eb
Explanation
Offer bladder training lasting for a minimum of 6 weeks as the first-­line treatment
m

m
to women with urgency or mixed urinary incontinence.
Reference
t

t
NICE. Urinary incontinence in women: management. NICE Clinical Guideline (CG 171).
e

ne

ne
September 2013.
.n

e.

e.
e

462. Answer  E  Supervised pelvic floor muscle training


re

fre

re
sf

sf
Explanation
ks
k

k
Offer a trial of supervised pelvic floor muscle training of at least 3 months’ duration
oo

oo

oo
as the first-­line treatment to women with stress or mixed urinary incontinence.
eb

eb

eb
Reference
NICE. Urinary incontinence in women: management. NICE Clinical Guideline (CG 171).
m

m
September 2013.

463. Answer  E 92%


t

et
ne

ne

Explanation

n
Women have used mechanical devices to reduce pelvic organ prolapse since
e.

e.

e.
ancient times, and the use of vaginal pessaries remains a simple and satisfactory
fre

fre

re
treatment. One study of 100 women using this method showed a 92% satisfaction
f
ks

ks

ks
rate in terms of prolapse symptoms.
oo

oo

oo

Reference
Jefferis H, Jackson SR, Price N. Management of uterine prolapse: is hysterectomy
eb

eb

eb

necessary? The Obstetrician & Gynaecologist 2016;18:17–23.


m

464. Answer  A 20%


Explanation
t

t
The incidence of asymptomatic bacteriuria during pregnancy is 2–5%, and if not
ne

ne

ne

treated, up to 20% of women will develop a lower UTI.


e.

e.

e.

Reference
re

re

fre

Asali F, Mahfouz I, Phillips C. The management of urogynaecological problems in


sf

f
ks

ks

pregnancy and the early postpartum period. The Obstetrician & Gynaecologist


k

2012;14:153–8.
oo

oo

oo

465. Answer  D Parity


eb

eb

eb

Explanation
m

Parity is associated with the greatest increase in risk of developing pelvic organ
prolapse.
t

et

t
ne

ne
n
e.

e.

e.
t

t
Urogynaecology and pelvic floor problems 257

ne

ne

ne
e.

e.

e.
re

fre

fre
Reference
f Asali F, Mahfouz I, Phillips C. The management of urogynaecological problems in
ks

ks

ks
pregnancy and the early postpartum period. The Obstetrician & Gynaecologist
oo

oo

oo
2012;14:153–8.
eb

eb

eb
466. Answer  A 2-fold
m

m
Explanation
Obesity appears to confer a 4-­fold and 2-­fold increased risk of urinary and anal
incontinence, respectively.
t

t
e

ne

ne
Reference
.n

Jain P, Parsons M. The effects of obesity on the pelvic floor. The Obstetrician &

e.

e.
e

Gynaecologist 2011;13:133–42.
re

fre

re
sf

sf
467. Answer  D Oxybutynin
ks
k

k
Explanation
oo

oo

oo
Do not offer oxybutynin (immediate release) to frail older women.
eb

eb

eb
Reference
m

m
NICE. Urinary incontinence in women: management. NICE Clinical Guideline (CG171).
September 2013.

468. Answer  E  Serotonin–noradrenaline uptake inhibitor


t

et
ne

ne

n
Explanation
e.

e.

e.
Duloxetine is a combined serotonin and noradrenaline reuptake inhibitor.
fre

fre

re
Adverse effects are largely related to increases in levels of noradrenaline and
serotonin, and include gastrointestinal disturbances, dry mouth, headache,
f
ks

ks

ks
decreased libido and anorgasmia.
oo

oo

oo

Reference
eb

eb

eb

Orme S, Ramsay I. Duloxetine: the long awaited drug treatment for stress urinary
incontinence. The Obstetrician & Gynaecologist 2005;7:117–19.
m

469. Answer  D  Within 6 months


Explanation
t

t
ne

ne

ne

Offer a follow-­up appointment (including a vaginal examination to exclude


erosion) within 6 months to all women who have had continence surgery.
e.

e.

e.

Reference
re

re

fre

NICE. Urinary incontinence in women: management. NICE Clinical Guideline (CG171).


sf

f
ks

ks

September 2013.
k
oo

oo

oo

470. Answer  A  Metabolic acidosis


eb

eb

eb

Explanation
Before augmentation cystoplasty, preoperative counselling for the woman or her
m

carer should include the common and serious complications: bowel disturbance,
metabolic acidosis, mucus production and/or retention in the bladder, UTI and
urinary retention.
t

et

t
ne

ne
n
e.

e.

e.
t

t
258 Module 18

ne

ne

ne
e.

e.

e.
re

fre

fre
Reference
f
NICE. Urinary incontinence in women: management. NICE Clinical Guideline (CG171).
ks

ks

ks
September 2013.
oo

oo

oo
471. Answer  A  M3 receptor antagonist
eb

eb

eb
Explanation
m

m
See Table 1 in the reference article.
Reference
Abboudi H, Fynes MM, Doumouchtsis SK. Contemporary therapy for the overactive
t

t
e

ne

ne
bladder. The Obstetrician & Gynaecologist 2011;13:98–106.
.n

e.

e.
e

472. Answer  E  Sacrospinous fixation


re

fre

re
Explanation
sf

sf
ks
Sacrospinous fixation is recommended if the vaginal vault is at the introitus at the
k

k
end of a vaginal hysterectomy procedure.
oo

oo

oo
Reference
eb

eb

eb
RCOG/BSUG. Post-hysterectomy vaginal vault prolapse. RCOG GTG No. 46. July 2015.
m

m
473. Answer  D  Percutaneous sacral nerve stimulation
Explanation
t

et
Offer percutaneous sacral nerve stimulation to women after a multidisciplinary
ne

ne

n
team review if:
e.

e.

e.
• Their overactive bladder has not responded to conservative management
fre

fre

re
including drugs and
• They are unable to perform clean intermittent catheterisation.
f
ks

ks

ks
Start treatment with botulinum toxin A only if the woman has been trained
oo

oo

oo

in clean intermittent catheterisation and has performed the technique


successfully.
eb

eb

eb

Restrict augmentation cystoplasty for the management of idiopathic detrusor


m

overactivity to women whose condition has not responded to conservative


management and who are willing and able to self-catheterise.
Reference
t

t
ne

ne

ne

NICE. Urinary incontinence in women: management. NICE Clinical Guideline (CG171).


September 2013.
e.

e.

e.
re

re

fre

474. Answer  D  Anterior vaginal wall 3 cm proximal to the hymen


sf

f
ks

ks

Explanation
k
oo

oo

oo

See Appendix 1 in the reference article.


eb

eb

eb

Reference
RCOG/BSUG. Post-hysterectomy vaginal vault prolapse. RCOG GTG No. 46. July 2015.
m

m
t

et

t
ne

ne
n
e.

e.

e.
t

t
Urogynaecology and pelvic floor problems 259

ne

ne

ne
e.

e.

e.
re

fre

fre
475. Answer  E 97%

f Explanation
ks

ks

ks
Colpocleisis has a short operating time and a low incidence of complications. One
oo

oo

oo
published study included 33 women and a second included 92 women. Success
rates of ≥97% have been reported.
eb

eb

eb
Reference
m

m
RCOG/BSUG. Post-hysterectomy vaginal vault prolapse. RCOG GTG No. 46. July 2015.
t

t
476. Answer  E Stress
e

ne

ne
.n

Explanation

e.

e.
e

A study of 565 patients with bladder pain syndrome was used to identify factors
re

fre

re
that can aggravate and alleviate this condition. Pain was found to be aggravated
sf

sf
by stress (61%), sexual intercourse (50%), constrictive clothing (49%), acidic
ks
k

k
beverages (54%), coffee (51%) and spicy foods (46%).
oo

oo

oo
Reference
eb

eb

eb
RCOG. Management of bladder pain syndrome. RCOG GTG No. 70. December 2016.
m

m
477. Answer  D 57–73%
Explanation
t

et
ne

ne

In a study of 565 patients with bladder pain syndrome, voiding was found to

n
relieve the pain in 57–73% of patients.
e.

e.

e.
fre

fre

re
Reference
RCOG. Management of bladder pain syndrome. RCOG GTG No. 70. December 2016.
f
ks

ks

ks
oo

oo

oo

478. Answer  B  Mild burning or bleeding during micturition


eb

eb

eb

Explanation
The risks associated with cystoscopy in women are as follows:
m

Common risks (>1 in 10):


• Mild burning or bleeding on passing urine for a short period after the
operation
t

t
• Biopsy of abnormal areas in bladder.
ne

ne

ne

Occasional risks (between 1 in 10 and 1 in 50):


e.

e.

e.

• Infection of the bladder requiring antibiotics.


re

re

fre

Rare risks (<1 in 50):


• Temporary insertion of a catheter
sf

f
ks

ks

• Delayed bleeding requiring removal of clots or further surgery


k
oo

oo

oo

• Injury to the urethra causing delayed scar formation


• Very rarely, perforation of the bladder requiring a temporary catheter or
eb

eb

eb

open surgical repair.


m

m
t

et

t
ne

ne
n
e.

e.

e.
t

t
260 Module 18

ne

ne

ne
e.

e.

e.
re

fre

fre
Reference
f
Lyttle M, Fowler G. Cystoscopy for the gynaecologist: how to do a cystoscopy. The
ks

ks

ks
Obstetrician & Gynaecologist 2017;19:236–40.
oo

oo

oo
479. Answer  B  Untreated UTI
eb

eb

eb
Explanation
m

m
The only true contraindication to cystoscopy is an untreated UTI, as outlined in
the British Association of Urological Surgeons (BAUS) guidelines.
Reference
t

t
e

ne

ne
Lyttle M, Fowler G. Cystoscopy for the gynaecologist: how to do a cystoscopy. The
.n

Obstetrician & Gynaecologist 2017;19:236–40.

e.

e.
e
re

fre

re
480. Answer  C  3 in 1000 women
sf

sf
ks
Explanation
k

k
oo

oo

oo
The risk of a pelvic abscess is 3 in every 1000 women (uncommon).
eb

eb

eb
Reference
RCOG. Vaginal surgery for prolapse. RCOG Consent Advice No. 5. October 2009.
m

EMQs
481. Answer  F  Digital assessment of pelvic floor contraction
m
t

et
ne

ne

n
Explanation
e.

e.

e.
Undertake routine digital assessment to confirm pelvic floor muscle contraction
fre

fre

re
before the use of supervised pelvic floor muscle training for the treatment of
urinary incontinence.
f
ks

ks

ks
oo

oo

oo

482. Answer  G  Filling and voiding cystometry


eb

eb

eb

Explanation
After undertaking a detailed clinical history and examination, perform
m

multichannel filling and voiding cystometry before surgery in women who have
had previous surgery for stress incontinence.
t

t
483. Answer  P  Urine dipstick test
ne

ne

ne

Explanation
e.

e.

e.

Undertake a urine dipstick test in all women presenting with urinary incontinence
re

re

fre

to detect the presence of blood, glucose, protein, leucocytes and nitrites in the urine.
sf

f
ks

ks

Reference
k
oo

oo

oo

NICE. Urinary incontinence in women: management. NICE Clinical Guideline (CG171).


September 2013.
eb

eb

eb
m

m
t

et

t
ne

ne
n
e.

e.

e.
t

t
Urogynaecology and pelvic floor problems 261

ne

ne

ne
e.

e.

e.
re

fre

fre
484. Answer  I Mirabegron

f Explanation
ks

ks

ks
Mirabegron is recommended as an option for treating the symptoms of
oo

oo

oo
an overactive bladder only for people in whom antimuscarinic drugs are
contraindicated or clinically ineffective, or have unacceptable side effects.
eb

eb

eb
Myasthenia gravis is a contraindication to antimuscarinics.
m

m
Reference
NICE. Mirabegron for treating symptoms of overactive bladder. NICE Technology Appraisal
Guidance (TA290). June 2013.
t

t
e

ne

ne
.n

485. Answer  F Duloxetine

e.

e.
e
re

fre

re
Explanation
Do not routinely offer duloxetine as a second-­line treatment for women with
sf

sf
ks
stress urinary incontinence, although it may be offered as second-­line therapy
k

k
oo

oo

oo
if women prefer pharmacological to surgical treatment or are not suitable for
surgical treatment. If duloxetine is prescribed, counsel women about its adverse
eb

eb

eb
effects.
m

m
486. Answer  D Desmopressin
Explanation
t

et
The use of desmopressin may be considered specifically to reduce nocturia
ne

ne

in women with urinary incontinence or an overactive bladder who find it a

n
e.

e.

e.
troublesome symptom. Use particular caution in women with cystic fibrosis and
avoid in those >65 years with cardiovascular disease or hypertension.
fre

fre

Reference f re
ks

ks

ks
NICE. Urinary incontinence in women: management. NICE Clinical Guideline (CG171).
oo

oo

oo

September 2013.
eb

eb

eb

487. Answer  O  Urinary tract infection (UTI)


m

Explanation
Urinary frequency or dysuria suggests a UTI, which is the most common cause of
haematuria in young women.
t

t
ne

ne

ne

488. Answer  A  Berger’s disease (IgA nephropathy)


e.

e.

e.

Explanation
re

re

fre

IgA nephropathy is the most common glomerulonephritis worldwide and tends to


sf

f
ks

ks

present in young adults within a few days of an upper respiratory tract infection.
k

Poststreptococcal glomerulonephritis would tend to present much later.


oo

oo

oo
eb

eb

eb
m

m
t

et

t
ne

ne
n
e.

e.

e.
t

t
262 Module 18

ne

ne

ne
e.

e.

e.
re

fre

fre
489. Answer  I  Polycystic kidney disease

f
Explanation
ks

ks

ks
Autosomal-dominant polycystic kidney disease is the most common inherited
oo

oo

oo
kidney disorder. Renal dysfunction may not present until after 40 years of age but
is often associated with hypertension. A renal calculus may give similar symptoms
eb

eb

eb
but would not be associated with hypertension and abnormal renal function tests.
m

m
Reference
Price N, Jackson S. Urogynaecology for the MRCOG and Beyond. 2nd edn. Cambridge:
Cambridge University Press, 2012.
t

t
e

ne

ne
.n

490. Answer   J  Open colposuspension

e.

e.
e
re

fre

re
Explanation
If conservative management for stress urinary incontinence has failed, offer one of
sf

sf
ks
the following:
k

k
oo

oo

oo
• A synthetic midurethral tape
• Open colposuspension
eb

eb

eb
• An autologous rectus fascial sling.
m

m
Reference
NICE. Urinary incontinence in women: management. NICE Clinical Guideline (CG171).
September 2013.
t

et
ne

ne

n
491. Answer  D  Augmentation cystoplasty
e.

e.

e.
fre

fre

re
Explanation
Restrict augmentation cystoplasty for the management of idiopathic detrusor
f
ks

ks

ks
overactivity to women whose condition has not responded to conservative
oo

management and who are willing and able to self-catheterise.


oo

oo
eb

eb

eb

Reference
NICE. Urinary incontinence in women: management. NICE Clinical Guideline (CG171).
m

September 2013.

492. Answer  E Colpocleisis


t

t
ne

ne

ne

Explanation
Colpocleisis is often reserved for elderly patients, in particular those with co-­
e.

e.

e.

morbidities that may render them unsuitable for the longer operating times and
re

re

fre

more invasive procedures associated with reconstructive surgery.


sf

f
ks

ks

Reference
k
oo

oo

oo

Jefferis H, Jackson SR, Price N. Management of uterine prolapse: is hysterectomy


necessary? The Obstetrician & Gynaecologist 2016;18:17–23.
eb

eb

eb
m

m
t

et

t
ne

ne
n
e.

e.

e.
t

t
Urogynaecology and pelvic floor problems 263

ne

ne

ne
e.

e.

e.
re

fre

fre
493. Answer  E  Bladder pain syndrome

f Explanation
ks

ks

ks
The widespread definition for bladder pain syndrome is that proposed by the
oo

oo

oo
European Society for the Study of BPS (ESSIC) in 2008 as ‘pelvic pain, pressure
or discomfort perceived to be related to the bladder, lasting at least 6 months, and
eb

eb

eb
accompanied by at least one other urinary symptom, for example persistent urge
m

m
to void or frequency, in the absence of other identifiable causes’.

Reference
t

t
RCOG. Management of bladder pain syndrome. RCOG GTG No. 70. December 2016.
e

ne

ne
.n

e.

e.
494. Answer  L  Urethral diverticulum
e
re

fre

re
Explanation
sf

sf
A urethral diverticulum may present with multiple symptoms. The historical
ks
k

k
classical triad of dysuria, postvoid dribbling and dyspareunia is only seen in
oo

oo

oo
a minority of patients. Lower urinary tract symptoms, namely frequency and
urgency, are present in 40–100% of cases.
eb

eb

eb
Reference
m

m
Archer R, Blackman J, Stott M, Barrington J. Urethral diverticulum. The Obstetrician &
Gynaecologist 2015;17:125–9.
t

et
ne

ne

495. Answer  M  Send urine for culture and sensitivity

n
e.

e.

e.
Explanation
fre

fre

re
If women do not have symptoms of a UTI but their urine tests positive for both
leucocytes and nitrites, do not offer antibiotics without the results of a midstream
f
ks

ks

ks
urine culture.
oo

oo

oo

496. Answer  E  Measure postvoid residual volume by bladder scan


eb

eb

eb

Explanation
m

Measure the postvoid residual volume by a bladder scan or catheterisation in


women with symptoms suggestive of voiding dysfunction or recurrent UTIs.
A bladder scan is used in preference to catheterisation on the grounds of
t

t
acceptability and a lower incidence of adverse events.
ne

ne

ne
e.

e.

e.

497. Answer  L  Refer to urologist


re

re

fre

Explanation
sf

f
ks

ks

Urgently refer women with urinary incontinence who have any of the
k

following:
oo

oo

oo

• Microscopic haematuria in women aged ≥50 years


eb

eb

eb

• Visible haematuria
• Recurrent or persisting UTI associated with haematuria in women aged
m

≥40 years
• A suspected malignant mass arising from the urinary tract.
t

et

t
ne

ne
n
e.

e.

e.
t

t
264 Module 18

ne

ne

ne
e.

e.

e.
re

fre

fre
Reference
f
NICE. Urinary incontinence in women: management. NICE Clinical Guideline (CG171).
ks

ks

ks
September 2013.
oo

oo

oo
498. Answer  L  Puborectalis muscle
eb

eb

eb
Explanation
m

m
The puborectalis muscle is the most caudal component of the levator ani complex
and is situated cephalad to the deep component of the external anal sphincter,
from which it is almost inseparable.
t

t
e

ne

ne
.n

499. Answer  B  Conjoint longitudinal coat

e.

e.
e

Explanation
re

fre

re
The anal sphincter complex consists of the external and internal anal sphincters
sf

sf
separated by the conjoint longitudinal coat.
ks
k

k
oo

oo

oo
500. Answer  E  Internal anal sphincter
eb

eb

eb
Explanation
The internal anal sphincter is innervated by the sympathetic (L5) and
m

m
parasympathetic (S2–S4) nerves and accounts for 50–85% of the resting anal
pressure.
t

et
Reference
ne

ne

n
Lone F, Sultan A, Thakar R. Obstetric pelvic floor and anal sphincter injuries. The
e.

e.

e.
Obstetrician & Gynaecologist 2012;14:257–66.
fre

fre

f re
ks

ks

ks
oo

oo

oo
eb

eb

eb
m

m
t

t
ne

ne

ne
e.

e.

e.
re

re

fre
sf

f
ks

ks
k
oo

oo

oo
eb

eb

eb
m

m
t

et

t
ne

ne
n
e.

e.

e.
t

t
ne

ne

ne
e.

e.

e.
Index
re

fre

fre
f
ks

ks

ks
oo

oo

oo
eb

eb

eb
abdominal pain, 66, 162, 168, antenatal care, 52–72, See also augmentation cystoplasty, 246,
m

m
181, 194 early pregnancy care 258, 262
abdominal pregnancy, 214, FGM and, 55, 66 auscultation of the fetal heart, 53
222 frequency of monitoring, 64 autosomal dominant, 89
abortion. See termination of haematological problems, 71 autosomal resessive, 89
t

t
e

ne

ne
pregnancy infections and, 69 azithromycin, 207
.n

Abortion Act 1967, 18 multiple pregnancy and, 65 azoospermia, 187, 191, 198

e.

e.
ACE inhibitors, 79 scan investigations/actions,
e

aciclovir, 110, 207 62 bacterial vaginosis, 180, 207


re

fre

re
acid–base balance, 34, 210 severe hypertension, 67 bacteriuria, 245
sf

sf
augmentation cystoplasty, thrombosis and embolism balanced chromosomal
ks
k

k
246 monitoring, 63 translocation, 212
oo

oo

oo
acute kidney injury, 96 antepartum haemorrhage, bariatric surgery, 22
acute renal failure, 144 55, 71 obstetric complications
eb

eb

eb
adenomyosis, 155, 184 anterior vaginal wall, 258 and, 56
adnexal cyst, 61 antibiotics basal FSH test, 193
m

m
adnexal mass, 215 intrapartum prophylaxis, benign tumours, most
air travel, and 59, 77, 82 common, 158
thromboprophylaxis, MRSA, 2, 5 benzodiazepines, 100
107, 120 perineal tear, 44 Berger’s disease, 261
t

et
ne

ne

alanine transaminase (ALT), 113 postpartum problems, 148 β-HCG test, 209, 215

n
alcohol prophylactic, 2, 4, 6 β-lactamase, extended
e.

e.

e.
cirrhosis and, 119 anticoagulants, 83, 84 spectrum, 148
in pregnancy, 57 anti-D immunoglobulin, 58 β-thalassaemia, 71, 94
fre

fre

re
α-fetoprotein (AFP), 104, 119 anti-D prophylaxis, 212 bilateral oophorectomy, 166,
amenorrhoea, 156, 162 anti-epileptics, 93, 100 182, 230 f
ks

ks

ks
aminosalicylates, 109 antihypertensives, 58, 98, 114 bilateral salpingo-
oo

oo

oo

amniocentesis, gestational age anti-K antibody, 58 oophorectomy, 31, 229,


and, 56 anti-Müllerian hormone 243
eb

eb

eb

amnioreduction, 74 (AMH) level, 187, 197 bilirubin, 113


amniotic fluid embolism, 99 antimuscarinics, 245, 261 biological syndrome, 154
m

amniotomy, 128, 131 antiretrovirals, 102 bipolar disorder, 95, 146, 149,
amylase, 113 antithrombin deficiency, 107, 163
anabolic steroid abuse, 197 121 bladder injuries, 21
anaemia, 71, 89 anti-Xa activity, peak, 95 caesarean section, 39, 43
t

t
ne

ne

ne

anal incontinence, 245 anxiety, 126 laparoscopy, 42


anal sphincter complex, 264 postpartum, 149 repairs, 31
e.

e.

e.

anal sphincter injury, anxiety disorder, 54 bladder management,


obstetric, 142 appendicitis, laparoscopy, 43 postpartum, 30
re

re

fre

analgesia appraisal, 12 bladder pain syndrome, 247,


sf

and OHSS, 185 artesunate, 110 263


ks

ks

in labour, 126–27 aspiration pneumonia, 146 bladder scan, 263


k
oo

oo

oo

regional, 111, 129 aspirin, 111 bladder training, 250, 256


androgens, 177 low dose, 115 blood loss, caesarean section,
eb

eb

eb

aneuploidy screening, 60, 80 asthma, 67, 144 140


angiotensin-receptor blockers atosiban, 124 blood transfusion
m

(ARBs), 79 atrophic vaginitis, 159, 160 intraoperative, 44


anorectal mucosa repair, 23 audit cycle, 15, 20 posthaemorrhage, 39

265
t

et

t
ne

ne
n
e.

e.

e.
t

t
266 Index

ne

ne

ne
e.

e.

e.
re

fre

fre
body mass index (BMI) >30 categorisation of traces, combined oral contraceptive
anal incontinence and, 245 127 pill (COCP), 145, 174,
f
ks

ks

ks
antenatal care and, 52 computerised, 75 181
cirrhosis and, 119 difficulty in interpreting, ovarian cancer and, 227
oo

oo

oo
endometrial biopsy, 155 136 combined screening test, 80
endometrial polyps, 157 failure to recognise complete molar pregnancy,
eb

eb

eb
glucose tolerance test, 101 abnormal, 128 212, 216
heavy menstrual bleeding, in labour, 123 complete septation, 237
m

m
164, 165 suspicious trace, 133 complex ovarian cyst, 237
hormone replacement catheter size, 22 compression duplex
therapy, 208 catheterisation, 246, 263 ultrasound, 84
postpartum headache, 147 ceftriaxone, 207 computerised CTG (cCTG),
t

t
e

ne

ne
proportion of women, 22 cell-free fetal DNA, 60 75
.n

subfertility problems, 188, cervical bleeding, 25 congenital malformations

e.

e.
e

196 cervical cancer, 230 anti-epileptics and, 93,


re

fre

re
thromboprophylaxis and, age ranges, 233 100
95, 107 squamous cell, 229 antihypertensives and,
sf

sf
ks
body weight, LMWH and, 95 stage 2, 234 58
k

k
borderline personality cervical dilation, 131 fusion in the female genital
oo

oo

oo
disorder, 154 cervical polyps, 175 tract, 160
botulinum toxin A, 258 cervical pregnancy, 217, 222, consent. See informed consent
eb

eb

eb
bowel obstruction, 229 224 conservative management
brachial plexus injury, 140 cervical screening, 232 gestational hypertension,
m

m
brachytherapy, 229 Charrière (Ch) gauge, 22 84
BRCA gene mutation, 229 chemotherapy, 108, 225 intramural fibroids, 193
breast cancer, 52, 91, 227 Chlamydia, 181, 214 twin pregnancy, 74
breastfeeding, type 2 diabetics, Chlamydia trachomatis, 201 contraception. See also
t

et
ne

ne

101, 117 chloroquine, 110 combined oral

n
breech birth chlorothiazide, 79 contraceptive pill
e.

e.

e.
emergency caesarean choriocarcinoma, 223 (COCP); family planning
section, 134 chorionicity, 65 emergency, 199
fre

fre

re
perinatal mortality, 134 chromosomal abnormality, perimenopausal woman,
buzz groups, 13 screening, 58 200 f
ks

ks

ks
chronic hypertension, 53, 98 copper intrauterine
oo

oo

oo

CA125. See serum CA125 chronic pelvic pain, 158, 168, contraceptive devices
caesarean scar pregnancy, 214 172 (IUCDs), 111, 144, 205
eb

eb

eb

caesarean section surgical management, 44, coroners, 20


bladder injuries, 21, 39, 43 45 corticosteroids, 110
m

breech birth and, 134 cleft lip, 100 crown–rump length, 72


category 2, 133 clindamycin, 110, 153, 207 ultrasound and, 85
emergency hysterectomy clinical governance, 14–20 cryoprecipitate, 151
and, 48 clinical skills, 1–6 CT, 237
t

t
ne

ne

ne

lactate levels and, 129 clinical trial, phases of, 15 CT pulmonary angiogram
postoperative infections, 32 clomifene, 189 (CTPA), 83
e.

e.

e.

stage three of labour, 134 clomifene citrate, 189, 196 CTG. See cardiotocography
stage two of labour, 134 Clostridium difficile, 44 (CTG)
re

re

fre

vaginal birth after, 135 cobalt isotopes, 240 cyclical pelvic pain, 175
sf

Caldicott guardian, 20 cognitive behavioural therapy cyst drainage, ultrasound-


ks

ks
k

candidiasis, vulvovaginal, 207 (CBT), 73 guided, 243


oo

oo

oo

carbamazepine, 94, 110 coincidental death, 115, cystic fibrosis, 261


carboplatin, 236 116 cystometry, filling and
eb

eb

eb

cardiac arrest, 99 colpocleisis, 51, 247, 262 voiding, 260


cardiac disease, 19, 91 colposcopy, 232 cystoscopy
m

atosiban and, 124 colposuspension, 51, 262 contraindications, 248


cardiotocography (CTG) combined hormonal risks of, 247
antenatal care, 73 contraception, 206 cytomegalovirus, 88
t

et

t
ne

ne
n
e.

e.

e.
t

t
Index 267

ne

ne

ne
e.

e.

e.
re

fre

fre
danazol, 181 early-onset group B episiotomy repair, 144
darifenacin, 246 streptococcus (GBS) epithelial ovarian cancer, 240
f
ks

ks

ks
death disease, 77 exercise in pregnancy, 52
classification of, 99, 146 early pregnancy care, 209–17, exogenous oestrogens, 176
oo

oo

oo
maternal. See maternal See also antenatal care expertise, determinants of, 7
death eclampsia extended-spectrum
eb

eb

eb
perinatal, 134 antenatal care, 67, 86 β-lactamase (ESBL), 148
death rates, extreme preterm postpartum problems, 146
m

m
births, 59 seizures and, 118 facial hair, excessive, 170
decelerations, CTG trace, 127, ectopic pregnancy faecal urgency, 44
132 abdominal, 214, 222 fallopian tube function, 184
deep dyspareunia, 172 cervical, 217, 222, 224 family history, ovarian cancer,
t

t
e

ne

ne
deep vein thrombosis (DVT), following IVF treatment, 239
.n

63, 84 214 Family Origin Questionnaire,

e.

e.
e

in pregnancy, 95 interstitial, 209 80


re

fre

re
deinfibulation, 66, 75, 86 management of, 215 family planning
delayed cord clamping, 133 salpingotomy after, 42 postnatal, and epilepsy, 94
sf

sf
ks
delivery, management of, tubal, 209, 222 postpartum, 144, 145, 200
k

k
133–39 Eisenmenger’s syndrome, 91 female genital mutilation
oo

oo

oo
depression electrolyte replacement (FGM), 2, 5
postnatal, 145, 149, 154 therapy, 1, 5 antenatal care and, 55, 66
eb

eb

eb
postpartum, 146 emergency contraception, 199 femoral neuropathy, 26, 38
dermoid cyst, 82, 169 emergency hysterectomy, 42 fenoprofen, 150
m

m
desmopressin, 261 enalapril, 98, 114 ferritin levels, 71, 89
desquamation of palms and endometrial ablation, 159 fetal abnormality, termination
soles, 77 second generation, 179 of pregnancy and, 61
diabetes endometrial biopsy, 160, 161, fetal blood sampling (FBS),
t

et
ne

ne

gestational, 101, 117, 138 171 123, 124

n
type 1, 96, 97 endometrial cancer. See fetal heart rate variation, 75
e.

e.

e.
type 2, 93, 101 uterine (endometrial) fetal heartbeat, absence of,
diabetic ketoacidosis (DKA), cancer 210, 215, 218
fre

fre

re
96, 97 endometrial hyperplasia fetal movements, reduced,
diamorphine, 111, 132 without atypia, 155, 160, 55, 60 f
ks

ks

ks
dichorionic diamniotic 161 fibrinogen concentrate, 151
oo

oo

oo

(DCDA) twin pregnancy, endometrial polyps, 157, 175 fibrinogen levels, 145
65 endometrial scratch, 186 fibroid uterus, 41
eb

eb

eb

direct death, 115, 152 endometriosis, 158, 181 fibronectin test, 142
DNA pox virus, 204 peritoneal, 156 Filshie clip, 3, 6
m

domestic violence, 61, 99 rectal, 190, 197 fishbowls, 13


donor insemination with subfertility and, 196, 190–91 fluconazole, 207
ovulation induction, 195 endometritis, 37 folic acid, 92
Down’s syndrome, 57 atrophic, 160 forceps delivery, 135
t

t
ne

ne

ne

genetics and, 88 enhanced recovery planning, bladder management,


drospirenone, 180 30 30
e.

e.

e.

Duchenne muscular Enterobacteriaceae, 27 Kielland, 142


dystrophy, 89 enterocele, 51 fosphenytoin, 116
re

re

fre

duloxetine, 245, 261 Entonox, 111 French gauge (Fg), 22


sf

dura mater puncture, 124, 152 epidurals, dura mater frequently occurring risk,
ks

ks
k

dural puncture headache, 152 puncture, 124, 152 34


oo

oo

oo

dyschezia, 190 epigastric pain, 104 full blood count (FBC),


dysgerminomas, 231, 239 epilepsy 179
eb

eb

eb

dyskaryosis, 232 anti-epileptics, 93, 100 fusion in the female genital


dysmenorrhoea, 164, 172, 179, maternal death, 99 tract, 160
m

190 opiates and, 94


dyspareunia, 163, 172, 253 pain relief in labour, 126 genital herpes, 93, 207
dysuria, 202, 253 postnatal contraception, 94 gentamicin, 148
t

et

t
ne

ne
n
e.

e.

e.
t

t
268 Index

ne

ne

ne
e.

e.

e.
re

fre

fre
germ cells, in testes, 191, 198 heavy menstrual bleeding, enhanced recovery
gestational age, 72 155, 156, 160 planning, 30
f
ks

ks

ks
amniocentesis and, 56 management of, 31, 160, laparoscopic assisted
cell-free fetal DNA, 60 164 vaginal, 47
oo

oo

oo
magnesium sulfate and, 122 tests taken, 165–66 most common serious risk,
multiple pregnancy and, 65 HELLP syndrome 39
eb

eb

eb
small for, 69, 76 acute kidney injury and, 96 perioperative
gestational diabetes, 101, 117, thrombotic complications, 41
m

m
138 thrombocytopenic postoperative
gestational trophoblastic purpura (TTP) and, 118 complications, 31
disease, 212 hepatic haemangioma, 118 serious complications from,
glibenclamide, 117 hepatitis B, 104, 119 22
t

t
e

ne

ne
glucose screening for, 59 vaginal, 246, 248
.n

monitoring, 93 hepatitis C, 119 vaginal vault prolapse post,

e.

e.
e

plasma glucose levels, 93, hepatocellular carcinoma, 119 46


re

fre

re
101, 117 hepatosplenomegaly, 69 hysterosalpingogram (HSG),
glucose tolerance test, 101 HIV infection, in pregnancy, 187
sf

sf
ks
glycine overload, 37 102 hysteroscopy, 238
k

k
GnRH analogues, 158, 174, hormone replacement therapy arterial blood gas levels, 35
oo

oo

oo
180, 181 (HRT), 200, 203 heavy menstrual bleeding,
gonadotropins, 196 hot flushes, 156 180
eb

eb

eb
gonococcal infection, 207 human epidermal growth pain relief after, 23
gonorrhoea, 181 factor receptor 2 (HER2), uterus perforation, 41
m

m
Graves’ disease, 113 91
ground E abortions, 82 Human Fertilisation and ibuprofen, 194
group A Streptococcus, 148 Embryology Authority ICSI with donated eggs, 195
group B Streptococcus (GBS), (HFEA), 194 idiopathic detrusor
t

et
ne

ne

57, 59, 82 human papillomavirus (HPV), overactivity, 252, 262

n
early-onset disease, 77 229, 232, 237 IgA nephropathy, 261
e.

e.

e.
growth scan, 62 Huntington’s disease, 89 immature teratomas, 239
gynaecological oncology, hydatidiform mole, 223 indirect death, 115, 116
fre

fre

re
225–43 hydralazine, 87 indomethacin, 150
5-year survival rates, 234 hydronephrosis, unilateral, induction of labour f
ks

ks

ks
age ranges, 233 230 and antibiotic prophylaxis,
oo

oo

oo

cervical screening, 232 hydrosalpinges, 185 81


stage of disease, 230 hyperandrogenism, 170, 187 and maternal age, 123
eb

eb

eb

tumour markers, 231 hyperemesis, 216 management of, 53


gynaecological problems, hyperemesis gravidarum, 96, vaginal examination, 77
m

155–70 211, 219 vaginal PGE2, 128


gynaecomastia, 191, 198 thyroid function tests, 211 infections. See also urinary
hypertension tract infections (UTIs)
haematuria, 254, 261 antihypertensives, 58, 98, antenatal care and, 69
t

t
ne

ne

ne

haemoglobinopathy 114 pelvic, 202


screening, 59 chronic, 53, 98 postoperative, 32
e.

e.

e.

haemophilia, 88 gestational, 115 surgical site, 22


haemorrhage postpartum treatment, 144 wound, 37
re

re

fre

antepartum, 55, 71 severe gestational, 64, 67, 84 inflammatory bowel disease,


sf

cystic mass, 216 treatment of, 151 92


ks

ks
k

postpartum, 145 hyperthyroidism, 96, 183 information technology (IT),


oo

oo

oo

requiring blood transfusion, hypoactive sexual desire 14–20


39 disorder (HSDD), 199 informed consent
eb

eb

eb

Hartmann’s solution, 1, 5 hysterectomy, 14, 155 obtaining, 18


Hasson (open) technique, 24 arterial blood gas levels, 35 presence of students, 14
m

headaches emergency, 42 process of, 3


postdural puncture, 124 endometrial hyperplasia, inguinal lymphadenitis, 202
postpartum, 147 176 inhibin, 241
t

et

t
ne

ne
n
e.

e.

e.
t

t
Index 269

ne

ne

ne
e.

e.

e.
re

fre

fre
insulin, 117 lactate dehydrogenase (LDH), liver, lesions of the, 104
insulin-like growth factor- 228, 241 losartan, 98
f
ks

ks

ks
binding protein-1, 141 lactational amenorrhoea low-molecular-weight heparin
intermenstrual bleeding, 165 method (LAM), 200 (LMWH), 63, 95, 111,
oo

oo

oo
intermittent auscultation, 141 lamotrigine, 110 120
interstitial ectopic pregnancy, laparoscopic-assisted vaginal lower urinary tract infection,
eb

eb

eb
209 hysterectomy (LAVH), 245
intra-abdominal pressure, 47 lymph nodes, 225, 230
m

m
laparoscopy, 43 laparoscopic ovarian
intrahepatic cholestasis of cystectomy, 42 M3 receptor antagonist, 258
pregnancy, 96 laparoscopy magnesium sulfate, 67, 86,
intramural fibroids, 184 actions/manoeuvres at 122, 142
t

t
e

ne

ne
intravenous fluid therapy, 23 incision, 45 malaria, 92
.n

maintenance volume, 24 appendicitis, 43 malaria prophylaxis, 54

e.

e.
e

intravenous rehydration, 211 bladder injuries, 31, 42 malignancy index (RMI)


re

fre

re
in vitro fertilisation (IVF) chronic pelvic pain, 44, 45 score, 157, 226, 242
ectopic pregnancy conversion to mini- malignant ovarian germ cell
sf

sf
ks
following, 214 laparotomy, 41 tumours (MOGCTs),
k

k
hydrosalpinges prior to, 185 depth below indented 227, 228
oo

oo

oo
maternal age and, 187 umbilicus, 24 malposition, reversion to, 137
OHSS and, 185 entry-related injuries, 29 malpresentation at term
eb

eb

eb
recurrent implantation Hasson technique, 24 spinal cord injuries and, 73
failure, 186 incision location, 45 manual rotation of fetal head,
m

m
vasa praevia and, 56 oophorectomy. See 143
iridium isotopes, 240 oophorectomy maternal age
iron supplements, 71, 89 ovarian drilling, 196 induction of labour and, 123
irritable bowel syndrome, 181 risks of, 3, 6 IVF treatment, 187
t

et
ne

ne

salpingectomy, 195 miscarriage and, 217

n
jaundice, 69 salpingotomy, 222 reproductive outcome and,
e.

e.

e.
laparotomy, 181 192
K antigen, 79 full-staging, 242 maternal death
fre

fre

re
karyotype 23,X, 223 mini, 41 causes of, 16, 99
karyotype 46,XX, 162, 223 ovarian tumours, 238 f
classification, 146
ks

ks

ks
karyotype 46,XY, 178 laser ablation, 74 congenital heart disease, 108
oo

oo

oo

karyotype 69,XXY, 223 late death, 116 definitions of, 115


karyotypes, 216 learning methods, 8 reporting, 20
eb

eb

eb

karyotyping of products of leg, pain and swelling, 63 types of, 99


conception, 221 levonorgestrel, 199 maternal medicine, 91–108
m

ketoacidosis, 96, 97 levonorgestrel-releasing maturation arrest, 187, 195,


kidney injury, acute, 96 intrauterine system 198
Kielland forceps (KF), 142 (LNG-IUS), 111, 155, Mayer–Rokitansky–Kuster–
Klinefelter’s syndrome, 198 156, 160, 164, 171, 174, Hauser syndrome
t

t
ne

ne

ne

knowledge 177 (MRKH), 177


importance of, 11 lichen planus, 156, 178, 237 MBRRACE report 2016, 16
e.

e.

e.

retention of, 8 lichen sclerosus, 178, 237 McCall culdoplasty, 51


Krukenberg tumours, 236 lichen simplex, 178 mebeverine, 181
re

re

fre

lichenification, 163, 178 medroxyprogesterone acetate


sf

labetalol, 64, 67, 98, 115, 136, 150 liquor volume, 62 (MPA), 111, 177
ks

ks
k

labour, management of, assessment, 73 mefenamic acid, 164, 179


oo

oo

oo

122–27, See also antenatal lithium treatment, 95 meiosis, in the testes, 184
care, See also caesarean litigation membrane sweeps, 56, 122
eb

eb

eb

section, See also preterm obstetric claims, 122 menstruation. See heavy
labour, See also induction shoulder dystocia, 134 menstrual bleeding
m

of labour liver cirrhosis, 119 mental health services,


pain relief, 126–27 liver function tests (LFTs), perinatal, 152
lactate, 123, 124 96, 104 meropenem, 153
t

et

t
ne

ne
n
e.

e.

e.
t

t
270 Index

ne

ne

ne
e.

e.

e.
re

fre

fre
mesalazine, 109 neurodevelopmental risk factors, 228
metabolic acidosis, 257 disability, 81 screening, 228
f
ks

ks

ks
metabolic alkalosis, 219 neuroprotection, 142 stage 3, 234
metformin, 35, 40, 101, 117, magnesium sulfate for, 128 ovarian cystectomy, 42, 181
oo

oo

oo
196 nifedipine, 142, 150 ovarian cysts, 155, 169, 182
methicillin-resistant nitrous oxide, 123 complex, 237
eb

eb

eb
Staphylococcus aureus nocturia, 250, 261 malignancy, 226
(MRSA), 2, 5, 148 non-epileptic seizure disorder, management options,
m

m
methotrexate, 222 118 234–43
methyldopa, 53, 98, 114, 145 non-steroidal anti- persistent, 230
microcephaly, 69 inflammatory drugs surgery, 41
micronised progesterone, 180 (NSAIDs), 28, 144, 179, ultrasound parameters, 157
t

t
e

ne

ne
midurethral tape procedure, 194 ovarian drilling, 196
.n

30 normal saline, 219 ovarian dysgerminoma, 231

e.

e.
e

migraine, 153 norpethidine, 111, 132 ovarian hyperstimulation


re

fre

re
Miller’s pyramid, 7 nuchal fold measurement, 53 syndrome (OHSS), 185
mini-laparotomy, 41 nuchal translucency analgesia and, 185
sf

sf
ks
mirabegron, 261 measurements, 80 IVF and, 185
k

k
miscarriage nucleic acid amplification test ovarian hyperthecosis, 182
oo

oo

oo
diagnosis of, 218 (NAAT), 206 ovarian reserve, 192
fetal heart activity, 210 overactive bladder, 245
eb

eb

eb
gestational trophoblastic obesity. See body mass index treatment of, 246
disease, 212 (BMI) >30 oxybutynin, 250, 257
m

m
maternal age and, 217 Objective Structured oxygen saturation, 126, 132
missed, 218 Assessment of Technical oxytocin, 128, 129, 131, 139,
recurrent, 137, 211, 212, 213 Skills (OSAT), 8 140, 143
molar pregnancy, 212, 216 obstetric anal sphincter injury oxytocin/ergometrine, 143
t

et
ne

ne

molluscum contagiosum, 208 (OASIS), 142

n
monochorionic diamniotic oestrogen, transdermal, 166, paclitaxel, 236
e.

e.

e.
(MCDA) twin pregnancy, 208 Palmer’s point, 50
54, 55, 65 oestriol, 159, 241 paracetamol, 194
fre

fre

re
antenatal care, 58 oestrogen receptor, 91 paranoia, 149
morphine, 2, 5 oestrogens, exogenous, 176 f
paraplegia, and pregnancy, 54
ks

ks

ks
mortality. See death oestrone, 173 partial molar pregnancy, 216
oo

oo

oo

moxifloxacin, 207 ofloxacin, 207 partial moles, triploid, 217,


MRI, 74, 87 oligohydramnios, 75 223
eb

eb

eb

myasthenia gravis, 250, 261 omental biopsy, 238 partner abuse, 99


myomectomy, 41, 193 oncology, gynaecological. See during pregnancy, 61
m

gynaecological oncology pelvic abscess, 248


naproxen, 150 oophorectomy pelvic floor muscle training,
National Health Service bilateral, 166, 182, 230 249, 250, 256
Litigation Authority bilateral salpingo-, 31, 229, pelvic infection, 202
t

t
ne

ne

ne

(NHSLA), 122 243 pelvic organ prolapse


nausea, 194 unilateral, 182, 238 aetiological risk factors, 245
e.

e.

e.

nausea and vomiting, 1 open abdominal vaginal pessaries, 244


acid–base balance, 210 sacrocolpopexy, 48 pelvic pain
re

re

fre

hyperemesis gravidarum, opiates, 194, See also morphine chronic. See chronic pelvic
sf

211 epilepsy and, 94 pain


ks

ks
k

intravenous rehydration, orchidopexy, 188, 196 cyclical, 175


oo

oo

oo

211 ovarian cancer, 225, 226, 230 subfertility and, 189, 190
peak of, 210 age ranges, 233 urinary urgency and, 253
eb

eb

eb

severity of, 210 bowel obstruction and, 229 pelvic ultrasound, 179
NCEPOD categories, 14 combined oral pemphigoid gestationis, 109
m

nephrotoxicity, 109 contraceptive pill and, Pendleton’s rules, 9, 12


nerve damage, postoperative, 227 percutaneous sacral nerve
33 germ cell tumours, 227, 228 stimulation, 258
t

et

t
ne

ne
n
e.

e.

e.
t

t
Index 271

ne

ne

ne
e.

e.

e.
re

fre

fre
perinatal mental health preconception counselling, preterm rupture of
services, 152 52 membranes
f
ks

ks

ks
perinatal mortality, 134 pre-eclampsia management of delivery,
perineal tear, repair of, 44 acute kidney injury and, 136, 141
oo

oo

oo
peritoneal cancer, 229 113 primaquine, 110
peritoneal endometriosis, 156 antenatal care, 67, 87 primary metabolic acidosis
eb

eb

eb
peritoneal washing, 230, 238 management of delivery, with respiratory
permethrin, 204 136 compensation, 40
m

m
peroneal neuropathy, 39 postpartum problems, 144 primary metabolic alkalosis
petechial rash, 69 tests at diagnosis, 84 with respiratory
pethidine, 111, 132 TTP and, 118 compensation, 40
pH, fetal scalp blood, 124 women at high risk, 115 primary respiratory acidosis,
t

t
e

ne

ne
phenytoin, 116 pregnancy. See also antenatal 39
.n

placenta care; early pregnancy primary respiratory alkalosis,

e.

e.
e

delivery of, 150 care; labour, management 39


re

fre

re
manual removal of, 139 of; twin pregnancy primiparity, and postpartum
retained, 133 alcohol during, 57 psychosis, 150
sf

sf
ks
placenta praevia, 87 bacteriuria in, 245 procidentia, 46
k

k
emergency hysterectomy caesarean scar, 214 progesterone
oo

oo

oo
and, 48 dating scan, 52 micronised, 180
plasma glucose levels, 93, 101, DVT in, 95 serum, 196
eb

eb

eb
117 ectopic. See ectopic progesterone-receptor
Plasmodium vivax, 92 pregnancy modulator-associated
m

m
pneumococcal vaccine, 92 exercise during, 52 endometrial changes
pneumonia, aspiration, 146 glucose monitoring in, 93 (PAEC), 157
polycystic kidney disease, 262 HIV infection management, progestogen-only implant, 206
polycystic ovarian syndrome 102 progestogens, 159, 174, 177
t

et
ne

ne

(PCOS), 35, 182, 195 intrahepatic cholestasis proteinuria screening, 60, 84

n
polycystic ovaries, 188 of, 96 prothrombin gene mutation,
e.

e.

e.
polydioxanone (PDS), 21, 28 molar, 212, 216 95
polyglactin (Vicryl), 28 partner abuse during, 61 pruritus, 202
fre

fre

re
polymorphic eruption of spinal cord injuries and, 54 psychosexual disorders, 205
pregnancy, 109 spontaneous intrauterine, psychosis f
ks

ks

ks
postablation tubal sterilisation 193 postpartum, 144
oo

oo

oo

syndrome (PATSS), 175 termination of. See puerperal, 154


postdural puncture headache, termination of pregnancy pubococcygeus spasm, 205
eb

eb

eb

152 pregnancy-unique puborectalis, 264


postmenopausal women quantification of emesis pudendal neuropathy, 38
m

bleeding, 160, 161 (PUQE), 219 puerperal psychosis, 154


cervical and endometrial premature ovarian pulmonary atelectasis, 38
polyps, 159 insufficiency, 156 pulmonary embolism (PE),
ovarian cysts, 155, 226 premenstrual syndrome, 158, 83, 84
t

t
ne

ne

ne

postnatal depression, 145, 149, 166 pyelonephritis, 148, 153


154 preterm labour pyrexia, 32
e.

e.

e.

postoperative care, 30–40 delayed cord clamping, 133


postpartum depression, 146 disabilities and, 59 quadruple test, 80
re

re

fre

postpartum haemorrhage, 145 magnesium sulfate and, 122 quality assessment criteria,
sf

postpartum problems, 144–49 management of, 129 9, 12


ks

ks
k

postpartum psychosis, 144 multiple pregnancies, 60 quality of life, 165, 176


oo

oo

oo

post-thrombotic syndrome poor prognosis following, quinine, 110


(PTS), 95 137 quinolones, 207
eb

eb

eb

post-traumatic stress disorder survival rates, 59


(PTSD), and pregnancy, thyroid disease and, 93 radioactive iodine, 183
m

54 preterm prelabour rupture of radiotherapy, 108, 229


postvoid residual volume, 263 membranes (PPROM), rectal endometriosis, 190, 197
potassium chloride, 1, 5, 219 57, 93 rectal examination, 197
t

et

t
ne

ne
n
e.

e.

e.
t

t
272 Index

ne

ne

ne
e.

e.

e.
re

fre

fre
regional analgesia, 111, 129 sodium valproate, 116 Swyer syndrome, 178
rehydration, intravenous, 211 Solomon technique, 74 Syntocinon, 133, 143
f
ks

ks

ks
remifentanyl, 132 somatic subtype, 154 Syntometrine, 143
renal failure, acute, 144 speculum examination,
oo

oo

oo
rescue cervical cerclage, 142 PPROM, 77 tachycardia, 63
research, 14–20 spermatids, 194 tremor, 130
eb

eb

eb
respiratory depression, 132 spermatogonia, 194 tamoxifen, 52, 108, 227
reversible cerebral spina bifida, 100 targeted biopsy, 238
m

m
vasoconstriction spinal anaesthesia, 2 teaching and assessment, 7–10
syndrome (RCVS), 153 spinal cord injuries, and teicoplanin, 153
Rhodes index, 219 pregnancy, 54 TENS, 111
rifampicin, 113 splenectomy, 94 termination of pregnancy
t

t
e

ne

ne
right ventricular dysfunction, spontaneous intrauterine congenital heart disease,
.n

108 pregnancy, 193 108

e.

e.
e

risk of malignancy index spontaneous rupture of fetal abnormality and, 61


re

fre

re
(RMI), 157, 226, 242 membranes, 59, 102, 125 ground E abortions, 82
risks of an operation, 34 squamous cell carcinoma testes, end product of meiosis,
sf

sf
ks
rubella, 88 cervical, 229 184
k

k
vulval, 225, 227 testicular failure, 183
oo

oo

oo
sacrocolpopexy, 48, 51 staphylococcal toxic shock, 56 testosterone, 161, 208
sacrospinous fixation, 51, 258 sterilisation, postablation tetralogy of Fallot, 91
eb

eb

eb
saline, 114, 219 tubal, 159 thalassaemia, 71, 89, 94
salpingectomy, 48, 195 steroids, 110, 231 thrombophilia, test for, 121, 221
m

m
salpingo-oophorectomy, abuse of, 197 thromboprophylaxis, 94, 95,
bilateral, 31, 229, 243 prophylactic, 137 107–8
salpingotomy, 42, 215, 222 stockings, anti-embolic, 120 thrombotic thrombocytopenic
Sarcoptes scabiei, 208 Streptococcus. See group A purpura (TTP), 118
t

et
ne

ne

SBAR format, 7 Streptococcus; group B thyroid disease, 93, 96, 183

n
schizophrenia, 95, 146, 149, 163 Streptococcus (GBS) thyroid function tests, 211
e.

e.

e.
seizures. See also epilepsy stress urinary incontinence, thyrotoxicosis, 220
causes of, 103 30, 244, 246, 249, 250, 252 tocolytics, 137, 142
fre

fre

re
self-harm, 149, 154 postoperative symptomatic, toxic shock syndrome, 56, 148
sensitivity of an assay, 78 46, 51 f
tranexamic acid, 164, 179
ks

ks

ks
serious risk, 34 stress, and bladder pain transcervical resection of the
oo

oo

oo

serotonin–noradrenaline syndrome, 259 endometrium (TCRE), 31


uptake inhibitor, 257 subfertility, 183–91 transdermal continuous
eb

eb

eb

serum AMH, 197 endometriosis and, 196, combined HRT, 208


serum CA125, 158, 171, 174, 190–91 transdermal oestrogen, 166,
m

230, 235 investigation options, 189 208


tumour marker, 226, 236 male factors causing, 185 transforming growth factor-β
serum FSH, 156, 162, 178, 187 treatment options, 188 (TGF-β), 231
serum ketones, 97 unexplained, 183, 193, 196 transobturator tape, 246
t

t
ne

ne

ne

serum LH levels, 197 subgaleal haematoma, 137 transvaginal ultrasound, 87, 129
serum progesterone, 196 suicide, 19, 154 early pregnancy, 209
e.

e.

e.

sexual and reproductive sulfasalazine, 92 ectopic pregnancy, 222


health, 199–204 surgery endometriosis, 197
re

re

fre

shoulder dystocia, 134 breast cancer, 108 miscarriage, 210, 218


sf

sickle cell disease, 80, 92 core skills, 21–29 tubal ectopic pregnancy, 218
ks

ks
k

sitagliptin, 101 ovarian cancer, 236 transverse vaginal septum,


oo

oo

oo

small for gestational age ovarian tumours, 228 157, 177


(SGA), 76 procedures, 41–47 trastuzumab, 108
eb

eb

eb

infections and, 69 surgical evacuation, 216 tremor tachycardia, 130


small group teaching, 10 surgical evacuation of the Trendelenburg position, 50
m

smoking, 107, 126, 202 uterus, 35, 44, 212 Trichomonas vaginalis, 200,
snowball groups, 13 suture materials, 23 208
sodium chloride, 28 absorbable, 21 trichomoniasis, 181
t

et

t
ne

ne
n
e.

e.

e.
t

t
Index 273

ne

ne

ne
e.

e.

e.
re

fre

fre
triplet pregnancy, 65 urinary catheter size, 22 vaginal vault level, 246
trisomy 21, 88 urinary incontinence vaginal vault prolapse, 42, 46
f
ks

ks

ks
trophoblastic disease, 212 obesity and, 257 posthysterectomy, 46
tubal ectopic pregnancy, 209, stress. See stress urinary vaginismus, 199
oo

oo

oo
222 incontinence vaginitis, atrophic, 159, 160
tumour markers, 226, 228, 231 with urgency, 244, 249, 250, valaciclovir, 207
eb

eb

eb
twin pregnancy 252, 253 vancomycin, 153
DCDA, 65 urinary tract infections vasa praevia, IVF treatment
m

m
delivery, 138 (UTIs), 251, 254 and, 56
MCDA, 54, 55, 58, 65 lower, 245 vasectomy, 164
preterm delivery, 60 untreated, 260 failures from, 3, 6
thromboprophylaxis, 107 urine dipstick test, 260 venous thromboembolism
t

t
e

ne

ne
ultrasound scanning and, 65 urogynaecology, 244–55 (VTE), 95, 107, 121
.n

twin-to-twin transfusion ursodeoxycholic acid postdelivery, 145

e.

e.
e

syndrome (TTTS), 54 (UDCA), 96 postpartum family


re

fre

re
type 1 diabetes, 96, 97 urticaria, 92 planning, 151
type 2 diabetes, 93, 101 uterine (endometrial) cancer, ventilation/perfusion (V/Q)
sf

sf
ks
236 scan, 83
k

k
UK Medical Eligibility Criteria age ranges, 233 ventouse, 142
oo

oo

oo
for Contraceptive Use radiotherapy, 229 Veress needle, 45, 50
(UKMEC), 199 stage 1, 234 vesicovaginal fistula, 37
eb

eb

eb
ulipristal acetate, 157 uterine fibroids, 23, 157 viral load, 117
ultrasound uterine hyperstimulation, 128 vitamin A, 59
m

m
adenomyosis, 184 uterine leiomyoma, 174 vitamin D, 53
compression duplex, 84 uterine perforation, 41 voiding, and bladder pain
crown–rump length and, 85 uterine septum, 175 syndrome, 259
cyst drainage, 243 uterotonic agents, 134, 138–39 vulval biopsy, 178
t

et
ne

ne

intramural fibroids, 184 uterovaginal prolapse, 248, vulval cancer, 230

n
multiple pregnancy and, 65 252 vulval intraepithelial neoplasia
e.

e.

e.
OHSS, 185 (VIN), 227
ovarian cysts, 157, 171 vagina, short, blind ending, vulval lichen planus, 156
fre

fre

re
pelvic, 179 162 vulval squamous cell
transvaginal. See vaginal birth after a caesarean f
carcinoma, 225, 227
ks

ks

ks
transvaginal ultrasound (VBAC), 135
oo

oo

oo

umbilical adhesions, 50 vaginal bleeding, 23, 209, 215, weight loss, subfertility
umbilical artery absent or 221, 227 problems, 196
eb

eb

eb

reversed end-diastolic vaginal breech birth, 134 workplace-based assessment,


velocity (AREDV), 75 vaginal discharge, 167, 202 7, 8
m

umbilical artery Doppler, 62 vaginal hysterectomy, 246 World Health Organization


umbilical hernias, 44 laparoscopic assisted, 47 (WHO), classification of
umbilical port, 47 pelvic abscess after, 248 death, 99
unilateral oophorectomy, 182, vaginal pessaries, 244, 252 wound infections, 37
t

t
ne

ne

ne

238 vaginal prostaglandin (PGE2),


ureteric injuries, 21 128 X-linked recessive, 88, 89
e.

e.

e.

urethral diverticulum, 263 vaginal spotting, 175


urgency urinary incontinence, vaginal tape, 249 zidovudine, 102
re

re

fre

244, 249, 250, 252, 253 vaginal vault bleeding, 25 Zika virus, 88
sf

f
ks

ks
k
oo

oo

oo
eb

eb

eb
m

m
t

et

t
ne

ne
n
e.

e.

e.

Das könnte Ihnen auch gefallen